Бигуди для извилин. Возьми от мозга все!

advertisement
Нурали Нурисламович Латыпов
Бигуди для извилин. Возьми от мозга все!
Текст предоставлен правообладателем
http://www.litres.ru/pages/biblio_book/?art=8371507
«Бигуди для извилин. Возьми от мозга все! / Латыпов Н. Н.»: АСТ; Москва; 2014
ISBN 978-5-17-087281-7
Аннотация
Почему умные люди на самом деле тупые? – Задает вопрос знаменитый интеллектуал
Нурали Латыпов. Без постоянных упражнений даже самый умный человек теряет хватку и
сообразительность. Автор предлагает всем, кто хочет оставаться в хорошей
интеллектуальной форме целый набор увлекательных задач, своеобразных снарядов для
умственного фитнеса.
Нурали Нурисламович Латыпов
Бигуди для извилин. Возьми от мозга все!
Научные консультанты:
профессор, доктор технических наук Николай Николаевич Карнаухов
профессор, доктор технических наук Николай Денисович Цхадая
Под редакцией Олега Вячеславовича Гаврикова
Выражаю искреннюю признательность моим коллегам и соавторам по предыдущим
книгам на тему развития изобретательских способностей – «Самоучитель игры на
извилинах» и «Турбулентное мышление. Зарядка для интеллекта» – Дмитрию Гаврилову и
Сергею Ёлкину за полезные обсуждения и конструктивную критику по ходу работы над этим
изданием.
Моя особая благодарность Сергею Тушеву, Вере и Владимиру Бреусам – за
неоценимую помощь в обработке материалов, использованных в процессе работы над
книгой, и, конечно, моей семье – Резеде, Дамиру и Тимуру Латыповым – за неизменную
поддержку в творчестве, которое выстраивает порядок в буквах и увы беспорядок в доме.
Предисловие
Широкомасштабные исследования работы человеческого мозга проводятся в последние
годы в США, странах Евросоюза, а также в Японии, Китае, Израиле. Гигантские финансовые
средства, которые были при этом задействованы, имеют как бюджетное, так и корпоративное
происхождение. Цель, в лучшем случае, – это создание искусственного мозга, в «худшем» –
«нейроморфных» компьютеров.
Вселенная, окружающая и включающая нас, имеет такую степень сложности, что
постижение этого Космоса возможно только с помощью инструмента, по сложности
превышающей саму Вселенную. Теорему Гёделя никто не отменял!
Таким инструментом может быть только человеческий мозг. Микрокосм, который он
создает, по богатству ничуть не уступает макрокосму, который его окружает. Задуматься
только, это розовато-серое студенистое образование может создавать миры, каждый из
которых отдельная Вселенная!
Крушение Советского Союза с его богатыми традициями в исследованиях мозга
совпали со всплеском интереса к этой проблематике со стороны стран Запада. Ученые США
и Европы даже объявили этот период «десятилетием мозга». Более сотни ведущих ученых
мира по результатам опроса самого авторитетного журнала мира «Science» в 2005 году
вынесли вердикт, что главными научными проблемами современности является проблема
строения Вселенной и проблема биологической основы сознания.
Решительный штурм второй проблемы возглавляет США. Барак Обама, при всех его
внешнеполитических ошибках, во внутренней политике придерживается разумной и
перспективной линии. Он заявил: «Настало время выйти на уровень научных исследований и
разработок, невиданный с момента пика космической гонки». Все секвестры американского
бюджета обходят перспективные научные разработки. Президент США считает, что сейчас
не время ужимать вложения в науку и инновации, и даже наоборот, следует «инвестировать
в великие идеи».
Такую Америку стоит догонять и перегонять! Ведь именно идеи являются отправной
точкой любого научно-технического рывка. Конкретная идея, конечно, не рождается в
коллективах института и лаборатории, она рождается в голове отдельно взятого человека.
Безусловно, речь идет о способных «человеках», а вот таких способных на протяжении
многих десятков лет США собирают со всего мира в своих лабораториях и университетах.
Разумеется, американцы не сгоняют этих одаренных людей силой, и даже высоченные
зарплаты не могут объяснить стремления мыслителей всех рангов вписаться в
инфраструктуру американской науки. Радость творчества в коллективах себе подобных
(«Счастье – это когда тебя понимают,» – написал в школьном сочинении одной строкой
герой фильма «Доживем до понедельника») возникает через глубинные структуры того же
мозга, через те же самые эндорфинные механизмы, которые обеспечивают более
приземленные радости (вроде секса и еды).
В свое время США не пожалели денег на грандиозный проект «Геном человека».
«Каждый доллар, который мы вложили в создание карты человеческого генома, вернул по
140 долларов в нашу экономику! Каждый! А сегодня наши ученые создают карту мозга». По
словам президента США новый мегапроект «Карта активности мозга» рассчитан на десять
лет и пока «тянет» на три миллиарда долларов. На «Геном…» потратили почти на один
миллиард больше, и это вернулось сторицей, что и констатировал Обама. Еще не успел
финишировать прежний проект, как уже американское агентство по перспективным
оборонным разработкам DARPA (Defense Advanced Research Projects Agency), а также
корпорации COOGLE и МICROSOFT «скидываются на троих» на новый. Участники проекта
«Карта активности мозга». планируют начать с «извилин» главной лабораторной героини
«дрозофилы», потом – рыбки, потом – мышки, и так далее – до человека. Проект настолько
же грандиозный, насколько и сумасшедший, поскольку, если брать только численность
нейронов нашего мозга, их порядка ста миллиардов, а соединений (синапсов) вообще
квадриллион. К тому же четкой специализации мыслительных процессов «по секторам» в
мозге нет, есть зафиксированные только частности, но скорее всего мозг работает как
целостная, единая система. Лауреат Нобелевской премии Нильс Бор говаривал: «Идея
недостаточно сумасшедшая, чтобы быть правильной». Поэтому тратить деньги на такого
рода сумасшедшие идеи считаю мудрым шагом, речь конечно же не идет об идеях всяких
докторов Петриков.
Пребывающий в экономической стагнации Евросоюз тоже раскошелился на один
миллиард евро – на проект «Человеческий мозг». Конечная цель – построение модели
человеческого мозга, или, так называемый «кремниевый мозг» – виртуальный ансамбль
виртуальных нейронов, обитающих в суперкомпьютере. Директор проекта, профессор
Швейцарского федерального технического института Лозанны Генри Маркрам уже создал с
помощью суперкомпьютера потомка DEEP BLUE, победившего в свое время Гарри
Каспарова, ансамбль из миллиона виртуальных нейронов. В швейцарской Лозанне полным
ходом идет строительство лабораторий, из которых вырастет город Нейрополис, и уже
двадцать две страны участвуют в этом проекте.
Пока же профессор Маркрам ведет исследования компьютерной корпорации IBM, и это
не случайность, поскольку самые крупные научные открытия происходят сейчас не в
государственных, а в корпоративных лабораториях. IBM. Кстати, вместе с вышеописанным,
там же осуществляется и проект SyNAPSE (DARPA’s Systems of Neuromorphic Adaptive
Plastic Scalable Electronics). Цель – «создание интеллектуальных компьютеров, способных к
самостоятельному усвоению новых знаний из различных источников, распознаванию
образов, продолжительному обучению, пониманию контекстуального значения
многозначной информации для решения сложных проблем в условиях реального мира на
основе способностей к восприятию, действиям и познанию».
По большому счёту все проекты такого рода, включая отечественные – ряда
лабораторий Курчатовского института, есть создание когнитивного нейроморфного
компьютера, который способен учиться и анализировать информацию «по-человечески» на
основе архитектуры, имитирующей наш мозг.
«С учетом нынешних темпов развития области искусственного интеллекта к 2028 году
компьютеры смогут обучаться на уровне человека, самостоятельно искать информацию,
перерабатывать ее и приобретать индивидуальные человеческие черты» – так считает Рэй
Курцвейл, футуролог и по совместительству директор отдела разработок компании Google, –
«Все это приведет к тому, что менее чем через два десятилетия вы уже будете не просто
использовать компьютеры, у вас с ними будут складываться своеобразные отношения, чемто напоминающие отношения между людьми».
Но вот что парадоксально: наш мозг настолько феноменален, что у него будут учиться
компьютеры будущего. Иными словами, завтрашние компьютеры пытаются «превратить» в
мозг. И в это же время большинство людей низводят лучший подарок, который они
получили от природы до уровня компьютера. Причём не в такой, который обыгрывает в
шахматы чемпионов мира, а в самый что ни на есть захудалый, с примитивными операциями
типа «стрелок» на клавиатуре.
«Проблема современности», – писал ещё Антон Павлович Чехов. – «Дело не в
оптимизме или пессимизме, а в том, что у девяноста девяти из ста нет ума»
Чтобы не войти в эти 99, нужно постараться. Как и каким образом стараться? Вместе со
своими коллегами я постарался дать в этой книге развёрнутый ответ на сей вопрос.
Послесловие к предисловию
Года два назад в американском журнале «Gizmodo» появилась любопытная статья,
название которой на русский переводится дословно как «Почему умные люди на самом деле
тупые». В частности говорилось, что группа изыскателей из Университета Торонто
проводила мониторинг способностей порядка пяти сотен студентов. Среди вопросов был и
такой: «Бейсбольная бита и мяч стоят доллар и десять центов. Бита стоит больше на
доллар, чем мяч. Сколько стоит мяч?»
Первоклассная задача, то есть для первоклассников. И автор этой книги, и целый ряд
его друзей, не торопились с ответом сходу, уже хотя бы завидев подозрительный заголовок
статьи. Между тем б олее 50 % студентов Гарварда, Принстона и Массачусетского
технологического института дали неверный. шаблонный ответ, вроде бы напрашивающийся
– 10 центов. Хотя уже упомянутый первоклассник, впрочем, советской школы, абсолютно
точно высчитал бы, что 5.
Объяснить этот феномен можно с опорой на последние достижения в области
исследования мозга с помощью функциональной магнитно-резонансной томографии,
которая позволяет фиксировать, в каких участках мозга наблюдается особенная активность
при решении тех или иных задач. Оказалось, что вобщем-то мозг – это своеобразный кот,
который гуляет сам по себе: может принимать и не принимать, решать то так, до эдак, а
главное, как и всё в природе, он стремится к экономии ресурсов. Мозг может потреблять от 5
до 60 % энергии, вырабатываемой человеческим организмом. Эволюционно экономия
собственной энергии оправдана, без неё ни человек, ни иной биологический вид, не выжили
бы на Земле. Опора на шаблоны и стереотипы – один из защитных механизмов по снижению
энергопотребления. Я читал, что американские нейробиологи проводили исследования
шахматистов. Мастера столь детерминированной игры зачастую, сами того не желая,
пропускали те ходы, которые вели к победе или уходу от поражения, будучи в рамках
привычной и столетиями расписанной схемы шахмат.
Так стоит ли заставлять мозг работать всегда и во всём? Как это сделать наилучшим
образом? Стоит ли вообще ссориться с мозгом? Или можно найти вариант мирного
сосуществования?
Леность ума, о которой упоминал классик, постигший со своим художественным,
образным восприятием глубины мира два века назад, теперь выявлена в натуральном виде с
помощью современных высокоточных научных средств.
Выходит, с одной стороны, нужно прилагать немалые усилия воли, чтобы заставить
собственный мозг не соскакивать на трафаретные ходы! С другой стороны, руководители
целых государств и корпораций должны иметь в виду, что отдача от научных и инженерных
кадров зачастую зависит от силы воли реализовать этот потенциал. Конечно, это может
приобретать, как учит история, и весьма жестокие формы. Вспомнить те же «шарашки» или
«атомный проект». Но результат налицо: и фантастически короткие сроки и фантастические
результаты!
«Почему умные люди на самом деле тупые»? Хотя и парадоксальный в части названия,
но далеко не единственный в части содержания материал, попавший в поле моего зрения за
последнее время.
В журнале «Trends in Genetics» опубликована статья генетика из Стэнфордского
университета доктора Джеральд Крэбтри, рассматривающего интеллектуальное угасание
человечества на фоне изменений в окружающей среде и неблагоприятных по совокупности
факторов мутаций нашего вида:
«Я готов поспорить, что если бы средний житель Афин 1000 года до нашей эры
неожиданно оказался среди нас, то он или она оказались бы среди самых умных и
интеллектуально живых людей современности, с хорошей памятью, широким спектром идей,
и ясным представлением о важных предметах. Более того, я предполагаю, что он или она
оказались бы одними из самых эмоционально стабильных личностей», – пишет учёный.
«Дарвиновская теория о «выживании наиболее приспособленных» менее применима
сегодня, поскольку те, кто обладает лучшими генами, не обязательно будут доминировать в
обществе так, как это было в прошлом…» – комментирует Крэбтри автор русского перевода.
В кризисных условиях, а человечество находится как раз в глубоком кризисе по многим
параметрам, выживает не умнейший, а более примитивный организм, чья задача получить
элементарные удовольствия биологического характера.
Впрочем, именно человеку, но отнюдь не обезьяне, свойственно получать
удовлетворение от решённой интеллектуальной задачи. И коль ныне в чести фабрики
всевозможных удовольствий, хотя бы из чувства самосохранения надо развивать и
культивировать чувство удовольствия от затрат на умственный труд. Иначе на человечестве
впору поставить крест.
Быть может потому, что зачатки такой радости от результатов интеллектуальной
деятельности, то есть очевидные признаки разума, хоть и иного разума, наблюдаемы у
дельфинов, Министерство окружающей среды и лесов Индии приняло решение запретить
содержание дельфинов в неволе и использование их в сфере развлечений по всей стране. В
заявлении от 17 мая 2013 Министерство посоветовало правительствам всех штатов
отклонить любые предложения о создании дельфинариев от любых лиц, организаций,
государственных учреждений, частных или государственных предприятий, которые
включают импорт, захват китообразных, организацию коммерческих развлекательных,
частных или государственных выставок, экспозиций или шоу.
Индия – страна с миллиардным населением, многотысячелетней традиционной
культурой, древнейшей религией. Быть может, в мире что-то и сдвинется к лучшему, если
остальная часть человечества задумается над свершившимся?!
«Принимая во внимание высокий по сравнению с другими животными интеллект и
чувствительность китообразных, который признается различными исследователями,
дельфинов следует рассматривать как «персон не-людей», наделённых их специфическими
правами. Поэтому держать их в плену для развлекательных целей морально неприемлемо», –
говорится в сообщении министерства.
Ric O’Barry, бывший дрессировщик дельфинов и нынешний директор американского
Earth Island Institute’s приветствовал принятие постановления: «Индийское правительство
высказалось не только против жестокости, оно инициировало важную дискуссию о природе
дельфинов, взглянув на них как на чувствующих и мыслящих существ, а не как на объекты
собственности, на которых можно делать деньги».
Но вернёмся к «хомо сапиенсам». Достойны ли они жизни, цель которой одни
развлечения?
***
Все наши органы имеют характерную форму. Самый же главный из этих органов имеет
форму, которую ему придаёт одно из самых главных его содержимых – желеобразная плёнка
толщиной около четырёх миллиметров, состоящая из нервных клеток – нейронов. Она
называется корой головного мозга. Чем больше «серого вещества», тем шире кора и тем
больше извилин. Если мысленно извилистую кору раскатать скалкой до гладкости, то мы
получим блин, площадью в три раза большей, чем внутренняя поверхность черепной
коробки.
При всей опосредованности извилин в качестве «индикаторов» интеллектуального
развития человека, в аллегорическом смысле они всё-таки получили неоспоримый статус
мерила человеческого ума. Собственно, поэтому они заняли место и в названии этой книги.
На её страницах будет приведен целый набор «бигудей» (методик, упражнений, задач) в
качестве своеобразных «снарядов» для умственного фитнеса, помогающих развивать
извилины и поддерживать их в должной интеллектуальной форме.
Возвращаясь к структуре человеческого мозга, нельзя не задаться вопросом, кто же
складывает кору такими извилинами, что они образуют рисунок одинаково подобный у всех
людей, от аборигенов Австралии до эскимосов Аляски?
Первые шесть месяцев развития эмбриона человека кора у него гладкая, а потом
нейроны активизируются, тянут отростки от одного к другому, цепляются друг за друга,
создавая нервные волокна, соединяющие одни участки коры мозга с другими. И как
резиновые жгуты они начинают кору стягивать за счёт сил натяжения. Мозг начинает
складываться в гармошку, а поскольку черепная коробка округлая, и внутричерепное
пространство криволинейное, то и складки искривляются. Такова гипотеза нейробиолога
Дэвида Ван Эссена (David Van Essen) из Университета им. Дж. Вашингтона в Сент-Луисе.
Сложный процесс – сложный результат.
– Мозг самое сложное, что есть во Вселенной, – говорит профессор Т. Черниговская,
один из лучших российских специалистов по изучению особенностей интеллекта. – Найти,
что-либо, по сложности сопоставимое с ним, не хватает никакой фантазии.1
Первым же исследователем извилин стал уроженец острова Кеос, ученик великого
Теофраста (прозванного «отцом ботаники») великий врач и исследователь Эразистрат. И
было это лет, примерно, за двести до нашей эры… Вот тогда-то Эразистрат и обнаружил в
коре головного мозга некие извилины и объяснил именно наличием разветвленной сети
извилин и борозд в полушариях мозга умственное превосходство человека над другими
живыми существами.
Кстати, он же впервые использовал слово «мозг» для названия мыслительного
1 Наука и жизнь. – 2012. – № 11. – С. 26–30.
устройства в человеческой голове.
Однако, человек ушёл в отрыв от остального животного мира не только по количеству
извилин. Ещё одним нау-хау творца стало разделение функций полушарий, причём с
широкой «автономией» каждого из них. Правда, у современных ученых уже нет
стопроцентного единогласия в трактовках этого подхода.
– Четкой локализации функций в мозге вообще нет, – считает директор Института
мозга человека РАН Святослав Медведев. – Вернее, её имеют интерфейсы взаимодействия с
миром, такие как механизм зрительного восприятия. А высшими формами деятельности
занимается мозг целиком. В итоге четкое картирование мозга наподобие картирования
генома человека невозможно. Одни поля коры мозга включены в какую-то человеческую
деятельность больше, другие меньше, но мозг работает как единое целое.
Английский исследователь Тимоти Кроу предположил в середине девяностых, что
появлению речи2 способствовала асимметрия мозга, то есть несимметричность функций
правого и левого полушарий.
Кроу исходит из того, что асимметрия полушарий есть результат генетической
мутации. Она способствовала резкому росту мощи мозга, но, одновременно заложила основы
серьёзных психических осложнений. Многие люди не справляются с управлением
асимметричным мозгом. Интеллектуальный локомотив, бывает, съезжает с пути. Происходит
своеобразная неврогенная катастрофа. Одна из статей Кроу так и называлась «Is
schizophrenia the price that Homo sapiens pays for language?», то есть «Шизофрения – цена,
которую homo sapience платит за язык?».
«Раньше была распространена демонстрация деятельности мозга с помощью рисунка,
где голова расписана как лоскутное одеяло – платоническая любовь отдельно, жадность и
дружба, вранье – отдельно: все расписано, – констатирует уже упомянутая мной доктор
филологических и биологических наук Т. Черниговская в одной из своих лекций.3 – Идея о
том, что в мозгу есть адреса и каждый из которых «своим делом» занимается, долго
держалась. Правда состоит в том, что они одновременно и есть, и нет. Если кирпичом дать
по зоне Брока, то человек говорить не будет. Если случится инсульт в зоне Вернике – не
будет понимать речь. Но если вы записываете с помощью все более усложняющихся
приборов мозговую активность любой сложной деятельности, то вы увидите, что этой
деятельностью занят весь мозг. Работает вся нейронная сеть: какие-то её куски больше,
какие-то меньше, но заняты этим все. До сих пор идет спор (в частности, в лингвистике) о
том, по какому принципу организован мозг – по модульному или по сетевому».
Хорошим подспорьем в картографии мозга могла стать структурная томография –
наука, хорошо оснащенная современными приборами, дающая очень точные и объективные
результаты.
Но, по мнению Криса Фритта, автора монографии «Мозг и душа», в психологических
исследованиях мозга помогли не структурные, а функциональные томографы, разработанные
через несколько лет после структурных. Они позволяют регистрировать потребление
энергии тканями мозга. «Бодрствуем мы или спим, 15 миллиардов нервных клеток
(нейронов) нашего мозга постоянно посылают сигналы друг другу. При этом тратится
немало энергии. Наш мозг потребляет около 20 % энергии всего тела, несмотря на то, что его
масса составляет лишь около 2 % от массы тела. Весь мозг пронизан сетью кровеносных
сосудов, по которым и переносится энергия в форме кислорода, содержащегося в крови.
Распределение энергии в мозгу очень точно отрегулировано, так чтобы в те участки мозга,
которые в настоящий момент наиболее активны, её поступало больше. Эта связь между
2 Timothy Crow, «A Darwinian Approach to the Origins of Psychosis», British Journal of Psichiatry (167) 1995;
Timothy Crow, «Is schizophrenia the price that Homo sapiens pays for language?», Schizophrenia Research, (28) 1997.
3 http://www.genlingnw.ru/person/Chernigovskaya.htm
активностью мозга и локальными изменениями кровотока была известна физиологам уже
больше 100 лет, но до изобретения функциональных томографов не было возможности
регистрировать подобные изменения, <…> указывающие на то, какие области мозга в
настоящий момент наиболее активны».
И, тем не менее, несмотря на разные мнения и подходы, «районирование» деятельности
мозга подтверждается результатами, полученными вполне натурфилософскими способами.
Так, Александр Лук, задавшийся целью описать связь между работой мозга и такими её
проявлениями, как юмор и творчество, пришел к выводу, что «расстройства остроумия
бывают при поражении лобных долей мозга – опухолью или абсцессом. При этом отмечается
не просто «выпадение». Ведь существует немало людей вполне здоровых и не склонных к
острословию. А при поражении лобных долей больные очень охотно шутят. Но шутки
становятся плоскими и грубыми. Приемы ограничиваются буквализацией метафоры и
двойным истолкованием в самой примитивной форме. Тематика сосредоточивается вокруг
физиологических отправлений организма. Этот «лобный юмор» настолько характерен, что
опытный невропатолог по одной только манере острить может заподозрить у таких людей
патологический процесс лобной доли мозга».
Ученые самых разных специализаций советуют учиться работать правым и левым
полушариями синхронно – это не только поможет использовать генетический потенциал
мозга, но и в значительной степени защитит вас от расщепления сознания, характерного для
упомянутой выше шизофрении.
А непревзойденным тренером для развития обоих полушарий является математика.
Упрощенно говоря, алгебра постигается одним полушарием, геометрия – другим. А вот
чтобы «пробить» аналитическую геометрию или тензорный анализ, нужна уже совместная
работа обоих полушарий. Математика – один из немногих учителей, способных научить
полушария мозга работать вместе, как единое целое.
Открытие известного французского представителя нейронауки Станисласа Дехене
(Stanislas Dehaene) тоже касается деятельности зоны мозга, которая становится активной,
когда специалист по математике смотрит на формулу. Но если ту же формулу будет
рассматривать человек, не сведущий в математике, то такая зона остается пассивной.
Проведя аналогичные исследования и процесса чтения текста, суть своего открытия
Дехене объясняет так: «Не существует единой зоны сегментации символов». В частности,
речь идет о хорошо известном речевом центре Брока. То есть способность внутренней
обработки символов принадлежит многим зонам мозга. себе в помощницы ученый
призывает цитату одного высказывания А. Эйнштейна: «Ни устный, ни письменный язык не
участвовали в формировании моих мыслей». Иными словами, язык и математика не связаны
между собой.
Но следует заметить: поскольку математика возникла как способ упорядочения всех
накопленных приёмов размышлений, она даёт каждому, кто удосужится погрузиться в её
основы, громадный набор не только готовых способов думания, но и приёмов дальнейшего
упорядочения всех собственных находок. То есть именно математика – главный инструмент
борьбы с энтропией (мерой хаоса) сознания.
Выдающийся отечественный философ Эвальд Васильевич Ильенков установил: в мозгу
человека практически нет встроенных структур с конкретными рефлексами и навыками
поведения. Зато необычайно – куда лучше, чем у большинства прочих животных – развита
способность к установлению взаимосвязей между малейшими крупицами накапливаемого
опыта.
Потому я и призываю читателя изучать математику – пусть даже не ради её самой, а
ради силы и красоты мысли. С твёрдой уверенностью могу заявить, что математика –
дизайнер мысли. Кстати математические упражнения оттачивают ещё и ОстроУмие! Да, да!
Не удивляйтесь!
Ещё Аристотель полагал, что остроумие – это дерзость, получившая образование. Этот
термин зачастую относят только к сфере сатиры и юмора, но ведь бывают и остроумные
решения проблем, остроумные доказательства. Вспомним хотя бы генеального чудака
Григория Перельмана: он, по меркам науки совершенно недавно, разрешил задачу другого
гения, Анри Пуанкаре, над которой учёные умы бились более сотни лет. Вот красивая
аналогия: для того, чтобы проткнуть прочный материал, нужно создать на его поверхности
область высокого давления. Какой инструмент нужен? Шило! Вот так и острый ум – в
нужном месте он создает область «высокого давления» на проблему. И проблема
«прокалывается»!
Александр Лук считал остроумие одним из компонентов продуктивного мышления и
удобным материалом для исследования, поскольку его проявление может быть оценено
людьми разных специальностей, «в отличие от решения, скажем, математической или
технической задачи. Изучая процесс синтезирования остроты, можно приблизиться к
пониманию некоторых закономерностей мышления и творчества».
Исследователь приводит факты, подтверждающие существование такого подхода ещё в
XVII веке: у испанского философа Грасиана один из его трактатов назывался «Остроумие,
или Искусство ума». В том же столетии итальянец Эммануэле Тезауро высказался по этому
вопросу так: «Остроумие заключает в себе два естественных дара: Прозорливость и
Многосторонность. Прозорливость проникает в самые дальние и едва заметные свойства
любого предмета… Многосторонность быстро схватывает эти сущности, их отношения
между собой и к самому предмету, она связывает их и разделяет, выводит одно из другого,
распознает одно по намекам другого и с поражающей ловкостью ставит одно на место
другого… Все это не что иное, как Метафора, мать Поэзии, Остроумия, Замыслов, Символов
и героических Девизов».
Еще одна выдающаяся придумка эволюции – зеркальные нейроны. Мы обращали на
неё внимание читателей ещё в прошлой книге – «Инженерная эвристика»:
«Около пятнадцати лет назад их открыла итальянская исследовательская группа
Джакомо Риззолатти, профессора Пармского университета, родившегося кстати в 1930-х
годах в Киеве. Это эпохальное открытие было сделано в какой-то степени случайно. Группа
Риззолатти регистрировала электрическую активность мозга макак. Им давали пищу,
упакованную в коробку, обезьянами вручили и некоторый набор инструментов. Как-то раз,
неосознанно, один из исследователей на глазах подопытной макаки вскрыл коробку таким
же подвернувшимся ему инструментальным набором. Обезьяна не шелохнулась, но датчик
показал, что кора мозга резко увеличила активность4, которая стала зеркальным отражением
другой, зарегистрированной – когда животное само проделывало эту процедуру.
Зеркальные нейроны, открытые командой Риззолатти у обезьян, вскоре были
обнаружены и у человека. Но кора головного мозга человека активизируется не только когда
он смотрит, но и в том случае, если он мысленно имитирует, моделирует, воображает ту же
процедуру. И если мы задумаемся о механизмах обучения, запоминания, то важнейшая часть
этого механизма – способность к подражанию, имитации. Зеркальные нейроны – это особые
клетки головного мозга, которые служат для понимания действий других, подражания или
сопереживания им, для обучения и трансляции знаний. Возможно с помощью этих клеток
человек постигает реальность не логической цепочкой размышлений, а цельным
чувственным пониманием».
В этой связи стоит вспомнить ещё одного выдающегося исследователя. Более ста лет
назад, то есть задолго до современных нам открытий по части строения и развития мозга,
французский криминалист и социолог психологической школы Габриэль Тард (1843–1904),
основываясь на собственном опыте работы, предвосхитил многие выводы профессора
Риззолатти, сделанные с помощью мощного электронного инструментария.
Согласно теории Тарда, изложенной им впервые в работе «Законы подражания» в 18904 Хотя обезьяна не выполняла действие сама, клетки её мозга отреагировали, когда знакомое действие
совершил человек.
м году, вся история общества есть научение по подражанию. Вывод: истину мы можем
постигнуть не только с помощью точнейших и тончайших изысканий, но и
натурфилософским осмыслением. Поэтому значительная часть сегодняшних открытий это
фактически экспериментальное подтверждение многих озарений прошлых поколений
учёных.
Да и многие методы, освещённые в нашей книге, разработаны исследователями разных
времён, но они не теряют своей ценности даже в потоке новейших, сенсационных открытий
в нейрологии.
Примечательна и статья Маркуса Райхла «Тёмная энергия мозга»5, на которую мы
также обращали внимание читателей в нашей книге «Самоучитель игры на извилинах» пару
лет назад. Автор – профессор радиологии и неврологии в медицинском колледже при
Университете имени Вашингтона в Сент-Луисе. Он уже много лет возглавляет группу
исследователей процессов человеческого мозга с помощью томографии: «Активность мозга
в те моменты, когда человек бездействует или грезит наяву, может стать ключом к
пониманию природы неврологических заболеваний и даже самого феномена сознания…» –
утверждает автор. Далее он формулирует важный вывод: «Долгое время нейрофизиологи
считали, что когда человек находится в покое, его мозг неактивен. Однако эксперименты с
применением методов томографии показали, что в мозге поддерживается постоянный
уровень фоновой активности. Этот пассивный режим, возможно, необходим для
планирования будущих действий…»
Мозг, скорее всего, восстанавливает недостатки информации об окружающем мире с
помощью своеобразных вычислений, поскольку из внешней среды на центральный «сервер»
поступает в конечном счёте мизерное количество сведений. Например, из 6 млн. бит,
проходящих через зрительный нерв, только десяток тысяч битов достигает участков коры,
ответственных за визуальную информацию. Дальше – меньше. Всего несколько сотен бит
участвуют в зрительном осознании. Поэтому от 60 до 80 % энергии, используемой мозгом,
уходит на «разработку» внутренней информации, восполняющей дефицит внешней.
Крис Фрит утверждает, что психические события, определяющие наши движения,
происходят не одновременно с физическими событиями: «Мозговая активность, связанная с
тем или иным движением, начинается до того, как мы осознаём свое намерение совершить
это движение, но движение «запускается» после того, как мы осознаём, что запускаем его.
Намерение и начало движения отделены во внутреннем времени нашего сознания меньшим
промежутком, чем в реальном времени.
Из этого открытия следует вывод, что, измеряя активность вашего мозга, я могу узнать,
что у вас возникнет желание поднять палец раньше, чем об этом узнаете вы сами.
Мы думаем, что делаем выбор, в то время как на деле наш мозг этот выбор уже сделал.
Следовательно, ощущение, что в этот момент мы делаем выбор, не более чем иллюзия. А
если ощущение, что мы способны делать выбор, есть иллюзия, то такая же иллюзия – наше
ощущение, что мы обладаем свободой воли.
Эти наблюдения демонстрируют, что наше тело может превосходно взаимодействовать
с окружающим миром даже тогда, когда мы сами не знаем, что оно делает, и даже тогда,
когда наши представления об окружающем мире не соответствуют действительности. Может
быть, наш мозг и связан с нашим телом напрямую, но поставляемые нам мозгом сведения о
состоянии нашего тела, похоже, носят такой же косвенный характер, как и поставляемые нам
сведения об окружающем мире. Мозг может не сообщить нам, что наше тело движется не
так, как мы хотели. Мозг может обмануть нас, заставив думать, что тело находится не там,
где оно находится на самом деле. И все эти примеры относятся к взаимодействию здорового
мозга со здоровым телом. Когда с человеком не все в порядке, его мозг способен вытворять и
не такое».
5 В мире науки. – 2010. – № 5. – С.22–27.
Не раз упомянутая здесь профессор Т. Черниговская вообще считает, что мозг просто
морочит нам голову, поскольку большая часть работы он проделывает сам и отнюдь не с
помощью сознания, а вне его. «Он, вообще, какой-то снобисткий, самостоятельный, и делает
что хочет. А это не очень приятно».
«Мы стали называть эту внутреннюю активность «тёмной энергией» мозга, – словно
вступает в перекличку Маркус Райхл. – В астрономии тёмная энергия – невидимая,
гипотетическая энергия, которая составляет значительную часть так называемой скрытой
массы Вселенной».
Я уверен, что именно эта «тёмная энергия» и есть фундамент пресловутой интуиции,
это и есть то самое подсознание. Извилины безусловно надо напрягать, но не случайно
озарения наступают в моменты, когда казалось бы ты о проблеме не думаешь,
расслабляешься. И само сознание расслабляется, а где-то там, внутри, дозревает и
кристаллизуется мысль, и вот в такой-то момент и начинает действовать вулкан информации
из подсознания.
Известные натурфилософы современности, например, Ганс Селье, полагали, что
озарение наступает в пограничном состоянии, «где-то» между сознательным и
бессознательным. Именно Селье заметил, что переходы от сна к бодрствованию и обратно,
недомогание и выздоровление меняют работу мозга, переключают его. Извилины,
безусловно, надо напрягать, но чтобы мысль закрутилась, чтобы она выкристаллизовалась, а
это происходит в подсознании, необходимо дать расслабиться сознанию!
Сколько параллелей, между фитнесом тела и тренировкой сознания! А как все же
оценить результат? Во всяком случае, справедливости ради, от себя замечу, что один из
общепринятых способов – тест IQ – рассчитан на людей с преимущественно
алгоритмическим или причинно-следственным мышлением. При этом очень сложно
анализировать интеллектуальные возможности людей с другими типами мышления, включая
вихревой или аналоговый. Обладатели этих типов мышления не мыслят строгими
цепочками, от причины к следствию, и не организуют мыслительные потоки по нескольким
направлениям – их «котелок» варит иначе. Так что тест IQ если и не совсем порочен, то
показывает в высшей степени условные результаты, не подходящие для справедливой
оценки.
Учёные проанализировали данные по 1983 представителям 99-ти популяций всех
населённых континентов. И полученные данные свидетельствовали о гибридизации
кроманьонцев, наших ближайших предков, вымершими видами неандертальцев.
«Одна из интереснейших целей генетики и молекулярной биологии – загадка
нейроэволюции, – говорит Константин Анохин – нейрофизиолог профессор, членкорреспондент РАН. – Я называю этим словом феномен, открытый в последние годы
молекулярной генетикой. Простой, но поразительный факт: чтобы создать любой из наших
внутренних органов, требуется всего три, пять, иногда семь процентов всех генов. А для
создания мозга у человека и других млекопитающих предназначено более половины генов в
составе генома. Задумайтесь: каждый второй из наших генов работает для обеспечения
функций мозга. Это в корне меняет представления о содержании эволюции. Нежная ткань
мозга не сохранялась у ископаемых животных, и палеонтологи более двух веков изучали в
основном строение тела, скелета, черепов и костей. Но сегодня мы начинаем осознавать, что
основные усилия эволюции мира животных были затрачены именно на создание мозга…
Мозг – орган, экспрессирующий наибольшее число генов в организме».
– Кому-то может повезти, и по наследству от бабушки с дедушкой он получит
замечательный мозг, который хорошо обучается и у которого быстрые биохимические
процессы. Но если ребенок при этом оказался в ситуации Маугли, или его плохо учат, то
толку от его замечательного мозга, который от предков достался, никакого. Какой бы
инструмент ни появился у вас в голове, на нем нужно научиться играть, – говорит Татьяна
Черниговская.
Да, генетически люди не равны, уже хотя бы потому, что по наследству могут
передаваться те или иные заболевания. Генетика – фундамент, но таковой фундамент
имеется у любого здорового «хомо сапиенс». Но даже с самого нижнего уровня развития, с
любой ступени можно подняться весьма высоко. Было бы только желание отстроить свой
интеллект, и методика, и система. Вот и надо сконцентрироваться на том, как нам
реализовать данный Богом ли, Природой ли человеческий потенциал. Это, собственно,
первое, на что я хотел бы обратить внимание читателя. Именно поэтому написана эта книга.
Перефразируя поговорку, нечего на генетику пенять, коли система обучения крива.
И последнее, пожалуй. Когда мы говорили о зеркальных нейронах, то прояснили, что
именно они являются биологической основой важнейшего компонента интеллекта –
воображения, фантазии.
Корней Чуковский, Самуил Маршак, Даниил Хармс, Борис Заходер, Льюис Кэрролл и
другие замечательные детские писатели убедительно показали своим творчеством, что дети
любят всякие «искажалки», перевертыши, несимметричные стихи с логикой уже
неподвластной закостенелому уму взрослого. Но именно эта способность необычного
видения мира формирует у детей неординарность мысли, даёт им потенциал действовать с
выдумкой, нестандартно, результативно. Мы уже останавливась на этом в нашей книге
«Турбулентное мышление», поэтому здесь позволим себе некоторые повторы, но
конспективно.
Корней Чуковский делится следующим своим наблюдением: «Взрослые, кажется,
никогда не поймут, чем привлекательны для малых ребят такие, например, незатейливые
деформации слов, которые я позаимствовал в английском фольклоре:
Жила-была мышка Мауси
И вдруг увидала Котауси.
У Котауси злые глазауси
И злые-презлые зубауси и т. д.
Дети именно потому и смеются, что правильные формы этих слов уже успели
утвердиться в их сознании.
Мою песенку очень бранили в печати за «коверкание родного языка». Критики
предпочитали не знать, что такое «коверкание» с незапамятных времён практикуется
русским фольклором и узаконено народной педагогикой».
В одном из первых критических откликов на работу писателя «О детском языке» (1909
года), т. е. ещё «до исторического материализма», некая барыня гневалась: «Что касается
детского языка, то советую вам почитать Библию; там вы узнаете, как три тысячи лет назад
премудрый Соломон доказал, что детского языка нет. А я, как мать многих детей, могу вам
доказать, что дети по недостатку развития своих внешних чувств и своего ума умеют только
картавить, то есть коверкать недослышанные слова взрослых. Сбоку приписка: «Вы забыли,
что яйца курицу не учат».
Но чему-то учит хотя бы исторический опыт!
Чуковский приводит ещё один следующий случай из своей практики:
«Я получил такое письмо: «Стыдно, т. Чуковский, забивать головы наших ребят
всякими путаницами, вроде того, что на деревьях растут башмаки. С возмущением
прочитали мы в вашей книжонке такие фантастические строки:
Жабы по небу летают,
Рыбы по полю гуляют,
Мыши кошку изловили,
В мышеловку посадили
(«Путаница»).
Зачем вы извращаете реальные факты? Детям нужны общеполезные сведения, а не
фантастика насчёт белых медведей, которые будто бы кричат кукареку. Не того мы ждём от
наших детских писателей. Мы хотим, чтобы они разъясняли ребёнку окружающий мир, а не
затемняли его мозги всякой путаницей!»
Я прочитал это письмо, и мне стало не то, чтобы грустно, а душно.
Какое затхлое и безнадежное невежество! Дело не во мне и не в моих бедных стишках,
а в огромном вопросе о принципах детского чтения, который нельзя же решать при помощи
одного только обывательского «здравого смысла», потому что «здравый смысл» нередко
бывает врагом всякой научной теоретической истины.
Признаться, я даже почувствовал к своему обличителю жалость: взять бы его за руку,
вывести на солнечный свет и объяснить ему от души, без запальчивости, самыми простыми
словами то, чего он не может понять в своем обывательском погребе.
Если бы, кроме «здравого смысла», у него были какие-нибудь другие ресурсы, он
увидел бы, что «путаницы», которые кажутся ему такими зловредными, не только не
мешают ребёнку ориентироваться в окружающем мире, но, напротив, укрепляют в нём
чувство реальности, и что именно в интересах реалистического воспитания детей следует
культивировать в детской среде такие стихи. Ибо так уж устроен ребёнок, что в первые годы
его бытия мы можем насаждать в его душе реализм не только путём ознакомления с
окружающим миром, но чаще и успешнее всего именно при посредстве фантастики.»
Фёдор Михайлович Достоевский: «Фантазия есть природная сила в человеке, тем более
во всяком ребёнке, у которого она, с самых малых лет, преимущественно перед всеми
другими способностями развита и требует утоления. Не давая ей утоления, или умертвишь
её, или обратно, – дашь ей развиться, именно чрезмерно (что и вредно) своими
собственными уже силами. Такая же натуга лишь истощит духовную сторону ребенка
преждевременно».
А уж уметь придумать нелепицу в духе барона Мюнхгаузена – это для взрослого
просто искусство, хотя для ребёнка – невеликий труд. И моему взрослому читателю я
рекомендую, в том числе, и такой способ тренировки.
Иван Бунин высказал мысль, что, «выдумать и уметь сказать хорошую нелепость,
хорошую шутку могут только очень умные люди, те, у которых ум по всем жилушкам
переливается».
Ну, и конечно, не надо лениться вчитываться в парадоксальные миниатюры, например,
того же Станислава Ежи Леца («В каждом веке есть своё Средневековье») или нашего
Андрея Кнышева («Велик человеческий разум. Кое-кому»)! Тренируйте воображение, рисуя
или рассматривая хорошие карикатуры – смещение сознание гарантируется! Особенно, если
рисунок не сопровождён текстом. Старайтесь осмысливать увиденное самостоятельно, а уже
только потом читать комментарии а не читать таблички! По крайней мере, учитесь у Козьмы
Пруткова не верить глазам своим, увидев на клетке с быком надпись «Тигр». Думайте сами,
решайте сами! И на ваших извилинах – «датчиках интеллекта» – это обязательно скажется.
Как сказал Александр Лук, чтобы оцепить смешное, надо уметь рассуждать и надо
научиться отличать добро от зла: «Шотландский философ из Абердина Джеймс Битт обратил
внимание, что остроумие не всегда вызывает смех, что есть и «серьезное остроумие».
Остроумно и смешно – эти понятия соприкасаются, но не совпадают.
И. Кант не отождествлял способность суждения и способность остроумия. Он разделил
всех людей на четыре группы. Умные обладают и способностью суждения и остроумием.
Способность суждения, не подкрепленная остроумием, создает так называемых умников, а
остроумие без способности суждения – столь же утомительных остряков. Наконец,
четвертые не обладают пи способностью суждения, ни остроумием – это дураки».
(Александр Лук, «Юмор, остроумие и творчество»).
«Иными словами, «синтезировать» блестящую остроту – это доступно редким людям.
Гораздо легче, оценив возникшую ситуацию как аналогичную, вовремя припомнить и «к
месту» привести чужую шутку. Наконец, есть ещё «остроумие взаймы» – утомительный
обмен анекдотами, когда говорят наперебой и каждый спешит «захватить трибуну» (там же).
В любом случае, не ленитесь напрягать извилины, ведь как говаривал тот же Ежи Лец:
«До глубокой мысли надо подняться», и получайте удовольствие от собственного подъёма на
новую интеллектуальную ступень.
Часть 1. Процесс и продукт работы мозга человека
1. Что умеет мозг
– А ты не печалься так! – хрипло сказала она мне. – Если бы у
тебя были мозги в голове, ты был бы как все люди! Мозги –
единственная стоящая вещь у вороны… и у человека!
Вот так-то я и узнал, что у людей бывают мозги…
А. Волков «Волшебник Изумрудного города»
Думать – значит, находить ответы на различные вопросы. Вспомним один из мудрых
рассказов Роберта Шекли. На некоей планете во Вселенной неважно кем построен Ответчик.
Обратите внимание: «он был построен, чтобы действовать столько, сколько необходимо, что
очень большой срок для одних и совсем ерунда для других». Если говорить о размерах, то
«одним Ответчик казался исполинским, другим – совсем крошечным. Это было сложнейшее
устройство, хотя кое-кто считал, что проще штуки не сыскать». В двух фразах – вся
относительность размеров, времён, знаний. И Ответчик мог ответить на любой вопрос. Если
он поставлен правильно. И он ждал, чтобы к нему пришли и спросили. Но, как оказалось,
Ответчик, обладая по сути бесконечным запасом знаний, никому не мог помочь: ведь, чтобы
получить знания – ответ, нужно задать вопрос, а чтобы задать вопрос правильно, нужно
знать большую часть ответа!
Кстати, о Вашей внимательности и желании научиться мыслить по-настоящему. Не
кажется ли Вам «Мыслитель» Родена овеществлённым символом процесса мышления? То
есть Вы вообще-то помните эту замечательную скульптуру? В глубокой задумчивости
человек сидит, опираясь локтем на колено, подпирая голову рукой. (Конечно, это не просто
«гигантская бесформенная глыба протоплазмы», как Мыслитель из рассказа Роберта Шекли.)
Итак – Вы никогда не замечали в скульптуре Родена ничего удивившего Вас, чего-то, что в
скульптуре кажется неправильным? (Хотя, кто знает…) Нет, не в том странность, что
человек вообще думает. И не в том, что процесс мышления – это и есть настоящий и
нелёгкий труд, как показал великий скульптор. Если Вам странно то, что в размышления
погрузился мужчина, не успев или забыв полностью одеться – это не ответ, которого я
ожидаю. Ну, Вы, батенька, невнимательны… Учтите: внимание к мелочам – тоже один из
признаков высокого интеллекта. Да что там говорить – великий Шерлок Холмс не стал бы
гением расследования, не будь он так наблюдателен (о дедуктивном методе поговорим
отдельно). Ничего странного не обнаружили? Ну, не переживайте, натренируетесь – всё
станете подмечать.
Ясно: не каждый человек рождается (или становится) творчески одарённым, умеющим
мыслить парадоксально и действовать нестандартно. Да ещё чтобы это проявлялось с
раннего возраста. Впрочем, иногда даже фактически стандартные действия представляются
чем-то, свидетельствующим о высоком интеллекте. Скажем, не так давно в члены известного
всемирного клуба интеллектуалов MENSA была принята трехлетняя малышка. (Впрочем,
гений не имеет возраста, как полагал капитан Немо.) В качестве одного из указаний на её
весьма высокий уровень интеллекта – ссылка на то, что она может очень аккуратно
нарисовать практически идеальную окружность! Неплохо бы, конечно, разобраться: это –
результат точного знания свойств окружности, сообщённого взрослыми (плюс свойственная
ребенку
тщательность
в
выполнении
задания),
или
интуитивное
(читай:
сверхинтеллектуальное) понимание этих свойств?
Известный психолог Чарлз Везерол отмечает, что, рассматривая аналитические и
творческие способности мозга, всегда имеют в виду его способности: логические, речевые,
математические, пространственного восприятия, память, умение формировать понятия.
Креативность – творческие способности – Можно изучать, разглядывая с разных сторон.
Каждый специалист найдёт что-нибудь «вкусненькое». Физиологи находят радость в
исследованиях структуры и функций нервных клеток, химических и физических процессов,
обеспечивающих деятельность мозга. Специалисты по информатике пытаются извлечь
сведения о процессах обработки и генерации информации, формулируя принципы
построения и работы нейронных сетей мозга. Психологам интереснее изучать: что является
стимулом к творчеству, что переживает человек в процессе создания нового, как побудить
его к творчеству, каковы индивидуальные особенности этой деятельности и что в ней можно
считать общим. Вот, например, как психологи понимают креативность – как «внутренний
потенциал и способность к творчеству», а «творчество – процесс, в котором проявляется эта
способность и раскрывается творческий потенциал». Здорово, да? И главное – возможно ли
вообще разорвать этот замкнутый круг определений?
Причём иногда креативность и интеллект сводятся, а иногда – не сводятся друг к другу.
У каждого подхода есть свои почитатели и защитники. Так это ещё и не всё! Всего этих
подходов (пока!) двенадцать: четыре основных и восемь специфических. И всё это – чтобы
обсудить природу, сущность и причины, по которым человек желает и способен создавать
нечто новое.
Специалисты по логике рассматривают творчество как систему закономерного
развития знания. Философы интересуются методами творческих исследований, проблемой
истинности и ценности получаемых новых знаний. Все, в общем, при деле.
Коротко говоря, в поисках интересного занятия разнообразные специалисты со всех
сторон набросились на проблему производства новой информации в мозгу человека. Значит,
если разберёмся, как человек мыслит, сможем помочь ему мыслить лучше, быстрее, вернее?
Хм-м. Ну, попробовать стоит… Можно было бы назвать это наше занятие, скажем, тренинг
креативности? Но, пожалуй тут уже всё, как ныне говорят, «схвачено»: социальной
психологией уже разделены на четыре типа все возможные, с точки зрения этой науки,
тренинги креативности. Это:
1) тренинги, опирающиеся на решение интеллектуальных творческих задач;
2) тренинги с использованием различных ролевых игр;
3) тренинги, в которых главное внимание уделено сензитивности, спонтанности;
4) тренинги, использующие средства искусства и близкие к методам арт-терапии.
Пожалуй, всё-таки лучше оставить за нашим занятием определение «фитнес для
интеллекта», как и было заявлено в начале книги, а в качестве «снаряда» выбрать «бигуди
для извилин». Чтобы сформировать свои извилины в соответствии с «эстетикой мыли» и её
содержательностью, нужно подвести теоретические обоснования к нашим мозговым
упражнениям (наподобие того, как атлет, приступая к действию, уже четко ориентируется в
том, какие группы мышц будут при этом работать, как будет строиться ритм его дыхания и
т. п.). И с чего начнём? Попробуем пока «всего лишь» понять (или дать определение –
зачастую это некая имитация понимания, но зато научно!), что означает «мыслить». Что за
«агент» действует в организме человека, когда он обдумывает сюжет и композицию новой
картины или книги, мысленно прослеживает развитие мелодической темы в симфонии? Что
и где происходит, когда возникает «из ниоткуда» решение научной проблемы, конструкция
нового удивительного механизма, ясный способ действий в неожиданной жизненной
ситуации? Почему гениальный футболист во мгновение ока отдаёт единственно верную
передачу? Кто или что подсказывает: с чего начать и как двигаться мысли во всех этих
случаях?
В «Сказке о Тройке» братьев Стругацких бодрый старикашка-изо-бре-татель объяснял
высокоумным заседателям из «Тройки»6, как и почему у коробки, присоединенной к
пишущей машинке, возникает «внутре синекдоха отвечания», если коробке задаются
вопросы. Всё предельно просто – в коробке есть Думатель и Анализатор! Изобретатель
вводит вопросы в Думатель лично, путём печатания на машинке. Ответы также печатает он
лично – поскольку ещё не всё полностью автоматизировано. И модернизировано. Кстати, в
реальной жизни различные «изобретатели» и «тройки» с подозрительной/поразительной
регулярностью воспроизводят ту же схему взаимоотношений. Но для нас в этом сюжете
любопытно иное: как просто, оказывается, можно промоделировать работу мозга, если
использовать этот самый мозг для получения не научного результата, а чисто практического.
Итак, значит, и наш мозг работает, «кодируя помаленьку», как объяснено у Стругацких.
Точнее, чтобы мозг работал, ему нужна «пища» – информация. Чтобы сохранить в памяти
мозга какую-то информацию, нужно уже обладать какими-то знаниями об окружающем
мире. Эти знания мозг для себя сохраняет, кодируя их для удобства сохранения и
использования: а) зрительно, б) акустически, в) семантически. Причем зрительное
кодирование опережает акустическое или слуховое. А семантическое кодирование – это
запоминание осмысленной информации. Для кодирования в этом случае необходима связь с
уже сохранёнными понятиями.
Фрит Крис так говорит об эволюции моделей мозга в развитии науки: «Когда-то вы
считали, что у нас в голове фотоаппарат. Теперь вы считаете, что там компьютер. Даже если
у вас получится заглянуть внутрь этого компьютера, вы останетесь всё с той же избитой
моделью. Конечно, компьютеры умнее фотоаппаратов. Может быть, они и способны
узнавать лица или механическими руками собирать яйца на птицеферме. Но они никогда не
смогут рождать новые идеи и передавать их другим компьютерам. Им никогда не создать
компьютерной культуры. Такие вещи не по силам машинному разуму».
Татьяна Черниговская также утверждает, что «компьютер» в нашей голове отличается
от любого из тех, который человечеству известен. В нашей черепной коробке, конечно,
также происходят вычисления. Но это не единицы и не нули, он работает по другому
принципу. Возможно, что он использует другой тип математики…»
Большим числом ученых овладевает пессимизм, отражающийся в одной фразе: «Наука
не может исследовать сознание». Почему же не может?
С моделью мозга в науке все обстоит непросто, но обратная связь от нее к
усовершенствованию реальных компьютеров прослеживается.
С 1970-х быстродействие компьютеров превзошло прогнозы в миллионы раз, а умнее
они не стали. Чтобы компьютеры поумнели и стали функционировать быстрее, ученые
решили изменить основные принципы их работы. Например, создавать их на основе
мемристоров. Речь идет об электронных аналогах синапсов – соединений между нейронами
мозга (каждый нейрон связан с остальными тысячами синапсов). Чтобы в компьютере
получить подобие нейронных сетей мозга, мемристоры должны уметь образовывать новые
связи, но при этом мало что известно о том, как этот процесс происходит в настоящем мозге.
Во всяком случае идея создания нейроморфных, или, как их ещё называют, когнитивных
компьютеров живет и, возможно, когда-нибудь приведет к пересечению техногенного и
человеческого интеллектов: мозги людей и компьютеров свяжет нейронет – сеть, которая с
помощью нейроинтерфейсов, позволит управлять внешними объектами силой мысли. Но
пока не появится понимание того, как думает человек, прогресс в развитии этого
направления вряд ли будет заметен.
Пока что мы можем лишь констатировать, что внутри нас непростой – да ещё и
6 Наши фантасты намекали на «особые тройки» (обычно первый секретарь региональной организации
коммунистической партии, начальник регионального управления внутренних дел, прокурор региона), в 1937–
38-м годах наделённые правом выносить приговоры – вплоть до смертных – на основании рассмотрения
следственного дела, без прений сторон. Впрочем, нынешние парламентские комитеты и комиссии зачастую
также не утруждают себя рассмотрением вопросов по существу.
непрестанно действующий – Думатель и Анализатор: структурированное вещество,
упрятанное в черепной коробке, устроенное сложным, всё ещё до конца не понятым, образом
и соединённое миллиардами связей со всеми частями тела. Часть целостной нервной
системы человека. Управляющий и контролирующий механизм. Вместилище информации.
Её получатель, хранитель и переработчик. А также и отправитель – в форме
электромагнитных волн? Или в неизвестной нематериальной (в привычном смысле слова
«материя») форме? Вот там «внутре» мозга зарождается и выдаётся наружу «синекдоха
отвечания» – «моя последняя книга…», «эта музыка – Мои воспоминания о…», «суть и
доказательство проблемы Пуанкаре вполне можно объяснить даже школьникам, а
именно…». Может, главное – задать ему правильный вопрос?
Как-то раз немецкий патолог Рудольф Вирхов демонстрировал студентам
физиологический опыт. Когда он удалил у жабы часть мозга, её тельце стало дергаться в
конвульсиях. Студенты засмеялись. Желая остановить неуместный смех, Вирхов, как ни в
чём не бывало, объявил:
– Итак, господа, наш эксперимент блестяще подтвердил, как мало мозга надо для того,
чтобы развеселилась целая аудитория.
На что же способен наш мозг? Можно ли расширить пределы его возможностей? И
как? Можно ли понять, каковы механизмы и приёмы работы ума, постичь структуру
вырабатываемого знания? Можно ли использовать наши знания об устройстве и принципах
деятельности мозга для повышения эффективности его работы?
Впрочем, можно и повторить слова Оливера Хэвисайда: «Разве должен я отказываться
от обеда лишь потому, что не понимаю, как происходит процесс пищеварения?» В самом
деле, перестанем ли мы креативно мыслить, не зная, что творится в черепной коробке?
Тем более что сегодня (точнее, уже лет сорок) все знают о существовании и
сотрудничестве двух полушарий мозга. Зачем-то сигналы от левой половины тела
обрабатываются правым полушарием, от правой половины – левым. Может, так решается
задача устойчивости системы. Или это что-то вроде того, как школьники из одной школы
идут писать ЕГЭ в другую? Чтобы объективизировать результат, так сказать.
Техническое решение, осуществлённое при сборке первого человека, гениально просто:
нервные окончания, сигнализирующие о положении дел во всех органах и участках тела, в
головном мозге перекрещиваются. Так что в правое полушарие попадают сигналы от левой
половины тела и наоборот. Поэтому поражение одной половины мозга – например,
вследствие инсульта – вызывает паралич противоположной стороны тела.
Известно, что левое и правое полушария головного мозга имеют разные
специализации. Левое управляет процессами, в которые вовлекаются числовые величины,
используются логические цепи, последовательно прослеживаются и анализируются ряды
фактов и событий. Правое же полушарие оперирует образами, содержит пространственные и
временные7 целостные картины. Для левого полушария естественна вербальная (т. е.
словесная), дискретная форма «записи» исходной и конечной информации – а также, повидимому, промежуточных результатов переработки одной в другую. В правом полушарии
возникающие и исчезающие образы бессловесны и непрерывны.
Но, конечно, оба полушария исследуют каждую проблему совместно, пытаются
обработать любую информацию. Ведь оба они узнают о новой задаче одновременно. Так в
нашей жизни о происшествии узнают сразу несколько различных служб: и журналисты, и
милиция, и пожарные. А дальше – каждый действует по своему разумению, делает, что
может.
Опять же психологи утверждают: именно левое полушарие человеческого мозга имеет
намного больше стимулов и возможностей для развития, чем правое. Объяснить это
несложно: в обыденной жизни человек гораздо чаще сталкивается с необходимостью
7 Возможно, и надвременны€е – не отсюда ли интуиция, предвидение, предчувствие?
использовать для общения речь, записи, всё время должен анализировать события, факты,
делать выводы и совершать поступки. Человек вовлечён в непрерывный процесс получения,
обработки и обмена дискретной информацией. Необходимость взаимодействия с нею
«включает» главным образом левое, логическое, полушарие. Правое полушарие заметно
развивается лишь у тех, кто интенсивно занимается творческой деятельностью – ведь при
этом необходимо мыслить целостно, образно, использовать подсознание, интуицию,
озарение. Правая часть мозга отвечает за художественные, музыкальные и математические 8
способности.
Любопытно, сказываются ли этнические характеристики человека на его способностях
в математике? Или здесь играют определяющую роль, например, религиозные, культовые
особенности, принятая система воспитания? Скажем, с древних времен известно: индийцы,
как правило, превосходные математики. Их отношение к числам совершенно особое: все
изучаемые в начальной школе числа (вплоть до четырёхзначных) имеют для индийских
школьников индивидуальные особенности – рассматривается подробно, каковы делители
чисел, их внутренняя симметрия, составная структура. Один индийский учёный на вопрос,
не является ли номер такси, на котором он ехал, неблагоприятным, ответил: этот номер,
число 1729, напротив, весьма интересен, поскольку может быть записан в виду суммы двух
кубов двумя способами! (Например, 12 в кубе плюс 1 в кубе. Вторую пару попробуйте найти
сами!) У нас тоже не всё так плохо: например, то, что квадрат любого натурального числа
можно записать в виде суммы подряд идущих нечётных чисел, (давно известный
математикам факт) обнаружил наш знаменитый соотечественник Андрей Николаевич
Колмогоров. Впрочем, тогда будущему академику было всего лет 6–7. И это своё первое
«открытие» он тоже сделал вполне самостоятельно.
Установлено: для мужчин естественнее лучшее развитие и использование правого
полушария. Не зря говорят: на одну женщину-математика приходится 13 математиковмужчин. Но о сравнительных особенностях мужского и женского интеллекта мы подробнее
поговорим дальше. А вот Эйнштейн утверждал: «Математика – единственно совершенный
метод, позволяющий провести самого себя за нос». Выходит, и в этой области сильному
полу гордиться нечем!
Кстати, об Альберте Эйнштейне. Вот уж пример несомненного, подлинного гения. А
известны ли какие-то уникальные особенности его мозга? Изучая мозг Эйнштейна (точнее –
фотографии 240 срезов мозга, сделанных с помощью микротома в 1955-м году),
американский антрополог Дин Фальк обнаружила особо развитые теменные доли мозга
(значительно превосходящие по размеру среднестатистические), сильно развитые
«музыкальные шишки», контролирующие движения левой руки (что характерно для людей,
музыкально одарённых), и необычный рисунок бороздок в затылочной области. Ранее в
качестве необычного показателя отмечалась лишь малая масса мозга – 1230 г, что близко к
нижней границе нормы.
Это если говорить о структуре мозга гения. А с точки зрения состава мозга Эйнштейна,
исследователи из Швейцарии установили: в ассоциативной области коры, ответственной за
высшие формы мыслительной деятельности, присутствует необычайно большое количество
вспомогательных элементов нервной ткани – клеток нейроглии. Эти клетки контролируют
передачу информации всем разделам мозга. Они, образуя дополнительную сеть в мозгу,
связывают все мозговые структуры воедино и задействуют весь мозг целиком для
«обсуждения» и принятия решения. Теперь известно: чем более число таких клеток
превышает число нейронов в мозгу (а этих тоже может быть немало – от 15 до 100
миллиардов!), тем более серьёзные интеллектуальные задачи способен решать мозг.
Немецкий ученый Оскар Фогт во время официального открытия Института мозга в
8 За математические, впрочем, скорее лишь в части, связанной с работой на очень высоких уровнях
абстракции, где нужно оперировать целостными структурами.
1927 году сказал, что в коре головного мозга Ленина обнаружено скопление пирамидальных
клеток, и связал с этим ярко выраженные ассоциативными способности Ленина (таковые
отличали и Эйнштейна, если вспомнить знаменитый мысленный эксперимент с лифтом).
Увы, исследование структурных особенностей не дает окончательного ответа о
природе гениальности. Это – во-первых. А во-вторых, даже зная о роли глиальных клеток,
разве мы можем неким образом увеличить в своем мозгу их количество? Неизвестно. А,
может быть, заставляя свой мозг чаще размышлять, рассуждать, анализировать, мы
увеличиваем хотя бы число связей между нейронами в мозгу? Задумаемся и над этим
вопросом?
От природы человеку дано многое – но кому много дано, с того много и спросится.
Человек действует в условиях многообразных контактов с внешней средой, в немалой
степени состоящей из других людей. Чтобы полноценно реализовать себя в жизни, нужно
максимально развивать в себе способности, заложенные от рождения.
А заложено, кстати, немало. У каждого из нас двое родителей, четверо бабушек и
дедушек… Если уйти назад всего на 30 поколений – лет на 700–800 – наберется больше
миллиарда предков. Но ведь в ту пору на всей Земле жило куда меньше народу. Значит, все
мы ныне живущие – родственники. Каждый из нас унаследовал те же гены, которые создали
великих воинов, прославленных поэтов, искусных художников, вдохновенных актеров…
Подаренные ими задатки – надежная опора для нашего творческого роста.
Надо только не пренебрегать собственными способностями.
Между прочим, не только Альберт Эйнштейн, но ещё один великий физик – Макс
Планк – был превосходным музыкантом. В юности он долго не мог сделать выбор между
карьерой пианиста и физика-теоретика9. А первый директор Объединённого института
ядерных исследований в Дубне, известный физик-теоретик, академик Дмитрий Иванович
Блохинцев был превосходным художником. Примерам такого рода несть числа. Бертольд
Брехт сказал: искусство требует знаний. Видимо, верно и обратное… Хотя, может быть, дело
лишь в том, что искусство восполняет недостатки природы, как считал Джордано Бруно.
Итак, выдающиеся представители так называемых точных наук демонстрируют явную
способность – если не сказать «талант» – к художественному творчеству. С другой стороны,
архитектура – область бесспорно художественной культуры – непредставима без умения
чётко и правильно вычислять. Так что примеров обоюдовыгодного сотрудничества и
взаимовлияния обеих частей мозга множество. С этой точки зрения холистические –
целостные – представления о мозге, как о сильносвязной системе, выглядят вполне логично.
Из любви к классифицированию и желая образованность показать, определим:
✓ логическое мышление как способность мозга анализировать окружающее
рациональными методами, т. е. видеть взаимосвязи между явлениями и предметами и
системно их упорядочивать;
✓ речевые способности, подразумевая, что мышление позволяет не только понимать
смысл отдельных слов, но и находить смысл предложений, сконструированных с помощью
слов;
✓ пространственное восприятие – способность различать предметы по форме и
размеру, а также соотносить одни формы с другими;
✓ математические способности – умение оперировать не только количественными
понятиями, но и создавать и изучать абстрактные конструкции – Математические и
натурфилософские структуры;
✓ умение формировать понятия – эта способность абстрагирования близка
математическим способностям и позволяет получать обобщённые представления о
9 Но всё же сделал правильный – с точки зрения физики и не только физики – выбор. Представьте себе, что
знаменитая физическая константа называлась бы не «постоянная Планка», а, скажем, «константа
Пфердшнауценбюгля»? Бедные наши студенты и школьники…
предметах и категориях;
✓ память – способность воспринимать, хранить и обрабатывать информацию.
Писатель Милорад Павич сказал: память человека тем отличается от памяти животных,
что способна истолковывать то, что хранит. По поводу людей Ларошфуко едко заметил:
«Все жалуются на свою память, но никто не жалуется на свой ум». Конечно, только
взаимодействие всех вышеназванных способностей даёт возможность мыслить ясно,
продуктивно и творчески.
Утверждается (и не без оснований): мозг, как и мускулатура, поддаётся эффективной
тренировке. Правда, в отличие от мускулов, мозг при тренировке количественно не растёт:
отрабатываются лишь процессы переработки информации в нём. Между тем многие
стремятся нарастить именно количество информации, хранимой в памяти. Тем более, что
такими накоплениями можно блеснуть, как накачанными мускулами на пляже: сообщить
изумлённым слушателям, какова длина какой-то железной дороги или высота горы…
Артур Конан Дойл устами своего главного героя – Шерлока Холмса – сравнил
бессвязную эрудицию с захламленным чердаком, где всё свалено без разбору и ничего
нельзя найти.
Развивая же аналогию мозга с мышцами, можно сказать: эрудиция без навыков анализа
и синтеза подобна мускулам культуриста – они накачаны бессчётными однообразными
упражнениями, но не способны к реальной работе. Между тем куда красивее и гармоничнее
фигуры пловцов или гимнастов: их мышцы также изящны и сильны, но ещё и работать
умеют – координируют усилия, способны и к статической, и к взрывной нагрузке.
Более того, на запоминание и понимание некоего закона или правила уходит не больше
сил, чем на заучивание одного факта, а вывести из этого закона можно сотни и тысячи
фактов – по мере надобности. Так что тренировать надо прежде всего интеллект –
способность понимать правила и выводить факты.
Замечу: при тренировке интеллекта обязательно «наработается» немалая эрудиция. Но
факты будут не просто свалены в памяти, а освоены, приведены в систему, логически
выстроены… Словом, ничего не будет ни выпирать, ни обвисать – эрудиция станет рабочей.
2. Что имеем – сохраним?
Столько он всего знал и был такой умный, что стал ужасно
рассеянным. Он всё забывал. Вы мне, наверно, и не поверите, если я
вам скажу, что он забывал порой даже своё собственное имя…
П. Трэверс. «Мэри Поппинс»
Ещё Платон считал, что человек ещё до рождения узнаёт всё и обо всём – просто не всё
успевает вспомнить за краткую жизнь. Но что означает «помнить»? Это означает уметь
усваивать, сохранять и повторно извлекать информацию. Не помнить – не справиться со
всеми или с какой-либо одной из этих задач.
«Наш мозг способен к извлечению своего содержания из памяти и порождению
аналогов, к вынесению информации в социум, в культурное пространство – это одно из
крупнейших прорывов человечества. Как-то человеку пришла в голову мысль вынести
информацию за пределы человеческого субстрата, чтобы она не погибла в голове вместе с её
обладателем. Это так называемая выносная память (книги, папирусы и библиотеки, музеи,
Интернет и т. д.). А человеческие языки – речь, математика, музыка и т. д. – это инструменты
для таких манипуляций и строительства внешней среды», – считает Татьяна Черниговская.
Откуда берется разум? Оптимистов и пессимистов в науке в попытке ответить на этот
вопрос объединяет вера в то, что разум и сознание возникают сами собой из работы
нейронной сети.
«Я – это мой коннектом» (характер связей между нейронами мозга), – говорит
Себастьян Сеунг, профессор Массачусетского технологического института и один из
ведущих разработчиков карт нейронных соединений.
Накоплено немало данных, подтверждающий эту гипотезу. Например, ещё в 90-х было
экспериментально доказано существование «нейрона Билла Клинтона», то есть нейрона,
который активируется при воспоминании об экс-президента, независимо от того, читаем ли
мы его имя, видим ли его портрет или воспринимаем понятия: Хиллари, Моника, саксофон,
«вдыхал, но не затягивался» и т. п.
По этой гипотезе в мозгу человека есть нейрон, отвечающий за кодирование
конкретных объектов, включая его предков (теорию, ещё в 60-х предсказавшую
существование таких нейронов называли «теория нейрона моей бабушки»).
Но не все экспериментальные данные согласуются с идеей о том, что все воспоминания
и аспекты личности человека закодированы в соединениях нейронов. Тем более, что мозг
состоит не только из нейронов, и из уже упомянутых глиальных клеток, которых в десятки
раз больше и они, как установлено, активно взаимодействуют с нейронами. Поэтому вряд ли
стоит все роли отдавать нейронам.
Кроме того, обучаться и запоминать информацию в определенной степени способны не
только нейронные сети, но и одноклеточные организмы (амеба, инфузория). Тогда,
возможно, секрет разума внутри клетки, а не снаружи?
А знаменитый математик Роджер Пенроуз вообще считает, что сознание и разум не
могут чудесным образом возникнуть из обмена сигналами в в какой бы то ни было сети.
«Люди думают, что сознание происходит из какого-то сложного аспекта вычислительной
активности, – объясняет Пенроуз. – Я смотрю на эту проблему совершенно иначе. По-моему,
в мозгу происходит много вычислительной активности, но это – бессознательное. А
сознание, на мой взгляд, это что-то принципиально иное. Понимание – это не вычисление.
Происходит что-то ещё. Я в-ерю в науку и считаю, что все происходящее у нас в голове
подчиняется тем же законам, которым подчиняется Вселенная вокруг нас. Но эти законы ещё
не до конца поняты нами. Я пытаюсь нащупать этот пробел в наших знаниях, это «что-то
ещё».
Многие ученые считают, что сознание подобно ветру: ветер увидеть нельзя, но
результаты его деятельности зачастую впечатляют.
Человеческая память селективна: мозг отбирает, сортирует и хранит лишь наиболее
важную информацию. Именно это свойство избирательности, наряду со способностью
забывать, и даёт мозгу возможность работать. В противном случае мозг «утонул» бы в
потоке непрерывно поступающих сигналов. При этом память – не пассивная система, не ряд
стеллажей, откуда снять нужный том или папку может только некий «читатель» – внешняя
по отношению к мозгу структура.
По характеру проявления память может быть образной, словесно-логической,
механической, эмоциональной и условно-рефлекторной. По типу восприятия – зрительной,
слуховой, обонятельной, двигательной и весцилярной (осязательной).
Мозг сам, как хороший библиотекарь, вводит в себя структурированные блоки данных,
хранит их в порядке, известном ему одному, и на полках, пронумерованных им самим,
способен в зависимости от задачи отыскать нужную папку на нужной полке. «Кто-то помнит
хорошее. Кто-то плохое. Наша память избирательна, как урна…» – писал Сергей Довлатов.
Следуя оценке, сделанной нейробиологами, принято считать: информационная ёмкость
коры головного мозга у человека равна примерно 3х108 бит. Полагая, что в среднем
информационный поток составляет 20 бит/сек10, найдём: за 70 лет при длительности
активного дня 16 часов общее поступление информации составит 3х1010 бит. Если для
хранения 1 бита требуется (по некоторым оценкам, учитывающим непревзойдённую
помехоустойчивость нашей памяти) 10 нейронов, то поступающая в мозг информация в 100
10 Оценка очень скромная: например, первичная пропускная способность зрения составляет миллионы
бит/сек. Так что здесь учитывается уже переработанная, довольно плотно упакованная информация.
раз больше его информационной ёмкости! Правда, тонкости работы нейронов ещё не
изучены, так что может оказаться: даже один нейрон может накапливать несколько битов.
Но с другой стороны нейроны заняты не только хранением информации, но и её обработкой.
Так что в любом случае в мозге может храниться не более 1 % от общего потока
информации11. Упомянутое выше внимание призвано прежде всего устранить избыточность
сенсорного потока и даже подавить многие сенсорные входы.
Современные исследования мозга подтверждают: память – свойство мозга как системы
в целом12, а не его отдельных молекулярных и клеточных компонентов.
Память человека спроектирована и создана предусмотрительно и логично, с
соблюдением иерархического принципа, когда информационные сигналы передаются
последовательно из одного воспринимающего или обрабатывающего участка в другой.
Каждый участок «знает свой манёвр».
Лучше всего мозг запоминает новую для него информацию. Да ещё и интересную 13 –
выделенную по каким-то признакам. Это, конечно, означает: на входе мозг сразу же
производит первичную обработку поступающей информации, сравнивая её с уже
хранящейся в нём. Такой «фейс-контроль» на входе: вот Вы, барышня, проходите, а Вас мы
уже тут видели, и Вы нам не понравились. А тебя, приятель, сразу сдадим скучающему
милиционеру – и больше не появляйся! Ну, а, установив степень новизны, мозг сортирует и
направляет отдельные блоки данных на разные «полочки» в «складе»-памяти.
По временным характеристикам память делят на:
✓ сенсорную, или ультракороткую (длительность хранения – Менее одной секунды),
✓ первичную (несколько секунд),
✓ вторичную (от нескольких минут до нескольких лет),
✓ третичную (информация хранится всю жизнь).
Сенсорная, автоматическая память как раз и зависит от эффективности работы органов
чувств: в ней запечатлевается всё, что они воспринимают. (Почти 60 % людей используют
главным образом зрительную, визуальную память; по типу восприятия память может быть
ещё и слуховой, обонятельной, осязательной, двигательной.) Просмотр и селекция
информации начинается при переходе из сенсорной в первичную память. Установлено: в
среднем человек запоминает 1/5 услышанного и 3/5 увиденного. Впрочем, если увиденное
одновременно и объясняется, то запомнить удастся до 4/5. Вот почему студентам имеет
смысл ходить на лекции – поглядывая на доску, можно одновременно ещё и что-то
услышать, а что-нибудь даже понять!
Сенсорную и первичную память называют кратковременной, вторичную и третичную –
долговременной памятью. Каждый вид памяти использует собственный механизм и
приспособлен для хранения информации разных типов. Различна и информационная ёмкость
видов памяти.
Кратковременная память имеет небольшой объём и обеспечивает хранение
информации от нескольких секунд до десятков минут, однако надёжно она работает лишь в
11 По расхожему мнению, картина прямо противоположная: средний человек использует не более 1 %
возможностей мозга – разве что гении задействуют 4–5 %. Очевидно, при этом не учитывается даже реальный
объём информации, которую мозг должен обрабатывать. Другое дело, что в каждый данный момент мозг
действительно включает в работу далеко не все свои структуры. Поэтому рост степени параллелизма их работы
способен заметно повысить общую выходную мощность.
12 Заметьте: целостный взгляд на сложную систему возникает всегда, как только оказывается очевидно
непонятен способ совместного функционирования – а зачастую даже само предназначение – большого
количества отдельных её компонентов. То есть в тех случаях, когда целое больше суммы всех своих частей.
13 Игроки «Что? Где? Когда?» чаще всего очень любознательны. Потому и помнят многое.
течение нескольких секунд. Примерно через 10–12 секунд восприятие информации уже
затруднено, а секунд через 25–30 невостребованная информация «стирается»14. Такая
оперативная память способна одновременно удерживать 7±2 (т. е. от 5 до 9, но чаще всего 7)
различных фрагментов информации. Любое воздействие, способное создать большие
флуктуации в работе материальных носителей памяти – нейронных сетей (наркоз,
электрошок), нарушит работу этого вида памяти. Информация будет утеряна.
Из кратковременной памяти в долговременную информация передаётся благодаря
преобразованию и упорядочиванию её материальных следов (так называемых энграмм),
вследствие чего возрастает вероятность запоминания информации. Внутренний
«библиотекарь» переносит коробки с записями – энграммы – на полки в другом, более
просторном и реже посещаемом (хотя и более надёжном) хранилище. Возможно, по дороге
он переписывает данные с хрупких старых листов на новую «бумагу» или «магнитную
плёнку». Заодно объединяет сведения, близкие информационно и по смыслу, в блоки. Что-то
может и потерять по дороге. Либо заложит в совсем уж дальний угол – вот и забылось чтото, когда-то важное. Может быть, сенсорная (или первичная) память сразу отбросит
ненужные сведения прямо «на пороге» мозга – ведь забывание начинается сразу же с
момента восприятия окружающей среды.
Информация, обработанная в сенсорной памяти, переносится в первичную. Человек
производит такой перенос на вербальном (словесно оформленном) уровне. При этом сигналы
получают своё словесное выражение и затем комбинированным образом, словесно –
сенсорно, закрепляются в памяти. Установлено: этот способ характернее для взрослых, чем
для детей. Другой способ переноса сенсорной информации в первичную – невербальный – до
конца пока не изучен. Этот путь – единственная возможность преобразования сенсорной
памяти в первичную для животных и маленьких детей.
Объём первичной памяти меньше объема сенсорной. Часть информации первичной
памяти вытесняется (забивается и от этого забывается) вновь поступающей информацией,
часть переходит во вторичную память. Считается: информация, не закодированная в виде
слов, не задерживается в первичной памяти и прямо переходит во вторичную. Только этот
вид информации может быть извлечён через значительный отрезок времени.
Вторичная память имеет большую ёмкость и длительность хранения. В отличие от
первичной вторичная память организуется на основе смыслового значения информации.
Если при извлечении слов из первичной памяти обычная ошибка – смешение сходных звуков
(например, «Г» и «Х», «П» и «Б»), то из вторичной при ошибках извлекаются разные слова,
но одного и того же смысла. Информация из первичной памяти извлекается с большой
скоростью, из вторичной – из-за необходимости перебора различных вариантов –
Медленнее.
Третичная память прочно фиксирует прошлый опыт: информация сохраняется даже
при серьёзных заболеваниях и массивных поражениях мозга, когда другие виды памяти
исчезают. В то же время информация оттуда извлекается с высокой скоростью. Вторичная и
третичная память – стабильные формы хранения информации.
Для сознательной работы памяти лучшим временем считается промежуток от 10 до 12
часов дня или после восьми часов вечера, когда организм максимально устойчив к
кислородному голоданию – зеваете меньше!
Более того, новая информация, вырабатываемая мозгом во время парадоксальной –
связанной с высокой активностью нервной деятельности – фазы сна, даёт возможность
разрешить любую проблему. По-видимому, мозг продолжает работу, стремясь уничтожить
или хотя бы сгладить очаг возбуждения, порождённый непрестанно «прокручиваемой»,
эмоционально волнующей проблемой.
14 Точнее, удаляется в некий «долгий ящик» в огромном хранилище мозга. Совершенно неожиданно её
фрагменты могут всплывать в будущем, через неопределённо долгое время, повинуясь неожиданным – иной
раз неосознанным – ассоциациям.
Успокоится он, только найдя некое приемлемое решение или объяснение. Видимо, мозг
мог бы сказать о себе словами Гёте: «Когда у меня нет новых и новых идей для обработки, я
точно больной». Ночная работа активно использует подсознание, хранящее почти 95 %
информации. Информационные обмены с подсознанием протекают в другой фазе – фазе
быстрого сна, характерной, в частности, быстрым движением глаз. Вот и математикам снятся
полезные вещи. Однажды Анри Пуанкаре, работая вечером, так и не смог решить сложное
дифференциальное уравнение. Отложив работу, он лёг спать. Под утро Пуанкаре увидел сон,
будто он читает студентам лекцию по теме своих вечерних занятий и легко вычисляет на
доске нужный интеграл. Итак, главное – не забыть утром результаты работы мозга во сне.
Увы, память человека слабеет пропорционально росту его беспокойства. Пытаясь
ускорить процесс вспоминания, человек начинает нервничать, испытывает стресс, что
тормозит и затрудняет работу систем памяти. Безусловно, лучше всего в этой ситуации
переключить внимание на какой-либо другой предмет, постараться сосредоточиться на иной
мысли. Тогда включается подсознание, и мозг сам выбирает удобный режим поиска нужной
информации. Например, задремали Вы в кресле у камина, и приснилось Вам, что Вы – химик
Фридрих Кекуле, который, находясь почти на грани отчаяния от безуспешности понять
структуру бензола, задремал в кресле у камина в своей лаборатории. И снится химику
Кекуле сон, а во сне видит он структуру бензольного кольца – основы органической химии.
Почти так оно, собственно, и было. Правда, приснилась ему змея, кусающая собственный
хвост – вот оно, кольцо!
Удивительной и неожиданной может оказаться природа информации, «всплывающей»
в сознание из подсознания. Особенно непонятен выбор момента её передачи. Подсознание
человека активно работает во время сна, раскладывая на части и перегруппировывая
громадные массивы информации. Исследования показывают: интенсивность работы
подсознания повышается в каждом следующем цикле парадоксальной фазы сна. Выше всего
эта интенсивность примерно за два часа до пробуждения. Вот не знаю только, в какое время
ночи Сэмюэлу Колриджу, принадлежащему к Озёрной школе (не путать с дачным
кооперативом), приснились образы поэмы «Кубла-хан». Но результат-то великолепен!
Получилось именно то, что сам Колридж сформулировал как определение: поэзия –
наилучшие слова, поставленные в наилучшем порядке.
Экспериментально доказано: в однородном материале лучше всего запоминаются
начало и конец, хуже – середина. Поэтому самое нецелесообразное и неэкономичное –
непрерывное заучивание вплоть до полного запоминания. Лучше запоминать такой
материал, делая перерывы – в это время подсознание самостоятельно обрабатывает его по
частям.
Не исключено, что при этом срабатывает так называемый «эффект Зейгарник» –
обнаруженное ещё в 1920-е годы влияние завершённости или незавершённости действия на
запоминание. Когда решение некоей задачи прерывается, она запоминается лучше, чем
задача, благополучно доведенная до завершения15. Причём доля запомнившихся
прерванных задач оказалась почти вдвое больше доли запомнившихся завершённых задач.
Как и полагал граф Биконсфилд – а по совместительству премьер-министр Англии
Бенджамин Дизраели: если помнить выгодно, никто забыт не будет. Но в чём же тут выгода?
Как раз в том, чтобы снять в конце концов высокий уровень эмоционального напряжения, не
получившего разрядки, если решение задачи было прервано.
«Память – это то, что определяет нас как личность. Мы – это то, что мы помним. Как
только мы перестаем помнить – нас нет», – отмечает Татьяна Черниговская. «Все на свете
ищут, все хотят знать, где в мозгу что где находится – где чашки, где ложки, где
французский, где английский и где все это хранится. Да нет этих мест, нигде не находится,
15 Похоже, после решения задачи мозг стирает промежуточные данные, понадобившиеся по ходу работы, и
зацепок для запоминания почти не остаётся.
потому что память – это не картотека, это не система ящиков, это – процесс. Каждый раз,
(нельзя два раза вспомнить одно тоже), каждый раз, когда вы вспоминаете – у вас идет
новый процесс, новые ассоциации. Материальным выражением, измерением памяти является
изменение эффективности связей между нейронами в сети».
Из памяти, по-видимому, ничто не исчезает – просто многое не может быть вызвано в
оперативную память из долговременной без особой эмоциональной встряски или без
специального запроса. Джером Клапка Джером «пошутил»: «Память подобна населённому
нечистой силой дому, в стенах которого постоянно раздается эхо от невидимых шагов. В
разбитых окнах мелькают тени умерших, а рядом с ними – печальные призраки нашего
былого «я»». Такой вот английский юмор.
Особенно проблематично что-то вспомнить, когда непонятно, как оформить запрос на
вызов информации из собственного мозга.
Интересный мнемонический приём – использование определенной последовательности
зрительно-пространственных образов для запоминания – Можно найти в книге Милорада
Павича «Внутренняя сторона ветра»: «Этот иностранец… учил их искусству запоминания,
мнемотехнике, разработанной на примерах речей Демосфена и Цицерона… Чтобы запомнить
текст, следовало, как рекомендовал русский учитель, вспомнить, как выглядел фасад одного
из часто попадавшихся на пути зданий, внешний вид которых застревал в памяти. Затем
нужно было представить себе, что по порядку открываешь все окна и двери этого здания и в
каждый проём, включая бойницы или слуховые окошки, проговариваешь одну из длинных
фраз Цицерона. Таким образом, мысленно обойдя всё здание и произнеся перед каждым
окном или дверью по одной фразе, можно было к концу воображаемой прогулки запомнить
всю речь и даже повторить её без особого напряжения». Писатель почти дословно пересказал
методику знаменитого мнемониста С.В. Шерешевского, зафиксированную выдающимся
психоневрологом А.Р. Лурия, много лет исследовавшим его память.
Мнемоника – набор приёмов запоминания, основанных на ассоциативных
«привязываниях» запоминаемых сведений к неким визуальным (как в вышеприведенном
примере), словесным или логическим рядам16. Мнемонические приёмы человек часто
придумывает сам для себя, запоминая телефоны, даты, важные сведения.
Ещё один важный этап в работе с памятью – научение, т. е. намеренное усвоение
определённого материала так, чтобы его можно было легко извлекать из памяти и
эффективно использовать. Научение отличается от простого заучивания или запоминания,
поскольку одновременно вырабатывается умение обрабатывать и использовать информацию
определенного типа. Известную с древнеримских времён поговорку учёные
перефразировали: не повторение – Мать учения, а применение!
У всех людей мозг имеет одинаковую глобальную структуру, однако каждый человек
уникален. Один имеет развитую знаковую память, другой – эйдетическую (образную).
Объём памяти многих великих людей поражает. Сенека мог запомнить и повторить до 2000
совершенно не связанных между собой слов. Великий математик Леонард Эйлер помнил все
степени, вплоть до шестой, чисел от 1 до 100. Академик Абрам Иоффе не заглядывал в
таблицу логарифмов, потому что помнил её наизусть. Вряд ли великие учёные зазубривали
горы чисел специально – скорее всего всё запомнилось в ходе употребления.
У некоторых людей память как будто вообще не имеет пределов. Например,
вышеупомянутый мнемонист С.В. Шерешевский легко запоминал огромные объёмы
произвольной информации: длинные – 100 и более – ряды цифр, слов, слогов, иностранных
16 Знаете, как в дореволюционных гимназиях учеников учили запоминать число «пи»? Очень просто: нужно
только запомнить простую фразу «Кто и шутя и скоро пожелаетъ «пи» узнать число, ужъ знаетъ» (здесь мы
учли старорусское написание слов со знаком «ъ»). А теперь подсчитайте число букв в каждом слове этой фразы
и запишите их. Вот и получилось с неплохой точностью число «пи»: 3.1415926536. Конечно, же «каждый
охотник желает знать, где сидит фазан» – лёгкое запоминание последовательности цветов в спектре.
слов, формул. Причём запоминал практически навсегда: возвращаясь к заданиям 15–
20‑ летней давности, он легко и безошибочно всё вспоминал. Приём, упомянутый выше в
пересказе Милорада Павича – лишь один из простейших в арсенале Шерешевского. Многие
технологии он употреблял вовсе неосознанно, и только многолетние кропотливые
исследования А.Р. Лурия17 позволили их сформулировать.
Есть люди, мгновенно – со скоростью, сравнимой с быстродействием компьютеров –
выдающие результаты арифметических и алгебраических действий над огромными числами.
Как устроен их мозг, что там модифицировано, улучшено, как сконструировано? Это пока
загадка.
Если мы что-то не можем вспомнить, нам часто кажется: искомая информация утеряна
безвозвратно. Правда, она может всплыть перед нами, когда уже совершенно не нужна:
нужный номер телефона кажется окончательно забытым, когда требуется срочный звонок,
но через много дней, во время случайного разговора, какая-то малозначимая деталь вдруг
извлекает этот номер из памяти. Или, скажем, вспомним (!) известный чеховский рассказ
«Лошадиная фамилия». Ассоциативный поиск фамилии зубного врача в закоулках памяти не
приводил к успеху, пока мужики не заговорили о продаже овса – вот тогда и всплыла нужная
фамилия: «Овсов! Овсов, Ваше превосходительство! Посылайте за Овсовым!».
Невозможность сохранить информацию практически неотличима от невозможности её
извлечь. Для успешного извлечения данных из памяти недостаточно, чтобы информация там
где-то хранилась. Важно, что она должна быть записана таким способом, чтобы каждый блок
был помечен особым образом, отличался от других похожих событий. Некий механизм
наклеивает специальные ярлыки на отдельные фрагменты, сортируя и классифицируя их.
Интереснейших вопросов об устройстве нервной системы так много, что одно их
перечисление и то займёт немало времени. Особенно же интересен вопрос, что происходит в
мозге при обучении. Некоторые учёные говорят: при откладывании информации в память
происходит активация клеток. Другие утверждают: есть гены, препятствующие
откладыванию в память информации – и запоминание происходит только тогда, когда
эмоциональные раздражители подавляют работу этих генов-блокаторов.
Теперь некоторая свежая информация к размышлению. Ученые из израильского
Института Вейцмана совсем недавно – в 2013 году – выяснили, что структуры нейронов
способны создавать в коре нашего головного мозга «архивы» из более ранних впечатлений
по мере добавления новых. Эти базы данных помогают сделать нам выбор в трудной
ситуации, то есть подключают к процессу принятия решения жизненный опыт.
Тал Хармелех (научный сотрудник департамента нейробиологии Института Вейцмана»
уточняет:
«Механизм, который формирует так называемые архивы впечатлений, и его влияние на
функции коры головного мозга до сих пор почти не изучены. Мы задались целью раскрыть
значение этих хранилищ информации.
Можно сказать, мы обнаружили окно в личную историю каждого человека. Выяснили,
что информация о впечатлениях включается в связи между сетями нервных клеток коры
головного мозга и может быть замечена при активации этой части мозга, которая, кстати,
приводит к длительным изменениям в связях между нейронами. Наши впечатления,
встраиваясь в эти соединения, создают буферные зоны (зоны ожидания), которые
активизируются при любом виде умственной деятельности. Это позволяет человеку
предвидеть результат в зависимости от его прошлого опыта.
17 Одной из самых сложных задач, решённых Лурия, оказалось обучение Шерешевского забыванию.
Колоссальные потоки информации, всплывающие по малейшему поводу, захлёстывали мозг, мешая ему
эффективно обрабатывать сиюминутные события. Только длительные тренировки по изобретённым Лурия
методикам позволили Шерешевскому отключаться от собственной памяти, чтобы без помех ориентироваться в
«здесь и сейчас».
Активированные с помощью воспоминаний нервные клетки коры головного мозга,
образующие «архивы», запечатлевались как яркие пятна на снимках магнитно-резонансного
томографа в течение 24 часов. Особенно важно, что этот эффект весьма надежен и может
быть применен к каждому отдельному событию в жизни человека.
Запущенные благодаря воспоминаниям «архивы» позволяют ученым диагностировать
психические расстройства и патологии головного мозга, а также выявлять качества, черты
характера, привычки и таланты человека». 18
Основную часть информационного потока обрабатывает сенсорная или
кратковременная память. Ученые полагают: зрительная сенсорная память – запоминание
угасающего следа зрительного изображения, сохранение довольно подробной информации в
течение нескольких секунд – обслуживает поздние стадии переработки информации,
сохраняя образ столько, сколько нужно для завершения этой стадии. Затем необходимая
часть данных – по некоторым оценкам, 1/10–1/4 исходного потока – переносится для более
долговременного хранения и последующего использования.
Сенсорная память работает в автоматическом режиме: внешние и внутренние факторы
слабо влияют на её операции. В первичной памяти с хранящимися данными производятся
некоторые действия: перезапись отдельных фрагментов, избирательное их сохранение и
усложнение материала в результате подключения дополнительных внешних и внутренних
информационных потоков. Во вторичной памяти мы можем запускать или прекращать
процессы, вести поиск в разных направлениях. Извлечение информации из одного места
может привести к запуску каких-то новых подсистем памяти, переключению на другие
нейронные сети. Всё это в конце концов направляет мысль по совершенно новому пути.
Озарение – как раз выход в сознание результатов подсознательной работы.
Но и такая работа сильно зависит от сознания – ведь именно из него поступает в
подсознание и цель умственной работы, и обрабатываемые сведения. Поэтому очень важна
степень увлеченности решаемой проблемой. Чем ярче доминанта, тем большая доля
подсознания (а не только сознания) занята решением. Чтобы подсознание справилось с
проблемой и наступило озарение, должна накопиться критическая масса сведений,
отправленных в подсознание, и критическая мощность раскрутки его сознанием
Кто как говорит, тот так и мыслит. И наоборот.
Как считал Эйнштейн, «можно стимулировать появление глубоких и оригинальных
мыслей, предоставляя полную свободу своему воображению, не ограничивая его
традиционными условными запретами. Он и относит открытие теории относительности не на
счёт своего особого дарования, а, напротив, – на счёт собственного так называемого
«задержавшегося» развития.
«Нормального взрослого никогда не станут беспокоить проблемы пространства и
времени, рассуждал Эйнштейн. – Есть вещи, о которых задумываешься только в детстве. Но
моё интеллектуальное развитие задержалось, в результате чего я начал размышлять о
пространстве и времени, будучи далеко не юным.»
В своих последних автобиографических записках великий физик вспоминает озарение,
которое привело его к созданию специальной теории относительности. Оно явилось
неожиданно, когда шестнадцатилетним юношей он просто мечтал о чём-то. «А что, если. –
подумал он тогда, – лететь рядом с лучом света с его же скоростью?»
Нормальные взрослые, как резонно заметил Эйнштейн, обычно заглушают в себе
подобные вопросы, а если они всё-таки возникают, то быстро забываются. Видимо, именно
это и подразумевал Уинстон Черчилль, когда говорил, что «много людей спотыкаются о
великие открытия, но большинство их них просто перешагивают и идут дальше».19
18 http://www.rusrep.ru/article/2013/07/10/okno
19 Венгер Вин, Поу Ричард. Неужели я гений? – СПб.: Питер Пресс, 1997. – С. 21–22.
Конечно же, и выдающиеся умы не всегда способны сразу по достоинству оценить
достижения современников.
Существует исторический анекдот на эту тему. Лорд Келвин знал, с каким трудом даже
признанные учёные усваивают новые идеи, и не обижался на это. И когда специалист по
магнитным компасам, королевский астроном Эри осмотрел компас, изобретённый Келвином,
он мрачно изрёк: «Не будет работать». Келвин добродушно заметил: «Это слишком
серьёзные слова, чтобы их можно было считать мнением королевского астронома».
Да что там! Сам Рене Декарт, один из величайших мыслителей в истории, «доказывал с
полным логическим обоснованием, что открытый Торричелли эффект давления воздуха
невозможен. Однако Торричелли, вопреки утверждениям этого маститого учёного,
удерживал столбик ртути на метровой высоте. Кроме того, он поставил опыт, показавший,
что если выкачать воздух, заполняющий пространство между двумя медными тарелками, то
даже четыре лошади будут не в состоянии растащить их», – приводил пример Эдвард де
Боно.
Основатель Общества когнитивной науки (науки о познании) Доналд Артур Норман
предложил определение: память – информационная система, где путь, приводящий к ответу,
подбирается и определяется самой формой вопроса, т. е. нужная информация сама содержит
в себе подсказку, где её искать20.
Можно сделать вывод: постановка вопроса, проблемы, формулировка задачи должна
осуществляться в форме, соответствующей типу искомой информации. Поиск данных для
получения ответа задаётся типом вводимой исходной информации. То есть направление
движения по коридорам памяти указывается – хотя бы приблизительно – самой формой
вопроса.
3. Эрудиция, ум, интеллект – сравним
Я рассказал им, Кто, Когда,
и Почему, и Отчего,
сказал, Откуда и Куда,
и Как, и Где, и Для Чего;
Что было Раньше, что Потом,
и кто Кого, и Что к Чему,
и что подумали о Том,
и если Нет, то Почему?
А.А. Милн. «Винни-Пух и все-все-все»
Всего лишь несколько десятков лет тому назад, считалось, что общепринятым
индексом интеллекта нечто, что присущее людям, но при этом отсутствует у самых
приближенных к человеку по умственному развитию животных – это и есть интеллект.
Люди, которые занимаются этим вопросом, исследователи гораздо взвешеннее и
осторожней в своих оценках. Интеллектом они наделяют и животных.
Считается, что интеллект вороны или попугая может быть на уровне малолетних детей,
хоть о нем и говорят: «глупая птица». То есть, интеллект есть некая универсалия, присущая
не только человеку. Но наши братья меньшие – наше отражение. Наше зеркало. Они же и
позволяют нам понять, что есть интеллект, и как его развивать. За последние 20–30 лет наша
отечественная наука очень сильно отброшена назад. С болью я смотрел на пустующее
20 В «Что? Где? Когда?» особо ценятся вопросы, содержащие в себе подсказку, требующие для ответа не
столько экзотических знаний, сколько изощрённого анализа.
гигантское здание Института Нейрокибернетики, в котором я получал образование. Сейчас,
все начинает потихоньку оживать, но западные ученые ушли далеко вперед в области
нейробиологии. Хотя есть факты наверстывания упущенного отечественной
нейробиологией, взять к примеру работы профессора, член-корреспондент РАН и РАМН
Константина Владимировича Анохин, внука академика Петра Кузьмича Анохина.
Исследование интеллекта животных нельзя проводить лишь в сравнительном режиме –
кто из них умнее. Например, обезьяна может банан достать с помощью палки, а дельфин нет.
Заведомо в неравные условия ставить нельзя. Это пищевые предпочтения, разные среды
обитания, разные конечности и так далее. Поэтому профессор Леонид Викторович
Крушинский предлагает термин «рассудочная деятельность», позволяющий избежать
отождествления мыслительных процессов у животных и человека. Безусловно, вершиной
творчества Леонида Викторовича Крушинского является создание учения об элементарной
рассудочной деятельности животных как предыстории интеллекта… По его определению,
рассудочная деятельность – это выполнение животным адаптивного поведенческого акта в
экстренно сложившейся ситуации. Какое-то животное в экспериментах умнее, какое-то
глупее. Можно сказать: более интеллектуальное, менее интеллектуальное, более развитое,
менее развитое. Нам же интереснее всего исследования, объектом которых является
внутривидовое разделение.
То, что не содержится – или не может быть быстро найдено – в памяти, можно
получить самому, используя механизмы не памяти, а мышления. Коротко говоря, сила мысли
может компенсировать недостаток информации или плохую работу памяти. Мышление в
принципе способно превзойти эрудицию. Психологи (Ф. Левинсон-Лессинг) даже различают
учёных-эрудитов – «ходячие библиотеки» – и творчески продуктивных учёных, не
перегруженных стандартными знаниями, зато обладающих высокоразвитой фантазией и
способностью быстрой реакции на мгновенно мелькнувшую новую мысль или информацию.
Джордано Бруно сказал: «Особенностью живого ума является то, что ему нужно лишь
немного увидеть и услышать для того, чтобы он мог потом долго размышлять и многое
понять». Эрудиту же может помешать некоторая «захламленность» памяти. Льюис Кэрролл
считал, что такие «умы, торопливо пробегающие книгу за книгой, не дожидаясь, пока их
содержание будет усвоено или классифицировано», хотя и наполнены всевозможными
сведениями, частенько неспособны дать содержательный ответ. Кэрролл пишет: «…
несчастный владелец такого ума весьма начитанный человек. О чём его ни спросить, всё
знает. Но обратитесь к нему и задайте вопрос, например, из английской истории. Эрудит
добродушно улыбается, делает вид, будто ему всё известно, и ныряет в дебри своего разума
за ответом. Выныривает он с горстью многообещающих фактов, но при проверке
выясняется, что все они относятся не к тому столетию. Он улыбается ещё шире и вновь
ныряет…» Ответ опять типа «в огороде бузина, а в Киеве – дядька».
Интеллект и память – функции головного мозга человека. Они просто заложены в
организм человека генетически, но ещё развиваются и совершенствуются на протяжении
всей его жизни. И от уровня их развития зависит, станет ли человек личностью, способной к
творческим свершениям, как использует в жизни эту способность, в конечном счете – будет
ли счастлив.
В известном определении: «Интеллект – способность человека к обучению и
осмыслению опыта, способность воспринимать и усваивать знания» трудно увидеть
различие между интеллектом и эрудицией. Нам представляется, что эти категории всё же
различны, а основа отличия – способность получения нового знания, нового, как минимум,
для носителя мысли
При решении проблемы можно рассчитывать лишь на ту информацию, которую можно
воспринять в данный момент, или на ту, которую сумеешь извлечь из памяти. Есть
утверждение: преимущества при решении получает не тот, у кого эрудиция богаче, но тот,
кто быстрее извлечёт из памяти нужную информацию. Этот тезис, вообще говоря,
представляется спорным, поскольку недостаточно только разложить перед собой все нужные
материалы – важнее определить правильный их порядок, последовательность, перебрав
различные комбинации. При этом важно как можно скорее отбрасывать те, которые явно не
имеют отношения к проблеме, не приближают к выходу из лабиринта, а уводят от него 21.
Время для получения ответа определяется не скоростью извлечения материалов из памяти, а
скоростью мышления.
Извлечение необходимых сведений – самостоятельный навык, поддающийся отдельной
тренировке. Вот пример. Создадим для изучения какого-то языка две группы, действующие
по разным методикам. Одна – куда придётся, очевидно, отобрать людей с хорошей памятью
– просто заучивает слова. Скажем, 5–10 тысяч. Заодно, конечно, изучаются и
грамматические правила – но тренируется только запоминание и вспоминание отдельных
слов. Другая же группа ограничена словарём в 100 самых употребительных слов и
выражений – но постоянно упражняется в их применении, построении фраз и целых
диалогов. Можно не сомневаться: реальные носители этого языка сочтут знающей язык
только вторую группу.
Из этого примера видно: знание делится на динамическое и статическое. Статическое
знание – просто эрудиция: накопленные сведения без попыток работы с ними. Конечно, без
такого сырья мысль работает вхолостую. Но на одном сырье, без переработки, ничего
серьёзного не разовьёшь22. Динамическое знание – умение применять накопленные
сведения – в какой-то мере опирается на статику. Но только динамика способна порождать
новое в человеческой жизни.
Немалая часть конфузов проистекает как раз из путаницы между этими двумя видами
знаний. Если тренироваться только в статике – динамика окажется непреодолимым
барьером.
Некоторые психологи полагают: экономное символическое обозначение понятий и
отношений между ними – важнейшее условие продуктивного мышления. В качестве
примера, с которым трудно не согласиться, приводится резкое упрощение операций
арифметики при переходе от римских чисел к арабским. Вообще математика и теоретическая
физика в этом аспекте предоставляют огромное множество эффектных примеров – таких, как
компактная форма классических уравнений теории гравитации Альберта Эйнштейна23:
всего лишь одна строка символов имеет огромный информационный объём, суммируя наши
знания о природе пространства и времени. Простые на первый взгляд понятия симметрии –
инвариантности характеристик системы относительно специальных преобразований её
элементов – и калибровки – инвариантности относительно выбора единиц измерения – лежат
в основе всех современных взглядов на устройство мира и материи.
Другой вопрос: насколько прост для мышления выход на следующий этаж понимания и
осмысления? Чтобы выйти за рамки стандартных и привычных факторов и данных
наблюдений, чтобы окинуть взглядом весь горизонт, нужно забраться достаточно высоко по
ступеням абстракции. И для этого надо научиться не только использовать ресурсы памяти,
но и соединять известные понятия в неожиданных, нестандартных, неизвестных ранее
комбинациях. То есть научиться нестандартно творчески мыслить.
21 Впрочем, если выход так и не найден, приходится возвращаться к отброшенным сведениям в надежде,
что с их помощью прояснится форма лабиринта.
22 Это, кстати, относится не только к интеллекту. Например, нынешняя сырьевая ориентация российской
экономики, возникшая в значительной степени из-за недомыслия политических и хозяйственных
руководителей позднесоветских времён и сохраняющаяся по сходным свойствам значительной части
постсоветских руководителей, заведомо бесперспективна. В частности, и потому, что сама порождает
процессы, закрепляющие интеллектуальную недостаточность.
23 Да и развитие электродинамики резко ускорилось после того, как Хевисайд – при поддержке Герца –
преобразовал уравнения Максвелла, сведя два десятка исходных выражений к четырём.
Приведём развёрнутую цитату из книги выдающегося английского философа и
психолога Майкла Полани «Личностное знание». В этом тексте мы найдём не только
прекрасный пример связи между словами и восприятием. Скорее Полани хорошо
иллюстрирует процесс «строительства» мышлением нового знания, нового смысла. Так,
долго бродя в темноте по незнакомым комнатам, мы внезапно натыкаемся на выключатель и
– свет озаряет наш странный и извилистый путь.
Итак, «представим себе студента-медика, посещающего курс рентгеновской
диагностики легочных заболеваний. В затемнённой комнате он изучает неясные следы на
флуоресцентном экране, расположенном перед грудной клеткой пациента, он слушает
профессиональный язык комментария врача-эксперта, объясняющего своим коллегам
значение этих теней. На первых порах студент в полном замешательстве. На рентгеновском
снимке он видит только тени сердца и рёбер с паутинообразными пятнами между ними.
Слова эксперта кажутся плодом его фантазии; студент не видит того, о чём он говорит. В
течение нескольких недель он ходит на лекции, внимательно рассматривая каждый новый
снимок различных заболеваний, и в какой-то момент к нему приходит понимание;
постепенно он забывает о рёбрах и начинает видеть лёгкие. В конечном итоге, при
достаточном упорстве перед ним открывается широкая панорама мельчайших деталей:
физиологические различия, патологические изменения, рубцы, хроническая инфекция и
признаки обострения. Так он открывает новый мир. По-прежнему он видит лишь небольшую
часть целого, но снимки и комментарии к ним начинают обретать значение. Он близок к
пониманию предмета; это подобно щелчку выключателя. Изучение языка лёгочной
рентгеноскопии ведёт к пониманию рентгеновских снимков. Проблема имеет две стороны –
непонятный предмет и непонятный текст, его комментирующий. Мы направляем наши
усилия на соединение двух половин проблемы; в поисках концепции, заключающей в себе
понимание неразрывности слова и предмета, решение приходит внезапно».
Существенная часть концепции Полани – разделение человеческих знаний и
механизмов мышления на артикулированные и неартикулированные. Скажем, мысль «Я
должен пойти в магазин и купить там хлеба» – артикулированная: вполне выражаемая
словами, не содержащая невыразимого остатка. Но есть множество вещей, которые человек
знает и умеет, но не может вполне передать словами.
Например, опубликовано несметное множество рецептов узбекского плова. Но ещё
никто не приготовил хороший плов только по рецепту. Но если раз десять побудешь
подмастерьем у настоящего мастера плова – вдруг начнёшь чувствовать, что, сколько и в
какой момент класть, как добавлять воду и регулировать огонь… Значит, в этом искусстве
есть, помимо артикулированного рецепта, какой-то остаток, невыразимый словами.
Неартикулируемое знание ценнее артикулируемого. Что можно выразить словами –
Можно и формализовать, и в конечном счёте переложить на плечи компьютера.
Неартикулируемое – именно то, что человек должен (может) делать сам.
Конечно, формализованная часть любого искусства тоже очень важна. В ней можно
применять готовые приёмы, не решая заново, «с нуля». Значит, больше сил остаётся на
творческую часть задачи. Правда, неформальным творчеством занимается в основном
подсознание – но ведь сознание снабжает его материалом для работы и ставит цель. Поэтому
так ценится любой способ формализации – перевода какой-то части знания на
артикулированный уровень.
Для эффективной работы нужно сочетать продвижение на обоих полях – и вовремя
переходить на то, где очередной этап проходит быстрее. Например, перевод арифметической
задачи в алгебраическую форму, позволяющий отвлечься от её содержания и найти
формальное решение, как правило, упрощает работу. Например, знаменитое решение Полем
Дираком придуманной им задачи о рыбах:
«Три рыбака легли спать, не поделив улова. Проснувшийся ночью первый рыбак решил
уйти, взяв свою долю. Но число рыб не делилось на три. Тогда он выбросил одну рыбу, а из
числа оставшихся забрал треть. Второй и третий рыбаки поступили аналогично (выбросили
по одной рыбе и взяли треть из оставшихся). Спрашивается, какое наименьшее количество
рыб может удовлетворить условию задачи»?
Ответ Дирака хорошо известен: минус две рыбы. Думаю, каждый сможет
воспроизвести рассуждения Дирака. Впрочем, в решении великому физику, видимо, помогла
его особая привязанность к античастицам.
Но каждый из нас не раз сталкивался с задачами, где именно содержательный анализ –
кратчайший путь к успеху. В этой книге приведено несколько таких задач – и любой сборник
головоломок изобилует ими.
Прекрасно сочетал формальную и содержательную работу гениальный физик
Вильгельм Конрад Рентген. Причём не только в науке, но и в живом общении. Рассказывают:
однажды он получил почтой просьбу: прислать несколько рентгеновских лучей с
инструкцией по использованию. У автора письма не было времени приехать самому, хотя он
и мечтал избавиться от застарелой револьверной пули, сидевшей в грудной клетке. Рёнтген,
не лишенный юмора, написал в ответ: «Увы, к сожалению, в настоящий момент у меня нет в
наличии Х-лучей, к тому же их пересылка – дело очень трудное. Давайте поступим проще:
пришлите мне вашу грудную клетку».
Тот же Рёнтген, отвечая позднее на вопрос репортёра: «Что вы подумали, увидев
вспышку флуоресцирующего экрана?» – сказал: «Я исследовал, а не думал». Это – пример
серьёзности и глубины научной работы. Открытое явление было столь новым, что сведения
для плодотворных размышлений могли появиться только после кропотливых экспериментов.
Рёнтген сразу понял это и не позволил себе уходить в заведомо бесплодные раздумья. Не
слишком ли часто у нас сначала «думают», а исследовать забывают?
Хрестоматийный пример накопления критической массы размышлений на заданную
тему, приводящего, в конце концов, к возникновению нового качества (структуризации
информации) – периодическая система элементов. Д. И. Менделеев исследовал различия
свойств химических элементов, размышляя над возможными закономерностями в этих
различиях, около 15 лет. Решение же пришло в течение одной ночи24! Когда критическая
масса накопилась – процесс развивается взрывными темпами.
Естественно, поиск решения облегчается при использовании для информационных
блоков разного типа символических обозначений: развешивание «ярлычков» на «ящички» с
различными материалами и сведениями25 облегчает их анализ и комбинирование. Мозг
использует разные типы кодов: зрительно-пространственный, словесный, буквенный,
цифровой, акустический и т. д. Интеллектуальная «мощность» мозга в значительной степени
определяется его способностью к созданию и применению различных типов кодов на разных
уровнях абстракции. Эффективность кодирования зависит и от того, насколько адекватно
способ кодирования соответствует форме и содержанию анализируемых данных26.
Важный вывод: одна из ключевых задач развития интеллекта – именно в том, чтобы
определить, какие типы кодирования информации наиболее эффективны для данного
человека с учётом особенностей его мышления, а также научить его самостоятельно
выбирать формы записи и обработки данных, наиболее продуктивные для конкретной
задачи.
Нужно, впрочем, добавить: всё вышесказанное относится к оценке и возможной
24 Кстати, во сне. Это напоминает, сколь велика в подобных качественных переходах роль подсознания.
25 Вспомните трюки Шерешевского!
26 В теории информации учёт даже такой простой характеристики, как вероятности кодируемых вариантов,
позволяет сократить суммарный объём закодированного материала во много раз. На подобных идеях строятся
программы сжатия данных, позволяющие, например, заметно увеличить пропускную способность каналов
связи.
корректировке работы лишь части мыслительного механизма мозга – сознания. Однако в
действительности гигантскую работу проводят системы подсознания. Возможность и умение
подключать их к поиску решения, к анализу вопроса могут сыграть решающую роль при
развитии интеллекта.
По результатам исследований, проведенных американскими учеными, люди вполне
могут жить до 200 лет, но для этого следить за тем, чтобы в организм человека постоянно
поступали необходимые ему полезные вещества.
Китайские ученые, как будто подхватив эстафету, доказывают, что потребление
продуктов, богатых магнием, таких, как орехи, полезно для мозга и памяти. У большинства
населения в странах с развитой промышленностью в организме наблюдается дефицит
магния, который с возрастом увеличивается. В результате формируются проблемы с
памятью, и предотвратить или уменьшить эффект можно с помощью дополнительного
магния. По мнению ученых, увеличение потребления магния может быть способом
предотвращения эффектов, которые оказывает на нервную систему старение, в частности,
прогрессирующего ухудшения памяти.
Сейчас известны разнообразные методики развития мышления, опирающиеся на
известные сведения об устройстве и особенностях функционирования головного мозга. Вот
некоторые сведения о таких методиках, предлагаемых известными специалистами в этой
области – в частности, психологами Л. Уилсон и А. Спирсом.
Основа так называемого «brain-based learning» (обучения, основанного на мозге) –
современные знания о структуре и функциях человеческого мозга в различных его
состояниях. Это позволяет применять результаты новейших достижений нейрологии и
биологии при формулировке методик обучения эффективному творческому мышлению. При
этом оказывается возможным понять и набор реакций, и поведение обучаемого человека.
«Brain-based learning» основано на следующих принципах:
✓ мозг – своеобразный «параллельный процессор», т. е. может одновременно
выполнять различные виды работ;
✓ в восприятии информации о целостном объекте и его частях участвуют
одновременно и весь мозг, как единое целое, и отдельные его части;
✓ информация хранится в различных структурах мозга и вызывается в «оперативную
память» благодаря действию многочисленных нейронных сетей и специальных механизмов
памяти;
✓ обучение затрагивает организм в целом; в его процессе активизируются все системы
организма, в том числе органы движения, питания, происходит концентрация внимания;
✓ поиск человеком смысла – качество, имманентное человеку; поиск смысла
происходит путём создания образов; при этом особую роль играют эмоции, сильно
влияющие на внимание, осознание и память;
✓ знание намного важнее просто информации;
✓ мозг – явление общественное: он лучше развивается в процессе общения с себе
подобными;
✓ весь комплекс обучения тормозится в условиях стресса, улучшается при
фокусировке внимания и периферического восприятия;
✓ каждый мозг имеет индивидуальные особенности.
Так может быть, описанный комплекс принципов поможет вам выбрать стратегию
победы и в следующей известной издавна игре? Играют два человека, по очереди
выкладывая куриные яйца на квадратную салфетку. Яйцо, уже уложенное на стол, больше
нельзя трогать или передвигать – ни тому, кто его положил, ни его противнику.
Выкладывают яйца по очереди, пока вся салфетка не будет покрыта яйцами. Выигрывает
последний, кому удаётся положить яйцо. Так вот, при правильной стратегии первый игрок
всегда может выиграть! Найдите такую стратегию (учтите: яйца – трёхмерные материальные
объекты неправильной округлой формы.) Да, и подсказка – иногда эту игру называют
«Колумбово яйцо» – поможет вам при выборе стратегии.2
4. Как оценить уровень интеллектуальных способностей человека
– Все кошки, уверяю вас, знают всё!
– Отлично! – проворчал Король. – Тебе придётся это доказать!
Раз ты такая умная, как говоришь, я задам тебе три вопроса. И мы
увидим… то, что увидим!
П. Трэверс. «Мэри Поппинс»
Вначале общее утверждение: прежде чем обучать человека чему-нибудь, нужно знать,
на каком уровне интеллектуального развития этот человек находится.
Ещё в 1950-е годы американский психолог Ч. Спирмен установил: интеллект человека
развит неоднородно. Пространственный интеллект наиболее развит у архитекторов,
художников, инженеров. Вербальный или семантический – у философов, писателей,
естествоиспытателей, склонных не к формальному, а к образному отражению мира.
Интеллект такого рода оперирует суждениями, понятиями, ему свойственно метафорическое
мышление. Поэтому и разработаны специализированные тесты для установления
«профориентации» каждого отдельного человека. В Интернете такого «добра» полным
полно. Помощи, впрочем, от этих наскоро слепленных тестов немного.
Читатель наверняка хорошо представляет себе, что нужно делать в таких тестовых
заданиях: продолжить числовую последовательность, исключить лишнюю фигуру или слово,
вставить число, слово, фигуру, определить связь между наборами букв или чисел, используя
аналогию, и т. д. За каждую задачу начисляются баллы либо просто подсчитывается
количество правильно решённых заданий. Сумма этих баллов по специальной шкале
пересчитывается в коэффициент интеллектуальности КИ (в оригинале IQ – Intellectual
Quotient). В ведущих психологических лабораториях мира, изучающих структуру
интеллекта, КИ-тесты условно делятся на две группы: тесты «скорости» и тесты «уровня». В
первом случае вводится жёсткий лимит времени. Для проведения тестов второго типа
характерно нарастание сложности заданий, быстроте же их решения особого значения не
придаётся. Некие математические расчёты переводят данные тестирования в коэффициент
интеллектуальности.
Словом, некоторые психологи всерьез предполагают, что интеллектуальные тесты
измеряют мыслительные способности человека. Но находятся и разумные скептики:
«Мы выдумываем нечто и смотрим, как испытуемые выполняют наши трюки. При этом
мы почему-то уверены, что они играют именно в эту игру, придуманную нами» (Р. Серпель).
Так что ответить на вопрос «что мы имеем в виду, говоря о «человеческом
мышлении»», непросто. Можно ли однозначно установить критерии, качественно
выделяющие мышление человека из набора мыслительных действий других живых существ?
Неужели отличия и превосходство носят только количественный характер? Вот, скажем,
такая история об исследованиях интеллектуальных способностей.
Человекообразные обезьяны имеют достаточно высокий уровень интеллекта.
Раскалывая камнями орехи, шимпанзе демонстрируют явное представление о целях своих
действий, способны планировать деятельность на несколько шагов вперёд, что соответствует
уровню детей 4–5 лет. Опыт деятельности передаётся: матери учат детёнышей навыкам
раскалывания, старшие братья обучают младших. Разные популяции шимпанзе различаются
по способам употребления орудий, причём это не врождённые различия, а приобретённые
(приматологи в этом случае безоговорочно говорят «культурные»). Карликовый шимпанзе
Кензи к 8 годам по уровню понимания обычной речи превзошёл двухлетнюю девочку,
потому что, в отличие от неё, безошибочно различал субъектно-объектные отношения.
Правда, его генеративная способность соответствовала таковой у ребенка полутора лет. Зато
высказывания
обезьян,
освоивших
вспомогательные
коды,
обнаруживают
«перемещаемость», то есть могут относиться к чему-то, что отсутствует в момент речи.
Человеческий детёныш научается этому к трём годам. Я уж не говорю о воронах – явные
признаки их высокого интеллекта обнаруживаются хотя бы в том, как они, держа в клюве
грецкий орех и забравшись повыше на провода над шоссе, бросают орехи на проезжую часть
и дожидаются, пока проезжающие машины расколют твердую скорлупу. Умной птице
остается лишь спустить за едой на дорогу. Так что интеллект проявляется именно в
добывании еды. Как это начиналось и у человека.
Очень может быть, что в рамках тестирования изучается вовсе не умение человека
мыслить вообще и уж тем более не глубина мышления, а именно – и только – его
способность мыслить быстро, решая определённое число задач за ограниченное время. То
есть беглость мышления – среднее количество мыслей, приходящих в голову в единицу
времени. Но гибкость мышления может быть важнее – лучше одна нестандартная мысль, чем
десяток тривиальных. Думаю, что тесты не могут дифференцировать тонкие характеристики
интеллекта. Как и в музыке: беглость пальцев исполнителя не сможет компенсировать
отсутствие творческих композиторских способностей или хотя бы способности
прочувствовать композиторское творчество.
Да и вообще, в тестах тоже можно просто натренироваться: тот, кто много раз
тестировался, действует лучше, чем новичок. Но эффект тренированности исчерпывается
после 4–5 прохождений подготовки к тестам: десятое испытание не даёт лучших
результатов, чем пятое.
Конечно, результат интеллектуального тестирования зависит не только мыслительных
возможностей, но и от всех психологических и физиологических особенностей. Поэтому
развитие интеллекта возможно при тренировке сосредоточенности, внимания, концентрации
мыслей на одном предмете. Внимание – первичный фактор всякого знания. Не давай своим
мыслям разбегаться, направляй их, держи их в узде точных формулировок.
Строго говоря, избыток внимания тоже опасен: он может обернуться полной
концентрацией на своём внутреннем мире, отключением от внешней реальности. В крайней
форме это становится уже болезнью – аутизмом (от греческого «аутос» – сам: имеется в виду
сосредоточенность на самом себе). Всем нам памятен аутист Рэймонд, блистательно
сыгранный Дастином Хоффманом27 в фильме «Человек дождя» (Rainman по созвучию с
Raymond). Но известный советский (ныне, как водится, американский) учёный Александр
Моисеевич Хазен отмечает: некоторые несомненные признаки аутизма являли многие
великие учёные – от Ньютона до Эйнштейна. Очевидно, именно такая сверхконцентрация на
собственных мыслях позволяет им продумывать свои теории в мельчайших деталях,
находить неожиданные принципиально новые концепции, убеждать недоверчивых
коллег28… Да и не столь знаменитые аутисты зачастую обладают недюжинными
способностями29 – хотя и ценой потери множества других, порою куда более важных для
повседневной жизни.
27 Хоффман до боли отчётливо представляет себе проблемы такого человека. Ведь он и сам страдает
психической проблемой – дислексией, то есть неспособностью обучиться чтению. Все свои роли он учит со
слуха – их ему зачитывают коллеги.
28 Математический аппарат специальной теории относительности ещё до Эйнштейна создали Хендрик
Антон Лоренц, Анри Пуанкаре и Герман Минковский. Но никто из них не рискнул объявить новые формулы не
просто трюками для расчёта в некоторых особых случаях, а следствием принципиально новых физических
законов. Именно смена аксиоматики – бесспорная и единоличная заслуга Эйнштейна.
29 Например, среди них немало мастеров сверхбыстрого счёта. Недаром персонаж Хоффмана помогает
брату выиграть в казино немалую сумму в покер. В этой игре ключевую роль играет оценка вероятности
появления значимых комбинаций карт, остающихся в игре, с учётом карт, уже сброшенных в предыдущих
турах.
Хазен считает аутизм своеобразной вариацией на тему теоремы Гёделя. Этот немецкий
математик прославился доказательством того, что в любой непротиворечивой системе
аксиом (достаточно обширной, чтобы описывать не только логику, но и математику) можно
построить утверждение, средствами этой системы недоказуемое и неопровержимое. Такое
утверждение можно считать истиной и даже добавить к системе аксиом – но в новой,
пополненной, системе появятся новые недоказуемые истины.
Теорему Гёделя иногда считают принципиальным ограничением способности человека
к познанию. На самом же деле она гарантирует, что познание будет бесконечным. Всё тот же
Хазен пишет: «Теорему Гёделя часто трактуют как доказательство невозможности познания
природы человеком. Это ошибка. Наоборот, существование логически недоказуемых
утверждений означает, что мозг человека всегда способен найти принципиально новые
решения. Какие бы трудности ни накапливались в науке, всегда согласно теореме Гёделя
существует принципиально новое решение проблем. Это гарантирует отсутствие тупикового
предела в познании природы мозгом человека».
Применительно же к аутистам Хазен отмечает: «Одна из формулировок теоремы
Гёделя говорит о возможности бесконечно длинных логических доказательств. Это и есть
аутизм в тех случаях, когда он не связан с патологией… Известный российский генетик
Владимир Эфроимсон в «Предпосылках гениальности» писал о том, что в истории от
первобытных племён всегда существовал культ нестандартных людей. Диапазон: от
альтруистов до крайних форм – юродивых и дурачков. Аутичные личности, не переходящие
пределов патологии, обладают большими возможностями к интуитивным находкам в
познании природы».
Культ нестандартных людей нашёл свое продолжение много позже, примером чему
киники в Греции, суфии на Востоке, блаженные на Руси. Образцы нетрадиционного
поведения стали своего рода приёмами смещения традиционного сознания. Вы сами можете
оценить, сколь обыденно, сколь сковано ваше собственное миропонимание, упражняясь в
ответах на чань30-буддистские коаны или поставив себя на место, например, ходжи
Насреддина.
Купил ходжа31 утром три оки мяса и, занеся домой, пошёл по своим делам. А жена
позвала приятельниц и устроила им великолепное угощенье. Когда вернулся ходжа, ему она
подала плов из булгура – на воде. Ходжа сказал: «Если у тебя не было, положим, времени,
чтобы приготовить мясное блюдо, неужели ты не могла бросить в булгур несколько жирных
кусков для придания вкуса?» На это жена ответила: «Хотела, да тут вышла история. Когда я
была занята мясом, ненадолго вышла с кухни, а тут выскочила откуда-то твоя любимая
полосатая кошка и всё мясо съела. Я пришла, смотрю – она облизывается». Ходжа принёс
весы. Затем вытащил из-под мангала кошку и взвесил её; вышло ровно три оки. Тогда он
сказал жене: «Ах ты бесстыжая! Если это – Мясо, куда же делась кошка? А если кошка, – где
мясо?»
Кстати, к аналогичным эффектам приводит и чувство повышенной ответственности.
Это заметнее всего при ухудшении результатов, показываемых участниками каких-либо
ответственных соревнований, будь то спортивная игра или олимпиада по математике, перед
которой тренер (учитель) долго «накачивает» участников, объясняя, что произойдёт в случае
неудачи. Каждой проблеме должен соответствовать свой уровень и характер мотивации.
Мне довелось испытать этот закон на себе. Попав в телепередачу «Что? Где? Когда?», я
прошёл отборочные туры блестяще (это не хвастовство, а оценка, данная организаторами
30 У нас это китайское название философской секты чаще произносят по-японски: дзэн.
31 Это не имя легендарного остроумца, а всего лишь почётное свидетельство исполнения им священного
долга каждого мусульманина: участия в хадже – ритуальном путешествии в святой город Мекку. Потому и
пишется «ходжа» с малой буквы.
отбора). Но как только сел за игровой стол – Меня буквально замкнуло. Я посчитал это
досадным сбоем, тем более что за пределами стола игра шла по-прежнему легко – я
расщёлкивал любой вопрос. Опять сажусь в телеигру – и вновь жёстко торможу. Точно по
поговорке, популярной в нашей игре: «Вероятность ошибки обратно пропорциональна
расстоянию до стола».
Это игровое бессилие стало буквально наваждением. В конце концов я решил, что из
игры надо уходить. Но тут Андрей Каморин и Сергей Царьков пригласили меня во вновь
созданную команду. И я решил: сыграю последнюю игру. А в последней, понятно,
ответственность меньше: всё равно на дальнейшую судьбу она не повлияет. Но при счёте 0:3
накопилась спортивная злость – и в условиях, когда мотивация уже не давила, игра наконецто пошла. Психологический порочный круг разомкнулся – и я наконец заиграл в полную
силу.
Закон Еркеса – Долсона может проявиться и на экзамене, и в срочной работе…
Высокое напряжение в нейронной сети, как в электрической, пробивает изоляцию и создаёт
опасные замкнутые контуры. Чтобы избежать пробоев, желательно заранее готовить себя к
действиям под давлением мотивационного стресса – тренироваться в условиях, как
говорится, максимально приближенных к боевым.
Александр Лук считает, что остроумна не только шутка. «Остроумным может быть
решение трудной проблемы, техническая идея, научная гипотеза. Творческое решение
научной задачи тоже связано с неожиданным сопоставлением отдаленных явлений, внешне
ничем не связанных. Отсюда вытекает необходимость искать те общие признаки, которые
есть в комическом и некомическом остроумии; а также определить роль остроумия в
научном мышлении, его место в системе творческих способностей.
Перенесение термина «остроумие» для характеристики технических решений и
научных теорий – не терминологическая вольность и не метафора. К любой задаче можно
подойти с разных сторон. И лишь неожиданное, оригинальное и в то же время простое
решение называют остроумным. Остроумие в данном случае не сопровождается смехом, но
несомненно вызывает эстетическое наслаждение. Этот критерий и служит подсознательным
основанием считать решение остроумным.
Термин «остроумный» относится не к оценке истинности научного результата, а к
оценке способа, которым получен результат.
В научном творчестве огромную роль играют подсознательные процессы, или
интуиция. Однако творчество нельзя считать полностью подсознательным процессом.
Предварительное накопление материала и критическая оценка результата осуществляются
под контролем сознания. Это те 99 % потения, о которых писал Эдисон. Но между этими
стадиями лежат подсознательные этапы инкубации и озарения, как раз тот единственный
процент вдохновения, в котором и состоит секрет творчества. Момент осознания идеи
наименее поддается регулированию с помощью волевых усилий. Все же многие
исследователи и художники из личного опыта знают, какие условия наиболее
благоприятствуют творчеству. Здесь возможны большие индивидуальные различия.
Например, некоторым очень важно высокоинтеллектуальное окружение, включающее
общение не только с коллегами, но также и с людьми далеких профессий. Если обмен
информацией с ними по-настоящему интересен, не тривиален и требует умственных усилий,
он нередко стимулирует творческую активность и оказывается полезен для решения чисто
профессиональных задач, содержание которых не имеет прямого касательства к тематике
бесед. Процесс работы над статьей, процесс изложения помогает иногда решению задачи,
потому что способствует систематизации накопленного материала и уяснению проблемы
«для самого себя».
Продуцирование остроты, как и любой творческий процесс, связано с выходом за
пределы формальной логики, с освобождением мысли от тесных рамок строгой дедукции.
Еще в прошлом веке один математик рассказал о процессе решения трудной задачи: «Я
читаю условие задачи, смотрю на него, ещё раз читаю – до тех пор, пока не приходит в
голову решение». Размышления над задачей, поиски путей решения происходят по
некоторым законам, правилам, алгоритмам, и нужно разработать методы, позволяющие
проникнуть «внутрь» творческого процесса. Математик Дьердь Пойя посвятил этой теме ряд
серьезных исследований. Он убедительно показал, что, если пассивно ждать, решение может
вовсе никогда не прийти. Человек активно ищет его, но не осознает программу поиска.
Здесь можно усмотреть аналогию с созданием остроты. Она как будто сама рождается в
голове, однако существуют – должны существовать – правила, даже упорядоченные наборы
правил, по которым происходит синтез остроты и которые определяют её архитектонику».
5. Что мы знаем (или предполагаем)
– Я начинаю чувствовать себя мудрым! – гордо сказал
Страшила. – Только бы мне научиться пользоваться моими новыми
мозгами, и я стану знаменитым человеком!
– А почему из твоих мозгов торчат иголки? – спросил Железный
Дровосек.
– Это доказательство остроты его ума, – догадался Трусливый
Лев.
А. Волков. «Волшебник Изумрудного Города»
Важный этап работы интеллекта – установление связей и отношений между старой –
известной – и новой – полученной в процессе мышления – информацией.
Интеллект человека нельзя считать простым – «одномерным» – устройством с чётко
фиксированным набором функций. Есть точка зрения, будто практически любые проявления
активности мозга однозначно определяются специфическими индивидуальными
характеристиками интеллекта. Так что каждому типу информации соответствует тип и/или
способ её обработки мозгом.
Поскольку эта книга рассчитана на широкий круг читателей, сделаю лирическое
отступление. Прочитав нижеследующий исторический анекдот, каждый волен сделать
выводы по своему разумению. И, надеюсь, читатель не будет в обиде на автора, если
некоторые рассуждения покажутся скучными. Не заслуживают они, по вашему мнению,
внимания – смело пролистывайте «лишние страницы».
Так вот, знаменитый лорд Кельвин – проницательный, ясно понимающий суть дела
учёный – с пренебрежением относился к всякого рода искусственным классификациям, коим
так привержены посредственности. Однажды на лекции он сказал: «Говорят, есть три вида
рычагов. Я не помню, какие из них первого, какие второго, а какие третьего рода. Да это и не
важно. Во всех трёх случаях рычаг поворачивается вокруг точки опоры и этого
достаточно…»
Теперь вернёмся к классификациям. И если читателю угодно проверить себя, пусть он
попробует втиснуться в эти формальные рамки – что, впрочем, не лишено смысла.
С древних времён рассказывают о четырёх типах людей:
Тот, кто кое-что знает и знает, что знает – Мудрец: учись у него.
Тот, кто кое-что знает, но не знает, что знает – спящий: разбуди его.
Тот, кто ничего не знает, но знает, что не знает – открыт: научи его.
Тот, кто ничего не знает и не знает, что не знает – глупец: избегай его.
Строго говоря, последний пункт охватывает не только глупцов, но и многочисленных
людей, просто не заглянувших в бездну премудрости. Перевод их в состояние,
соответствующее предпоследнему пункту – задача учителей начальной школы и
преподавателей курсов для начинающих.
Хочешь что-нибудь новое сказать в науке об интеллекте? Придумай собственную
классификацию изучаемых явлений. Скажем, психолог Говард Гарднер утверждает, что
можно выделить восемь различных типов интеллекта, и каждый из этих типов имеет
собственные системы программирования (обучения), обработки информации (мышления,
решения проблем), хранения данных (память)32. Он даже полагает, что они полностью
автономны, хотя и должны время от времени работать совместно – но чаще на уровне
глубокого подсознания, чем осознанно. У всех людей какой-то из этих типов интеллекта
развит лучше, какой-то хуже. Но возможно развить и остальные типы интеллекта: оцените
свои способности, найдите слабое место своего интеллекта и укрепляйте его
целенаправленными занятиями. Вот и всё.
Общество, где большинство ничего не хочет слышать о различных нововведениях,
какой бы сферы жизни они ни касались, невозможно представить себе состоящим из
творчески развитых личностей. Именно поэтому психологи так серьёзно изучали и изучают
способы и методы развития интеллекта обычного человека. При этом приходится обсуждать
роль накопленных знаний, воспитания, этнической среды, мотивы для творчества,
психологические особенности, способные послужить толчком к росту активности.
Примем за доказанный следующий тезис: поднять средний уровень развития
интеллекта возможно, креативность – вполне демократичное качество. Представляется, что
даже в племени неандертальцев любой желающий мог попробовать свои силы в настенной
живописи Лучшие произведения сохранялись на века, а не очень удачливые живописцы
искали утешения в искусстве охоты или изготовления посуды, например. Сублимация,
однако…
Роберт Стернберг определяет творческую личность как активно устремлённую вперед,
настроенную на успех и удачу, способную выдвигать новые подходы и идеи – они вначале
не воспринимаются обществом, но со временем их значимость получает достойную оценку.
С этой точки зрения человек начинает креативно мыслить, когда им движет желание
максимального успеха. Приводя примеры из бизнеса, образования, искусства, политики,
Стернберг описывает и такое качество креативной личности, как умение, опираясь на
интуицию, предсказывать ведущие тенденции развития области науки, искусства или
общества. В понятие креативности входит и способность личности осуществлять
задуманное, несмотря на сопротивление традиционалистов.
Тот же Р. Стернберг, обсуждая роль интеллекта при определении креативных
способностей, утверждает: традиционные измерения IQ зачастую не соответствуют
результатам творческой деятельности, сопровождающейся генерацией новых, продуктивных
идей. Такие данные он со своими сотрудниками получил при анализе материалов
исследования групп студентов в рамках предложенных им программ развития креативности.
Как показано, школы обращают внимание почти исключительно на способности к
запоминанию и – в довольно ограниченной мере – анализу материала, но не развивают
оригинальное творческое мышление. В исследованиях также отмечается роль накопленных
ранее знаний, а также необходимость учёта различных типов мышления, индивидуальных
особенностей и различной мотивации при развитии креативности. Правильное сочетание и
баланс всех этих факторов необходимо обеспечить, если желательно в результате занятий
получить творчески активную, развитую личность. С главным утверждением стоит
согласиться: каждый человек может стать творчески развитым, и этому можно научиться. И
этому можно научить.
Успешным интеллектом Р. Стернберг называет способность индивида мыслить таким
образом, чтобы развить своё мышление и способности и достичь продвижения, успеха в
своём деле. Основными частями такого тройственного, триархического (по терминологии
Стернберга) интеллекта являются:
✓ аналитический интеллект – способность анализировать проблемы, находить их
решения и обсуждать их, иными словами, умение сознательно задавать направление
ментальных процессов для достижения осознанных решений проблем;
32 То есть три из шести способностей, выделенных Везеролом, Гарднер считает всего лишь инструментами
интеллектуальных механизмов, выделенных им.
✓ креативный интеллект – способность формулировать новые идеи, основанные на
них оригинальные подходы к решению задач в самых различных областях;
✓ практический интеллект – способность эффективно применять высказанные идеи в
практике жизни и творчества.
Предложив такую версию структуры интеллекта, Стернберг постулировал:
способности человека не фиксированы раз и навсегда – скорее они достаточно гибки. По
представлениям Стернберга интеллект – результат обработки информации, применяемый в
процессе освоения жизненного опыта для адаптации, подстройки и выбора внешнего
окружения. Как и любое другое качество личности, интеллект не задан жёстко, а может – и
должен! – формироваться целенаправленно. Следовательно, необходимо строить процедуру
любого обучения так, чтобы в ней использовать и развивать все типы мышления33.
Успешный интеллект – как целостная система – наиболее эффективен, когда все три
его составляющие сбалансированы. Индивиды, обладающие гармонично развитым
интеллектом, знают, что делать и каким образом нужно действовать в разных случаях, как
включить на предельную мощность свои возможности и как попытаться выйти за эти
пределы. Такие индивиды ясно мотивируют свою активность, умеют её контролировать,
осознают особенности задачи, ситуации и применяемых методов, независимы в своей
деятельности. И самое для нас важное – они подготовлены для творческого анализа и
использования явлений, предметов и событий.
Сейчас процесс обучения построен так, что преподаватели стараются обнаружить и
задействовать те типы мышления, которые у учеников наиболее развиты и максимально
успешно используются. Однако при этом не подключаются – и соответственно не
развиваются – иные типы мышления, т. е. весь интеллект в целом. Так тормозится развитие
всего комплекса способностей. Ну, и правда – в век ЕГЭ, в эпоху полной коммерциализации
образования, во времена, когда обладатель интеллекта вынужден скорее стыдиться его…
Другие способности нужны в XXI веке? Не будем спешить становиться пессимистами, коечто зависит и от нас. Умный человек способен на многое.
Примером может служить одна замечательная история, рассказанная выдающимся
математиком Джоном Литлвудом. Он вспоминает, как в незапамятные времена один
упорный, но весьма дотошный аспирант до такой степени надоел своему научному
руководителю излишними вопросами, что тот в сердцах поручил этому аспиранту
самостоятельно разработать процедуру построения циркулем и линейкой правильного
многоугольника с числом сторон 2 в 16-й степени плюс 134! «Пока не закончите, не
появляйтесь мне на глаза», – сказал рассерженный шеф. Аспирант ушёл. И вернулся через
двадцать лет. Работа была завершена, он принёс искомое построение. Говорят, эта работа и
сейчас хранится в архиве Гёттингенского университета. Как вы думаете, это история о
глупости? Об упорстве? О возможностях интеллекта? Обо всём понемногу…
При гармоничном сочетании всех типов мышления в процессе обучения можно
обнаружить, как быстро возрастает интеллектуальная мощь обучаемых, их способность к
различным видам креативной деятельности. Обратите внимание: современные зарубежные
33 Учитывая падение интеллектуального потенциала нашего общества, следует признать этот вывод особо
важным для нас.
34 Ещё «король математиков» Гаусс показал: циркулем и линейкой можно построить правильный
многоугольник с числом сторон, равным увеличенной на 1 степени двойки, показатель которой – также степень
двойки (и, конечно, любой многоугольник с числом сторон, полученным удвоениями этого исходного числа. То
есть число сторон = 2N*(2L+1), где L=2J. Заданное аспиранту число = 65537. Понятно, метод построения
включает десятки тысяч действий. Для сравнения: предыдущий многоугольник, соответствующий условию
Гаусса – с 17 сторонами; метод его построения разработал сам же Гаусс; соответствующий чертёж есть на его
надгробии.
технологии образования35 ориентированы именно на развитие его интеллектуального,
творческого компонента. Здесь корень многих проблем общества: хоть сколько-нибудь
улучшая основные характеристики мышления, имеющаяся в наличии традиционная
образовательная система, как правило, не формирует личность интеллектуально развитую в
целом, готовую к творческой деятельности. В связи с этим представляется недостаточным
знать лишь IQ конкретного человека. Нужно понять, каким образом этот человек организует
и обрабатывает информацию, относящуюся к разным областям жизни. То есть понять, каким
типом мышления обладает личность.
Тип мышления – всего лишь предпочтительный метод использования способностей.
Это не сама способность как таковая, а именно предпочтение, выбор в конкретной ситуации
метода действий и подходящей под ситуацию мыслительной процедуры. Как считает
Стернберг (и я с этим полностью согласен): у каждого имеется природная наклонность к
определённому типу мышления, однако человек не ограничен в таком выборе.
Тип мышления можно подстраивать к разным задачам или ситуациям. Например, тип
мышления, требуемый для понимания литературного произведения – не тот, который нужен
при чтении руководств или инструкций. Тип мышления, используемый при решении
алгебраической задачи, отличается от нужного для доказательства геометрической теоремы.
Даже у математиков-профессионалов есть разделение на геометров с хорошим визуальным
воображением и алгебраистов с более развитым комбинаторным мышлением.
Далее, стили мышления варьируются в зависимости от времени, от той роли, которую
человек играет в различных ситуациях. Однако в силах человека самостоятельно менять,
перестраивать, развивать своё мышление. Не так уж трудно определить, как организовано
Ваше собственное мышление – ведь почти всегда мы склонны к одному из этих типов
поведения и мышления. Но ведь их можно менять в зависимости от задачи и ситуации!
Хорошо решающий задачи – «король» на уроках математики и физики – старается
отсидеться в уголке, когда нужно подтягиваться на турнике или обточить деталь на токарном
станке.
Выдающийся математик Анри Пуанкаре, будучи в конце прошлого века студентом
Политехнической школы, уже великолепно понимал математику, но совершенно не успевал
по черчению. Ученый совет университета постановил: освободить Пуанкаре от черчения.
Многие ли наши университеты поступят так же?
Тот, кто на работе раздаёт «ценные указания» и требует беспрекословного подчинения,
уверенный в эффективности своего руководства (таким может быть и преподаватель), может
дома безоговорочно следовать сложившимся правилам поведения и вообще не «возникать»
без повода36. Каждый отдаёт предпочтение чему-то одному, но должен уметь в разных
ситуациях подстраивать своё мышление под ситуацию, находить наиболее продуктивный
способ справиться с задачей, к какой бы области науки или жизни она ни относилась.
Мы все неодинаковы по типу мышления: он зависит от многих факторов, которые
нужно знать, чтобы расширить возможности своего интеллекта. Невозможно в приказном
порядке заставить человека мыслить по иному; если нужно обучить человека, в первую
очередь сам учитель должен владеть всем арсеналом методов мышления и обучения. Тогда
этими же методами и стилями овладеют и ученики.
Очень хорошо по этому поводу высказался академик Юрий Цолакович Оганесян,
директор Лаборатории ядерных реакций Объединённого института ядерных исследований в
Дубне: «Мы всё время говорим о том, что и как у нас будет в будущем. Но чтобы
прогнозировать, как будет, надо посмотреть назад, как было. Вспомните, сразу после
35 Например, упомянутые выше методики по Уилсон и Спирсу.
36 Особо жёсткие руководители зачастую склонны к мазохистским развлечениям, чтобы хоть ненадолго
отдохнуть от бремени принятия самостоятельных решений.
революции, в холодном и голодном Петрограде люди стояли в очереди за картошкой,
чернила застывали в чернильнице, но существовали блестящие школы литературы, физики,
был расцвет науки и искусства. Откуда взялись тогда эти таланты, когда в стране было очень
тяжёлое время? Мне кажется, что расцвет творчества просто совсем не связан с
материальным комфортом, это какая-то чисто духовная субстанция. Когда мы хотим что-то
создать или воссоздать, мы должны, прежде всего, создать творческий климат. А для этого
недостаточно просто выделить деньги и сказать «сделайте мне, пожалуйста, такой климат»…
Сюда (в Дубну) постоянно приезжали не только учёные, но и художники, музыканты,
актеры, литераторы. Они в физике ровным счетом ничего не понимали, это ведь были люди
искусства. Просто здесь был творческий климат, а бацилла творчества должна иметь
определённую среду, чтобы развиваться».
Давайте хотя бы попробуем создать такую среду – вокруг себя, среди своих друзей, на
работе. Давайте предложим окружающим такой стиль жизни и мысли! Пусть
распространяется эта бацилла творчества, и да здравствует великая и славная болезнь –
желание выдумывать, фантазировать, творить!
Часть 2. Процесс и продукт работы интеллекта человека
6. Новое знание: «Откуда ты, прелестное дитя?»
– Ну, – сказала Сова, – обычная процедура в таких случаях
нижеследующая…
– Что значит Бычья Цедура? – сказал Пух. – Ты не забывай, что
у меня в голове опилки и длинные слова меня только огорчают.
А.А. Милн. «Винни-Пух и все-все-все»
Человек изучает этапы появления нового знания, закономерности его возникновения,
интерпретацию и осмысление, начиная с того момента, когда впервые задумался о
возникновении Мысли в нём самом.
Специалисты, изучая проблему мышления, то рассуждают, можно ли охватить
мышлением само мышление, то погружаются в анализ информационных процессов, то ищут
«материальный» отпечаток мысли в структурах мозга.
И верно, для мышления мозг необходим. Но недостаточен. Нужно ещё что-то для
создания сложной системы связей внутри мозга. И это «что-то» – опыт. Опыт наблюдения,
научения, осознания. Опыт описания, сопоставления, сравнения, простейшего
умозаключения. Опыт развития интеллекта, осваиваемый мозгом, начиная с самого
рождения37.
Количественные и качественные параметры усвоенного опыта зависят от того, чем и в
какой мере занимается данный мозг. Как показал – на основе опыта обучения
слепоглухонемых детей – советский философ Эвальд Ильенков, почти все поведенческие
реакции, которые мы привыкли считать врождёнными, появляются у человека в результате
взаимодействия с внешним миром и активного приспособления38 к нему.
37 До 80 % нейронов формируется уже после рождения.
38 Кстати, именно для того, чтобы приспособление было активным, инвалиды – даже столь тяжёлые –
нуждаются не только в уходе, но и в поощрении самостоятельной их деятельности. А иногда – даже в
принуждении к самодеятельности. Создатель и директор одного из лучших интернатов для инвалидов Шамиль
Тауфикович Шакшакбаев говорит жёстко: «Мир не будет приспосабливаться под тебя. И если ты не хочешь
остаться в стороне от жизни, если хочешь полноценно участвовать в жизни – стисни зубы и приспосабливайся к
этому миру». Сам он в полной мере последовал своему совету: перенеся множественные ампутации
конечностей на почве системной склеродермии, он не только продолжил занятия спортом (правда, из
Одной из первых – и наиболее удачных – попыток проникнуть в течение мысли
оказались сформулированные Аристотелем правила формальной дедуктивной логики.
Силлогистическая схема стала опорой и критерием истинного мышления, объявив, что:
1) в ходе рассуждений нельзя подменять один предмет другим;
2) нельзя признавать истинными два взаимоисключающих высказывания;
3) из двух предшествующих утверждений может быть выведено третье, новое
утверждение, при этом частное следует из общего;
4) из прямого утверждения, вообще говоря, не следует справедливость обратного, так
что его требуется доказывать.
Вот этот, последний пункт схемы рассуждений исключительно важен в математике.
Даже в школьной. Помните, для многих теорем по геометрии приходилось учить ещё и
обратную теорему, разбирая отдельно её доказательство? И доказательство обратной
теоремы иногда бывало сложнее доказательства прямой. Например, кто сразу сформулирует
(хотя бы!) теорему, обратную к теореме Пифагора? И большинство, к сожалению, не
продвигаются дальше: «Если «Пифагоровы штаны» во все стороны не равны».3
Да и пункт 2 можно проиллюстрировать множеством способов. Начиная от очевидного:
если я тут, значит, в другом месте меня нет. Вроде как «там хорошо, где нас нет»39. Кстати,
а как будет звучать обратное утверждение?4 И всегда ли оно верно?
Есть и более строгий вариант применения метода «от противного». Разберём такую
задачу: Некий врач должен был обследовать в полевых условиях трёх инфекционных
больных. Было точно известно, что все трое поражены разными заболеваниями,
передающимися при прикосновении. Да и сам врач не уверен в своём здоровье. У врача было
лишь две пары стерильных перчаток. Тем не менее, он справился с задачей, не заразился сам
и не перенёс болезнь одного пациента другому. Как ему удалось это сделать?5
Дедукцию, как метод установления истины, в «обычной» жизни эффективно и
эффектно применял сыщик всех времён и народов Шерлок Холмс. Знание психологии
человека вообще – то есть общих закономерностей поведения – позволяло Холмсу делать
умозаключения о поведении конкретного человека в конкретной ситуации. Весьма помогло
Артуру Конан Дойлу понимание того, что мотивы негодяев практически везде и всегда
неизменны: деньги, власть, страх разоблачения. Отсюда уже нетрудно экстраполировать
общие закономерности на данного типа или типчика.
Многие сотни лет после Аристотеля Роджер Бэкон заявил: для познания мира, для
получения новых знаний куда важнее метод индуктивной логики, позволяющий двигаться в
обратном направлении – из частных фактов получать общие утверждения.
Впрочем, позднее стало ясно: законы мышления не сводятся только к правилам логики.
Тем не менее не следует искать новое знание совсем уж бессистемно. Декарт писал по этому
поводу: «… уж лучше совсем не помышлять об открытии каких бы то ни было истин, чем
делать это без всякого метода, ибо несомненно то, что подобные беспорядочные занятия и
тёмные мудрствования помрачают естественный свет и ослепляют ум».
Но как же человек приходит к умозаключению? Самым простым способом объяснить
процесс мышления было бы сказать: новые выводы – всего лишь результат припоминания и
комбинирования старых сведений, заложенных в память ранее. Новая проблема порождает в
баскетболиста стал стрелком и вошёл в первую десятку стендовиков Казахстана, а теперь единственной
уцелевшей рукой ставит рекорды по толканию ядра из коляски), но и занялся бизнесом (и преуспел настолько,
что расширяет интернат за свой счёт).
39 Позднесоветский анекдот. К всесоюзному старосте Михаилу Ивановичу Калинину приходит старушка с
просьбой: разрешить ей выезд в Америку. Изумлённый председатель президиума Верховного совета СССР
спрашивает, зачем ей столь дальнее и тяжкое путешествие (в ту пору Атлантику пересекали только на
кораблях, и поездка занимала несколько дней – чаще всего с непрерывной качкой). Она объясняет: там жить
хорошо. «Эх, бабушка, там хорошо, где нас нет» – говорит Калинин. «Вот и я говорю, Михал Иваныч: хорошо
там, где Вас нет».
мозгу некоторую цепь ассоциаций, и та, в конце концов, «вытаскивает» из памяти нечто
похожее на решение. Конечно, умственное усилие стремится в поисках решения задачи
использовать все предметы и понятия, находящиеся в «пространстве – времени проблемы».
Мысль располагает предметы и события в соответствии с логикой задачи или своей
собственной. Даже простой детский лепет может оказаться для мысли источником
размышлений.
Итак, попробуем применить вышесказанное, чтобы сделать первые витки на наших
умозрительных тренировочных снарядах.
Бигуди № 1
Вот послушайте, что однажды сказала маленькая Лиза, сидя на крылечке рядом с
приятелем из детского сада: «Когда «послезавтра» станет «вчера», то «сегодня» будет так же
далеко от воскресенья, как и тот день, который был «сегодня», когда «позавчера» было
«завтра». Но в какой же день недели это было сказано? Взрослый человек легко
упорядочивает события, ему нетрудно уложить в правильном порядке пронумерованные дни
недели, месяца, года. Так что вы, скорее всего, и без труда разберётесь в этом детском
лепете.6
Однако анализировать особенности мышления лишь на субъективном уровне – изнутри
самого себя – затруднительно. В начале XX века стало ясно: для объективного описания
процесса мышления нужно изучать «человека мыслящего» со стороны. Оказалось: решение
любой проблемы представляет собой хаотические «блуждания» по лабиринту вариантов,
отбрасывание ошибочных, тупиковых ответвлений и случайный выход к цели.
Такой анализ процесса мышления тоже казался неудовлетворительным. Как выразился
М. Полани, «жизнь слишком коротка, чтобы мы могли себе позволить проверять миллионы
ложных гипотез в надежде наткнуться на одну истинную». В науке случаи, где число
вариантов исчисляется миллиардами и миллиардами миллиардов – как при полном переборе
всех возможных ходов шахматной партии – называют «переборным взрывом». Желательно
найти какой-либо разумный принцип, чтобы сократить число перебираемых вариантов.
Вы, скажем, собираетесь в поход, и, перед вами гора предметов, которую требуется
запихнуть в рюкзак. Перебираете, поставив ограничение: «не беру того, без чего можно
обойтись». Вот вам простые эвристические ограничения.
Сравните с известным рецептом Микеланджело: «Для того, чтобы создать скульптуру,
нужно взять подходящий кусок мрамора и при помощи молотка и резца отсечь от этого
куска всё лишнее». Что именно отсечь – вот это вы и выбираете.
Ещё одну попытку объяснения процесса мышления предприняли представители
гештальтпсихологии. После серии длительных экспериментов они указали: мышление
способно выделять из ситуации сразу некий целостный образ (гештальт),
трансформирующийся в процессе его обработки мышлением. Вначале мысль пытается
обозреть всю задачу, представляя её структуру как целого, собирая и располагая отдельные
характерные черты и данные, стараясь установить внутренние взаимосвязи между
элементами целостной картины. Мышление непрерывно уточняет, перестраивает общую
картину, пока не получит результат, согласующийся с внешними или внутренними
критериями истинности решения. То есть происходит поиск решения методом проб и
ошибок, в процессе которого «мы чувствуем путь к успеху и можем производить нужную
коррекцию нашего действия относительно цели, не зная даже, каким образом мы это делаем»
(М. Полани). Учёные называют этот метод решения задачи методом «научного тыка». Но не
будьте наивны: не так уж бессмысленно и наугад производятся попытки продвинуться в
решении проблемы.
время, затрачиваемое на решение аналогичных задач в аналогичных условиях,
постепенно уменьшается. И после тренировки решение (таких и только таких) задач
становится почти мгновенным.
Более поздние исследования показали: метод проб и ошибок не полностью хаотичен и
нецелесообразен, как считал Торндайк – всё же опыт и определённый уровень развития
мышления позволяют ввести ограничения для перебора вариантов «тыка». Конечно,
приятно, что при использовании этого метода можно почти ничего не знать, лишь бы Вы
смогли распознать среди многочисленных результатов верный. Причём при удачном
стечении обстоятельств можно обойтись и не слишком большим числом проб.
А вот известный рассказ Ярослава Гашека «Я варю яйца всмятку» демонстрирует
противоположную ситуацию, когда в методе проб и ошибок неверно выбрана исходная
точка. Ситуация проста: в подарок родная тетя передала племяннику 60 яиц, и он стал искать
указаний, как варить яйца. Гашек рассказывает: «Трудно поверить, что такой сложный
вопрос мог совершенно выпасть из поля зрения ученых. Я не нашёл ничего, имеющего к
нему хотя бы отдалённое отношение. Правда, в «Научном словаре» есть сведения о том, что
яйца идут в пищу и приготовляются разными способами, но как именно – это осталось для
меня загадкой даже после трёхчасовых поисков в «Научном словаре». Есть там, правда,
места, приближающиеся к этой проблеме. Например, такое: «В Англии яйца употребляются
в пищу большею частью сырыми, либо варёными вкрутую или всмятку. В каждой
порядочной английской семье к завтраку обязательно подаются яйца всмятку. Их едят по
каждому поводу». Но что с ними делают, перед тем как подать на стол, я так и не узнал. Мне
не осталось ничего другого, как произвести опыт, самому нащупать теорию варки яиц,
самостоятельно добиться нужных результатов, хотя бы ценой утраты нескольких яиц,
которые, возможно, придется выбросить. Я купил спиртовку, пять литров спирта и Папенов
котёл40, употребление которого было мне знакомо с юности, из гимназического курса
физики, и приступил к делу. Налил в Папенов котёл воды, положил туда десять яиц и зажёг
спиртовку. Через четверть часа я вынул яйца из Папенова котла. Расколупал одно –
оказалось, оно ещё твердое; другое – тоже твёрдое. Все были ещё твёрдыми! Тогда я совсем
облупил их, бросил опять в Папенов котел и варил ещё час. Они оказались по-прежнему
страшно твёрдыми. Я варил их до утра – они остались твёрдыми…».
В качестве главного недостатка метода проб и ошибок я бы отметил: процесс
мышления при этом практически отсутствует – не анализируются никакие вообще приёмы и
возможные алгоритмы решения, а иногда перебор вариантов приобретает почти хаотичный
характер. Поиска закономерностей в потоке ответов нет. Непонятно также, нужно ли вообще
над чем-то задумываться или лучше не мешать судьбе?
Говорят, есть правило: первое пришедшее в голову решение – слабое41. Честное слово,
не всегда! Говорю это со знанием дела, как человек, имеющий опыт в игре «Что? Где?
Когда?». Многое зависит от собранности, внимательности, «заряжённости» на ответ. Скорее,
наоборот – первая реакция (при соблюдении указанных условий) оказывается весьма
близкой к истине. Во всяком случае, «свет в конце тоннеля», как правило, возникает.
Теорию мышления изучали и продолжают изучать не только психологи. Выдающийся
русский физиолог И.М. Сеченов выделил в качестве двух главных и взаимоопределяющих
стадий мышления анализ и синтез. Лауреат Нобелевской премии И.П. Павлов показал, как
формируются временные нервные связи, отвечающие за ассоциативные каналы. Приходя к
умозаключению, организм (аналогично прочим своим занятиям), получая сигналы о каждом
своём действии, непрерывно корректирует свою деятельность. То есть мысль непрерывным
потоком «обтачивает» задачу-камешек, разворачивая её, изучая и анализируя. Мозг
выдвигает всё новые предположения, получая дополнительную информацию и проверяя
40 Сейчас этот прибор широко известен как скороварка.
41 Легендарно беспринципный остроумец и великий дипломат Талейран учил: бойтесь первого побуждения
– оно самое благородное.
гипотезы, пока не будет выстроено решение, удовлетворяющее неким внутренним
критериям правильности.
По словам уже не раз упомянутой мной Т. Черниговской, принятие решения у мозга
занимает секунд 7–20, и принимается оно до того, как вы об этом узнаете и будете оставаться
в уверенности, что это ваш самостоятельный выбор. Речь, конечно, идет о простых
ситуациях, а не о жизненно важных решениях. Но это не избавляет от возникновения
правомерного вопроса: «Так кто же кому принадлежит – я мозгу или мозг – Мне?».42
Знание – «активное постижение познаваемых вещей, действие, требующее особого
искусства. Акт познания осуществляется посредством упорядочения ряда предметов,
которые используются как инструменты или ориентиры, и оформления их в искусный
результат, теоретический или практический… В каждом акте познания присутствует
страстный вклад познающей личности и эта добавка – не свидетельство несовершенства, но
насущно необходимый элемент знания» (М. Полани).
Майкл Полани – учёный-химик, получивший дополнительно и медицинское
образование – уже упоминался в предыдущей части книги. Он создал выдающийся
философский труд «Личностное знание», где проанализировал как различные аспекты
процесса получения знания, так и основные характеристики самого знания. Один из
важнейших тезисов его работы: «искусство познания и искусство действования, оценка и
понимание значений выступают… как различные аспекты акта продолжения нашей
личности в периферическом осознании предметов, составляющих целое. Структура этого
фундаментального акта личностного познания диктует необходимость как участвовать в его
осуществлении, так и признавать универсальное значение его результатов. Этот акт является
прототипом любого акта интеллектуальной самоотдачи».
Речь идёт всего лишь о том, что конкретные знания могут быть получены только в
процессе мыслительной деятельности. Познающий человек лично участвует в актах
понимания. Бесстрастная, безличная истина познаётся и формулируется только в процессе
мышления, носящем глубоко личностный характер, так что любая новая информация несёт
на себе отпечаток её создателя. Человек (субъект) своими умственными усилиями постигает
независящее от него объективное знание – оно, как говорит Полани, позволяет установить
«контакт со скрытой реальностью». Лично от человека зависит форма, в которой он сумеет
изложить это знание, глубина постижения, скорость мышления также определяется
индивидуальностью. Конечно, мышление – совершенно индивидуальный процесс. Как и
любое творчество. Попробуйте начать думать и творить, ведь «кто испытал наслаждение
творчества – для того всех других наслаждений не существует!», как искренне полагал
Антон Павлович Чехов. А уж он-то знал толк и в других наслаждениях…
В процесс мышления вовлечён человек как целое. При этом сама возможность
осуществления мыслительных действий определяется именно наличием у него центральной
нервной системы, мозга с особой структурой, характерной для данного биологического
вида43. Но тип умозаключений, их содержание очень сильно зависят от способа
формирования информационной структуры мозга. Проще говоря, определяются влиянием
среды, культурой, образованием, общим строем мысли (как принято говорить – и в широком,
и в узком смысле – Менталитетом). Личность должна быть готова принять новое знание,
включить его в свою систему ценностей. «Потому что во многой мудрости много печали;
и кто умножает познания, умножает скорбь» (Екклезиаст44).
42 Наука и жизнь. – 2012. – № 11. – С. 26–30.
43 Иногда об этом говорят как о принадлежности к виду Homo Loquens (Человек Говорящий).
44 Довольно долго шли споры: действительно ли под псевдонимом «проповедник» – Мы его знаем и в
древнееврейском звучании «когелет», и в древнегреческом «екклезиаст» – писал легендарный мудрец,
иудейский царь Соломон. В последние десятилетия большинство исследователей признало: в текстах
Мышление – в простейшем понимании – предназначено для решения тактических и
стратегических задач существования. Как решать – уже другой вопрос. Уровень
информационной готовности для выработки алгоритмов решения можно назвать
компетентностью. Иными словами, компетентность есть сумма знаний, определяющих
пределы успешности выполнения задачи (и для животных, и для человека). Естественно,
если компетентность – в том числе и генетическая – равна нулю, то никакие побуждения не
могут вызвать выполнение данной задачи. Как-то прочитал почти по этому поводу:
«Информация поступила в голову и безнадёжно ищет мозги»… Врачи знают: существуют
больные, ввиду специфического повреждения мозга не способные различать пальцы на
своих руках. Следовательно, они не могут научиться считать. Так что получать информацию,
например, видеть – очень хорошо, но перерабатывать входные данные, понимать, что вижу –
уже иной уровень мышления.
Ещё один важнейший вопрос: какую роль в процессе мышления играет человеческая
речь. В качестве опорного можно принять утверждение: потенциальная возможность
говорить зависит от генетических факторов, а реальная речевая продукция – от опыта. Что
первично в этом процессе взаимообогащения и взаимосоздания – трудно сказать.
Определённая логика есть как в утверждении Н. Хомского, что мозг создал язык, так и в
тезисе «язык создал человеческий мозг» (Д. Дикон). Наверняка можно утверждать лишь:
процесс мышления сложно зависит от внутренней речи – научно говоря, от соотношения
«вербализованных и невербализованных слоёв мысли» – и речи, как таковой вообще (Л.С.
Выготский и Ж. Пиаже).
«Последние исследования показали: неандертальцы нам родственники. Хотя в этом
долгое время сомневались. Как-то страшновато было представить, что сотни тысяч лет назад
на планете жили одновременно разные хомо. Представьте: есть кошка и некая НЕДОкошка.
Она не лиса, не волк, она из тех же, но не вполне. Соседствовали разные виды хомо, и мы
оказались самыми сильными и победили. Но почему НЕДОчеловеки (или «другие
человеки») погибли? Ясно, что мы их переиграли, в том числе и с помощью языка. Гомо
сапиенс научился разговаривать звуками, а неандертальцы, насколько нам известно, освоили
только язык жестов.
Самое виртуозное, что у нас есть это язык. Это сложнейшая система и устроена она так
ужасно, что все лингвисты мира не могут с этим справиться, несмотря на все усилия». (Т.В.
Черниговская).
При этом невербализованный уровень мышления относительно самостоятелен, хотя и
испытывает постоянное и весьма существенное влияние и со стороны процесса
вербализации, и со стороны уже хорошо оформленных словесно – как бы отшлифованных –
Мыслительных структур. Но непременным компонентом мышления является «внесловесная
мысль». Она объективно существует, записана в мозге в «кодах определённого типа,
отличных от кодов внутренней речи» (Д.И. Дубровский). Иначе говоря, новое знание,
оригинальная мысль зарождаются не в сфере внутренней речи (хотя и не без её содействия),
не тогда, когда человек проговаривает, проборматывает свои рассуждения. Бессловесное, на
уровне «мычания» зарождение нового хорошо известно поэтам и художникам, музыкантам и
математикам.
М. Полани ввёл понятие артикулированных (получивших словесное, речевое
оформление) и неартикулированных способностей человеческого мышления. Он полагал:
неартикулированные – потенциальные – способности, благодаря которым человек
превосходит животных и которые, создавая речь, объясняют его интеллектуальное
превосходство, сами по себе почти незаметны. Артикуляция же – преобразование
интуитивно ясной, но невыражаемой в точных речевых образах, информации – всегда
прославленнейшего в истории пессимиста есть многие тонкости, ощутимые только с позиции высшей власти,
когда между человеком и его представлением о боге нет никакого посредника и начальника. Тоже серьёзное
открытие!
остаётся неполной: как говорит Полани, наши словесные высказывания никогда не могут
заменить немых интеллектуальных актов. Вместе с тем в интеллектуальном отношении мы
очень многим обязаны артикуляции, хотя она главным образом занимается понятиями, а
язык играет при ней лишь вспомогательную роль, Полани пишет: «понятия, предлагаемые
речью, позволяют нам осознать как то, каким образом наша речь обозначает определённые
вещи, так и то, как эти вещи устроены сами по себе».
Интересна интерпретация той же Татьяной Черниговской процесса освоения языка
ребенком. Он приходит в мир, не зная про него ничего. Он должен все про него узнать. В
этом ему помогает его мозг, благодаря генетической программе, которая знает, как вынимать
из мира информацию. Теперь представьте, что ребенок должен овладеть языком тогда, когда
его никто этому не учит. Вы можете возразить, что родители и все вокруг разговаривают. Но
ребенок – это все что угодно, только не магнитофон.
Ребенок должен вычислить: что есть язык, а что не есть язык. Он слышит много разных
звуков. Соседи сковородками кидаются, кошки мяукают, собаки лают, машины скрипят
тормозами. Что из этого язык? Ему никто не говорит. Более того, ему никто не сообщает
никаких правил ни про падежи, ни про окончания. Он сам пишет так называемую карту
языка. Ребенок выполняет задачу, которую не могут выполнить лучшие лингвисты Земли.
Как он это делает – Мы хотим знать. Стараемся из всех сил.
«Кроме того, ребенок окружен языком, состоящим из сплошных ошибок. Если сейчас
записать мою устную речь – там будет масса сбоев. Не потому, что я неграмотная, просто
язык «не то сказал». Ребенок же умудряется из хаотической информации вынуть правило.
Только мощнейший компьютер, которым является мозг, позволяет ребенку это сделать».
Так малыш, ещё не знающий слова и понятия «собака», описывает их (в рассказе
Чехова «Гриша»): «большие кошки с задранными вверх хвостами и высунутыми языками».
Полани формулирует своеобразный «закон бедности языка»: язык должен быть
настолько беден, чтобы можно было достаточное число раз употреблять одни и те же слова.
Только в этом случае удаётся уже известными и ограниченными символами языка
записывать новые, принципиально неограничимые категории и понятия. Иначе для каждого
нового знания было бы не только возможно, но и необходимо ввести новые обозначения.
Рост их числа катастрофически ограничил бы возможности анализа и комбинирования
«старых» знаний с целью получения новых45.
Мераб Мамардашвили говорил: «раз существуют слова, то из них можно создать
миллион умных вопросов…Всегда есть все слова, посредством произвольной комбинации
которых можно получить симулякр – ответ (мысленную конструкцию, не проверенную и не
подтверждённую ничем, модель, вариант ответа), тень ответа на любой ваш вопрос… Или,
по-другому, – всегда есть вербальный мир, который сам порождает псевдовопросы,
псевдопроблемы, псевдомысли, и отличить их от истинной мысли невозможно».
– Если бы основной функцией языка была коммуникация, то он становился бы все
более и более однозначным, – считает Татьяна Черниговская, – Меж тем все языки мира
демонстрируют ровно обратную картину. Все зависит от контекста. Для того, чтобы понять
другого человека, нужно знать, кто сказал, когда сказал, что было до этого, что будет после,
что остальные про это думают. То есть основная задача человеческого языка – НЕ
коммуникация, а мышление.
«Карл Пятый, римский император, говаривал, что гишпанским языком с Богом,
французским – с друзьями, немецким – с неприятелем, италианским – с женским полом
говорить прилично. Но если бы он российскому языку был искусен, то, конечно, к тому
45 Древние системы счисления – еврейская, римская, греческая и выросшая из неё старославянская –
использовали для единиц, десятков, сотен и т. п. разные обозначения. Это делало даже простейшие
арифметические действия сложными задачами, разрешимыми только после длительного обучения: так,
умножать учили в университетах. С переходом к позиционным системам, где в каждом разряде используются
одни и те же цифры, арифметика стала детской игрой.
присовокупил бы, что им со всеми оными говорить пристойно, ибо нашел бы в нём
великолепие гишпанского, живость французского, крепость немецкого, нежность
италианского, сверх того богатство и сильную в изображениях кратость греческого и
латинского языков», – утверждал патриот Ломоносов, мастер поэтических од и придворной
игры. Но то, что русский язык стал изворотливым – не случайно. Слишком стеснённой была
жизнь народа в условиях абсолютной монархии, чтобы не извернуться по-эзоповски для
собственного выживания. Голь на выдумки хитра!
Язык как орудие общения всегда является средством отображения мира вокруг нас, он
развивается по мере того, как мы познаём мир, Он эволюционирует, подобно обычным
инструментам, оставаясь орудием универсальным.
Из слов – символов языка – Можно составлять новые конструкции, слова. Такие
интеллектуальные игры (скажем, шарады) полезны и забавны.
Бигуди № 2
Шарада – загадка, обычно составленная в стихах. Задуманное слово распадается в ней
на отдельные части: каждая представляет собой самостоятельное слово (как правило,
односложное). Разгадав каждое слово, можно сложить их вместе, узнав исходно задуманное
слово. Первые шарады вошли в моду во Франции ещё в XVIII веке, а в средневековье
шарадами называли повозки на двух колёсах. «Целый воз болтовни» – неплохой перевод
слова шарада. Итак, пример шарады, которую предлагается разгадать:
Я замерзаю в холоде полярном,
Хоть и лежу с запасами тепла.
Мой первый слог – название собаки
Иль это столб, как острая игла;
Второй мой слог – одна из форм рельефа,
Но, правда, на немецком языке,
А целое – найдёте Вы на карте в далёком уголке.7
Можно потренироваться и в разгадке метаграмм. Угадайте слово (или хотя бы набор
букв без начального смысла) и заменяйте в нём буквы, чтобы получались новые слова с
указанным смыслом:
С «Г» – полезное растение,
С «Ш» – пугает нас порой,
С «П» – несёт нам разорение,
С «В» – навалено горой.8
Может быть, именно возможность переписывания сведений о прошлом опыте с
использованием символической записи информации обеспечивает и объясняет
интеллектуальное превосходство человека. Причём человек может формально или мысленно
реорганизовать эти символы в целях извлечения новой информации, новых знаний. Как
пишет Полани, усиление наших интеллектуальных способностей с помощью удачно
выбранной символики убедительно показывает: простое манипулирование символами само
по себе никакой новой информации не даёт46. Оно эффективно лишь постольку, поскольку
содействует реализации неартикулированных мыслительных способностей, считывая
результаты их применения. Так что процесс артикуляции даёт колоссальную по
46 Многие исследователи придерживаются несколько иного мнения. Они полагают: удачная символика если
и не создаёт новую информацию, то хотя бы открывает интеллекту эффективный доступ к ней. То есть для нас
эта информация нова.
эффективности помощь нашим врождённым мнемоническим способностям. В качестве
примера можно привести, скажем, записную книжку изобретателя – она представляет собой,
по сути, его «карманную» лабораторию.
Изобретательность – свойство мышления, проявляющееся во всех сферах жизни.
Быстрый умом Пётр I сразу понял: первую петербургскую газету никто читать не будет – не
Европа, не приучены. Заставлять читать силой? Так всех не заставишь, а хотелось бы, чтоб
читали побольше людей. И Пётр сообразил: куда народ ходит непременно и часто? В
трактир! Следует распоряжение Петра: в трактирах кормить бесплатно тех, кто читает газету.
Разумно? Да. Действенно? Может быть. Надолго ли хватило? А вот это уже другой вопрос…
Можно утверждать: большей частью своего знания образованный ум обязан
вербальным источникам – но сложно структурированная информация, чью запись в обычном
языке представить совершенно немыслимо, может быть сформулирована отчётливо лишь с
использованием логических символов.
Сделаем лирическое отступление и поговорим о символах. В связи с этим придётся
обратиться к математике – именно в этой науке символьная запись информации имеет
важнейшее значение.
Операции над символами, вообще говоря, не могут быть полностью описаны в
терминах некоей «инструкции» – скорее они могут представлять собой:
✓ поиск «впотьмах», наощупь – результаты такого поиска должны быть впоследствии
скорректированы в свете достигнутого «молчаливого понимания», как говорит Полани;
✓ предвосхищающую интуитивную догадку – она также должна смениться
молчаливым осмыслением полученного знания, которое в дальнейшем сможет получить
словесное, артикулированное выражение.
Как именно сказано, как зашифровано в символическом виде – это уже вопрос
компетенции мозга и его способности работать на различных уровнях абстракции. Полани
формулирует принцип непрестанной деятельности мозга так: «Наш разум живёт в
постоянном действии; всякая попытка сформулировать основания этого действия приводит к
появлению набора аксиом, которые сами по себе ничего не сообщают нам о том, почему мы
их приняли».
Мозг неустанно обтачивает то одну, то другую мысль, плетёт сети рассуждений и
создаёт логические цепочки, оформляя результаты своего труда в виде всё новых «побегов»
на общем информационном «древе». Далеко не все озарения и строгие выводы имеют право
быть включёнными в общую систему знаний под вывеской «Новое, оригинальное, ценное».
Мозг, впрочем, не выбрасывает даже «стружки» и «обрезки»: покопавшись по случаю,
можно и там невзначай обнаружить подходящее «жемчужное зерно».
Способность активно поставлять в мировую систему знаний нетривиальный
интеллектуальный продукт под маркой «Это придумал я», количество и качество этого
продукта определяются уровнем Вашего мышления, развитием творческого потенциала,
умением использовать закономерности эвристики – науки о творчестве.
7. Творчество и творческие способности
– Сова, – сказал Пух, – Я что-то придумал.
– Сообразительный и изобретательный медведь! – сказала Сова.
Пух приосанился, услышав, что его называют Поразительным и
Забредательным Медведем, и скромно сказал, что да, эта мысль
случайно забрела к нему в голову.
А.А. Милн. «Винни-Пух и все-все-все»
Науку, устанавливающую и изучающую законы творчества, ещё в начале XX века
развивал П. Энгельмейер. Он назвал её эврилогией47 и выделил несколько этапов
творческого акта. Вначале – на психологической стадии – формируется замысел, возникает
идея, предчувствие мысли, интуитивное ощущение. Логический этап, осуществляемый в
рамках рациональных мыслительных процедур, приводит к непосредственному получению
знания. На третьем этапе – конструктивном – сформулированная чётко и доказанная мысль
получает конкретное воплощение, реализуется в материальной форме.
Творчество имеет двойственную природу: генерация новой идеи содержит
необходимый компонент интуиции, подсознательного мышления, а её обработка и
«прививка» к подходящей ветви единого системного «дерева знаний» невозможна без
участия педантичного и чёткого логического аппарата. Но если логика обеспечивает
непрерывное и планомерное «сканирование» информации и её осмысление, то интуиция
нарушает непрерывный ход мысли. Акт интуиции странным образом меняет ход
биологического времени48: происходит своеобразное «сжатие» алгоритмических процедур
во времени, «перескакивание» на иные уровни абстракции с введением обобщающих
понятий и погружение нескольких вариантов алгоритмов в подсознание. Два типа мышления
– интуитивное и логическое (дискурсивное) – не взаимоисключают, а взаимопроникают друг
в друга.
Лишь в процессе интеллектуальной работы:
✓ выделяется и перестраивается (по мере необходимости) информационное
хранилище;
✓ подчиняясь мгновенным командам интуиции, посылаются и затухают
информационные запросы-импульсы;
✓ образуются временные локальные нейронные сети (новые связи в мозгу),
усиливающие или ослабляющие сигналы мозга при проверке правильности решения
проблемы.
Высказана и такая гипотеза: чем сильнее влияние нового знания на ранее накопленное,
чем интенсивнее их обратная связь, тем выше творческий потенциал мышления. Новое
знание не существует отдельно от уже известного – помните слова о «прививке» новых
побегов? Известный философ М.А. Розов определил новое знание как ту часть неизвестного,
которая сводится к известному. Сводится, т. е. осваивается, осмысливается, «приживляется»
к уже существующему в мышлении системному «дереву» понятий и сведений.
Например, Вы что-то этакое помните из школьной геометрии. Может ли это Вам
пригодиться, скажем, при поездке в Египет? Ещё как! Особенно если во время экскурсии к
великим пирамидам рядом с Вами окажется милая незнакомка, не так хорошо
подготовленная, как Вы. Вот тогда на её восхищение и изумление при виде великой
пирамиды Хеопса вы сможете непринужденно заметить: ещё Фалес Милетский (живший в
VI веке до н. э.) знал, как можно легко определить высоту пирамиды по её тени. Достаточно
лишь взять палку, поставить её вертикально и дождаться, когда её тень на песке станет9…
Тут возможны варианты. Либо Вы продолжите мысль Фалеса и разъясните прекрасной, но
наивной незнакомке, в чём же состоит его идея. Либо предложите ей самой догадаться, как
можно узнать высоту пирамиды без вертолёта, альпинистского снаряжения или лазерного
угломера. Выбирайте!
Вопрос же о возникновении замысла – локальная часть более общего вопроса: о
глобальной цели познания. М. Полани сформулировал тезис: «Искусство познания и
искусство действования, оценка и понимание значений выступают как различные аспекты
акта продолжения нашей личности в периферическом осознании предметов, составляющих
целое. Структура этого фундаментального акта личностного познания диктует
47 От греческого «Эврика!» (нашёл!): помните, что воскликнул Архимед, выскочив голышом из ванны?
48 То есть необыкновенно быстро устанавливает новую систему взаимосвязей между узлами-понятиями.
необходимость как участвовать в его осуществлении, так и признавать универсальное
значение его результатов».
Эта мысль близка точке зрения академика А.Д. Сахарова на жизнедеятельность (в том
числе, безусловно, и на мышление) как на непрерывную экспансию, расширение самого себя
– не в физическом, а в психологическом, интеллектуальном плане – за пределы своей
личности, желание не сузить мир до своих размеров, а расширить себя до размеров мира.
Увы, впрочем, сейчас находится достаточно представителей Homo Novus, понимающих
тезис об экспансии как смысле жизни куда прагматичнее и примитивнее.
Сейчас для обозначения всей науки, анализирующей процесс творчества, и раздела
психологии, посвящённого изучению закономерностей получения новых знаний, принят
термин «эвристика». Часто этим же термином обозначают специальные методы решения
нестандартных задач, способов организации творческой деятельности (включая и
коллективную – тот же метод мозгового штурма), приёмов обучения творческому
мышлению. Можно сказать: «эвристика изучает, как делаются открытия, как
устанавливаются новые истины, как решаются нешаблонные задачи, т. е. как протекает
продуктивная умственная деятельность, приводящая к оригинальным результатам» (И.М.
Розет).
Общие принципы, методы и законы эвристики имеют динамическую природу,
поскольку опираются на нелинейные процессы взаимодействия информации с информацией.
В результате такого «сложения» количеств рождается новое качество – информация, которой
ранее не было.
Эвристика не количественная наука и ничего не может сказать ни о самих
информационных взаимодействиях, ни о конкретных их результатах – результатах
творчества. Но она помогает найти тот кратчайший путь к творческим достижениям,
движение по которому наиболее продуктивно. То есть качество – степень новизны,
оригинальность и полезность для дальнейшего применения – информации, получаемой на
этом пути, наивысшее. Значит, эвристика помогает сформулировать и активно использовать
некий «принцип наименьшего действия», обеспечивающий движение мысли в «пространстве
проблемы» по кратчайшему пути и с наибольшей выгодой – получением максимального
количества нового знания.
Как обычно происходит поиск решения? Чаще всего путём проб и ошибок, благодаря
которым мы «чувствуем путь к успеху (интуиция? озарение? акт подсознания?) и можем
производить нужную коррекцию нашего действия относительно цели, не зная даже, каким
образом мы это делаем. Так постигается принцип плавания или езды на велосипеде» (М.
Полани). В процессе отбора нужного материала для работы мысль тоже способна выбрать
пути и понятия, в данной ситуации полезные и эффективные, даже если сами эти действия
мозга – а тем более их причины – неведомы ему.
Как отмечено выше, слишком большой информационный запас несколько утяжеляет и
замедляет движение мысли. Если на плечах лежит коромысло, то быстро развернуться с ним
непросто – примерно так и огромный запас знаний эрудита иногда не позволяет быстро
«развернуть» своё мышление. До какого-то момента, до некоторой «критической массы»
эрудиция действует как хорошая смазка, но в слишком большом количестве информации
мысль может просто завязнуть.
Бигуди № 3
Подумайте, как помочь двум приятелям из рассказа знаменитого фантаста Роберта
Шекли. Два друга – основатели «Антиэнтропийного Агентства», зарабатывающие на
исправлении дефектов экологии различных планет – купили у известного старьёвщика,
торгующего всевозможным межпланетным хламом, удивительную машину – синтезатор,
способный сделать всё! Имея в руках такой аппарат, не приходится задумываться даже о
пропитании: кажется, машина сделает на заказ любое блюдо, продукт. Правда, только в
одном экземпляре: синтезатор так сложен, что почти разумен, и повторяться ему скучно. В
полете всё шло хорошо, в том числе и благодаря разнообразию меню. Но при посадке на
очередную планету космический корабль получил серьёзное повреждение. Выяснилось, что
для ремонта нужно сделать десяток одинаковых деталей. Как поступить?10
Известно определение: талант – это совокупность психофизиологических и
интеллектуальных качеств, необходимых для достижения таких решений, когда заранее
неизвестен набор правил и операций, последовательность которых приводит к цели.
Определение вполне научное и, вообще говоря, понятное. Вот только не ясно, сколько и
какой работы должен совершить мозг, прежде чем упомянутые решения будут достигнуты.
Тем более, что путь к ним совершенно ничем не освещён. В этих условиях мысли
приходится идти наощупь, непрестанно выдвигая, проверяя и отбирая наиболее
эффективные и адекватные искомой цели идеи, подходы и способы. Человек тем
талантливее, чем короче путь к цели, чем более интересной и многообещающей выглядит
найденная им дорога. О таланте говорит также число неожиданных связей с известными
понятиями и способов использования различных «правил и операций».
Талант – скорее духовный и интеллектуальный потенциал. Непосредственное же
приложение этого потенциала – процесс его использования, в результате которого создаётся
новое знание, в самом широком смысле этого слова – Можно называть творческим
мышлением.
Существует много различных определений того, что мы называем творческим
мышлением, творческой одарённостью, способностью к творчеству. Ныне всё это чаще
обозначают собирательным понятием «креативность»: от латинского глагола «creare» –
творить, создавать – и существительного «creatio» – творение. Смысл понятия
«креативность» проще всего определить так: введение в человеческую практику чего-то, что
ранее в ней не присутствовало, будь то предмет, способ действий или мысль. Человек
креативен, если пишет действительно новую музыку, а не повторяет в тысячный раз
музыкальную комбинацию из трёх аккордов.
По словам Татьяны Черниговской, основой креативности и «вообще основой всего,
благодаря чему наша цивилизация создалась – является как раз непредсказуемость, на – этом
стоит вся наука и вообще вся культура. Предсказуемые ходы никому не нужны Размытость,
неточность, приблизительность описания – самые хорошие примеры здесь – кулинарные
рецепты или то, как мы объясняем друзьям как пройти к нам, к нашему дому».
Человек креативен, если доказывает новую теорему, изобретает новый кулинарный
рецепт или ракетный двигатель, проходит по давно изъезженным местам ранее неизвестным
путём и при этом обнаруживает то, что никогда и никем не замечалось. Даже если Вы
самостоятельно изобретёте что-то, что когда-то уже было известно, но о чём Вы не знали,
вспомните, что «всё новое – это хорошо забытое старое49» и утешьтесь: всё равно Вы
убедились, что способны извлекать новое и упорядоченное из хаоса, способны изменить
свои представления о вещах, явлениях или способах объяснения этих явлений. Даже если Вы
всего лишь объясните незнакомой девушке, почему очень лёгкое тело трудно бросить на
большое расстояние. Предварительно непременно процитируйте Михаила Жванецкого: «Вы
пробовали когда-нибудь зашвырнуть комара далеко-далеко?». Проявить эрудицию – не
только полезно, но и приятно. Только не забудьте всё же объяснить всерьёз (можно и
попозже…), почему трудно далеко забросить, например, комара!11
49 Фраза принадлежит модистке французской королевы Марии-Антуанетты. Мастерица не поспевала за
балами, где легкомысленная супруга Луи XVI стремилась щеголять каждый раз в новом наряде. Может быть,
если бы высокопоставленная клиентка чаще следовала – и свою бесчисленную свиту заставляла следовать –
этому разумному совету, страна не дошла бы до разорения, а вследствие него до Великой Французской
революции, стоившей венценосным супругам голов, а всей Европе – Многих миллионов жизней, потерянных в
четвертьвековой войне.
В мифах народов мира – стран от Запада до Востока и обратно – Мы обнаружим
множество персонажей, объединённых архетипом «трикстер». Эти персонажи –
катализаторы действия и мысли, креативные личности, живущие парадоксом, то есть
радостью мысли. Среди них и суфий Насреддин, и Иван-дурак, и злой шутник Гермес, и
проказник Локи. Умелая шутка в исполнении трикстера, как мне представляется, есть способ
подачи мысли в парадоксальной форме, выворачивание наизнанку самого образа мысли.
Трикстер вводит даже самого неподатливого ученика в состояние восприятия им учителя.
Правда, дорогой ценой – умаляя у пока ещё малограмотного или зашоренного человека (в
том числе порою у себя самого) чувство значимости и гордости собой. Вот пример – от
хорошо знакомого всем нам ходжи Насреддина:
Некий дервиш-мелами по имени Шейяд-Хамза – человек просвещённый, совершенный,
идущий по верному пути, живущий праведно – сказал ходже: «Ходжа, неужто-таки твоё
занятие на этом свете одно шутовство? Если ты на что-нибудь способен, так покажи своё
искусство, и если есть в тебе какая учёность, прояви её нам на пользу». Ходжа спросил у
него: «А у тебя какое есть совершенство и какая в тебе добродетель, и людям какая от неё
польза?» – «У меня много талантов, – отвечал Шейяд, – и нет счёту моим совершенствам.
Каждую ночь покидаю я этот бренный мир («мир элементов») и взлетаю до пределов
первого неба; витаю я в небесных обителях и созерцаю чудеса царства небесного». –
«Хамза, – заметил ходжа, – а что, в это время не обвевает ли твоё лицо нечто вроде
опахала?» Хамза, радостный, подумал: «Ну, напустил я на него туману», – и сказал: «Да,
ходжа». – «А ведь это – хвост моего длинноухого осла», – сказал ходжа.
Не удивительно, что маску трикстера время от времени примеряет на себя и по сей день
иной учёный, если все иные способы донести своё мнение до общества пробуксовывают.
Есть немало исторических анекдотов, достойных трикстера. Будучи умело вставлены в
речь, они, как отмечает Д.А. Гаврилов в двух своих монографиях на тему лицедея, шута и
перевёртыша, то есть трикстера, в фольклоре народов мира, «поворачивают сказанное к
слушателю совсем с неожиданной стороны, подцепляя его внимание на крючок любопытства
и умаляя его самооценку, чтобы подготовить к Новому».
Некая почтенная дама обратилась с каверзным вопросом к польскому писателю и
блистательному афористу Станиславу Ежи Лецу: «Это очень трудно – выдумывать всё из
головы?» – «Что Вы, – ответил он незамедлительно. – Из ноги было бы гораздо труднее
К писателю В.В. Вересаеву – Медику по образованию – обратился его коллега по перу
за медицинским советом. Вересаев, осмотрев пациента, молвил: «Знаете, я ведь царь-врач».
«Что это значит?» – удивился больной. «Это весьма просто, – ответил Вересаев. – Царьпушка не стреляет, царь-колокол не звонит, а царь-врач не лечит».
«Есть вещи настолько серьёзные, что о них можно говорить лишь шутя», – любил
повторять Нильс Бор. Он со своими молодыми (в основном студенческого возраста)
учениками за считанные годы создал квантовую механику, противоречившую всем
привычным тогда физическим канонам. Вряд ли это удалось бы без созданной им в
коллективе атмосферы иронического отношения к вечным истинам.
Известный германский физик, открыватель 3-го начала термодинамики, Вальтер
Нернст в свободное время разводил карпов. Кто-то из его знакомых заметил: «Странный
выбор. Кур разводить и то интересней». На это Нернст убеждённо возразил: «Я развожу
таких животных, которые находятся в термодинамическом равновесии с окружающей
средой. Разводить теплокровных – значит, на свои деньги обогревать мировое
пространство».
Об известном физике Филиппе Франке рассказывают такую историю. Однажды на
первой лекции по истории науки он обратился к слушателям со словами: «Первый вопрос,
который мы рассмотрим, касается пространства и времени. В какой аудитории мы будем
заниматься и по каким дням?» Согласитесь, после такого предисловия всё, что будет сказано
о пространстве и времени в самом лекционном курсе, запомнится надолго.
Следующий анекдот в Шотландии приписывают лорду Келвину, а в Москве –
профессору Умову. На экзамене Умов спрашивает студента: «Что такое электричество?»
Студент после долгого раздумья говорит: «Простите, профессор. Утром помнил, а сейчас
забыл». «Вот, господа, – обращается Умов к другим студентам, – величайшая трагедия
физики нашего века: один-единственный человек на свете знал, что такое электричество, да
и тот забыл!»
Мышление действует по собственным эстетическим «законам прекрасного»: по
выражению М. Мамардашвили «мысль безусловно связана с радостью, иногда с
единственной радостью человека». Собственная мысль – достижение, прозрение,
вычисление Вашего собственного мозга, та конструкция из догадок, сомнений и убеждений,
которая отражает специфику именно Вашего типа мышления – даже если речь идёт о
повторном открытии чего-то известного. По словам того же философа, если что-то
помыслено Вами – оно Ваше, даже если это совпадает с мыслью другого человека, даже если
это совпадет с мыслью великого мыслителя. Собственно, это и имел в виду Сенека, сказав:
«Всё что сказано хорошо – Моё, кем бы оно ни было сказано». Естественно, под этими
словами, следуя приведённой логике, следует тоже приписать «Моё».
Бигуди № 4
Насчёт эстетических законов мышления – вот пример, когда эстетические «законы
прекрасного» весьма далеки от этических норм. Я имею в виду, когда хитроумное
человеческое мышление находит лазейки в неписаных правилах жизни общества или
писаном своде законов. Цель мышления в этом случае – чаще всего захват власти,
обогащение, т. е. по сути устранение препятствий. Пример из истории: римский император
Лициний (не очень приятная личность, если посмотреть поближе) своим указом установил
точную величину месячного налога и потребовал от граждан ежемесячных выплат. Личные
планы императора были, однако, столь обширны, что собранных с народа денег не хватало
на их выполнение. Повысить величину ежемесячного налога? Неприлично собственный указ
нарушать, народ обидится. Что же делать? А почему бы не изменить частоту – насчёт этого
ещё указов не было! И Лициний…. Какой же указ издаёт император Рима?12
Изменяет ли человек, порождая новое знание, только свой собственный мир? Видимо,
творческие способности означают создание чего-то нового скорее для самого человека, чем
для его окружения. Действительно, новые элементы вносятся в окружающий мир не только
тогда, когда мы это делаем целенаправленно, но даже тогда, когда мы делаем открытия,
казалось бы, лишь для себя. На самом деле меняется мировая информационная среда, и в мир
постепенно встраивается новая информация.
Под творчеством часто понимают и создание чего-либо нового, принимаемого в
определённой ситуации и в определённое время за нужное и полезное.
Приведём несколько определений понятия «креативность», данных весьма известными
людьми. Итак, креативность – это:
✓ Появление нового, производного по отношению к уже известным, продукта,
возникающего как следствие уникальности некого индивидуума (Карл Роджерс).
✓ Возникновение такого сочетания (предметов или идей), которое является и новым, и
ценным (Генри Миллер).
✓ Способность создавать новые комбинации социально значимых ценностей (Джон
Хафеле).
✓ Создание нового смысла путем синтеза (Майрон Аллен).
✓ Беглость, гибкость, оригинальность, а иногда – совершенствование (Торранс).
Любой мыслительный процесс, в результате которого формируются и могут быть выражены
определенным образом оригинальные образы (Гарольд Фокс).
✓ Способность видеть (или осознавать) и реагировать (Эрих Фромм).
✓ Результат подсознательного конфликта побуждений и потребностей,
сублимированный благодаря усилиям эго в результат, полезный как для творца, так и для
общества (Зигмунд Фрейд).
А вот ещё одно изящное неявное определение, предложенное в качестве примера
известным зарубежным психологом Л. Уилсон: «Креативность напоминает кошку,
пытающуюся догнать свой хвост. В каждом акте творчества или при решении проблемы мы
часто ходим по замкнутому кругу, желая поймать мысль. Иногда верный ответ или решение
прямо перед нашими глазами, но мы не можем его увидеть. Чтобы найти ответ задачи,
сообразить, чего мы не учитывали, понять, в чём решение проблемы, нам достаточно всего
лишь посмотреть на то, что хорошо известно, с другой стороны или совершенно по-новому».
Итак, креативность, коротко говоря – способность к творчеству. А процесс, в котором
эта способность находит своё выражение, будем называть креативным (творческим,
эффективным, продуктивным) мышлением.
Можно сказать и по-другому: креативное мышление – процесс, который мы
используем для того, чтобы прийти к новой идее. Или – эвристический акт. Или – процесс
рождения нового знания, ранее не существовавших идей.
Что значит «ранее не существовавших»? Если ты ничего не сеял, то вряд ли будет что
собирать. В начале решения задачи мозг создаёт новые комбинации из уже имеющихся
блоков информации. Каждая такая заготовка подвергается проверке с привлечением
различных критериев: имеет ли отношение этот «информационный эмбрион» –
своеобразный зародыш решения – к анализируемой задаче, каковы его внутренние свойства,
связи его элементов между собой и с другими структурами и т. д.
Но самое важное, что совершает наше мышление, – развивает каждую из мыслейэмбрионов. То есть строит модельную ситуацию, используя все входные данные. Цель –
проследить, как меняются мысль, предмет или способ при варьировании (в пространстве и
времени) связей между окружающим миром и информационной моделью. Оценивая
параметры такой эволюции, мозг принимает решение о степени соответствия новой идеи
искомому решению.
Иногда генерацию новых идей сводят к примитивному утверждению: новые идеи
формируются в нашем мозгу, только развивая уже имеющиеся, новые идеи появляются лишь
в контексте того, что мы уже знаем. В процессе мышления мозг способен порождать новую
информацию, новые креативные идеи, только опираясь на имеющийся запас данных и свои
комбинаторные и аналитические способности. Ещё в XVII веке один из основоположников
современной педагогики философ Джон Локк сформулировал это чеканной латынью: nihil
est in intellectu, quod non fuerit in sensu, nisi intellectus ipse – в сознании нет ничего, чего бы не
было раньше в ощущениях, кроме самого сознания. Поэтому придумать что-либо при
анализе задачи, вопроса, для разрешения «безвыходной» ситуации мы можем, лишь
перебирая записи в своей «записной книжке».
Бигуди № 5
Из записной книжки писателя (изобретательного и остроумного Роберта Шекли). Те же
два вышеупомянутых приятеля на сей раз приобрели потрясающую подводную лодку,
обеспечивающую их всем необходимым (как заявил продавец – всё тот же старьёвщик) и
гарантирующую полную безопасность при исследовании океана на чужой далёкой планете.
Однако быстро выясняется, что друзья практически «выкопали себе яму»: лодка держит их
взаперти (это ещё полбеды!), при температуре не ниже 30 градусов Цельсия, кормит только
чем-то вроде смеси глины с машинным маслом, а воду считает безусловным ядом. Потому
водой не поит, так как своим долгом считает сохранение жизни своих пассажиров. Именно
такие параметры жизнедеятельности в неё заложены её инопланетными конструкторами.
Поскольку вода – яд, то и выйти на берег друзьям невозможно: лодка вообще не подходит к
берегу, а выпустить их в воду не желает, поскольку обязана беречь их жизнь! Может быть,
вы знаете, как следует действовать заложникам заботливой компьютерной системы?13
Кстати, уровень развития современной технологии позволяет весьма эффективно
использовать компьютер в качестве помощника и тренера для организации и повышения
продуктивности своего мышления, для выработки по настоящему творческих и
осмысленных идей и решений. В качестве одного из примеров отметим комплекс программ
«MindManager» в любой из его редакций. Президент Компьютерного клуба «Golden Triangle»
Д. Фэррис говорит о нём: «Этот инструмент позволяет вам стать более коммуникативным,
более творческим и более продуктивным. Развивая и увеличивая возможности известных
методик по организации мышления, программа даёт вам возможность увидеть, где вы
находитесь сейчас, куда вы хотите прийти, и выбрать наиболее эффективный способ
движения. Вы сможете использовать все возможности, которые предоставляет мозговой
штурм, быстро генерировать новые идеи, анализировать их во всех деталях, структурировать
информацию для более удобного её запоминания, ускорять реализацию ваших идей».
Созданы специализированные варианты таких программ, предназначенные для бизнесменов,
менеджеров, маркетологов и пр. Все эти программы опираются на методики, предложенные
и разработанные известным специалистом в области креативной психологии Т. Бузаном и
его последователями.
Такого рода «интеллектуальные тренажёры» становятся сейчас достаточно популярны.
Но ведь всё дело в том, хочет ли сам человек развить своё мышление, хватит ли у него
терпения ходить с помощью «электронных костылей». Как Вы думаете, часто ли бизнесмен,
менеджер или политик будет прибегать к такому интеллектуальному помощнику, который
не скажет сразу, что делать, а будет предлагать лишь один за другим разные варианты – не
решения, нет! – только пути, по которому можно было бы попробовать двигаться! Но ведь
нет иного способа решать действительно творческие – а потому каждый раз новые – задачи.
Есть анекдот: изобретена универсальная машина-парикмахер, идеально стрижёт и бреет
любого. Вставляешь голову в отверстие – дальше всё сделает сама машина. Различия в
форме головы и чертах лица существуют только до первого сеанса…
Один из критериев различения типа творческого процесса – степень его
целенаправленности. Он может быть неорганизованным (случайным), преднамеренным или
непрерывно обусловленным, т. е. идущим одновремено с выполнением какой-либо работы.
«Многие из открытий, которые обычно считают случайными, на самом деле родились
благодаря огромной силе воображения, мгновенно рисующей разнообразные приложения
случайного наблюдения. Вот несколько классических и наиболее часто упоминаемых
примеров таких «случайных» открытий.
Двое физиологов – фон Меринг и Минковский50 – изучали функцию поджелудочной
железы при пищеварении. Для того чтобы посмотреть, как будет протекать процесс
пищеварения в отсутствие этой железы, они удалили её хирургическим путем. И вот
однажды служитель, ухаживающий за их подопытными животными, пожаловался, что не в
состоянии поддерживать чистоту в лаборатории: моча собак с удалённой поджелудочной
железой привлекает полчища мух. Подвергнув мочу анализу, Минковский обнаружил в ней
сахар. Это послужило ключом к установлению связи между действием поджелудочной
железы и заболеванием диабетом и явилось основой последующего открытия инсулина.
Выдающийся французский физиолог Шарль Рише51, плавая на прогулочной яхте
принца Монакского, вводил собакам экстракт из щупальцев актинии, определяя токсичную
50 Оскар Минковский (1858–1931) родился в Каунасе (тогда – Российская империя), барон Йозеф фон
Меринг (1849–1908) – в Кёльне. Исследования поджелудочной железы они совместно вели в Страсбурге, тогда
входившем в Германскую империю.
51 Шарль Рише (1850–1935) – французский физиолог и бактериолог, лауреат Нобелевской премии за 1913 г.
дозу. Однажды, при повторном введении собаке того же экстракта, он заметил, что очень
маленькая его доза приводит к немедленному летальному исходу. Этот результат был
настолько неожиданным, что Рише отказался в него верить и поначалу не приписывал своим
действиям. Но повторение эксперимента показало, что предварительное действие этого
экстракта вызывает повышение чувствительности к нему, или сенсибилизацию. Таким путем
Рише открыл явление анафилаксии, о возможности которого, по его собственным словам, он
никогда бы не подумал.
Основоположник биохимии Гоуленд Хопкинс52 давал своим студентам в качестве
упражнения хорошо известный тест на белок. К его удивлению, ни один из студентов не
получил положительной реакции. Исследование показало, что тест даёт такую реакцию
только в том случае, если используемый при этом раствор уксусной кислоты содержит в
качестве случайной примеси глиоксиловую кислоту. Этот вывод вдохновил Хопкинса на
дальнейшее исследование, приведшее в итоге к выделению триптофана – части белка,
вступающей в реакцию с глиоксиловой кислотой.
Когда Луиджи Гальвани у себя дома в Болонье увидел, что лягушачьи лапки, висевшие
в ожидании поджаривания на железной проволоке, периодически сокращаются, он после
внимательного наблюдения сделал вывод, что сокращение мышц происходит в том случае,
когда лапка одной своей частью касается железной проволоки, а другой – куска медной
проволоки, случайно прикрученного к концу железной. Именно это наблюдение привело его
к конструированию так называемой биметаллической дуги, что в итоге выразилось в
понимании природы электричества и последующем изобретении элемента Вольта.
Немецкий физик В. Рёнтген экспериментировал с электрическими разрядами в высоком
вакууме, используя платиноцианид бария, чтобы обнаружить невидимые лучи. Ему и в
голову не приходило, что эти лучи способны проникать сквозь непрозрачные материалы.
Случайно он заметил, что платиноцианид бария, оставленный вблизи вакуумной трубки,
начинает флуоресцировать, даже если его отделить от трубки чёрной бумагой. Позднее он
скромно объяснил: «По воле случая я обнаружил, что лучи проникают сквозь «чёрную
бумагу»». На самом же деле нужна была величайшая сила воображения, чтобы не только
увидеть этот факт, но и осознать его огромные последствия для науки».
Во втором случае можно эффективно и полезно применять существующие методики
организации креативного мышления – например, мозговой штурм (см. ниже). В этом случае
творческий процесс идет быстрее и создаётся пространство для рождения новых идей.
Многие используют эту методику неосознанно, не зная о ней заранее.
В третьем случае творческое мышление – непрерывный процесс, в котором генерация
вопросов и анализ ответов на них происходят благодаря накопленной информации,
тренировке, развитию правильной организации мышления. Можно даже сказать, что в этом
случае важен определённый, непрестанно совершенствуемый, склад ума. Результаты
творческого мышления такого типа, как правило, гораздо весомее, чем при креативном
поиске первых двух типов. Однако для обеспечения непрерывного прогресса следует – в
качестве первого шага – обучиться методикам развития креативного мышления. Т. е. нужно
не просто «запустить» свой мозг – генератор идей, а понять, в каких режимах он работает
лучше всего, и повышать путём тренинга свои интеллектуальные показатели. Может статься,
постепенно, в процессе деятельности, Вы поймёте: количество новых идей у Вас столь
велико, что никакие способы интеллектуальной «зарядки» Вам больше не нужны.
В освоении способов управления процессом своего мышления, хорошим упражнением
может быть так называемый обратный мозговой штурм. Его используют для решения
конкретных задач. На первом этапе всё внимание концентрируют на выявлении
всевозможных недостатков объекта. Анализ вскрывает недостатки, ограничения, дефекты и
52 Фредерик Гоуленд Хопкинс (1861–1947) – английский биохимик, один из основателей витаминологии,
лауреат Нобелевской премии за 1929 г.
противоречия, имеющиеся в конкретной идее или техническом объекте, который требуется
разработать или усовершенствовать. Предварительную их оценку проводят участники
сессии, более тщательную – эксперты, которые вычеркивают явно ошибочные утверждения,
уточняя тем самым перечень обнаруженных недостатков. На втором этапе обратного штурма
ведут поиск путей ликвидации недостатков, причем используют правила обычного
мозгового штурма. Один из основных недостатков мозгового штурма – отсутствие времени
на глубокое осознание задачи. Кроме того, для многих людей эффективнее процесс
индивидуального творчества.
Бигуди № 6
Раз Вы уже стали достаточно креативным, помогите сотрудникам «Рено» справиться с
проблемой «боя»: более 3 % ветровых стекёл доходят с заводов-изготовителей до
сборочного конвейера разбитыми! Причина проста – неосторожное обращение грузчиков. И
это несмотря на предупреждающие надписи на упаковке. «Рено» предпринимает срочные – и
тоже креативные! – действия. Результат налицо – «бой» уменьшился в несколько раз.
Надеюсь, Ваш совет в этом случае не хуже того, что придумали креативщики из фирмы
«Рено»?14
Известны различные критерии определения или выделения личностей с высоким
творческим потенциалом. Рассмотрим некоторые из них, а в качестве определения таких
личностей установим: «Творчески мыслящие личности характеризуются тем, что они
способны представить обычные, хорошо известные вещи и предметы в новом свете,
погрузиться под поверхность очевидного, чтобы обнаружить там ранее неизвестные образы,
обнаружить связи между, казалось бы, совершенно несвязанными друг с другом явлениями»
(Р. фон Эйк).
Приведенные ниже критерии сформулированы так, чтобы каждый человек, пытаясь
что-то понять о себе, указывал конкретный временной фактор, например, «большую часть
времени», «иногда», «очень редко». Тогда в конце этого самоисследования можно понять, к
какой группе людей себя относить: творчески одарённых – или не слишком. Чем больше
будет ответов типа «большую часть времени», тем выше вероятность того, что вы относитесь
к творчески одарённым личностям.
Итак, считается, что творческие личности:
✓ способны обнаруживать множество удивительных свойств у различных предметов,
постоянно задают вопросы обо всём, имеют широкую область интересов в разных областях,
часто собирают коллекции необычных вещей;
✓ легко генерируют большое число идей, вариантов решения задач, находят для
решения необычные, нестандартные, зачастую даже уникальные приёмы и методы;
✓ могут быть несдержанны в выражении своего мнения, радикальны и упрямы,
чрезвычайно упорны, особенно при отстаивании своего мнения;
✓ склонны к рискованным действиям, даже приключениям, любят получать новые
неожиданные и ранее неизвестные впечатления;
✓ интересуются различными интеллектуальными играми, легко погружаются в
фантазии, мечты, размышления, уходят в своё воображение; говорят: «Интересно, что было
бы, если…», или «Что, если мы изменим…»; манипулируют идеями, без труда их
видоизменяя, адаптируя к другим условиям, модифицируя свои оригинальные или чужие
мысли;
✓ демонстрируют живой, острый юмор, видят смешное в тех предметах и ситуациях,
которые другим не кажутся смешными: их юмор может даже показаться слишком
эксцентричным или несоответствующим ситуации;
✓ чрезвычайно эмоциональны и чувствительны, склонны к проведению
самостоятельных эстетических опытов (в живописи, литературе, музыке);
склонны к нестандартному поведению, более готовы к иррациональным поступкам;
бывает, что они обладают чертами, не свойственными своему полу, – например, повышенное
чувство независимости у девочек, какие-то девичьи интересы у мальчиков; они не
стесняются показать все противоречия своей натуры;
✓ допускают беспорядок, хаос в своём окружении или ситуациях, не интересуются
деталями, которые считают неважными; проявляют себя как неконформисты, считаются
эгоистами, индивидуалистами; их невозможно отнести к «серой массе», большинству, они не
задумываясь при необходимости противопоставляют себя большинству;
✓ ничьи категорические утверждения не признают без собственной проверки,
подвергают всё общепризнанное критическому пересмотру, но, как правило, конструктивно;
смелость ума позволяет им следовать своей интуиции.
Попробуйте проверить себя – каков, примерно, Ваш уровень креативности? Заодно
помогите крупной фирме сэкономить на лифтах.
Бигуди № 7
В новом высотном здании крупной и, вообще говоря, не бедной фирмы (скажем,
«Газпром») установлено несколько лифтов. Но служащие жалуются: ждать лифта
приходится долго, народ раздражается, ехать приходится в тесноте. Особенно это
чувствуется в часы пик: с утра, в обед, перед уходом с работы. Руководство фирмы, как и
следовало ожидать, не желает ни менять лифты на скоростные, ни увеличивать количество
лифтов, ни устанавливать компьютер для оптимизации их движения. При этом руководство
понимает: просто уволить всех недовольных сотрудников и набрать новых, покладистых –
решение временное, неудовлетворительное. Ребята, помогите бедному руководству фирмы
(скажем, «Газпром») советом! Кто чем может! Подайте бывшему… (продолжите фразу Кисы
Воробьянинова).
Приведём (следуя Гоу и Вудворту) ещё одну интересную классификацию творческих
типов личности, отметив заранее: в творческом коллективе, при проведении
крупномасштабных исследований или, например, при «мозговом штурме» важно
присутствие всех нижеперечисленных типов (кстати, к какому типу творцов вы могли бы
отнести себя?):
✓ «пионер» – Мощный творческий ум, создатель новых школ и направлений, успевает
сделать практически всё главное, щедро даря последователям множество неразработанных
идей, неоконченных экспериментов и плодотворных мыслей;
✓ «фанатик» – увлечён до одержимости одной идеей или целым направлением; часто
это вредит творчеству, так как отсутствует «перекрёстное опыление» с другими областями
знания;
✓ «эрудит» – образован и начитан, но стимул творческого поиска у него ослаблен или
вообще отсутствует; он довольствуется тем, что знает и так «всё обо всём»;
✓ «диагност» – критичен, обладает зорким умом, легко находит малые и большие
недостатки, уязвимые места теорий, скорее разрушитель, чем созидатель;
✓ «техник» – успешно решает поставленные задачи, доводит «голые» идеи до
завершения, результата;
✓ «эстет» – любит утончённые интеллектуальные проблемы, ценит красоту и
изящество в их решении, не станет начинать никакую, пусть и многообещающую
деятельность, если не чувствует к ней духовного влечения;
✓ «методолог» – придаёт важность не самой проблеме, а методикам её решения,
находит в этом полноту и законченность, любит обсуждать проблемы с коллегами;
✓ «независимый» – одиночка, не любит работать в коллективе, предпочитает
самостоятельно выбирать тему творческого поиска, никому не подчиняется и ни с кем не
согласовывает свои решения, опирается только на свои силы, совершенно уверен в себе.
Слова «творчество», «творческое мышление», «творческий подход», конечно, означают
далеко не просто комбинирование известных идей. Из множества их сочетаний, как мы уже
говорили, требуется отобрать по-настоящему ценные и продуктивные – или вообще не
создавать новых конструкций из старых мыслей, а дождаться, пока количество мыслей
перерастёт в новое качество. В новую информацию.
Тот же Полани писал: «Способность делать открытия не похожа на удачу азартного
игрока. Она зависит от природных дарований, развитых в ходе обучения и направляемых
интеллектуальным усилием. Она сродни достижениям в искусстве и, как они, не поддаётся
точному анализу».
Как для выбора, так и для генерирования идей совершенно нового качества нужны
вполне определённые параметры мышления. Для выбора необходимо точное логическое
прослеживание видимых и скрытых связей и отношений между различными явлениями и
предметами. Кроме хорошего логического мышления требуется надёжно действующая
память, наблюдательность, цепкое внимание – эти факторы обеспечивают логический
аппарат информацией. Однако же одних этих факторов недостаточно для творчества.
Компенсировать недостатки памяти и внимания может развитая интуиция. Она и важна
при генерировании, когда возникающая мысль не является явной комбинацией уже
известных. Напомню, что наиболее перспективно соединять в разных вариантах идеи и
выводы, принадлежащие даже не к смежным, а к максимально удалённым друг от друга
областям. Естественно, для этого нужна определенная дерзость ума. Но недаром Аристотель
говорил: остроумие – это дерзость, получившая образование.
Александр Лук отмечает присутствие в слове «остроумный» понятия «острый», то есть
отточенный, хорошо режущий или колющий. А в древнерусском языке слово «острый»
имело и другое значение – быстрый, что отображается и в некоторых документах тех времен
(«А люди тоя страны зело учтивы… и ко учению остроумныя», 1731 год).
Мотивом для мышления, для появления замысла, с «обычной» точки зрения может
служить не совсем ясное представление об «интересном». С точки же зрения философа,
интересное – то, что «раздражает мысль, вашу, мою, вот здесь, сейчас, а не останавливает
мышление, не даёт мысли остаться в привычных клише историко-философских,
лингвистических, культурно-исторических или каких угодно ещё концепций или
идеологических конструкций. Интересное для меня то, что изменяет тенденцию мышления в
отношении мыслимых им объектов: так объект А, только что мыслимый как А, сейчас
мыслится как В» (А.М. Пятигорский). Можно сказать вслед за М. Мамардашвили: мысль
рождается из удивления вещам как таковым, из душевного потрясения. Потрясение, некий
тонкий дисбаланс между информационными посылками и есть удивление. И это называется
мыслью. Мысль не есть исчисление… Выразить письмом мысль нельзя, мысль невыразима
(и это вновь о роли неартикулированного знания!)
Процесс появления новых идей сильно обусловлен и работой подсознания. Интуиция
извлекает из хаоса подсознания те идеи, в этот момент представляющиеся сознанию
удовлетворяющими нужным критериям, и упорядочивает их. Конечно, подсознание активно
участвует и в процессе логического мышления. Иногда различают глубинный (интуитивный)
и поверхностный (логический) уровни подсознания. Отмечая исключительно важную роль
интуиции в научной деятельности, великий математик А. Пуанкаре называл интуицию
«проницательным предчувствием». Ганс Cелье добавил: «Логика составляет основу
экспериментальных исследований точно также, как грамматика составляет основу языка.
Однако мы должны научиться пользоваться математикой и статистикой интуитивно, т. е.
неосознанно, так как у нас нет времени для того, чтобы на каждом шагу осознано применять
законы логики».
Интуицию чаще определяют как «способность постижения истины путем прямого её
усмотрения, без обоснования с помощью доказательства». В познании мира интуитивное и
логическое не исключают, а диалектически дополняют друг друга, помогая выбрать главное,
существенное, особенное.
Часто «инстинктивное чувство интуиции» относят к числу способностей, заложенных в
человека Природой, но не получивших полного развития. Примерно того же типа, что и
телепатия. Например, при использовании концепции единого энергоинформационного поля
человечества, популярной в некоторых кругах, можно было бы построить вполне
механистическую модель работы подсознания и интуиции, как некоего «транспортёра»,
непрерывно подающего блоки информации («мысленную материю» или «мысленную
субстанцию», как говорил М. Мамардашвили – субстанцией можно назвать то, что дальше не
имеет никакого другого носителя, ни к чему не сводимо и носитель самого себя) из поля в
отдельный мозг. Запросы на эти фрагменты мозг передаёт во всемирное хранилище в
соответствии с характером решаемой задачи и в связи с полным набором возникающих при
этом ассоциациативных связей53. Отметим, что в этом случае память выступает и как склад
информации, получаемой индивидом лично, и, возможно, как механизм, промежуточный
между «процессором» мозга и «глобальной информационной сетью».
Задействовать работу подсознания на полную мощность, пробудить интуицию – одна
из сложнейших задач при активизации творческого мышления. И, наверное, самая главная.
А. Пуанкаре считал: творческий процесс представляет собой чередование сознательных и
подсознательных процессов в коре головного мозга. Неудачные попытки сознания получить
решение задачи «включают» механизмы подсознания. Не исключено, что «включение»
носит своеобразный пороговый характер: подсознание начинает перебор своих вариантов в
рамках заданных сознанием условий и критериев только после достижения структурами
мозга определённого уровня возбуждения54.
В выработке остроты мышления очень помогает решение обратных задач. Даже из
повседневной практики известно, что вычитать труднее, чем складывать, делить сложнее,
чем умножать, извлекать корень сложнее, чем возводить в степень и, наконец, интегрировать
труднее, чем дифференцировать. Обратные задачи в области поиска противоречий дают
практически неограниченные возможности для самосовершенствования. Ведь объектов
вокруг нас несчётное множество, и каждый возник в результате разрешения некоторого
противоречия. Если вы освоите этот метод, то вам больше не понадобятся многие задачники.
Вы сможете тренировать свой ум всюду, независимо от того, есть ли у вас под руками
бумага и карандаш, книга или учебник.
Альберт Эйнштейн в одном из писем сказал: «… соединение интуиции и воображения
является, возможно, основной отличительной чертой продуктивной мысли, которая уже
значительно позже оформляется в логический порядок слов или других знаков, посредством
которых можно передать её другим. Привычные слова и другие знаки с усилием
отыскиваются уже во вторую очередь, когда упомянутая ассоциативная игра уже достаточно
закреплена в памяти и может быть по желанию воспроизведена вновь». Но он же и
признался: «Я думал и думаю месяцами и годами. Девяносто девять раз заключение неверно.
В сотый раз я прав».
Заметьте: здесь вновь встречается мысль о переходе информации из
неартикулированной в артикулированную форму. И что ещё важно: сама возможность или
53 Впрочем, описанная модель лишь переформулирует в более современных терминах концепцию,
популярную ещё у творцов индуизма и иудаизма, а в европейской традиции связанную с именем Платона. Если
считать, что душа человека содержит все сведения, накопленные до рождения (в предыдущих земных
существованиях или в потустороннем мире), то интуиция сводится к вспоминанию фрагментов этих сведений.
Общий же их объём неограничен: душа может располагать в неявном виде сведениями обо всех Вселенных.
54 Или же, возможно, после прохождения нервного импульса заданной формы – «ключа», означающего
примерно: «у меня не выходит, давай подключайся».
невозможность повторить, закрепить, уточнить промелькнувшую мысль обусловлена типом
её «записи», характером и глубиной артикуляции. Впрочем, Луи Пастёр отметил: интуиция,
«случай помогает лишь умам, подготовленным к открытиям путём усидчивых занятий и
упорных трудов».
Вернёмся к классификации понятий «креативности». Вот какая она по Р. Лэйну:
✓ Креативность в науке обычно ассоциируется с познанием неизведанного, изучением
того, что не совпадает с предсказаниями общепринятой теории, или в том случае, когда две
теории приводят к разным результатам (автор считает: креативность в этом аспекте близка к
понятию открытия).
✓ Выделение (восприятие) образа (сигнала) из сложного фона. В научном творчестве
возможно получение новой информации – открытие – при выделении из общего фона
уникального образа или сигнала.
✓ Восприятие, обнаружение нового в старом, известном55.
✓ В процессе синтеза, имеющего место после проведения анализа, происходит
креативное комбинирование элементов задачи или вообще новых элементов, выявляемых
уже этим анализом56.
✓ Изобретение в общем смысле – это сочетание различных форм креативности, но при
оформлении патентов требуется уточнить некоторые специальные аспекты57. Отметим, что
изобретатели исходят из воображаемого использования прибора, устройства, технологии.
Конкретные технические приёмы имеют при создании прибора небольшое значение: важнее
воображение, связывающее необходимость иметь прибор такого типа и назначения с
возможностями его создания.
✓ Решаемая проблема содержит и требует задействовать все аспекты креативности – в
том числе и фиксированное восприятие конкретной проблемы. А именно, мозг должен
представлять себе альтернативы, уметь отвечать на вопрос типа «как это может быть
иначе?»; естественно, эта способность связана и с возможностью мыслить абстрактно.
✓ Использование аналогий. Метафора и сходство – лишь два типа аналогий. Другие
появляются при переносе, например, экономических парадигм в политику, при выявлении
исторических аналогий: скажем, поведение России в конфликте в Сараево в 1997 г. было
похоже на её поведение в 1914 г. Такие аналогии могут быть креативны, но нужно
учитывать: аналогии не полны – то, что схоже в одном аспекте, может быть совершенно
иным в другом.
✓ Восприятие очевидной задачи: обнаружение и изучение иррелевантных, не
относящихся к делу аспектов ситуации, выделение определяющих, доминирующих
факторов.
✓ Проникновение в мысли и чувства других людей58.
55 Например, художник рисует всё тот же дом и сад, но искривляет вертикальные линии, нарушает законы
перспективы, пишет оранжевым то, что в действительности зеленеет… А на готовой картине вы неожиданно
обнаруживаете новую информацию о хорошо известных предметах, их новое соотношение и новые
возможности, видите доселе скрытые аспекты (ту самую скрытую реальность, о которой говорил Полани!).
56 Пример из политики: известны все элементы кризисной ситуации, военные возможности сторон, их
экономические и национальные интересы, история отношений противников – но что конкретно делать в этой
ситуации и что посоветовать делать противникам? Ответ зависит от синтеза элементов в согласованную
полную картину, чья полнота и адекватность проверяются с использованием критериев общественнополитического характера, и набора предсказаний-прогнозов.
57 Например, если в предложенном изобретении использована модификация ранее известных приборов –
это кумулятивное изобретение и т. д.
58 Эту форму креативности автор полагает простирающейся далеко от науки или экономики – к семье,
Раз уж мы так глубоко погрузились в науки, добавлю: С. Смит, Т. Уорд и Р. Финке
предлагают свой анализ вклада в креативные способности личности, привносимого
подсознательными, интуитивными процессами. При этом креативность (они считают: это
просто решение проблем, формирование концепций и понимание) порождается
ментальными процессами, включая работу подсознания, интуицию. При таком описании
креативного мышления познание рассматривается как результат срабатывания интуиции,
озарения, инсайта, подсознательного прорыва. Ну, конечно, особо важны и накопленные,
априорные знания при креативном решении проблемы.
Бигуди № 8
Скажем, удастся ли Вам, не зная законов физики, не проиграть пари в следующей
ситуации: Вы находитесь в баре, рядом с вами попивает пиво из кружки Ваш случайный
знакомый, который утверждает, что способен сделать так, чтобы монета упала на стол
ребром и осталась стоять на ребре! Но Вы-то хорошо знаете: вероятность падения монеты
«на ребро» очень мала – Монета всегда падает на стол вверх «орлом» или «решкой» с
вероятностью, почти точно равной 1/2. Да ещё и чтобы монета продолжала стоять на ребре!
Итак, пари? Конечно! И всё же не ставьте слишком много, Вы рискуете проиграть. Но
почему? Разве одной теории вероятностей недостаточно для анализа ситуации и победы?
Обратите внимание на все детали в условии задачи и не забывайте: кроме силы тяготения на
монету может действовать ещё какая-то сила, стоит только кое-что сделать с этой монетой…
Продолжайте мысль! Зря, что ли, там пиво?15
Говорят: если невозможно получить аналитическое решение проблемы, можно найти
ответ путём инсайта, озарения, в основе которого лежит эффективная работа подсознания.
Попробуйте и так.
Ещё одна модель описания креативности носит красивое название – Модель
шестиконечных снежинок (Д. Перкинс). Основными элементами креативности при этом
считаются:
✓ наличие сильного внутреннего побуждения, обязательства перед собственными
эстетическими принципами: творец склонен к усложнению, реорганизации, асиметрии; он
получает наслаждение, бросая вызов хаосу и пробиваясь к разрешению и синтезу;
✓ способность выходить за рамки при решении проблем: так, например, учёные
считают «хорошими» лишь вопросы, дающие интересные – т. е. неожиданные – ответы, что
позволяет взглянуть на проблему с иной стороны, получить креативное решение или сделать
открытие;
✓ ментальная мобильность, дающая творческим личностям возможность находить
новые перспективы при решениях традиционных или возникающих проблем: такие личности
склонны думать и рассуждать «напротив» или «вопреки»; рассуждая метафорически и по
аналогии, они в конце концов приходят к предположениям, ведущим к решению;
✓ готовность рисковать и ошибаться: творческие личности способны учиться на своих
ошибках; «работая» на краю возможностей, там, где велик риск ошибки, креативно
одарённые люди с большей вероятностью выдают и новые креативные результаты;
✓ приятие сторонних точек зрения: креативные личности склонны не только
критически рассматривать и судить собственные идеи или предложения, но также адекватно
воспринимают чужое мнение или критику, они объективны, что означает выход за пределы
своего «Я», поиск и учёт советов от знающих коллег, проверку своих идей;
друзьям, в общество. Далее он отмечает: для выделения этих особенностей нужно говорить о социальном
интеллекте, который практически ортогонален навыкам, проверяемым при изучении IQ – это также отмечал Р.
Стернберг.
✓ внутренняя мотивация: творцы вовлечены в процесс работы по собственному
желанию, не из-за внешних материальных стимулов, их работу катализирует наслаждение,
удовлетворение и полезная отдача от работы.
А. Тэйлор в книге «Природа творческого процесса» определяет пять уровней
способностей к творчеству, отмечая: первые три уровня могут быть достигнуты каждым при
соответствующей мотивации и упорстве (возьмите это на заметку!). Последние два уровня
доступны далеко не всем, но характерны для личностей, способных переживать вдохновение
или сверходарённых от природы – то есть гениев.
1. Уровень примитивной и/или интуитивной экспрессии. Творчество на этом уровне
представляет собой непосредственное и простейшее выражение чувств, эмоций и мыслей,
обычно присущее детям и подросткам, не занимающимся специально и постоянно
искусством. Это наивное и примитивное искусство (творчество), тем не менее, наполнено
чувствами и переживаниями. Процесс и результаты такого «наивного» творческого процесса
предназначены в основном для собственного удовольствия. Так что – пишите стихи и песни,
рисуйте или вышивайте. Если вам это нравится – замечательно!
2. Академический и научно-технический уровень. Люди, находящиеся на этом уровне
креативности, обладают обширными познаниями в разных областях, освоили методики и
приёмы создания нового знания в различной форме, имеют достаточно опыта и умения для
реализации своих замыслов и идей, используя широкий круг приёмов и методов. В этом
случае совершенное владение своими знаниями, ремеслом и инструментом сочетается с
творческой энергией. Вот в этом случае уже можно пытаться опубликовать результаты своих
трудов или устроить выставку.
3. Изобретательский
уровень.
Для
человека-творца
открыта
возможность
экспериментировать в рамках своих знаний или деятельности (ремесла), эксплуатировать
различные способы использования известных инструментов, предметов, подходов. При этом
творцы-изобретатели используют стандартные приёмы, общеизвестные факты, устойчивые
навыки лишь в качестве опоры, исходной точки для выработки новых идей. Для этого уровня
характерно разрушение повседневных правил, выход за границы академических традиций.
Это уже серьёзный уровень, тут обитают хорошие учёные и специалисты, неплохие писатели
или кинорежиссеры, вполне интересные художники.
4. Уровень инноваций. В этом случае артисты, писатели, музыканты, изобретатели и
мыслители ещё оригинальнее. Разрушая все границы, они предлагают методы, идеи, знания,
качественно отличные от стандартных. Представления предыдущего уровня сохраняются
лишь на уровне субструктуры, подсознательного мышления, руководящего этими
креативными усилиями. А чтобы работать на этом уровне, нужно быть воистину новатором,
двигаться, не обращая внимания на то, что остаётся позади, и не сокрушаясь по этому
поводу. Генрих Гейне заметил: «Если бы римляне изучали все исключения из правил своей
грамматики, то у них не осталось бы времени на покорение мира».
5. Уровень гениальности. Он присущ редким индивидуальностям, чьи идеи и открытия
в искусстве и науке вообще не могут быть истолкованы или представлены в виде
комбинации идей, порождаемых на предыдущих уровнях творчества. Гений находится на
уровне интеллектуального и творческого развития, необъяснимом и, возможно,
недостижимом сознательными усилиями. Он уникален по своей природе, от природы. Что
тут скажешь? Моцарта не воспитаешь из обычного прилежного маленького мальчика. Если
маленькому Андрюше Колмогорову нравилась математика, так он ещё в 7–8 лет обнаружил:
квадраты натуральных чисел всегда представляются в виде суммы подходящего числа
последовательных нечётных чисел.
Родоначальник эврилогии П.К. Энгельмейер предлагал особо выделять технических
гениев по их возможностям интуитивного извлечения из смутного хаотического потока
мысли основной идеи изобретения. Таланты же в этой области превосходно справляются с
дальнейшей разработкой сформулированной идеи – это 3–4-й уровни по вышеприведенной
классификации. Для конструктивного выполнения достаточно уже быть просто
высококвалифицированным работником (2-й уровень).
Любопытно, впрочем, что психологическое изучение изобретателей, проведенное ещё
во времена Советского Союза, показало: а) продуктивность зависит от уровня интеллекта,
внимания, научно-технической эрудиции, но б) основным фактором, выделяющим подлинно
крупных изобретателей, представляется набор личностных качеств, помогающих отстаивать
свои идеи, создавать условия для их реализации, организовывать производство. Несомненно,
такие черты были специфичны для изобретателей тех времён: не пробьёшься – не добьёшься.
Но и в другие времена для осуществления идей нужно не меньше усилий, чем для их поиска.
Томас Алва Эдисон – один из величайших изобретателей прошлого – полагал:
изобретательский гений – это 99 % упорства и сидения и 1 % таланта. А в США – с учётом
насыщенности рынка не только товаров, но и идей – популярна поговорка: $1 тому, кто
придумал; $10 тому, кто сделал; $100 тому, кто продал.
Умственные усилия, которые совершает мозг в творческом процессе, обладают
эвристическим действием, соединяя в единое целое (смысл природного явления,
произведение искусства, схему прибора и т. д.) всё, что представляется подходящим для
создания целостного образа. Обработка (анализ и синтез) всего, что входит в круг, по
выражению Полани, «периферического знания», ведётся, пока не удастся констатировать:
«Процесс генерации умозаключения завершён. Мысль оформлена в надлежащем виде с
использованием требуемых подручных средств (записана словами, формулами, нотами,
красками), помещена в память и в любой момент может быть вызвана для её модернизации,
реализации или в качестве элемента при построении более общего образа». Как только будут
получены неизвестные данные, появятся новые замыслы, возникнет представление иного,
более высокого уровня абстракции, произойдёт следующий творческий акт, формирующий
уже новые понятия и опять связывающий их между собой.
Бигуди № 9
Некий английский часовщик из города Киддерминстер решил воспользоваться для
одной цели обычным лимоном. С одной стороны изобретатель вставил в него гвоздь, с
другой – найденную в кармане медную монету. Что же получил часовщик?16
8. Научное творчество
Как лететь с земли до звёзд,
Как поймать лису за хвост,
Как из камня сделать пар, —
Знает доктор наш Гаспар.
Ю. Олеша. «Три толстяка»
О научном творчестве стоит немного поговорить хотя бы для того, чтобы отчётливее
увидеть, как реализуется в жизни эвристический акт, как осуществляется «наше
эмоциональное заинтересованное личностное участие в поиске и приобретении
знаний…интеллектуальный порыв» (М. Полани), результатом которого становятся
Понимание, Осмысление и Знание.
Труд учёного – творческий, поскольку непрерывно меняет наше видение мира, то
уточняя детали отдельных явлений, то ломая целиком всю устаревшую систему наших
представлений о мире. Радость мышления, счастье открытия, эстетическое чувство красоты
полученных результатов – эмоциональные компоненты эвристического действия,
необратимо меняющего как каждый индивидуальный интеллект, так и общемировой фонд
знаний и идей.
Интеллектуальные акты эвристического типа создают некоторое приращение знания, и
именно в этом смысле они необратимы, в то время как следующие за ними рутинные
действия совершаются внутри уже существующего массива знания и потому обратимы59.
Ощущение интеллектуальной гармонии способно вести нас к познанию реальности,
порождая не просто веру, но убеждённость в истинности и глубине результатов
мыслительного процесса. Научные открытия достигаются страстными и напряжёнными
усилиями сменяющих друг друга поколений великих людей, которые сумели покорить всё
человечество силой своих убеждений, поскольку «интеллектуальное усилие, приведшее к
открытию и руководившее его верификацией, преобразуется в энергию убеждения,
утверждающую истинность этого открытия» (М. Полани).
Шаги творения
Как можно описать ход научного исследования? Основная его ткань – это «фантазия, в
которую вплетены нити рассуждения, измерения и вычисления», как сказал знаменитый
биофизик А. Сент-Дьёрди. Один из величайших математиков XX века Анри Пуанкаре
выделил в научном творчестве четыре стадии: подготовку, вызревание, озарение и проверку.
✓ Подготовка – стадия, на которой главную работу выполняет сознание, вооружённое
Логикой и Анализом и ведомое Сомнением и Предчувствием Гармонии (о последнем оно
может и не догадываться). При этом, как говорил Эйнштейн, творческий интеллект
выискивает (среди множества эмпирических фактов) то, что может повести в глубину, и
отбрасывает всё остальное – то, что перегружает ум и отвлекает от существенного.
✓ Затем, когда сознание сформулирует необходимость постройки нового мостика в
область неведомого, можно предоставить всё дело подсознанию. На этапе вызревания мозг
совершает огромную работу, мгновенно – и, похоже, хаотически – создавая, проверяя и
отбрасывая всё новые конструкции мостиков, ведущих к островкам в архипелаге
неизвестного ещё знания. Этот этап А. Эйнштейн называл «интуитивным прочувствованием
фактов».
✓ В конце концов на какой-то вариант твёрдо и недвусмысленно укажет Предчувствие
Гармонии – и это будет озарение.
? Затем вновь всё возьмёт в свои руки сознание и будет тщательно – не поддаваясь
радостной и наивной уверенности интуиции – проверять и перепроверять надёжность
конструкции, заодно устанавливая, к какому именно острову ведет мостик, и нет ли от него
дороги ещё куда-нибудь.
Конечно, такое перечисление этапов выглядит несколько пафосным. Более строго
дедуктивный метод формулируется так.
1. Опыт. Результат опыта следует осмыслить и найти уже известные объяснения. Если
эти объяснения неудовлетворительны, переходим к шагу 2.
2. Формулируем гипотезу, новое объяснение. Полезно изложить его кому-то другому
или постараться записать.
3. Сделаем возможные выводы. Если гипотеза (шаг 2) истинна, что из неё следует с
точки зрения логики?
4. Проверка (верификация). Ищем факты, противоречащие каждому из выводов (см.
шаг 3), чтобы опровергнуть гипотезу (шаг 2).
Галилео Галилей для объяснения законов падения предлагал известный мысленный
эксперимент (гипотезу), однако без экспериментов с шарами, бросаемыми с Пизанской
59 Рецепты создания булатной стали многократно терялись и вновь переоткрывались. А открытие
металловедения вряд ли удастся «закрыть». И отработанные на его основе новые технологии булата тоже
навсегда останутся достоянием человечества. Не зря теперь на выставках художественных ножей найти
булатный клинок едва ли не проще, чем из легированной – по тем же правилам металловедения – стали.
башни он не смог бы обосновать свои предположения и завершить трактат «Беседы и
математические обоснования двух новых наук, касающихся механики и законов падения».
Шаги метода можно выполнять по порядку – 1, 2, 3, 4. Если по итогам шага 4 выводы
из шага 3 выдержали проверку, можно продолжить и перейти снова к 3-му, затем 4-му, 1-му
и так далее шагам. Но если итоги проверки из шага 4 показали ложность прогнозов из шага
3, следует вернуться к шагу 2 и попытаться сформулировать новую гипотезу («новый шаг
2»), на шаге 3 обосновать на основе гипотезы новые предположения («новый шаг 3»),
проверить их на шаге 4 и так далее. Справедливо говорится: научный метод не может
доказать истинность гипотезы (шаг 2). Метод позволяет лишь доказать ложность этой
гипотезы. Именно поэтому происходит возврат к шагу 2.
Великий физик Ричард Фейнман отметил: «У нас всегда есть возможность
опровергнуть теорию, но, обратите внимание, мы никогда не можем доказать, что она
правильна. Предположим, что вы выдвинули удачную гипотезу, рассчитали, к чему это
ведет, и выяснили, что все её следствия подтверждаются экспериментально. Значит ли это,
что ваша теория правильна? Нет, просто-напросто это значит, что вам не удалось её
опровергнуть».
Вот те, кто работал в рамках такой схемы, действительно достиг вершин в науке своего
времени. Например, Абу Али ал-Хасан ибн ал-Хайсам ал-Басри – выдающийся арабский
учёный-универсал (965–1039 гг.) Занимая в Басре должность визиря, он оставил этот
высокий пост, когда понял, что на самом деле его интересует лишь наука. Халиф Египта алХаким пригласил ученого реализовать проект регулирования вод Нила, построив плотину
ниже Асуана – это была идея самого Ибн ал-Хайсама. Однако, убедившись, что проект
осуществить невозможно (в то время), учёный был вынужден притвориться сумасшедшим,
ибо халиф был намерен казнить его. Каково? Во все времена инакомыслие пряталось в
одежды слабоумных и юродивых…
Впрочем, после смерти халифа Аль – Хазен (так его называли в средневековой Европе)
жил в почёте в Каире до самой смерти, написав почти сотню научных трактатов, из которых
89 посвящены математике, астрономии, оптике и механике. Для нас существенно, что Ибн
ал-Хайсам сочетал в своих научных занятиях тщательные эксперименты со строгими
математическими доказательствами. В его честь назван кратер на Луне. Ученый пытался
доказать 5-й постулат Евклида, получил формулы для суммы последовательных квадратов,
кубов и четвёртых степеней и другие формулы для сумм рядов, почти обосновал появление
интегралов. Занимаясь оптикой, Ибн ал-Хайсам выдвинул теорию, согласно которой
«естественный свет и цветные лучи влияют на глаз», а «зрительный образ получается при
помощи лучей, которые испускаются видимыми телами и попадают в глаз». А вот ещё: он
высказал предположение о конечности скорости света! Гений? Пожалуй. Вот только знаний
на тот момент не хватало, чтобы сформулировать основные положения специальной теории
относительности, и Майкелсон с Морли ещё не родились.
Подготовка открытия, как и проверка – это творческие действия, ведущиеся
планомерно и систематически. О вызревании и озарении ничего определённого сказать
нельзя. Это и есть тот нелинейный процесс информационного взаимодействия, который пока
невозможно описать в известных терминах. Эвристический выбор пути в процессе научного
исследования характеризуется лишь ощущением растущей близости скрытой истины:
отдалённо похожее чувство направляет нас в попытках мысленно нащупать в памяти
забытое имя. И вдруг в памяти ниоткуда60 появляется ответ!
60 Или почти ниоткуда, как в рассказе А. Чехова «Лошадиная фамилия». Выше мы уже приводили этот
пример: «Надумал, Ваше превосходительство!… Надумал, дай бог здоровья доктору! Овсов! Овсов фамилия
акцизного! Овсов, Ваше превосходительство! Посылайте депешу Овсову!» – сообщает генералу, чей зуб
разболелся, восторженный приказчик. Он несколько дней пытался вспомнить фамилию некоего специалиста по
заговариванию зубов. Вспомнил же после того, как местный доктор спросил у него о цене на овёс. Конечно,
никакого открытия здесь не совершено. Достаточно было внешнего информационного толчка, чтобы цепь
ассоциаций замкнулась. Нужные сведения лежали в укромном уголке памяти и трудно было их всего лишь
Скачок над неведомым
Научное озарение – скачок, преодолевающий логический разрыв. Прыжок, в результате
которого и захватывается плацдарм на берегу неведомого, совершается внезапно. Но, повидимому, ведущим импульсом для него является информационный обмен внутри мозга –
Между сознанием и подсознанием.
Простейший пример – решение математической задачи. «Король математиков» Карл
Фридрих Гаусс не раз говорил: «Решение у меня уже есть давно, но я ещё не знаю, как к
нему придти». На «том берегу» разум уже побывал и знает, какие сокровища там хранятся.
Вопрос теперь лишь в том, как построить надёжную дорогу в те края, чтобы убедиться в
ценности научных сокровищ и ввести их в круг освоенных знаний.
Учёный должен рассчитывать и надеяться на рискованные скачки озарения,
разрывающие и разрушающие упорную систематическую процедуру изучения
действительности. Установленные правила умозаключения – дедуктивная логика Аристотеля
(руку к дедукции, впрочем, приложили и сам Платон, и Евклид, а также Роджер Бэкон,
Декарт, Паскаль, Спиноза и Лейбниц), индуктивная процедура Фрэнсиса Бэкона – указывают
пути извлечения из существующего знания разумных выводов. Первооткрыватель,
приходящий к своим оригинальным выводам, преодолевая логический разрыв, отклоняется
от общепринятого процесса рассуждения. Его действие оригинально в том смысле, что даёт
начало новому направлению, а способность начинать новое – это и есть свойственный лишь
немногим талант оригинальности. Чем обширнее зона контакта с неизведанным, чем
большее число проблем охватывает разум исследователя, тем мощнее его интеллект.
Пьер-Симон Лаплас подарил свою новую книгу «Изложение системы Мира»61 коллеге
по Институту Франции (так в эпоху революции называлась Академия Наук) Наполеону
Бонапарту62. Будущий император заметил: «Гражданин Лаплас, Ньютон в своей книге
говорил о Боге. В Вашей книге, которую я уже просмотрел, я не встретил имени Бога ни
разу». В ответ Лаплас сказал: «Гражданин Первый консул, я не нуждался в этой гипотезе»63.
Прорыв в неизведанное, научное озарение – одна из форм самообучения интеллекта,
способность переносить имеющийся опыт мышления или отдельные его элементы в новую
обстановку и использовать для решения новой задачи.
Альберт Эйнштейн на вопрос, как делаются великие открытия или изобретения,
ответил: «Очень просто. Все знают, что этого сделать нельзя. Но находится один невежда,
который этого не знает, он-то и делает открытие».
«Новая истина, – писал Жак Барзун, – неизбежно выглядит сумасшедшей, и степень
достать. Сходство с научным озарением, однако, в том, что логическим перебором информации достичь
результата не удавалось (этим перебором фамилий приказчик занимался несколько дней, а генерала всё это
время мучила зубная боль), «прозрение» произошло мгновенно при поступлении определённой информации
извне.
61 В двух томах размером с нынешний Большой Энциклопедический Словарь было подробно рассказано и
рассчитано, каким образом из ньютоновой теории тяготения вытекает всё наблюдаемое строение Солнечной
системы и поведение звёзд.
62 Великий полководец попал в академию за вполне серьёзные математические заслуги. Он, в частности,
решил несколько сложных геометрических задач на построение циркулем и линейкой – в ту пору эти задачи
были очень важны для прояснения самих оснований геометрии.
63 Вообще в науке стараются обходиться без понятия бога, дабы избавиться от соблазна объяснить его
вмешательством всё, что ещё не познано. Ломоносов говорил: легко выучить три слова – Бог сие сотворил – и
полагать их вместо всех причин. Каких бы убеждений ни придерживался учёный в повседневной жизни – на
работе он вынужден рассуждать как атеист.
этого сумасшествия пропорциональна её величию». А Ганс Селье сказал: «Для
непосвящённого имеется много общего между блестящими и сумасшедшими умами».
Впрочем, он тут же добавил: «Но важно рано распознать многообещающего
фундаментального исследователя, тогда, когда он нуждается в поддержке для развития своих
особых дарований. Культура, здоровье и мощь нации зависят прежде всего от её творческих
фундаментальных исследователей, от яйцеголовых. Точно так же, как каменный век,
бронзовый век и железный век характеризовались употреблением камня, бронзы и железа,
так и наш век, несомненно, войдёт в историю как век фундаментальных исследований».
Бигуди № 10
Между прочим, вышеприведенную мысль Эйнштейна хорошо иллюстрирует
следующий исторический факт: будучи ещё студентом, Джеймс Максвелл сдавал экзамен по
термодинамике и теории газов. Преподаватель, высоко оценивая способности Максвелла,
дал ему задачу, решения которой в то время не существовало, о чём сам студент не
подозревал. Просидев над задачей более часа, Максвелл подал профессору листок с ответом.
Так появилось знаменитое «распределение Максвелла». Попробуйте и Вы «воспарить над
неведомым» и разберитесь в следующем вопросе. Все видели, что во время соревнований по
легкой атлетике некоторые бегуны выбирают такую тактику бега: на дистанции они почти
всё время держатся позади противника, а у финиша вырываются вперёд (такой бурный
финиш называется «спурт»). В прошлом так часто побеждали легендарный бегун Владимир
Куц, великий финский стайер Пааво Нурми. Как Вы считаете, это чисто психологический
приём или есть и физическая причина для таких действий? Не забывайте и о том, что
стадионы находятся на Земле, а не в космосе!17
Гармонизация хаоса
Но какова же последовательность шагов по пути к открытию, с чего всё начинается? С
того, что нечто неясное или негармоничное раздражает мозг, как соринка мешает глазу. Так
появляется ощущение проблемы64. Убеждение, что «соринка» нарушает ранее известную,
устойчивую картину явления (справедливость теории, истинность теоремы), крепнет по мере
анализа ситуации.
«Беспокойство – это неудовлетворенность, а неудовлетворенность – первейшее условие
прогресса. Покажите мне совершенно удовлетворенного человека, и я открою в нем
неудачника», – сказал как-то по этому поводу тот же Томас Эдисон.
И вот уже начинается подготовка – задача увидена, круг внутреннего
интеллектуального «зрения» расширяется. Мозг действует как незрячий, нащупывая дорогу
своей тростью: она, натыкаясь на предметы, передаёт эти толчки мышцам руки и таким
образом преобразует механические колебания в информацию о предметах, на которые
наткнулась. Собирая информацию, мозг ищет способ «расправиться» с задачей, т. е.
выдвинуть гипотезу, идею – предположение, которое может быть правильным или ложным,
в зависимости от того, существуют ли в действительности заключенные в нём скрытые
возможности65.
64 Великий физик Дж. Дж. Томсон, удостоенный за свои труды титула лорда Келвина, на рубеже XIX и XX
веков заявил: стройное здание теоретической физики уже построено, и горизонт науки омрачают всего два
облачка – фотоэффект и опыты Майкелсона – Морли. Хотя гигант науки мыслил в концепциях века
уходившего, интуиция его не подвела. Из первого облачка выросла квантовая механика, из второго теория
относительности – две величайших физических концепции века нового.
65 Скажем, абсолютная система отсчёта Исаака Ньютона – как раз пример необоснованной гипотезы.
Уверяя, что «не измышляет гипотез», Ньютон, конечно, не лукавил: такая система отсчёта – с раз навсегда
запущенными часами и неизменными осями Вселенной – вполне удовлетворяла его религиозные чувства. Но
Нильс Бор сказал об идее, выдвинутой молодым физиком: «Это безумная идея. Вопрос
в том, достаточно ли она безумна, чтобы быть верной»66. Так вот, сущность научного
метода и заключается в выборе для последующей проверки таких гипотез, которые должны с
большей вероятностью оказаться верными. Ставить в ходе научного исследования «хорошие
вопросы» – признак научного таланта. А «хороший вопрос» – такой, что ответ на него
расширяет пределы истинного знания. «Безумие» же идеи означает: новое знание должно
качественно – в ключевых деталях – изменить привычную и уже неудовлетворительную
картину, поставив на её место нечто радикально иное.
Представьте, что Вы решаете задачку по математике и ответ получен «почти
правильный» – ну, всего на одну десятую не сходится. Тот, кто неплохо соображает, поймёт:
нельзя закрыть глаза на такую «мелочь». Такое несоответствие означает какую-то
существенную ошибку. Решение нужно перестраивать с самого начала, менять в нем всё 67!
Каждая «безумная» идея ставит всё «с ног на голову» только на первый взгляд. Потом
удивительным образом оказывается: именно так всё в природе и устроено. Основные идеи
квантовой механики тоже вначале казались безумными – а теперь не только вся
полупроводниковая электроника строится на этих «безумных» идеях: мы разрабатываем
квантовые компьютеры на основе полного их воплощения и предполагаем, что лет через 20–
30 с их помощью будет создан искусственный интеллект.
Разве не гармонизацией хаоса является порядок, установленный Бором в «квантовых
скачках» – экспериментально известных спектральных закономерностях, которые казались
чем-то совершенно непонятным, хотя и точно математически выверенным. Заодно Бор
заложил основы квантового мировоззрения, разрушив классический хаос в теории атома: на
место положительно заряженного «кекса» с вкрапленными «изюминками» – электронами –
пришла изящная аналогия с Солнечной системой.
Мозг выбирает среди тех картин, которые ему хорошо известны? Или природа именно
эту нужную информацию о себе и передает мозгу? Как понять?
Известный американский физик-теоретик Ф. Дайсон рассмотрел исторические примеры
болезненного процесса восприятия и признания принципиально новых идей в физике. Он
делает вывод: «Новый способ рассуждений и новые представления должны быть отысканы
ощупью и в темноте. Это медленный и мучительный процесс… Великое открытие, когда оно
только что появляется, почти наверняка возникает в запутанной, неполной и бессвязной
форме. Самому открывателю оно понятно только наполовину. Для всех остальных оно –
полная тайна. Поэтому любое построение, которое не кажется на первый взгляд безумным,
не может иметь надежды на успех». Однако Дайсон особо подчёркивает: творения
многочисленных ниспровергателей основ науки больше всего страдают недостаточной
«безумностью» и «отклоняются редакцией не потому, что их нельзя понять, а именно
потому, что их можно понять».
Сегодня нам хорошо известно: почти все вещества, встречающиеся в живых
организмах, – соединения углерода с заложенной в них асимметрией окружения атомов
углерода. Скажем, молекула метана – простейшая из всех углеводных молекул, простейшая
органическая молекула, в ней четыре атом водорода и один углерода. Но эта молекула
симметрична – Ле Бель и Вант Гофф представили её как тетраэдр, где все атомы водорода
располагаются на равных расстояниях от центрального атома углерода. А вот если вместо
водородов в такой структуре будут другие, различные атомы, то… Но когда Вант Гофф и Ле
Бель независимо друг от друга высказали предположение, что асимметричное
ведь мир оказался устроен «немного» сложнее!
66 Перекликается со словами Барзуна, не правда ли?
67 Правда, тот, кто соображает ещё лучше, вообще не ошибётся в решении.
тетраэдральное строение молекулы – причина её оптической активности (такие молекулы поразному ориентируют в пространстве электромагнитные волны), это вызвало скептицизм и
насмешки со стороны коллег-учёных. Один из них даже назвал взгляды Вант Гоффа «жалкой
спекулятивной философией». Однако именно Вант Гофф и Ле Бель оказались правы!
Бигуди № 11
Известный поэт Уильям Оден, большой поклонник книжек про Алису, в стихотворении
«Беспокойный возраст» описывает, как правша-ирландец, сидя в нью-йоркском баре и
любуясь своим отражением в зеркале, произносит:
Двойник мой, близнец, дорогое моё отраженье,
Каков на вкус ликёр в рюмке,
Которую…….?
Что там с этой рюмкой и почему Одена интересует вкус ликёра
И ещё один любопытный вопрос. Эксперты утверждают: среди профессиональных
шулеров левши встречаются очень часто. Рассмотрим, например, распространённый метод
мошенничества: подсмотреть, что за карта лежит сверху колоды, и соответственно сдать её
или тайком «придержать». Достаточно ли этого описания, чтобы объяснить, почему левше
проще, чем правше, это сделать? Как помечают карты – вы, я думаю, знаете.18
Идеи для будущего
Знаменитый физик Дж. Дж. Томсон заметил: «Из всех услуг, какие могут быть оказаны
науке, величайшая – введение в её обиход новых идей». Вот для чего нужны в науке
творческие «безумцы», способные генерировать идеи – а затем ещё и доказывать их
справедливость и убеждать общественность в реальности своих открытий.
Например, о новой математической концепции можно сказать, что она реальна, если её
разработка ведет к возникновению обширного круга новых и интересных идей. За сто лет до
Николая Лобачевского итальянец Саккери исследовал геометрии, где не содержался
евклидов постулат о параллельных прямых (не забудем также и более ранних математиков:
Омара Хайяма, Ибн ал-Хайсама и многих других68). Но он не смог осознать, что эти
геометрии могут быть истинными. Он только надеялся привести их к противоречию и тем
самым доказать – превратить в теорему – неподатливую аксиому. Лобачевский же – и
одновременно с ним Янош Больяи69 – приняли, как новую реальность, возможность более
чем одной прямой, проходящей через заданную точку, не пересекаться с заданной прямой, и
создали на основе неевклидовых аксиом новую область математики. Несколько позже
Бернхард Риман выстроил геометрию, где непересекающихся прямых вовсе нет. И, что
существенно, в конце концов научная общественность оказалась вынуждена признать: новые
концепции обладают той же степенью реальности, какую приписывали ранее системе
Евклида. Ну, хотя бы потому, что в евклидовом пространстве существуют поверхности, где
выполняются аксиомы Лобачевского или Римана70. А главное – в основе современных
68 Есть основания считать, что и сам Евклид не был уверен в очевидности и надёжности этого постулата.
Не зря он сформулировал это утверждение очень неуклюже, в отличие от изящной отточенности остальных
своих аксиом.
69 Почти одновременно с Лобачевским – а именно в 1825-м году. Свои выводы Янош Больяи опубликовал
позднее – в 1832-м году – в качестве приложения к учебнику геометрии отца, Фаркаша Больяи. Больяи-старший
списался со своим другом – королём математики Гауссом. «Я побоялся крика беотийцев!» – признавал Гаусс,
прочитав труд Яноша, содержащий основы неевклидовой геометрии. – «Чрезвычайно поражён случившимся…
Очень радует, что именно сын моего старого друга таким удивительным образом меня предвосхитил».
70 На плоскости отрезок прямой – кратчайший путь между заданными точками. Если это свойство счесть
физических представлений об устройстве мира теперь уже очевидно лежит неевклидова
геометрия пространства-времени.
Замечу: Лобачевский и Больяи не дождались признания своих идей. К сожалению,
такая судьба постигает многие новые разработки, и каждый творец должен считаться с
риском невостребованности. К счастью, творческая работа сама по себе приносит немалое
удовлетворение. И к нему добавляется сознание того, что рано или поздно человечество
извлечёт из неё плоды.
Ганс Селье говорит: «Исследование может быть полезным для человечества и принести
удовлетворение самому учёному, только если оно выполнено в такое время, когда может
встретить интерес и понимание. Атомная теория материи в том виде, как она была выражена
Демокритом, оказалась преждевременной и, стало быть, для того времени бесплодной, так
как не существовало никаких практических средств доказать либо опровергнуть её. Позже, в
эпоху средневековья, когда стало возможным доказать существование химических
элементов, алхимики, движимые надеждой на преобразование одного элемента в другой,
пытались получить золото. Недавно корректность этой идеи была доказана, и всё же к
алхимикам по справедливости относились как к эксцентричным фантазёрам, поскольку в их
время предпринимаемые ими попытки были не более чем мечтой. Все фантазии содержат в
себе зерно истины; гениальность же состоит в том, чтобы суметь распознавать такие
фантазии, из которых это зерно можно извлечь».
Роджер Бэкон (1214-после1292), казалось бы, всё время шёл против здравого смысла (с
точки зрения современников), и даже пятнадцать лет провёл в заключении, но на века
опередил других гениев – Парацельса и Леонардо да Винчи. За 300 лет до Коперника Роджер
Бэкон подверг сомнению правильность геоцентризма Птолемея, доказывал, что Луна светит
отражённым от Солнца светом, а Млечный Путь – это скопление звёзд, подобных дневному
светилу, расположенных от Земли неимоверно далеко.
Бэкон привёл доказательства, что Земля по форме – шар на основании наблюдений за
горизонтом во время плавания из Англии во Францию: линия горизонта представлялась ему
дугой, но не прямой. «Опыт, – писал Роджер Бэкон, – один дает настоящее и окончательное
решение вопроса; этого не могут сделать ни «авторитет» (который не дает «понимания»), ни
отвлеченное доказательство. Полезно и необходимо изучать также математику, которую
ошибочно считают наукой трудной, а иногда даже и подозрительной, потому что она имела
несчастье быть неизвестной отцам церкви». С её помощью он хотел проверять данные всех
остальных наук, и считал доступной каждому. Бэкон подразумевал, что есть действительный
жизненный опыт и «опыт-доказательство, полученный через внешние чувства». Но наравне с
опытом «материального» толка, он предлагал опираться и на духовный опыт, через
«внутреннее озарение». Его идеи предвосхищают понимание значимости творческой
интуиции и эвристических методов.
Те, кто сотни лет назад мечтал о полётах на Луну, не могут претендовать на приоритет
в области современных космических исследований. Даже если в будущем станет возможным
продлить человеческую жизнь на несколько веков, не стоит считать великим пророком
современного врача, предсказывающего это.
Ситуация несколько меняется, если фантазия представляет собой не просто банальное
благое пожелание, а интуитивное предвидение некоей скрытой плодотворной истины, до
поры до времени недоступной другим. Если этот истинный факт, не являясь в данный
момент объектом исследования, достаточно близок к уровню знаний своего времени, он
может побудить других специалистов создать соответствующие средства и методы.
В первом своём варианте – в формулировке Фракасторо – идея о переносе заразы
определением, то на любой поверхности можно строить местные аналоги прямых – геодезические, т. е.
кратчайшие, линии. В частности, на сфере прямыми по этому определению оказываются большие круги –
пересечения сферы с плоскостями, проходящими через её центр. И для таких «прямых» выполняется геометрия
Римана. А для геометрии Лобачевского подходит седлообразная фигура – гиперболический параболоид.
невидимыми крохотными существами не могла получить экспериментального
подтверждения, поскольку не существовало микроскопов или других средств её проверки. И
всё же эта интригующая мысль подспудно тлела до тех пор, пока ею не занялись Пастёр и
Кох, дав, вероятно, толчок последующему развитию микробиологии. Сформулированная
второй раз идея неразличимых под микроскопом переносчиков инфекции была
непрактичной, но несомненно послужила стимулом для поиска вирусов на третьей стадии
исследования, когда стали доступны методы ультрафильтрации71 и электронной
микроскопии.
Простое хотение (полностью непрактичная фантазия) отличается от интуитивного
предвидения чего-то, что ещё далеко не очевидно, но что имеет шансы стимулировать
дальнейшие исследования в тот момент, когда это интуитивное предвидение ещё не
выражено или по крайней мере ещё не забыто.
Открытие Грегором Менделем основных принципов генетики игнорировалось в
течение тридцати пяти лет после того, как о нём не только был сделан доклад на заседании
научного общества, но даже опубликованы его результаты. По мнению Р. Фишера, каждое
последующее поколение склонно замечать в первоначальной статье Менделя только то, что
ожидает в ней найти, игнорируя всё остальное. Современники Менделя видели в этой статье
лишь повторение хорошо к тому времени известных экспериментов по гибридизации.
Следующее поколение поняло важность его находок, относящихся к механизму
наследственности, но не смогло полностью оценить их, поскольку эти находки, казалось,
противоречили особенно горячо обсуждавшейся в то время теории эволюции. Позвольте,
кстати, добавить, что знаменитый статистик Фишер перепроверил результаты Менделя и
заявил: при обработке современными статистическими методами выводы отца генетики
демонстрируют явное смещение в пользу ожидавшихся результатов.
Бигуди № 12
1610-й год оказался временем великих и счастливых открытий для Галилея. 25 июля
Галилей в свой телескоп снова видел «Юпитера утром на Востоке вместе с его свитой». Но
затем он обнаружил «другое необычайнейшее чудо», сообщение о котором, как и требовала
научная
традиция
того
времени,
представил
в
виде
анаграммы
«Smaismrmielmepoetaleumibuvnenugttaviras». Ни в коем случае не беритесь расшифровывать
её – слишком много вариантов! Правда, Кеплер попытался это сделать и, выбросив две
«лишние» буквы, получил ответ в виде фразы «Привет вам, близнецы, Марса порождение».
Может быть, Галилей открыл два спутника Марса? Но нет, истинная расшифровка фразы
была другой: «Высочайшую планету тройную наблюдал» (Altissimum planetam tergeminum
observavi). Позднее Галилей написал: «Я нашёл целый двор Юпитера и двух прислужников у
старика, они его поддерживают и никогда не отходят от его боков». Что ж это за
«прислужники» и у какого «старика»? И почему эта планета «высочайшая»? Эти открытия
Галилея в дальнейшем уточнены многими исследователями. В качестве подсказки – почти
через 50 лет после наблюдения Галилея Христиан Гюйгенс открыл у этого «старика»
большой спутник, на котором по предположению учёных НАСА (вновь высказанному в
2010-м году) существуют элементарные формы жизни. Хотя, кажется, об этом писал в конце
1970-х – начале 1980-х годов один из популярных советских детских журналов, освещая
полёты «Вояджеров» 1 и 2.19
71 Существование вирусов впервые доказал русский биолог Ивановский, пропустив сок табачных листьев,
поражённых мозаичной болезнью, через фильтр из неглазурованного фарфора. Мельчайшие поры этого
материала заведомо не пропускают ни одну живую клетку, включая бактерию. Тем не менее фильтрованный
сок продолжал вызывать у табачных кустов болезнь. Так выяснилось: существуют возбудители болезней, куда
меньшие, чем клетка.
Непрерывная перестройка
В любом научном исследовании, по словам Эйнштейна, присутствуют явная и неявная
стороны: вторая становится первой при сопоставлении нового высказывания со старым.
Будет ли при этом полностью или частично отброшено устаревшее знание, ушедшее в тень,
ставшее неявным – зависит от радикальности перестройки всей системы знаний, от глубины
её творческой переработки72.
Отбрасывание – одна из основных черт творческой деятельности – не только имеет
внешний, философский аспект (отбрасывание устаревших концепций и т. д.), но и является
важным инструментом творческой деятельности. По пути к решению проверяются и
отбрасываются различные гипотезы – в этом плане удаление из «оперативной памяти»
неудовлетворительных вариантов укорачивает и расчищает путь.
Да, конечно, в науке решение одной частной проблемы практически сразу же приводит
к возникновению другой, зачастую более глубокой и общей – но в этом и заключается
непрерывность процесса познания.
Этапы большого пути
Можно ли зафиксировать основные этапы решения научной проблемы так чётко, чтобы
это творческое действие могло быть осуществлено «по инструкции»? Вряд ли – учитывая тот
хорошо известный факт, что чуть ли не самой важной «пружиной» в творческом процессе
является своеобразная одержимость задачей. История науки знает огромное число примеров
эффективной научной деятельности, завершающейся открытием лишь в результате
напряжённейшего процесса сосредоточенных размышлений. Например, тот самый «эффект
подсознания» сказывается лишь после загрузки в подсознание множества рабочих вариантов
и подключения всех, имеющих хоть малейшее отношение к проблеме, информационных
блоков.
Набор советов известного математика Д. Пойа помогает несколько упорядочить работу
на этапе подготовки: «Всмотритесь в неизвестное. Всмотритесь в конец. Помните о своей
цели. Не забывайте о ней. Удерживайте в уме то, чего вы добиваетесь. Всегда имейте в виду
цель, к которой вы стремитесь. Рассмотрите неизвестное. Рассмотрите заключение».
Призывы нечто внимательно рассмотреть и не забывать о цели самой деятельности,
повторяемые как заклинания, помогают сконцентрироваться именно на данной проблеме. Но
помогают только внушением. Они ничего не говорят о технике мышления, о том, как
конкретно – даже имея в виду цель, куда нужно стремиться – строить и исследовать ведущие
(или не ведущие) к ней пути.
Упрощая схему поиска решения, можно считать, что она содержит две совместно
выполняемые операции.
✓ Зафиксировать проблему в адекватных ей символах, т. е. на соответствующем языке.
Далее следует перестраивать способ представления задачи для того, чтобы выявлять в ней
новые аспекты, позволяющие или взглянуть на тот же предмет с иной стороны, или
проникнуть в глубину темы. Так расширяется запас данных: в растущих информационных
массивах мысль может прокладывать всё новые пути и устанавливать дополнительные связи.
✓ Покопаться в памяти – авось удастся вспомнить какую-нибудь аналогичную задачу,
решение которой уже известно. Вспоминание, конечно, неотделимо от попыток
72 Вообще в науке предпочитают сохранять старое знание как частный случай нового. Скажем, риманова
геометрия на сфере бесконечного радиуса превращается в евклидову. Физика Аристотеля получается из физики
Ньютона при учёте сил тяжести и трения. Но, конечно, такое сведение старого к новому возможно не всегда.
Например, утверждение того же Аристотеля о том, что у мухи 8 ног – тривиальная ошибка; правда, из почтения
к великому философу её без проверки переписывали из книги в книгу почти две тысячи лет.
самостоятельного исследования и поиска. Изменение исходных терминов и способа
описания, иная интерпретация первичных данных вполне может протекать, не будучи
однозначно оформленной в виде символов и слов. Это этап неартикулированной активности
мозга, нащупывание тропинок и постройка мостиков к островкам нового знания.
В дальнейшей работе мышление чередует вычисления и интуицию, отвоёвывая у
неизведанного всё новые территории. Это и есть тот спектр интеллектуальных операций,
посредством которых «артикуляция дисциплинирует и расширяет возможности
человеческого мышления» (М. Полани).
Зависимость хода мышления от узкого понимания контекста, погоню за
однозначностью, легко продемонстрировать одним из воспоминаний студенческих лет. Один
из моих друзей на пари утверждал, что может спрятать в комнате водку так, что её
невозможно будет найти. Остальные приятели, естественно, не могли в это поверить, так что
пари было заключено.
Итак, куплена и выдана моему другу бутылка водки. Все, кроме него, удалились из
комнаты на пятнадцать минут. Потом мой друг отворил дверь и жестом показал всем: прошу
входить! Ищите!
Поиски заняли почти час! Всё было перерыто, простукано и ощупано в лучших
традициях НКВД тридцатых годов. Найдено множество считавшихся ранее утерянными
вещей. Обнаружена и пустая бутылка из-под водки. Друг был обвинён в том, что попросту
выпил содержимое. Попросили дыхнуть. Но и само его поведение явно свидетельствовало об
ошибочности этого суждения.
В конце концов, придя в состояние полного изумления, «поисковая группа» возопила:
«Где же. это. искомое?» Мой друг спокойно взял со стола стоящий там всё время графин,
открыл и дал каждому понюхать. «Искомое» находилось у всех и каждого буквально «под
носом». Просто никто не сообразил: ищется не бутылка с водкой, а именно водка. Она и
была налита в графин вместо воды.
Это – типичная ошибка при поиске: нужные предметы, информация, сведения, данные.
разыскиваются где угодно – под подушкой, за шкафом, в секретном досье, в сейфе, в
тайнике. но только не на открытом месте. О разумности хранения важнейшей информации
на самом видном месте говаривали ещё Эдгар По и Артур Конан Дойль. Да и Гилберт Кийт
Честертон утверждал, что лист лучше всего спрятать в лесу.
Замечу, что сокрытие информации и её добыча – два противоположных, но
родственных направления разведывательной деятельности. Ибо зная, как прятать,
представляешь, и как искать.
Профи из разведывательных сообществ различных стран, безусловно, не совершают
наивных ошибок. Поиски, добывание, хранение, передача, защита важнейшей информации
входят в круг обязанностей информационно-аналитических служб. Для получения
информации о противнике зачастую оказывается вовсе не нужно ставить «жучки», делать
снимки в инфракрасных лучах, записывать спектры колебаний оконных стёкол. И уж тем
более не надо часами лежать с пистолетом в грязи под лавочкой или уметь водить всё, что
способно двигаться. Зато нужно уметь нечто иное: анализировать то, что на первый –
неискушённый – взгляд не может вообще содержать никакой важной информации.
Для извлечения интересующей информации из так называемых «открытых
источников» нужен серьёзный труд аналитиков и экспертов. Поиск сильно зависит от самого
ищущего: от его эрудиции, таланта и интуиции. И, в общем, оказывается, что обнаружить
нечто на самом деле важное с очень большой вероятностью можно, проводя поиск прямо
«под фонарём». Нужно «всего лишь» знать, что ищешь, чем искомое отличается от прочего
«мусора». И как потратить на поиски меньше времени. И при этом быть уверенным, что
ничего важного не осталось незамеченным. По оценкам профессионалов – разведчиков, как
западных, так и наших «ГРУшников», от восьмидесяти до девяноста пяти процентов всей
разведывательной информации составляет как раз та, что проистекает из открытых
источников.
Бигуди № 13
Давайте воспользуемся и мышлением, и интуицией, ещё раз прочтём схему поиска
решения. Может быть, этого будет достаточно, чтобы Вы сумели продолжить ещё один ряд,
предложенный членами MENSA: 1, 8, 70, 627, 5639…. Подсказка нужна? Есть такое поверье,
что у кошек… жизней. Это число вам пригодится!20
Единые правила
Существуют ли общие принципы и методы решения научных проблем?
Например, в естественных науках единым и универсальным считается принцип
подобия: различные по природе процессы или явления стремятся описать одинаковыми –
при представлении в безразмерной форме – уравнениями. Коэффициенты уравнений
понимаются как параметры – критерии сходства, подобия рассматриваемых процессов.
Ещё один вполне строгий метод – Математическая индукция: формализованный
логически способ выведения общего утверждения из частного. Исходный пункт при этом –
некое твёрдо установленное положение. Затем выдвигается гипотеза о форме и сущности
общего умозаключения. Её доказательство опирается на логическую процедуру,
позволяющую двигаться шаг за шагом, получая верный вывод на каждом шаге, а
следовательно, для всех шагов.
Любую сложную задачу имеет смысл свести к совокупности более лёгких задач.
Сначала следует упростить задачу до предела, оставив только её главные черты: на «дереве
проблемы» нужно ободрать листья, крону, оставив лишь ствол. Выяснить возможность
решения в предельных, частных случаях. Попытаться найти грубое качественное решение.
Наконец, на всех этапах следует пытаться опровергнуть полученный результат, используя
все соотношения, получаемые из конечного результата в разных предельных случаях.
Проверять необходимо и логическую структуру ответа: следует ли он из принятых посылок,
не противоречит ли неким общим утверждениям, верны ли границы применимости, можно
ли – и насколько – экстраполировать результат и т. д.
Известен некий «закон сохранения трудностей»: если при каком-либо подходе
выясняются принципиальные трудности, то они, как правило, должны проявиться и при
другом подходе к решению. Если трудности исчезли, надо понять, почему: либо есть прямой
способ, либо обходной манёвр неверен.
Результат решения оценивают ещё по одному критерию: ответ должен быть красив,
изящен, эстетичен. Если результат не таков – скорее всего, он неверен. На эту особенность
решения задачи всегда обращал внимание А. Эйнштейн, считая, что результаты
исследования человеком природы отражают внутреннюю гармонию и симметрию
устройства мира: «Без веры во внутреннюю гармонию нашего мира не могло бы быть
никакой науки. Эта вера есть и всегда останется основным мотивом всякого научного
творчества». А поскольку гармонию и симметрию человечество формализовало
математикой, не удивительно, что Эйнштейн сказал: «Бог говорит с нами языком
математики».
После получения научного результата нужно всесторонне его обдумать, стараясь
понять, какие смежные результаты он даёт. Излишняя концентрация внимания в одном
направлении иногда мешает получить лежащие рядом результаты! Стремление сначала
понять всё-всё до конца, а потом уже начинать работать – частая причина неудач в науке.
Прав был великий немецкий физик, физиолог и психолог Герман Людвиг Фердинанд
Гельмгольц: «Требуется хорошая систематизация, чтобы не потеряться безнадежно в
лабиринте учености».
Другой, противоположный, недостаток – желание на лету поймать ответ, предугадать
результат без упорной, длительной и сконцентрированной работы. Эта своеобразная
мозговая лень – причина весьма многих неудач не только в науке, но и в о всех сферах
жизни. Человек позволяет своему мозгу, мышлению, интеллекту, как пруду, заплывать
ряской, погружаться в тину.
Эта же лень опасна и ещё в одном отношении: человек может быть слишком уверен в
своих гипотезах – настолько, что не способен продолжать поиск и анализ. Однако для
научной работы (да и не только) нужно правильное сочетание уверенности и сомнения,
непреклонности и колебаний.
У всех методик есть свои границы. Хорош тот метод, которым можно свободно
пользоваться не обращаясь к инструкции, лучший из подходов тот, что дает результат, не
снижая, а повышая творческий потенциал личности.
В сущности, это простая диалектика жизни. А творчество в науке – часть жизни. И
принципы здесь одни и те же. Диалектика, гибкость ума, умение создавать новые
строительные «леса» мысли «раскачивают» ситуацию, заставляют мозг искать противоречие
и снимать его на ином качественном уровне. Вот что не даёт реке мышления стать прудом.
То есть труд мышления должен быть непрерывным и интенсивным.
Застойные явления в мышлении гораздо опаснее, чем даже в физиологии.
Анекдот почти на эту тему. Лесоруб в поисках работы приходит в лагерь
лесозаготовителей. Управляющий встречает его не слишком обнадёживающе. «Не знаю,
подойдёт ли тебе работа, – сказал он. – Мы здесь валим лес». Лесоруб обрадовался: «Вот это
дело как раз по мне». Управляющий решил испытать его в деле. «Бери топор, – сказал он. –
Посмотрим, сколько времени потребуется тебе, чтобы свалить вон то дерево». Лесоруб
подошёл к дереву и свалил его одним ударом топора. Управляющий потрясён, но не сдаётся.
«Великолепно, – сказал он, – теперь попробуй повалить вон то большое дерево». Лесоруб
подходит к огромному дереву и двумя ударами валит и его. «Потрясающе! – восклицает
управляющий. – B жизни не видал ничего подобного. Ты принят! Но где ты научился так
валить лес?» «О, я изрядно попрактиковался и набил руку в лесу Сахары», – отвечает
лесоруб. Управляющий в недоумении переспрашивает: «Ты хочешь сказать – в пустыне
Сахаре?» «Ну да, теперь там пустыня», – поясняет лесоруб.
Аналогии
В «Очерках организационной науки» создатель тектологии Александр Богданов
рассуждал так:
«Природа организует сопротивление многих живых организмов действию холода,
покрывая их пушистым мехом, перьями или иными мало проводящими тепло оболочками.
Человек тем же самым путем достигает тех же результатов, устраивая себе теплую одежду.
Стихийное развитие приспособило рыбу к движению в воде, выработавши определенную
форму и строение её тела. Человек придает ту же форму своим лодкам и кораблям, причём
воспроизводит и строение скелета рыбы: киль и шпангоуты в точности соответствуют её
позвоночнику и ребрам. Посредством «паруса» перемещаются семена многих растений,
животные с летательными перепонками и т. п. Человек усвоил метод паруса и широко
применяет его на памяти истории. Режущим и колющим природным орудиям животных,
например, клыкам и когтям хищников, были, вероятно, подражанием ножи и копья
первобытных дикарей, и т. п. В истории культуры можно найти сколько угодно таких
иллюстраций.
Самая возможность подражания, в сущности, уже достаточное доказательство того, что
между стихийной организующей работою природы и сознательно планомерною – людей нет
принципиального, непереходимого различия. Не может быть подражания там, где нет ничего
общего».
Ещё более общее находится между самими людьми. Действуйте, направленно
подражая лучшим образцам изобретательской мысли! Это при должной практике
сформирует набор эталонов, аналогий, инвариантов, к которым читатель сможет приложить
для сравнения и свои собственные наработки, личностное знание. Умение свести новую
задачу к предыдущей, уже решённой кем-то задаче, за счёт нахождения у них общего – один
из показателей развитого мышления.
В процессе научного творчества исключительно важным может оказаться умение
видеть и исследовать аналогии (можно найти и связь между количественным аппаратом
метода подобия и качественным изучением явления – по аналогии). Автор теории тепловых
двигателей Сади Карно уподобил тепловые двигатели давно известным водяным. Их
приводит в движение вода, падающая с высоты. Значит, так же переносит энергию
«теплота», отбираемая от нагревателя и «падающая» на холодильник73.
Неверная аналогия ставит преграду на пути к открытию. В XVII веке движение крови в
организме сравнивали с приливами и отливами – отсюда и специфические методы лечения, и
невозможность продвижения в познании анатомии человека. Из тупика анатомию вывел В.
Гарвей: он представил сердце в виде насоса – результатом стало открытие непрерывного
движения крови, большого и малого кругов кровообращения.
Полагая, что процесс горения подобен дыханию, химик Дж. Пристли экспериментально
доказал: растения восстанавливают кислород, израсходованный в процессе дыхания или
горения. Д. И. Менделеев вывел принцип периодичности и предугадал открытие некоторых
новых химических элементов, пользуясь аналогией со свойствами соседних, уже изученных.
Г. Лейбниц нашёл сходство между логическими доказательствами и вычислительными
операциями – позднее (правда, через два века) эта аналогия сработала при создании
математической логики. Помните, В. Кекуле во сне увидел змею, кусающую свой хвост, и
это натолкнуло его на идею структуры бензола в виде кольца – разве это не показывает и то,
как работает подсознание, и то, как оно ищет и демонстрирует аналогии в разной форме? А
атом в виде Солнечной системы, явившийся Нильсу Бору, по его словам, также во сне?
Как видим, аналогии в науке дают арсенал не только идей, но и решений. Новые идеи в
большинстве случаев – давно и хорошо забытые старые, но преобразованные,
перелицованные по-новому гипотезы и мысли, которые уже могут быть восприняты, для
которых подготовлена почва, пришло время. Для их обоснования подготовлен аппарат –
причём не обязательно в этой же области: аналогия, необходимая для прорыва, может
придти с совершенно иного направления. Основой такого сближения служит ещё и
единообразное устройство материального мира в различных его проявлениях. Говорят:
использование картин явлений и процессов по аналогии – твёрдая почва для
контролируемого риска.
Аналогия часто удобна для объяснения, облегчения понимания74. Без красивых
аналогий трудно описать сущность важного научного открытия. Причём аналогии должны
быть наглядны. Представление в упрощённой форме серьёзных научных трудов требует
таких же творческих усилий, как и сама научная работа 75. Многие глубокие научнопопулярные книги дают не меньший толчок развитию науки, чем оригинальные работы.
73 Впоследствии выяснилось: природа тепла принципиально отличается от природы жидкостей. Но
аналогия Карно оказалась столь удачна, что выведенные им формулы ничуть не изменились. Разумеется, дело
здесь прежде всего в том, что физик сразу же проверил свои расчёты на совпадение с реальностью. Стоит
какой-то аналогии дать хоть один явно неверный результат – и она сразу отбрасывается.
74 На международной конференции по физике многие коллеги В. Гейзенберга выражали сомнение в том,
что атомное ядро не содержит электронов. Как это может быть, говорили они, если электроны явно вылетают
из ядра, что хорошо видно в ядерных процессах? Не находя уже других аргументов, Гейзенберг закричал
коллегам: «Смотрите в окно, вот идут люди в пальто, входят в бассейн. Но они же не плавают в пальто в
бассейне! Откуда же уверенность, что из ядра выходят такие же частицы, что были внутри?»
75 Эйнштейн говорил: если учёный не может объяснить, что он делает, пятилетнему ребёнку – значит, он
шарлатан. Сам он такие объяснения давал не раз.
Примитивный пример умозаключения по аналогии из области права. По делу о
квартирной краже следователь обращает внимание на то, что преступники проникли в
квартиру в то время, когда хозяйка развешивала во дворе выстиранное бельё. Оказалось, что
несколько месяцев назад прокуратурой было приостановлено расследование по двум другим
делам о квартирных кражах, где преступники использовали аналогичное обстоятельство для
проникновения в квартиру. Догадка на основе аналогии подтверждается – квартирные кражи
совершены одной и той же группой.
Бигуди № 14
А вот ещё вопрос из области права. Двоих людей обвинили в совместном
преступлении. Если оба признают себя виновными, каждый получит лёгкое наказание. Если
это сделает только один, его освободят, а второго подвергнут суровому наказанию. Если оба
не признают своей вины, их обоих освободят от наказания – ибо прямых улик нет. Почему с
точки зрения отдельного обвиняемого лучше признаться, а с точки зрения обоих –
правильнее не делать этого?21
Опыт в уме
Чрезвычайно эффективный приём творчества – Мысленный эксперимент. Вот один из
его примеров. Мы уже вспоминали, как Галилей сбросил с Пизанской башни два пушечных
ядра разного калибра – чем наглядно доказал: все тела падают с одинаковым ускорением.
Этому предшествовал, как мы уже тоже вспоминали, изящный мысленный эксперимент –
тоже весьма наглядный.
Допустим, что прав Аристотель76: лёгкое тело падает медленнее тяжёлого. Возьмём те
же два ядра – лёгкое и тяжёлое – и свяжем их вместе. Получившаяся связка тяжелее любого
из исходных ядер. Значит, и падать должна быстрей. Но с другой стороны, ведь в эту связку
входит лёгкое ядро. Оно будет падать медленнее тяжёлого – и тем самым тормозить его.
Таким образом получится, что связка должна падать и быстрее, и медленнее тяжёлого ядра.
Очевидная бессмыслица. К такой же бессмыслице приведёт и предположение, что быстрее
падает более лёгкое ядро. Значит, приходится придти к выводу: они падают с одинаковой
скоростью. А эксперимент на натуре всего лишь подтвердил результаты мысленного.
Заметим: все рассуждения этого мысленного эксперимента вполне логичны. Логика
вообще позволяет вывести очень многое. Но логика в чистом виде формальна –
перемалывает всё, что будет предложено логически работающей мысли. Нужна подходящая
отправная точка. Её правильный выбор – это уже креативный момент.
Такое сочетание логики и креативности ближе к определению диалектической логики,
введенному в своё время выдающимся философом Эвальдом Ильенковым. На первый взгляд
термин «креативная логика» противоречив: ведь логика – строгое построение цепочки
мыслей по единым правилам, а креативность предполагает выход за правила. Но само
творчество тоже имеет определённые закономерности построения цепочек. Просто
закономерности эти сложнее – но если их удаётся постичь, творчество становится внятным и
логичным.
История развития науки свидетельствует о блестящих результатах применения
мысленного эксперимента, а современные тенденции развития превращают его в одну из
важнейших процедур познания. Мысленный эксперимент использовали Галилей и Ньютон,
Мах, Кирхгоф, Максвелл, к нему постоянно обращались Эйнштейн, Бор, Гейзенберг.
Правда, пока отсутствует единая терминология мысленного эксперимента. Его
76 Великий философ опирался на очевидные наблюдения: камень падает несравненно быстрее пера. Чтобы
выделить влияние сопротивления воздуха, понадобилось почти две тысячи лет развития науки.
называют умственным, идеализированным, воображаемым, теоретическим.
Мысленный эксперимент – познавательная деятельность, где важное место занимает
научное воображение. Д.П. Горский называет мысленным экспериментом метод,
«позволяющий прибегнуть к отвлечениям, в результате которых создаётся
идеализированный объект (абстракция, идеализация)». С другой стороны мысленный
(воображаемый) эксперимент – умственный процесс, строящийся по типу реального
эксперимента и принимающий его структуру. Это вид теоретического рассуждения,
реализующий одну из основных присущих человеку функций – поиск новых знаний.
Эксперимент77, осуществляемый практически, есть вид материальной деятельности,
имеющий своей целью исследование объекта, проверку полученных знаний и т д. Всякий
материальный эксперимент предполагает выбор определённого объекта исследования и
определённого способа воздействия на него. Воздействие осуществляется в строго
воспроизводимых
условиях,
что
обеспечивает
воспроизводимость
результата
78
эксперимента .
Мысленный эксперимент, в свою очередь, развивается из реального эксперимента. На
каких-то этапах развития эксперимента субъект не отделяет осмысление его течения от
объективного хода экспериментального процесса. Позднее появляется способность
проделывать эксперимент как бы про себя, в уме, не воздействуя материально на сам ход
эксперимента. Это отражает характерную особенность сознательной человеческой
жизнедеятельности: прежде чем производить непосредственно, субъект мысленно решает
различные практические и теоретические задачи, совершает сложные и разнообразные
мысленные операции, предвосхищающие непосредственное действие.
Мысленный эксперимент – вид познавательной деятельности, в котором структура
реального эксперимента воспроизводится в воображении. Между мысленным и
материальным экспериментом имеется определённая аналогия. Такая аналогия –
существенная черта умственного эксперимента. Исследователь мысленно вводит изучаемый
объект во всё новые и новые взаимодействия, ставит его в разнообразные условия, постоянно
учитывая возникающие причинно-следственные отношения, пространственно-временные и
другие изменения, которые должны при этом совершаться в объекте, и соотнося их с
первоначальными условиями и связями. Изучаемое явление многократно повторяется в
различном составе и порядке. При этом в нём обнаруживаются новые, ранее неизвестные
свойства и стороны.
Одно из самых впечатляющих применений мысленного эксперимента – установление
основ квантовой механики. Самым крупным её оппонентом был Эйнштейн. Хотя ему самому
довелось приложить руку к формулировке квантовой теории (и даже получить Нобелевскую
премию именно за эти труды, а не за теорию относительности), он никогда полностью не
разделял её идей, считая квантовую теорию либо ошибочной, либо в лучшем случае
«истинной наполовину». Известно его изречение: «Бог не играет в кости». Эйнштейн был
77 Как неоднократно подчёркивал Н. Бор, в научном познании экспериментом мы обозначаем ситуацию, в
которой мы нечто наблюдаем, осознаём и можем, пользуясь языком, сообщить о наблюдаемом другому. Вопрос
только – какому «другому»: такому же, как мы, или иному? Этот аспект сформулировал Гейзенберг и соотнёс с
интеллектуальным диалогом, который возник у него с Эйнштейном в конце 1920-х годов по поводу
методологических особенностей возникающей тогда квантовой теории. Как вспоминает Гейзенберг, Эйнштейн
подчеркнул своё понимание наблюдаемости так: «лишь теория решает, что наблюдаемо, а что нет». Таким
образом, в глазах Эйнштейна – а его точку зрения принял и Гейзенберг – принцип наблюдаемости не является
чисто эмпирическим. Однако, наверное, было бы неверным трактовать его и как только теоретически
нагруженный. Смысл этого принципа – в диалоге теории и практики.
78 Идея воспроизводимости долгое время считалась основой науки. Поэтому столь революционной стала
квантовая механика, показавшая: даже в идентичных условиях результаты эксперимента могут отличаться.
Понадобилось множество натурных и мысленных экспериментов, чтобы уточнить само понятие
воспроизводимости.
убеждён: за квантовым миром с его непредсказуемостью, неопределённостью и беспорядком
скрывается привычный классический мир конкретной действительности, где объекты
обладают чётко определёнными свойствами, такими, как положение и скорость, и
детерминированно движутся в соответствии с причинно-следственными закономерностями.
«Безумие» атомного мира, по утверждению Эйнштейна, не фундаментальное свойство. Это
всего лишь фасад, за которым «безумие» уступает место безраздельному господству разума.
Эйнштейн пытался найти это фундаментальное свойство в нескончаемых дискуссиях с
Бором – наиболее ярким выразителем взглядов той группы физиков, которые считали
квантовую неопределённость неотъемлемой чертой природы, не сводимой к чему-либо
другому. Эйнштейн с завидным упорством продолжал свои атаки на квантовую
неопределённость, пытаясь придумать гипотетические («мысленные», как принято говорить)
эксперименты, которые обнаружили бы логический изъян в официальной версии квантовой
теории. Бор каждый раз отражал нападки Эйнштейна, опровергая его аргументы.
Особенно памятен один эпизод на конференции, где собрались многие ведущие физики
Европы в надежде услышать о последних достижениях новой тогда квантовой теории.
Эйнштейн направил свою критику против варианта принципа неопределённости,
устанавливающего, с какой точностью можно определить энергию частицы и момент
времени, когда частица ею обладает. Эйнштейн предложил необычайно остроумную схему,
позволяющую обойти неопределённость энергии – времени. Его идея сводилась к точному
измерению энергии с помощью взвешивания: знаменитая формула Эйнштейна E = mc2
сопоставляет энергию E и массу т, а массу можно измерить взвешиванием.
На этот раз Бор был обеспокоен, и те, кто видел, как он провожал Эйнштейна в
гостиницу, заметили, что Бор сильно взволнован. Он провёл бессонную ночь за детальным
анализом рассуждений Эйнштейна. И на следующий день, торжествуя, обратился к
участникам конференции. Развивая свои аргументы против квантовомеханической
неопределённости, Эйнштейн упустил из виду один важный аспект созданной им самим
теории относительности. Согласно этой теории, гравитация замедляет течение времени. А
при взвешивании без гравитации не обойтись. Значит, эффектом замедления времени
пренебречь нельзя. Бор показал: при надлежащем учёте этого эффекта неопределённость
восстанавливается на обычном уровне.
Примеры из истории высокой науки можно дополнить иллюстрациями вполне
обыденными. Ведь мысленный эксперимент возможен и во вполне бытовых условиях.
Например, я в своё время работал в районе Политехнического музея в Москве. Мне надо
было часто ходить через Старую площадь – к улице Солянке и обратно. В сквере две
дорожки – формально совершенно равноценные. Выбор между ними мог со стороны
показаться задачей в духе Буриданова осла. Но если сами дорожки одинаковы, то
автомобили вокруг сквера движутся по-разному: с одной стороны вверх по скату, с другой –
вниз. Достаточно поставить себя на их место, чтобы понять: машина, идущая вверх, газует
куда сильнее – значит, с дальней от центра стороны площади выхлопных газов значительно
больше, и гулять здоровее по дорожке, что ближе к центру.
Такое рассуждение выглядит достаточно простым. Но именно поэтому вести
мысленные эксперименты можно в любых условиях, почти непрерывно. Голова должна
работать постоянно. Тем более что тренировка в мысленном эксперименте ещё и очень
приятна.
Потренируемся? Итак, вопрос: «Произойдёт ли затопление материков, если в
результате глобального потепления все льды, плавающие в Мировом океане, растают?»
Первое ощущение – нет же никаких необходимых для решения задачи данных и слишком
большая неопределённость в формулировке задачи: «А сколько льда плавало? В каких
широтах? На сколько высоко поднялась температура нижнего слоя атмосферы?». На эти
вопросы ответа нет. Но чем больше неопределённость, тем больше свободы для мысленных
экспериментов. Поставим мысленно сосуд с водой, где плавает кусок льда, на весы. Пусть
стенки сосуда будут достаточно прочными, невесомыми и плотно прилегают к поверхности
чаши весов, а дно сосуда отсутствует – вода непосредственно опирается на чашу весов. Но
давление воды на чашу не изменится, даже если весь лёд в воде растает – ведь вес
содержимого сосуда останется прежним. Так что чаше безразлично, что происходит в сосуде
– плавает кусок льда или уже давно растаял. Значит, не меняется и уровень воды в сосуде,
так как давление пропорционально высоте столба воды.
На самом деле всё не так просто. Ведь Земля – планета, а не банка с водой и даже не
блин на слонах, китах и черепахе. Толщина мирового океана в разных частях планеты (и
даже в соседних областях одного и того же моря) отличается, тогда как в идеализированной
модели сосуда до дна в любой точке поверхности воды одинаковое расстояние. Земля имеет
ядро: оно, судя по всему, не находится точно по центру планеты, а напоминает яичный
желток. Вокруг не вполне круглой планеты Земля двигается ещё и Луна, влияя своим
тяготением на приливы и отливы. Так что если средний уровень воды в океанах Земли и не
изменится от таяния всех айсбергов, плавающих в них, то вследствие разницы в
распределении льда по поверхности воды могут быть затоплены значительные (прибрежные)
территории. Это пример изначально некорректно сформулированной задачи и
идеализированной модели, далёкой от практики.
Бигуди № 15
А как обстоит дело с песочными часами, то есть зависит ли их вес от того, течёт в них
песок или нет? А будет ли одинаковым вес закрытого сосуда, в котором или спят на стенках,
или летают мухи? Хотите, проведите эти эксперименты на самом деле. Но можно обойтись и
мысленными.22
Победное ликование
Усилия, призванные решить задачу, предполагают большое эмоциональное
напряжение, а достигнутый результат разряжает это напряжение, доставляя большую
радость. Архимед в восторге выбежал нагим на улицы Сиракуз. Иоганн Кеплер впал в
религиозный экстаз, увидев как под его пером открывается великая гармония Вселенной.
Дж. Дж. Томсон устроил весёлую пирушку для друзей и учеников после того, как был
экспериментально открыт электрон.
Учёные – те же люди, только более любознательные! Нобелевский лауреат академик
Лев Давыдович Ландау сказал: «Без любознательности нормальное развитие человека, помоему, немыслимо. Отсутствие этого драгоценного качества зримо при всяком столкновении
с куцым интеллектом…»
Другой Нобелевский лауреат – академик Виталий Лазаревич Гинзбург – выдающийся
физик-теоретик, один из близких сотрудников Ландау, всю свою жизнь посвятивший работе
во вроде бы узкой области теоретической физики – прославился в самых разных
направлениях своей науки, так что считается бесспорным универсалом. Не природная ли
любознательность – корень тому?
Бескровные перевороты
Научное творчество не терпит жёстких рамок, ограничений и догм. Устаревшие
понятия отбрасываются, сменяясь новыми: важна лишь истина, проверяемая опытом,
логикой, мыслью. Никакой доказанный и установленный факт не может быть отброшен,
даже если он противоречит сложившейся системе взглядов – напротив, должна быть
изменена эта система.
Вообще системы взглядов время от времени подлежат пересмотру уже потому, что
отражают не только достижения, но и предрассудки эпохи своего становления. А.М. Хазен
справедливо отмечает: «Классики науки получали результаты исследований в определённых
исторических условиях. Поэтому самые бесспорные и выдающиеся достижения человечества
необходимо всегда оценивать в контексте времени и обстоятельств, когда они были созданы.
Подобное утверждение в научной литературе нередко считают крамолой, канонизируя в
современных условиях прошлые результаты вне области их применимости. В результате
часто спор о научном существе работ заменяется злоупотреблением цитатами, оторванными
от авторского и исторического контекста».
Томас Кун в книге «Структуры научных революций», совершившей в своё время
переворот в науковедении, отмечает: систему взглядов – парадигму – никогда не
пересматривают плавно и постепенно. Даже после выявления фактов, бесспорно
противоречащих старой парадигме, она долго ещё может господствовать. И потому, что всё
ещё способна давать разумные результаты в пределах области своей применимости (правда,
ограниченность этой области становится всё очевиднее). И потому, что бесчисленные
видные учёные обрели авторитет и титулы именно на ней. Чтобы старая парадигма в
одночасье превратилась в смешную старину, надо не только накопить критическую массу не
вписывающихся в неё фактов, но и начать на фундаменте этих фактов строить парадигму
новую – сознавая, что рано или поздно ей тоже предстоит сходная судьба. Как говорится,
каждая истина переживает четыре этапа:
1. Что за бред?
2. В этом что-то есть…
3. Ну кто же этого не знает!
4. Кому нужна эта древняя глупость?
Правда, даже у безнадёжно устаревшей парадигмы остаются приверженцы – и те, кто
обязан ей своими успехами, и вполне бескорыстные79. Первооткрыватель квантовой
механики Макс Планк грустно говорил: новая идея никогда не побеждает старую в
человеческих умах – просто поборники старого постепенно умирают, а молодёжь с самого
начала учится новому80.
Впрочем, желающих следовать без сомнений и раздумий за «передовой наукой» всегда
предостаточно: когда в XVIII веке Парижская академия наук отрицала факт падения
метеоритов, многие музеи природных явлений выбрасывали из своих коллекций эти ценные
экспонаты.
В оправдание академиков следует отметить: одновременно с постановлением об
отсутствии камней на небе они решили не рассматривать проекты вечных двигателей и
решения трёх старинных задач – о квадратуре круга, трисекции угла, удвоении куба – при
помощи только циркуля и линейки без делений. В следующем веке найдены строгие
доказательства неразрешимости этих трёх задач. А уже в XX веке математически обоснована
невозможность вечного двигателя: энергия сохраняется, пока и поскольку законы природы
не меняются со временем81. Так что интуиция почтенных вождей французской науки
оказалась в большинстве случаев верна. Репутация учёного определяется не его ошибками, а
лучшими достижениями – так же, как спортсмена судят не по падениям на тренировках, а по
рекордам.
Между прочим, бывали задачи, относительно решения которых мнения ученых
расходились кардинально Вот пример такой задачи. Её приписывают Льюису Кэрроллу.
79 В Великобритании доселе существует общество приверженцев плоской Земли.
80 Как тут не вспомнить библейского Моисея, сорок лет водившего свой народ по пустыне, чтобы вымерли
все рождённые в рабстве. Впрочем, расхожий израильский анекдот говорит: Моше все эти годы искал в Аравии
место, где нет нефти. Но и такие поиски можно интерпретировать как заботу об избавлении от рабства –
нефтяного, столь знакомого нам по печальному опыту последних трёх десятилетий.
81 Немецкий математик Эмми Нётер доказала: любой симметрии – то есть неизменности свойств при какихто переменах – соответствует закон сохранения.
С двух сторон неподвижного лёгкого блока на равных расстояниях от пола на
перекинутом через блок канате (для простоты невесомом) висят обезьяна и равный ей по
весу груз. Обезьяна начинает карабкаться по канату. Что произойдёт с уравновешивающим
её грузом с другой стороны блока? Не кажется ли вам, что обезьяна и груз будут в любой
момент времени находиться на равных расстояниях от блока? (Законы сохранения, равенство
сил, то-сё…)
Бигуди № 16
А что будет, если висящие на одинаковом расстоянии от пола с двух сторон блока две
обезьяны начнут одновременно подниматься вверх, причём одна из обезьян поднимается со
скоростью, вдвое большей, чем другая? А если та обезьяна, которая движется быстрее, имеет
и вдвое большую массу, то которая из них поднимается быстрее?23
Знания и нравы
Сомнения в науке тоже должны опираться на научные, а не сторонние аргументы. Это
неоднократно доказано от противного – а иной раз от очень противного.
Так, немецкие физики Штарк и Ленард устраивали публичные собрания, чуть ли не
митинги, с требованием запретить «неарийскую» теорию относительности вместе с её
создателем. Борьба за арийскую физику заметно затормозила разработку германского
ядерного оружия. Так что весь мир имеет некоторые основания для благодарности
поборникам имперской идеологической чистоты. Но повлиять на законы природы
невозможно – и недопонимание независимости науки от идеологии тоже кое-что говорит об
этих «учёных»82.
Безусловно, человеку мало уметь творчески думать – необходима нравственная основа
всякого рода знаний и открытий. Выдающийся философ А. Маслоу, говоря о необходимости
для человека и человечества непрестанно расширять пределы познания, имел в виду такое
научное творчество, которое бы включало в себя любовь и сотрудничество, то есть опыт
переживания любви и сотрудничества, аккумулированный в науке как продукте совместной
деятельности, сотрудничества людей во имя совместной любви к истине. М. Полани также
пишет: «Научная жизнь также может быть исполнена страсти, красоты, надежды для всего
человечества и нести откровения относительно нравственных ценностей». Тысячи примеров
вдохновенного творчества в науке подтверждают эти слова.
Сложный социальный фонд знаний может передаваться и развиваться только с
помощью обширной когорты специалистов. Их ведущая роль приводит к тому, что их мысли
и чувства до некоторой степени разделяются всеми членами общества. Однако пониманию –
данному изначально или воспитанному долгими размышлениями – научить нельзя. Передать
можно лишь осознанные и апробированные в научном творчестве и собственной практике
принципы и методы творческого мышления. Естественно, в процессе продолжительного
формального обучения усваиваются и другие – более высокого порядка – интеллектуальные
навыки. Новые способности и возможности интеллекта – это структурные изменения,
результат специального «мозгового фитнесса», интеллектуального тренинга.
Не стоит, конечно, думать, будто появление способности делать умозаключения
означает некую безошибочность мышления. Увы, эта способность зачастую позволяет
обосновывать ошибки. Да, конечно, уметь думать – это хорошо! И тем не менее давайте
последуем призыву Блеза Паскаля – будем же учиться хорошо думать! Это ещё лучше.
82 Впрочем, и Штарк, и Ленард были к тому времени авторами многих бесспорно значительных работ.
9. Обучение: развитие мозга и интеллекта
– Мне вот так и не удалось по-настоящему углупиться! Не
хватило меня на это, – сказал Деликатес со вздохом, – Так я и
остался при высшем образовании…
Л. Кэрролл. «Алиса в Стране Чудес»
Учиться нужно, но нужно бы не только учиться, но и научиться учить. Грегори Бэйтсон
рассказывает такую историю:
– Правда ли, что взрослые, многоопытные родители умнее своих детей? – спрашивает
сын. – И поэтому дети обязательно должны учиться?
– Да, конечно, – отвечает отец.
– Но, папа, почему же тогда теорию электромагнетизма создал Джеймс Клерк
Максвелл, а не его отец?
Может быть, потому, что маленький Джеймс не только слушал, чему его учили, но и
старался думать сам?
Среда стимуляции
Гениальный самоучка Оливер Хэвисайд приходился племянником Чарлзу Уитстону –
изобретателю «мостика» для измерения электрического сопротивления. А Чарлз Уитстон
был дружен с Уильямом Томсоном (позднее лордом Келвином) и Майклом Фарадеем. Лорда
Келвина и Хэвисайда также связывала многолетняя дружба. Это наводит нас на мысль о
необходимости для изобретателя стимулирующей среды общения.
Увы, обычное, стандартное школьное образование почти никогда не ставит такой цели
– развить в человеке творческие навыки, нешаблонное мышление. Стандарт нестандарту
враг – и везде, где может, старается унифицировать, обезличить, обрезать.
Исходно природа в каждого вложила разум, талант интуиции, способность мыслить. Но
развить всё это до того уровня, на котором уже возможны иной, свободный и глубокий
взгляд на мир, осмысление и преобразование этого мира, творческое сотрудничество с
природой, человеку придётся самому83. Тут стоит только начать: чем больше новых мыслей
будет возникать в голове, чем больше способностей откроется, тем больше захочется знать –
и с новой энергией мысль будет двигаться всё глубже, всё больше будет открываться
возможностей. Настоящий снежный ком!
Да, в своё время ещё незабвенный Козьма Прутков сказал бессмертные слова:
«Специалист, как флюс, односторонен». И бывает так неловко наблюдать, как взрослый
человек, выдающийся программист, вэб-дизайнер или даже нанотехнолог, словом, крупный,
состоявшийся специалист в своём деле оказывается косноязычным и не умеющим публично
выступать человеком, не умеющим связать пару слов без шпаргалки…
Александр Лук иронизирует по этому поводу: стоит ли столько мучиться, чтобы узнать
так мало, как сказал приютский мальчик, дойдя до конца азбуки.
Но как заинтересовать человека, пригласить его идти по пути познания и
совершенствования самого себя? Сначала придётся ему сообщить, что знает он пока очень
мало. «Мы знаем 10 % того, что читаем, 20 % того, что слышим, 30 % того, что видим, 50 %
того что одновременно и видим и слышим, 70 % того что обсуждаем с другими людьми,
80 % того, что испытали на опыте, 95 % того, чему учим сами» – говорил У. Глэссер.
Немецкий философ Эдмунд Гуссерль был склонен требовать от знания такой крайней
строгости и точности, какая не встречается даже в математике. Биографы Гуссерля с иронией
вспоминают в связи с этим случай, произошедший с ним в детстве. Ему был подарен
83 Ну, разве что с помощью тех, кто знает, как научить и как научиться мыслить, как путём тренировки
может быть отточен и развит интеллект.
перочинный ножик, и, решив сделать лезвие предельно острым, он точил его до тех пор,
пока от лезвия ничего не осталось.
С. Льюис, высмеивая подготовку, которую давал Уиннемакский университет своим
питомцам, сообщает, что их обучили «санскриту, навигации, счетоводству, подбору очков,
санитарной технике, провансальской поэзии, таможенным правилам, выращиванию
турнепса, конструированию автомобилей, истории города Воронежа, особенностям стиля
Мэтью Арнольда, диагностике кимопаралитической миогипертрофии и рекламированию
универмагов».
А можно ли построить процесс обучения не конкретным вещам, не обращению с
предметами, но работе с самим собой, со своим мышлением? Можно ли человека научить
думать продуктивно, творчески? И когда следует начинать учить? А как учить?
Бигуди № 17
Известный мудрец и богослов Настор-ад-Дин Шейх Мухаммад Мансур – учитель
Омара Хайяма – говорил своим ученикам: «У саранчи пять способностей, но ни одного
таланта. Она бежит, но не быстро; ползает, но только по земле; летает, но невысоко; плавает,
но недолго; копает, но неглубоко. В каждом из вас тоже заложены способности. Сколько их?
Я пока не знаю. И моя задача – довести до совершенства…». Какую же задачу желал решить
мудрец?24
Детство творчества
Очевидно, начинать учить творческому мышлению надо, как минимум, со школы –
если это не начали делать ещё в детстве родители. Потому что, лишая любого человека
возможности в будущем творить, мы не только отнимем у него огромную долю радости
жизни, не дадим ему реализовать свои способности и стать счастливым, но лишим и
общество, цивилизацию чего-то, возможно, исключительного, выдающегося, неповторимого.
Ведь творческие способности присущи индивидуальности каждого человека ещё с раннего
детства.
К примеру, большинство современных ученых склонны считать синестезию
(характеризующуюся, в частности, восприятием одного и того же явления разными органами
чувств) свойством человека, но не патологией и не болезнью. Среди проявлений синестезии
– способность слышать свет («цветной звук»). Большинство новорождённых подобным
образом и воспринимают первое время окружающий мир. Такое же мироощущение имели и
древние, что немало стимулировало их мифологическое мышление. Творческие люди в той
или иной степени сохраняют синестезическую способность «запечатлевать», а не
запоминать.
Суматошная среда современной цивилизации мешает нам ощущать мир, как это умеют
дети. В древних Афинах все было несколько иначе. И там вовсе не собирались величайшие
умы эллинского мира. Почти все афинские гении сформировались на месте, в результате
социальной преемственности, общения друг с другом, благодаря тому, что понимание и
«спрос» на их творчество встречалось не только в кругу ценителей, но и со стороны народа.
Но никакие генетические данные не позволяют думать, что афиняне наследственно
превосходили окружающие их современные народы. Секрет весь заключался именно в
стимулирующей среде».84 А предпосылки гениальности определялись особостью
мировосприятия – по-видимому, теми самыми синестезическими свойствами древнего
человека, которые на благодатной почве и давали свои результаты.
84 Эфроимсон В.П. Предпосылки гениальности (Биосоциальные факторы повышенной умственной
активности) // Человек, № 2–6. 1997; № 1, 1998.
Откройте, например, знаменитую книгу К. Чуковского «От 2 до 5». Разве не вся
философия искусства – во фразе совсем ещё маленького человека, но уже большого творца:
«Я так много пою, что вся комната делается большая, красивая…». Попробуйте сами
придумать новые точные слова так, как способны малыши – не зря Чуковский назвал их
величайшими умственными тружениками нашей планеты. Только живой творческий ум
ребенка может переоткрыть слово «никчемный» или создать «обутки» и «одетки»,
«подхализу» и «кучело». Куда же с годами уходят эти самые «высокие качества детского
разума»? Почему мозги становятся, по выражению пятилетней девочки, недодумчивые?
Кстати, влияние детских писателей на взрослую жизнь мы сейчас наблюдаем в
громадном масштабе: капитализм в нашей стране построен в точности по детской
сатирической книге Носова «Незнайка на Луне».
Эйнштейн говорил: если ученый не может объяснить, что он делает пятилетнему
ребенку – значит, он шарлатан.
В современной педагогике существует понятие образовательных целей: они
упорядочивают формирование у ученика – то есть у маленького развивающегося человека –
знаний, навыков и умений. Понятно, знания многогранны: ведь это – факты, общие понятия,
причинно-следственные связи, принципы и правила, законы, закономерности.
Можно поставить целью образования просто хорошее усвоение различных
компонентов знаний. Но знания об одном и том же могут различаться качественно,
проявляться по-разному. Качество знаний можно характеризовать их полнотой и глубиной,
осмысленностью или осознанностью, оперативностью и гибкостью, системностью и
систематичностью, свёрнутостью и развёрнутостью, конкретностью и обобщённостью,
прочностью и т. д.
Профессор Т. Черниговская, исследуя становление речи у человека, сравнивает его
речевой аппарат с часовым механизмом: до определенного критического возраста
(примерно, лет до пяти) ребенок должен получить «инъекцию» речи, а затем – запустить эту
машинку речи и развивать свои лингвистические способности дальше. В этих словах ученого
просматривается одно важное, далеко «недетское» превращение (хотя, в рассматриваемом
случае и удающееся именно детям намного успешнее, чем взрослым) – переход количества
накопленной информации в качественный скачок в обучении. «Мне кажется, что мы почти
во всех науках дошли до предела – предела накопления данных, – говорит далее Т.
Черниговская. – А что с ними дальше делать? Нужен прорыв! Должен придти гений,
который посмотрит на ситуацию со стороны».
Педагоги всегда ставят перед собой цель повышения качества знаний – но, конечно,
процесс этот неопределённо длителен и на отдельных его этапах речь идёт об улучшении не
всех, а лишь некоторых показателей качества. На различных стадиях процесса познания с
разной степенью глубины и осмысления человек учится (и его учат) таким разным вещам:
✓ узнавать;
✓ распознавать;
✓ воспроизводить;
✓ объяснять;
✓ преобразовывать;
✓ переносить;
✓ строить;
✓ конструировать;
✓ создавать
✓ и т. д.
Конечно, современная теория обучения ставит и так называемые развивающие цели –
следующие частные задачи умственного развития:
✓ проводить умственные действия: вычленять, соотносить, схватывать сущность
(идею), и т. д.;
✓ уметь выделять признаки (свойства), а среди них, что особенно важно,
существенные признаки;
✓ переносить знания в изменённые ситуации;
✓ переформулировать задачу (условия, требования);
✓ находить и выделять вспомогательную задачу;
✓ овладевать мыслительными операциями: анализом, синтезом, обобщением,
классификацией, систематизацией;
✓ овладевать обобщёнными приёмами решения задач;
✓ овладевать алгоритмическими, эвристическими, алгоэвристическими методами
осуществления интеллектуальных процессов.
Самое здесь интересное – слово «овладевать». Как конкретно «овладевать»? Ведь
понятно: после «овладения» всё пойдёт хорошо – и творчество, и учёба, да и жизнь вообще.
Но как научиться это всё делать, с чего нужно начинать? Эти вопросы как-то осторожно
обходятся. А в них ведь вся суть творческого интеллектуального развития. Конечно, все
люди индивидуальны, но можно сформулировать общие принципы эффективного
мышления, пригодные для каждого человека – в том числе и ребёнка (об этом подробнее в
части 3).
В любом случае конкретные знания, полученные при обучении, должны быть не
столько самоцелью, сколько полигоном для развития мыслительных способностей.
Например, знаменитый физик Макс фон Лауэ [10] сказал: «Не так важно приобретённое
знание, как развитие способности мышления. Образование есть то, что остаётся, когда всё
выученное забыто»85.
Бигуди № 18
Посмотрим, как Вы образованы! Слушайте: «Они мастерски управляли своими змеями
и драконами и наводили ужас на всех своим презрением к смерти!» О ком здесь идёт речь? И
что за змеи и драконы? На эти вопросы по истории можно легко ответить, если ещё
вспомнить, что «они» грабили Париж и Гамбург, бывали в Константинополе, основали
королевство в Ирландии, разрушили Лондонский мост. И так далее.25
В современной педагогике задаются и примерные формулировки воспитательных
целей обучения:
✓ развитие нравственных и эстетических чувств учащихся (вызвать сопереживание,
сочувствие, чувство гордости, восхищения, радости, уважения, презрения, негодования и
т. д.);
✓ формирование оценок (сформировать оценку…, подвести к пониманию…, подвести
к выводу…, научить оценивать различные объекты с позиций научного мировоззрения и
т. д.);
✓ формирование взглядов (сформировать…; добиться…; усвоить…, подвести к
пониманию…; подвести к выводу…);
✓ развитие в сфере межличностного взаимосогласия (вступить в контакт, выразить
мысль, выразить согласие (несогласие), ответить, поблагодарить, присоединиться,
сотрудничать, принять участие и др.).
Как видим, в методах и схемах обучения всё предусмотрено. Но, к сожалению,
известно, куда ведёт дорога, вымощенная такими бумажными благими пожеланиями. На
85 У нас фразу «Знание – то, что остаётся, когда всё выученное забыто» традиционно приписывают
известному советскому физику Исааку Кушелевичу Кикоину. Похоже, все физики этого поколения общались
между собою так активно, что многие великие мысли стали в их среде «бродячими».
деле, несмотря на все эти теоретические положения, практические разработки и
дидактические рекомендации, ситуация в российском образовании (особенно в важнейшей
его компоненте – обучении творческому мышлению) остаётся неизменно плохой.
Следовательно, опасной для будущего. Говоря высоконаучным языком, прикладная
российская теория обучения до сих пор не решила общую проблему формирования у
учеников школ, гимназий и лицеев, а также студентов колледжей, вузов и университетов,
различных структур интеллектуальных процессов. Короче говоря, что-то запоминать
научила (или заставила), а думать самостоятельно и продуктивно – нет. А тут ещё и
пресловутый ЕГЭ86…
В ЕГЭ можно отметить правильно заложены цели: уравнять по возможностям
периферию м центром. Но… Гладко было на бумаге, да забыли про овраги. Принцип,
аналогичный ЕГЭ применял ещё мой покойный отец – заслуженный учитель, работавший в
провинциально школе и очень много сделавший для разработки и усовершенствования
методик обучения. Несколько опередив ГАИ, которое ввело затем «трафаретный» способ
испытаний, он использовал шаблоны, позволявшие быстро на каждом уроке проводить
фронтально опрос: раздавал карточки с несколькими вариантами ответов. Ученики в них
ставили крестики, после чего на карточки накладывался шаблон. И никому нельзя было
укрыться! Все знали, что они будут проверены. Высочайший уровень, огромное количество
побед на олимпиадах, поступление в центральные вузы – одна из сторон результативности
его работы.
Эта упрощенная методика была хороша для текущей успеваемости, но хуже всего,
когда её подвергают бездумной экстраполяции. Ведь составить достойную задачу гораздо
труднее, чем решить её. Сейчас эти задачи в ЕГЭ зачастую просто за уши притянуты.
Исследования ТАССИС показали, что наши школьники по знанию формул, находятся в
передовых рядах. А по связи с практикой? Если ты не можешь связать знание с практикой, то
это уже не знание, а просто информация. Вот в такие резервуары для накачки информации и
превратили наших школьников. Причем в ЕГЭ как раз и проверяется, насколько ты накачан
информацией! Более того, и эту информацию сужают: появились и специальные
направления по сдаче ЕГЭ.
В конце концов это ведет к своеобразной «расфасовке» сознания, превращении его в
оцифрованную матрицу. Ребенка с таким мозгом-матрицей пророчески предвидел Альберт
Эйнштейн в одном из своих высказываний: «Дети в конце концов начинают думать, что бог
– это газообразное позвоночное».
Воистину беда не приходит одна.
Есть в теориях обучения некие формальные основания, которые определяют: достигнут
в учении успех или нет. В частности, алгоритмы обучения (и учения) требуется создавать с
учётом по меньшей мере следующих факторов:
1) учебного материала (что я учу);
2) учебной цели (как хорошо я это делаю);
3) учебной системы (например, какой учащийся);
4) системы обучения (какой учитель, используется ли компьютер, какая учебная книга
и т. д.);
5) алгоритма обучения (насколько точен и эффективен метод обучения);
6) окружения (где я учусь).
Таким образом, учащемуся сообщают вплоть до мелочей все детали того, что он
должен делать, если хочет данный учебный материал выучить и запомнить. Задача такого
обучающего алгоритма формулируется просто: каждый ученик должен уверенно и
максимально быстро дойти от своей начальной ступени (относительное незнание учебного
86 (Единый Государственный Экзамен – уточняю я для будущего поколения на случай, если к моменту
прочтения книги эту аббревиатуру уже забудут, а сам экзамен упразднят «пользы Отечества для»).
материала) до цели (мастерство) с точно определяемым временем запоминания. Причём
признак мастерства – не просто правильные ответы, а правильные ответы по прошествии
определённого времени. И без запланированных повторений. Время запоминания заранее
устанавливается в цели обучения.
Эта теория так называемых динамических алгоритмов отличается от общепринятого
подхода в обучении («учить давать правильные ответы»), где считается: «учиться =
запомнить учебный материал», «не запомнил = не выучил». По данным специалистов (К.
Бунг) применение в обучении таких динамических алгоритмов способствует повышению
успехов в учении почти в три раза. Очень здорово! Только надо ещё подумать, что
подразумевается здесь под успехами в учении. Ведь теория динамических алгоритмов
ориентирована главным образом на преодоление забывания и прочное сохранение в памяти
изучаемого материала. Такие способы обучения, скорее всего, могут быть названы
образовательными технологиями. Но при чём тут развитие творческого мышления? Не
кажется ли вам, что, формулируя «теории обучения» и «критерии успешности обучения»,
специалисты из Минобразования больше думают и рассказывают нам о дырке от бублика,
чем о самом бублике?
Как выразился античный учитель о своем ученике: «Его была пустая голова, доверху
набитая знаниями».
Бигуди № 19
Попробуем сделать «бублик» из твёрдой стали и нагреем его над костром. Понятно, что
сталь расширяется, внешний диаметр «бублика» растёт. А будет ли отверстие в «бублике»
увеличиваться, уменьшаться или же останется прежнего размера? Что нам на это скажут
товарищи специалисты по обучению, как Вы думаете?26
Алгоритмы, эвристики – и их объединитель
На этот вопрос отвечает алгоэвристическая теория психолога с мировым именем Льва
Наумовича Ланды. В ней чётко сформулированы (и проверены на практике) основные
принципы создания творческого мышления. Она ориентирована именно на умственное
развитие учащихся, освоение ими познавательных процессов во всех когнитивных сферах.
Посвятим некоторое время рассказу об этой теории и о целой науке, называемой на
Западе ландаматика – в честь её создателя.
Л.Н. Ланда всю жизнь занимался научным и практическим развитием передовых идей
психологии, ориентированных на практику обучения. Основную проблему он
сформулировал в названии одной из своих книг: «Умение думать. Как ему учить?».
Разработка этой проблемы стала делом всей его жизни. Первое исследование он завершил,
когда ещё учился в аспирантуре. Затем молодой учёный стал активно работать и как
экспериментатор, и как теоретик психологии обучения, связав свои исследования с
возникшей в то время передовой педагогической технологией – программированным
обучением. По убеждению Ланды, именно она впервые открывает реальные возможности
для индивидуализации – говоря языком кибернетики, адаптации – обучающих воздействий.
Используя новейшие достижения в разных областях науки – психологии, дидактики,
кибернетики, философии, логики, семантики, медицины и др. – Л.Н. Ланда создал строгую и
стройную теорию адаптивного обучения. В её основе – раскрытие и анализ неосознаваемых
или плохо осознаваемых людьми механизмов мышления, обеспечивающих успешное
решение задач, моделирование выявленных механизмов в виде предписаний
алгоритмического, эвристического и смешанного типов, обучение на их основе87. Таким
87 Особенно – обучение тех учащихся, кто не в состоянии самостоятельно открыть для себя такого рода
механизмы.
образом, предметом обучения впервые становятся непосредственно сами механизмы
мышления. Создаются условия, при которых каждый из обучаемых может овладеть не
только содержанием знаний, но и процессами успешного оперирования ими, поверить в свои
возможности, приобрести уверенность в себе. Так учитываются и психологические
механизмы.
Лев Ланда впервые ввёл в практику понятие алгоритма умственных действий. Взятое из
математики, оно позволило придать описанию психических процессов ту строгость и
определённость, которая обеспечивает эффективное управление учением и позволяет
разумно выстроить как структуры учебных предметов, так и весь учебный процесс.
Управление не только внешними, но и внутренними (умственными) процессами позволило
создать новый вариант программированного обучения, существенно отличающийся от всего,
что имелось в те времена и в Советском Союзе, и на Западе.
Составная часть теории адаптивного обучения – формирование мыслительных
процессов с заданными свойствами. То есть предполагается выявить систему параметров для
того, чтобы, следя за ними, можно было изучать, как происходят мыслительные процессы у
каждого учащегося. В зависимости от результатов выбираются и корректируются
обучающие воздействия.
Ещё один важный раздел теории – диагностика умственных процессов, формируемых у
учащихся. Л.Н. Ланда ввёл понятие диагностики механизмов (структуры) психических
процессов – в отличие от диагностики их уровней, составляющей предмет классической
тестологии. Направленная на обнаружение нерациональных – фактически дефектных по
своей структуре – процессов мышления, диагностика механизмов служит основой для
динамической подстройки стратегии обучения каждого конкретного учащегося. Значит,
удастся быстро и вовремя устранить психологические причины неэффективности или
ошибочности его умственных действий.
Бигуди № 20
Обычные методики тестирования учеников с нахождением некоего среднего
коэффициента успешности обучения, чем-то напоминают следующую задачку: имеется куча
одинаковых кирпичей и линейка; как, сделав всего один замер, узнать длину диагонали
кирпича?27
В СССР известный учёный, профессор Л. Ланда создаёт и возглавляет научноисследовательский институт, его методы и предложения пользуются известностью. Но – в
1975 г. сын учёного женится на американке, и в том же году профессор Л.Н. Ланда при
очередных выборах на должность в институте утверждается лишь старшим научным
сотрудником. Далее всё привычно: профессора Ланду – автора важнейшего метода обучения
эффективному мышлению – фактически выталкивают из страны. «Такое мышление нам не
нужно!» – подход весьма привычный для заскорузлых чиновничьих мозгов (и по сей день).
Даже если речь идёт о человеке талантливом, выделяющемся своими способностями и
идеями. Не имеет значения, что эти идеи и предложения исключительно важны для
будущего страны. Любое «серое» общество и соответствующее ему правительство и прочие
«вертикальные» власти отлично характеризуются тезисом Сергея Довлатова: «Скудость
мысли порождает легионы единомышленников».
Так что в 1976 г. Л. Ланда уезжает в США. Там учёный продолжает развивать основное
направление своей научной деятельности: выявляет механизмы мышления профессионалов,
представляет эти механизмы в виде легко воспринимаемых моделей алгоритмического или
эвристического типа и строит методики обучения на их основе.
Постепенно создаётся новая научно-практическая область – ландаматика. Один за
другим в США и других странах стали открываться центры по обучению рациональной
умственной деятельности в самых разных сферах человеческой практики. Ландаматика
применяется для подготовки операторов ядерных установок, банковских служащих,
менеджеров, страховых агентов, работников налоговых служб, специалистов по написанию
различного рода инструкций и документов, аналитиков в области рекламы и многих других.
Внедрение ландаматики – эксперты называют её «работающим чудом Льва Ланды» –
приносит потрясающие результаты. Фирмы и корпорации экономят колоссальные средства,
ранее тратившиеся на подготовку и доучивание работников, а также на устранение
последствий их ошибочной деятельности, которой не удавалось избежать при традиционном
обучении88.
Каковы же исходные позиции автора ландаматики?
В первую очередь надо учить думанию (школы с этой задачей пока справляются
плохо). Думание – это не знание (или далеко не только знание). Думание, по определению
Ланды – то, что вы делаете с и над знаниями, т. е. трансформации и операции над знаниями.
Они выполняются с помощью специфических умственных действий. Ну, а что же
происходит в наших школах и вузах? Учителя и преподаватели не знают, как учить думать,
потому что:
1) они часто не осознают разницы между знанием и умственными действиями, и таким
образом учат преимущественно знаниям;
2) если же они и осознают эту разницу, то часто не уверены в тех специфических
умственных действиях, которые необходимы для решения проблем определённых типов;
3) в случае, если они уверены в специфических умственных действиях, то не всегда
знают, как учить проводить такие действия, как их формировать.
Л. Ланда говорил: «Общеизвестно, что ученики часто обладают знаниями в
определённой области, но не умеют решать задачи. Психологи и учителя часто объясняют
этот факт, говоря, что их ученики просто не знают как правильно мыслить, они не способны
приложить свои знания, в их мышлении отсутствуют, не сформированы принципы и
процессы анализа и синтеза».
Ланда рассказывает, как учитель математики сообщил ему об одном из своих учеников,
получившем «неудовлетворительно» на экзамене по геометрии. Учитель разводит руками:
«Он не знает, как надо думать. Он не может представить, что хорда может быть рассмотрена,
как сторона вписанного в окружность угла». На вопрос «А почему он не может это себе
представить?» учитель ответил: «Он не может этого представить потому, что просто не
может представить. Вот и всё». После этого замечания учитель считает, что проблема
исчерпана, и вопрос может быть закрыт. На самом же деле, отмечает Ланда, решение этой
проблемы только должно начинаться.
Технология обучения, разработанная Л. Ландой, и предназначена для решения этой
проблемы. Так, он экспериментально проверил алгоритмическую методику развития
процесса мышления при изучении геометрии. Для проведения геометрических доказательств
в старших классах были чётко выделены отдельные операции – этапы, необходимые при
построении доказательства (в стандартных методиках обучения ученикам просто сообщают
основные концепции и теоремы и приводят примеры решённых задач). Даже когда учителя
для проверочных работ отбирали лучших – всё знающих – учеников, они сами
констатировали: и эти избранные ученики не могут решать задачи и не имеют навыков и
знаний общих методик мышления. Не используя методик Ланды, средние ученики решали
около 25 % задач, лучшие – до 40 %. После обучения по методикам Ланды в следующем
88 В Business Week сообщается: использование ландаматики для тренинга сотрудников способствует
повышению производительности труда на 75 % и подъёму уровня качества на 90 %. Так, компании «Форд» и
«Дюпон» снижают на миллионы долларов расходы на проверку операций – качество продукции заметно
возрастает и уже не требует тщательного контроля. Всё потому, что создают эту продукцию люди с эффективно
работающей головой. Вот и весь секрет.
тесте все ученики решили 87 % задач!
Не знаю, явился бы следующий вопрос трудным для учеников Ланды, но для Вас,
надеюсь, ответ найдётся без труда: существует ли кривая, образованная из множества точек,
равноудалённых от одной точки-центра, но при этом эта кривая – не окружность?28
Подобный эксперимент проведен в классе, изучающем русскую грамматику. В этом
случае применение методик ландаматики позволило снизить число ошибок в 7 раз, а 4летний курс стало возможным освоить за 3 года, причём при более высокой успеваемости
учеников89.
Далее из отдельных умственных операций (действий) формируются системы,
организованные в интеллектуальные структуры. К таким структурам относятся:
алгоритмические, полуэвристические и эвристические. Ланда считает, что обучение готовым
алгоритмам – бедное обучение: надо учить самостоятельно открывать и создавать
алгоритмы. Однако не все проблемы и задачи можно решать по определённым алгоритмам.
Многие задания – эвристические (творческие) и требуют соответствующих эвристических
методов думания.
Какие же процессы алгоритмичны? Те, которые можно считать вполне определёнными,
регулярными и однородными. Представьте, что Вам нужно позвонить по телефону или
завести машину. В обоих случаях вполне можно составить детальную инструкцию для
пошагового её исполнения (алгоритм) и всегда выполнять эти операции заранее известным
образом. То есть алгоритмический процесс состоит из ряда относительно простых действий,
которые выполняются некоторым регулярным и универсальным образом в определённых
условиях для решения проблем определённого типа. Предписание (инструкция) описывается
в виде алгоритма.
Если процесс действительно алгоритмизуемый, с ним может по алгоритму управиться
любой. В фильме «Воздушные приключения» прусский офицер вынужден впервые в жизни
лететь на самолёте, пользуясь только инструкцией – но по-немецки подробной. Достаточно
сказать: первый пункт внушительного тома, вручённого ему помощниками, содержит всего
лишь слова «Сесть в кресло». Аккуратно листая инструкцию, бравый вояка долетает от
Лондона до середины Ла-Манша. Но тут пролетающая птица выбивает инструкцию из его
рук – и приходится идти на вынужденную посадку. Потому что офицер, дисциплинированно
справляющийся с алгоритмом, не обучен эвристикам90.
Эвристический процесс – тот, где присутствуют новизна, избирательность, развитие.
Простейшие примеры: нужно решить некоторую техническую задачу, когда для этого нет
подходящих орудий, или найти путь математического доказательства не по стандартной
схеме, а самому выбрать необходимые для доказательства элементы геометрической фигуры
или вспомогательные теоремы. Итак, эвристический процесс состоит из ряда
неэлементарных – заранее неизвестных исполнителю – действий или из операций
элементарных, но выполняемых нерегулярным, неуниверсальным образом. Однако важно,
что алгоритмизировать можно не только простые навыки: даже для сложных комплексных
навыков, имеющих эвристическую природу, можно создать некое алгоэвристическое
предписание.
Конечно, эвристические и алгоритмические процессы тесно взаимодействуют.
Созданная Ландой алгоэвристическая теория охватывает как алгоритмические, так и
89 Может быть, по этой методике удастся научить русскому языку даже многих российских политиков? Не
говоря уж о представителях так называемого «делового мира».
90 Впрочем, сомнительно, что эвристика со всей своей креативностью – включая ТРИЗ с её принципом
«заранее подложенной подушки» – ему хоть как-то бы помогла лететь дальше. Обычно разве что русский
компьютерщик, если ничего не помогает, читает инструкцию, которую следовало бы изучить прежде.
неалгоритмические процессы и обеспечивает формирование как тех, так и других91.
Разумеется, для эффективности обучения нужно подбирать соответствующие –
подходящие для данной области – алгоритмы и эвристики. Практически во всех ситуациях
можно эффективно использовать три основных типа алгоэвристических предписаний:
✓ тем, кто уже исполняет некое действие, нужно указать, что делать, чтобы выполнить
работу на уровне специалиста, мастера;
✓ тем, кто ещё учится, нужно указать, что делать, чтобы научиться выполнять работу
так, чтобы… (см. выше);
✓ тем, кто учит, нужно указать, как развить алго-эвристическую активность у
исполнителей и учащихся, чтобы они могли действовать… (см. выше).
Один из основных принципов искусства думать и рассуждать – так называемый
«принцип снежного кома» (the snowball method). Это метод проведения мыслительных
действий шаг за шагом:
1) сначала: описание и объяснение некоторой алгоритмической (или нет) процедуры и
демонстрация того, как она используется при решении задач;
2) затем: выделение и описание особенностей первой операции или действия, с
которого начинается процедура;
3) после этого: указание задач, для решения которых необходима именно эта операция,
и предоставление возможности попрактиковаться в её применении;
4) теперь снова: выделение и демонстрация особенностей следующей операции;
5) и опять: указание задач, для решения которых необходимо совместное
использование первых двух операций;
6) описание следующей операции;
7) указание задач, для решения которых нужны все три операции.
И так далее. Как снежный ком, растёт – слой за слоем – набор методов и операций, с
помощью которых можно разрешить любую задачу определённого класса. Причём
одновременно с расширением арсенала методов постепенно возрастает и сложность
решаемых задач.
Таким образом, при формировании мыслительных операций этот принцип развивает
способность воспринимать и выполнять несколько различных операций как одну целостную
операцию, мозг создаёт собственные системы из последовательных действий, объединяя их в
блоки.
Важнейшая особенность метода не в том, чтобы выстроить и продиктовать каждому
начинающему набор последовательных операций, когда все они заранее известны. Сущность
в том, чтобы научить человека таким способам думания, которые позволят эти
последовательные операции, решающие задачу, открыть, создать, разбить на части и
придумать чёткий и ясный алгоритм их применения. Причём это нужно делать в ситуации,
когда ни учитель, ни исполнитель ничего заранее не знают о таких процедурах и могут даже
не знать о существовании процессов, в которых они применяются92.
В гуманитарных науках также необходимы алгоритмические процессы, хотя здесь
соотношение между алгоритмическими и эвристическими процессами иное. Ланда
91 Когда-то маленький Блез Паскаль самостоятельно переоткрыл все основные теоремы геометрии Эвклида
– сначала эвристически изучил свойства предметов окружающего мира, а затем создал правила работы с ними и
выведения всё новых связей между этими свойствами (теорем). Правда, он называл отрезки прямых «палками»,
а окружность – «колесом». Но это уже неважно.
92 Например, при обучении программированию можно увидеть такую картину. Студент отлично знает
синтаксис языка программирования, знает, что означает каждый его элемент, оператор. Но из этих элементов у
него не получается единая программа. Преподаватель в отчаянии может капитулировать перед этим фактом
непонимания и неумения превратить количество в качество. Процесс обучения заканчивается констатацией
факта «студент вообще не умеет думать».
показывает: и писателю, и художнику необходимы в качестве базы алгоритмические,
полуалгоритмические, полуэвристические процессы.
Возможно создать и такие эвристические схемы (алгоритмы), которые станут
инструментами для обучения операциям творческого мышления, ведущим к развитию
творчества более систематическим, верным и быстрым путём. Сама система алгоритмов
(эвристик) и представляет собой модель тех процессов, которым нужно учить.
И главное – планомерно, спокойно и последовательно проводите анализ любой
ситуации. Логика, только логика и одна только логика!
Воссоединение элементов
Естественно, для развития эффективного мышления нужно сообщить (или иметь)
фундаментальные, базовые компоненты знания и представлять умственные операции, из
которых состоят процессы мышления. Они и являются психологическими «атомами» и
«молекулами» умственного развития. Эти «кванты мышления» должны начать работать,
преодолевая пропасть между знанием и его применением. Всем известно, что пассивное,
мёртвое знание «в одно ухо входит, в другое выходит». То, что не работает, мозг быстро
задвигает в дальний угол. Но процесс приобретения знаний можно соединить с обучением
умственным операциям и их применением. И вот тогда этот процесс становится очень
активным и целенаправленным.
Это легко наблюдать, на примере неприятнейших, но ценных для научного познания
существ – крыс. Лабораторные крысы, научившиеся всяким интеллектуальным штучкам во
время экспериментов, тем не менее, не могут часто решать элементарные задачи из обычного
«бытового» набора крысиного сообщества. А крысы «из природы» – «серенькие» во всех
смыслах – эти задачи решают с легкостью. Для нашего осмысления это очень серьезный
показатель. Суть в том, что лабораторные крысы живут в ограниченных условиях, с
ограниченными сигналами и контактами – они не получают полной гаммы нагрузок на свой
интеллект, их Ойкумена гораздо уже, чем в естественной среде обитания, и их интеллект
работает по более зауженной программе. А крысы-пасюки живут в реальном мире, полном
опасностей (человек травит, хищники преследуют, капканы расставлены и прочее).
Опасность со стороны других видов и конкуренция внутри гораздо выше. Решается много
задач, и эти задачи, несомненно, богаче и шире. Ведь реальная практика – самая богатая.
В искусственной лабораторией под названием «ЕГЭ» мы подвергаем школьников
похожему процессу.
Хочется напомнить: фантазия, воображение, интуиция, т. е. то, что в первую очередь
обеспечивает творческое озарение, прорыв, связано главным образом с функциями правого
полушария мозга. Вот эти качества промоделировать, алгоритмизовать весьма трудно.
Но можно. Есть даже математическая основа для такой работы. Ещё в 1985-м году
американский математик Лофти Заде создал теорию нечётких (их ещё называют размытыми)
множеств. Сейчас этот раздел математики весьма моден. Даже юристы пытаются его
приложить к неким статистическим исследованиям. Уж не говоря об экономических,
экологических и прочих науках. Тех, где чёткая фиксация множества объектов затруднена,
где расплывчаты границы «рабочего поля», в котором мозг пытается найти логические связи
между элементами. Но даже такие структуры мыслящий мозг математика сумел укротить –
создать алгоритм их описания и анализа. Именно с такими множествами имеет дело мозг в
процессе творческого поиска. Умение с ними работать – основа креативных способностей.
Такое умение можно тренировать и развивать. Это и утверждает ландаматика. Об этом
говорит методика «brain-based learning». Таковы основные положения теорий Гарднера и
Стернберга и многих других специалистов. Именно это я прочувствовал на собственном
творческом опыте.
10. Обучение творчеству: общие принципы
– А какие у вас ещё были предметы? – спросила она.
– Ну, конечно, Истерия, – отвечал Деликатес, загибая лучи на
своих плавниках, – Истерия, древняя и новейшая, с Биографией.
Потом… раз в неделю приходила Мурена. Считалось, что она нас
учит Рисковать Угрём и прочей муре – ну, там, Лживопись, НатюрМорды, Верчение Тушею…
Л. Кэрролл. «Алиса в Стране Чудес»
Есть заповедь, своеобразно связанная с креативным подходом: создавая своё
окружение (community), ты изменяешь своё существование (Дж. Зипс). Этот принцип
активно используется в процессе обучения творческому мышлению (см. выше).
Предлагаемые, например, ландаматикой способы развития мышления могут эффективно
сочетаться с элементами интерактивного обучения.
Погружение в обучение
Например, в методике так называемого «оркестрованного погружения» обучающая
среда (то самое community) специально выбирается для «погружения» в неё учеников или
студентов. Неопытный ещё, начинающий преподаватель биологии изобразит дождливый лес
в классе, населит его живущими там животными, создавая всё это из бумаги и картона.
Учитель более высокого уровня отведёт учеников в школьный лес, чтобы обнаружить и
идентифицировать следы животных на снегу, заодно научит ориентироваться по компасу,
разводить костер. Но самый лучший учитель биологии или географии устроит экскурсию на
целый день. Учитель астрономии будет объяснять ощущение невесомости в плавательном
бассейне. Ощущения зрительные, тактильные, возможность прикоснуться, сравнить,
изменить своими руками – замечательные «входы» для информации, пробуждающей
мышление.
Рассказывают, как в студии одного художника шёл спор: что такое настоящее
искусство. Присутствующий тут же Шаляпин участия в споре не принимал, а слушал молча.
Через некоторое время он вышел незамеченным. Вдруг дверь мастерской открылась, на
пороге стоял Шаляпин:
– Пожар! – крикнул он.
Поднялась паника, а Шаляпин присел к столу и, как ни в чём не бывало, заметил:
– Вот это и есть настоящее искусство, а то, что вы спорили – сущие пустяки.
Выше уже говорилось: все люди различны по типу и стилю мышления, способностям,
обучаемости. Но вполне возможно создать такую обучающую среду, в которой их
мыслительный потенциал достигнет максимума. Для этого необходимо, чтобы обучающая
обстановка и все происходящие в ней процессы были совершенно безопасны. Дело в том, что
активное мышление обусловлено работой одной из частей мозга (неокортекса – новых, т. е.
образовавшихся на недавних этапах эволюции, частей коры), которая в условиях стресса или
боязни попросту «тормозит»: основная нагрузка перекладывается на более примитивные
части мозга, где не вырабатываются новые решения, а используются уже готовые – это
быстрее. Поэтому там, где возникает опасность, появляются сбои и ошибки. Конечно, «на
каждый глюк можно найти патч»93. Так вот, для отсутствия мозговых сбоев лучше всего
сразу же обеспечить удобную и спокойную атмосферу, в которой проходит обучение.
Таким образом, мы приходим к созданию некой комплексной технологии
алгоэвристического обучения, сочетающей все наиболее эффективные принципы развития
93 В переводе с жаргона программистов – каждый сбой можно устранить, подбирая соответствующее
«лечение», для каждой прорехи можно подобрать заплатку.
творческого мышления. В ней, помимо весьма действенного «принципа снежного кома» Л.
Ланды, нужно внимательно отнестись и к созданию обучающей среды, также
стимулирующей индивидуальное творчество. Например, чтобы эту индивидуальность
выразить, хорошо бы каждому учащемуся обеспечить отдельный маленький мир (рабочий
стол, доска, компьютер, хорошее освещение и т. д.), своё «творческое пространство». Но при
этом нужна и «кают-компания», где можно думать и работать вместе.
Причём место работы и размышления должно быть удобным! Многие знают, что
великий Лев Ландау предпочитал заниматься теоретической физикой не за столом, а лёжа на
диване. Как вы думаете, если бы ему запретили это делать и строго указали, где и как надо
сидеть, много ли придумал бы мозг, «выведенный из себя»? А если вы учите думать (а
значит, играть) маленьких детей, вспомните, насколько интереснее в детстве было под
столом!
Бигуди № 21
Поговорим ещё немного о детях. Представьте себе двух математиков, не достигших
пенсионного возраста (т. е. 60 лет), которые встречаются после долгого перерыва. Вот
фрагмент их диалога:
– Ну, а дети у тебя есть?
– Три сына.
– А сколько им лет?
– Если перемножить, будет как раз твой возраст.
– (После размышления.) Мне этих данных недостаточно.
– Если сложить их возраст, получится сегодняшнее число.
– (Вновь после размышления.). Всё ещё не понимаю.
– Кстати, средний сын любит танцевать.
– Теперь понял!
А у Вас получится так же быстро определить возраст каждого из сыновей? Учтите:
ответ вполне однозначный. И ещё: попробуйте метод перебора, для ускорения процесса
подскажу: математик старше Остапа Бендера.29
Подстройка под личность
Известно, что человек лучше понимает и запоминает те предметы, процессы или
информацию, которые воспринимает как полезные, удобные, приятные, нужные. То есть при
развитии мышления важно создать правильную эмоциональную установку – вообще говоря,
индивидуальную. От качества и расширения круга положительных эмоций (или от сужения
круга негативных) зависит и качество креативного мыслительного процесса (Б. Фредриксон).
Другими словами, креативные творческие процессы – сложная работа ума, при совершении
которой мозг сам отбирает нужную информацию в соответствии с задачей и конкретным
моментом времени. Этому моменту должны соответствовать и эмоции. Между прочим, роль
положительных эмоций и их уникальное воздействие на человека ещё недостаточно
учитываются для расширения мыслительных возможностей мозга.
Как пишет выдающийся исследователь Н.П. Бехтерева: «…большая радость и особенно
большая печаль могут нарушить нормальное течение мыслительных процессов, а в ещё
более яркой форме это происходит при болезненных эмоциональных реакциях и состояниях.
Дело в том, что в таких случаях физиологические сдвиги – здесь, пожалуй, уместно сказать
«грубые сдвиги» – происходят во многих зонах мозга. Это, естественно, приводит к
изменению свойств этих зон, их возможностей, вплоть до затруднения выполнения какойлибо работы, причем прежде всего мыслительной, т. е. той деятельности, которая, как
отмечалось выше, тем богаче, чем больше мозг может отпустить ей своих потенций…».
Фактор индивидуальности исключительно важен и в случае совместного творческого
поиска. Дело не только в том, что один лучше воображает и фантазирует, а другой мыслит
рационально и последовательно. Дело в личностной конкуренции, способной даже убить
оригинальность и креативность мышления: чтобы не рисковать, не прослыть невеждой или
болтуном, человек промолчит, не выскажет идею, все пройдут мимо нового и интересного
«ответвления» мысли.
Кстати, в творчестве должно найтись место и для людей с быстрым мышлением, и для
«тугодумов». Креативное мышление нельзя включить или выключить – требуется время,
чтобы успеть подумать и пофантазировать, чтобы поймать и представить всем свою идею.
Именно – свою, потому что человек творческий должен иметь собственную,
независимую, подчас даже парадоксальную точку зрения. Она может в будущем даже
совпасть с видением коллег, друзей, сослуживцев, но ни в коем случае нельзя поддаваться
лени в настоящем, следовать путем наименьшего сопротивления сейчас, нельзя действовать
по шаблону и отказываться от независимости мышления.
Бигуди № 22
Давайте, например, вместе подумаем, как помочь выжить человеку из известного
рассказа Шекли «Человек по Платону». Лагерь на неизвестной планете. Из всей экспедиции
в лагере на несколько дней остаются космолётчик Холлорен и робот Макс, охраняющий
лагерь от проникновения всего опасного неземного. Холлорен выходит за пределы лагеря
пройтись, робот уточняет: «А вы знаете пароль, мистер Холлорен?» Летчик отвечает: «Знаю,
Макс. А ты?» «Мне он известен, сэр» – отвечает робот. Вернувшись, Холлорен узнаёт, что
помнил старый пароль, а робот получил от командира экспедиции, вылетевшего в
длительный поиск, новый! Для робота знание пароля – единственное доказательство того,
что существо, поразительно напоминающее мистера Холлорена – землянин, и его можно
пустить в лагерь. Точнее, Макс знает: «земляне всегда знают пароль; неземляне никогда не
знают пароля, но всегда пытаются проникнуть в лагерь; существо, которое не пытается
проникнуть в лагерь, можно рассматривать как землянина – до тех пор, пока оно не проявит
настойчивое неземлянское желание туда проникнуть, не зная пароля». Да, ещё вводная: на
планете очень жарко, без воды долго не протянуть, вода только в лагере. Испробовав
множество вариантов проникновения в лагерь (лагерь нужно срочно переносить в связи с
грозящим землетрясением; я – инопланетянин и пленник Холлорена, которого нужно
немедленно напоить и др.), космолётчик придумал способ проникнуть в лагерь.
Вернувшиеся из поиска коллеги нашли его в лагере, без сознания. При этом робот Макс
утверждал: в лагерь никто извне не проникал, по его словам, Холлорен «просто вдруг уже
был там». Как это произошло?30
После того, как мы узнали о разном стиле поведения и деятельности двух полушарий
мозга, ясно: в процессе обучения креативным навыкам и приёмам необходимо «разминать»
обе части мозга. Поэтому и задачи, которые ставятся в процессе мышления, должны быть
совершенно разными. Когда ясно, что ученик знает, что помнит, чем интересуется, можно
уже показывать, как анализировать информацию, разбивая её на части, следуя различным
критериям, находя аналогии, противоречия, логические связи. Если не рассказать, как
устроен и работает мозг, что в нём происходит, когда человек мыслит, будет гораздо труднее
развивать навыки творческого мышления.
Комплексная тренировка
Для каждого человека процесс своего собственного обучения – тоже творчество, и
нужно дать каждому возможность создавать самого себя. То есть это всё тот же «фитнесс
мышления». Кстати, приступая к накачке мускулов, каждый человек тоже вначале
прикидывает, на что вообще он способен, и изучает, какие конкретно мышцы развиваются
при данном типе упражнений. Хорошо, когда при занятиях лёгкой атлетикой или плаванием
одновременно тренируются и различные группы мышц, и дыхание, и воля. Неплохо и когда
тренировка абстрактного мышления одновременно означает выработку опыта повседневной
деятельности, приложения на практике получаемых и вырабатываемых знаний.
Помните принципы ландаматики? Там ведь также приобретаемое умение мыслить
проверяется и используется в конкретных делах – неважно, доказательство теорем это или
изобретательская деятельность. И, знаете, важно научить не бояться ошибок: наиболее
ценные жизненные уроки приходят именно после пересмотра и исправления ошибок94. Куда
важнее уметь так хорошо мыслить, чтобы суметь эти ошибки исправить95. А по мере
развития мышления удастся вообще их не допустить «на автопилоте», не заботясь об этом
особо.
Пусть каждый, кто учится мыслить, научится фиксировать информацию в разных
формах. Можно писать, проговаривать, зарисовывать96. Можно тренировать свою память и
её способность «выдавать» нужную информацию в нужный момент.
Бигуди № 23
Например, попробуем расшифровать эпонимы (слова, происходящие от фамилии
человека, который изобрел, применил или изготовил указанный предмет) по следующим
определениям.
Очень жестокий, по имени греческого законодателя VII века до н. э.31
Вид свитера, по фамилии британского генерала XIX века.32
Духовное чувство, по имени греческого философа.33
Роскошные пиры, по фамилии римского военачальника I века до н. э.34
Хитрый и лживый, по фамилии флорентийского государственного деятеля XV–XVI
веков.35
Ну-ка, эрудиты, не осрамимся перед самим Анатолием Васерманом!
Публичная тайна
Впрочем, иной раз цель сложных ассоциаций – сокрытие информации. В старину
учёный, не уверенный в достоверности своего открытия, публиковал его зашифрованным97.
Если открытие подтверждалось, он сообщал ключ к шифру и тем самым доказывал свой
приоритет. Если выявлялась ошибка, репутация не страдала – ведь никто не знал, какой текст
94 Правда, Бисмарк говорил, что на своих ошибках учатся только дураки, а он сам предпочитает учиться на
чужих. Но он явно хвастался: своих ошибок ему тоже хватало для полноценного курса обучения.
95 Талантливый математик А. Безикович сказал: «Репутация математика определяется числом
предложенных им плохих доказательств». Он по собственному опыту прекрасно знал, что первооткрыватели
всегда неуклюжи: специалисты не столь творческого склада довели до совершенства доказательства
нескольких десятков открытых им теорем.
96 Меня часто спрашивают, в чём смысл какого-нибудь моего рисунка, какую информацию он содержит,
почему выполнен в данной форме или технике. И я попадаю в неловкое положение: сказать правду журналисту
или посетителю моей выставки было бы неделикатно. Правда же – в том, что мне просто не нужно было бы
рисовать, если бы я мог словами описать смысл рисунка. Я выбираю именно ту форму, в которой могу передать
– говоря словами Полани – личностное, неартикулируемое знание. В какой-то мере это и личное творчество. Я
нахожу способ передачи своих мыслей и чувств, который мне ближе и понятнее. Я так «записываю» свои
размышления и ведения о мире, людях, событиях. А для кого-то более запоминающимся может быть
ассоциирование информации с мелодией или набором танцевальных па. Но если моё творчество находит
отражение у зрителей, если мои выставки смотрят – значит, выбранная мною форма соответствует содержанию.
97 Выше приведен пример такого шифра – от самого Галилея!
опубликован!
Самый известный из таких научных шифров принадлежит Нострадамусу. Лишь в наши
дни доказано: его легендарные центурии – прогноз развития обстановки в Европе в случае
победы протестантизма в Испании, величайшей в ту пору католической державе. Понятно,
публиковать такой прогноз открытым текстом во Франции, охваченной религиозными
войнами, было по меньшей мере небезопасно.
Одним из последних официально использовал этот приём Роберт Гук. Он на
протяжении большей части своей научной карьеры ожесточённо боролся за приоритет.
Учёный секретарь Королевского общества был обязан еженедельно демонстрировать
академикам новые физические опыты и явления. Естественно, коллеги – включая президента
общества Исаака Ньютона – охотно пользовались находками своего главного ассистента и
творчески развивали их уже от своего имени. Поэтому, открыв закон упругих деформаций,
Гук опубликовал его в виде анаграммы высказывания на латыни: «Ut tensio sic vis» (каково
удлинение, такова и сила).
Между прочим, не всякую информацию – или её существенную часть – удаётся скрыть.
Иногда её выдаёт сам человек – своими эмоциями или действиями. Н.П. Бехтерева
напоминает о мудрости гувернантки, чьи воспитанники притихли в соседней комнате: «Дети,
сейчас же перестаньте делать то, что вы делаете!»
Плечи гигантов
Кстати, о Гуке и Ньютоне. Поскольку оба они работали над одними и теми же
проблемами, приоритетные споры между ними были особо остры и в конце концов перешли
в откровенную вражду. Поскольку Гук умер первым, его портреты до нас не дошли: Ньютон
лично позаботился об их розыске и истреблении. Известно только, что Гук был
низкорослым: знаменитая фраза Ньютона «Если я видел дальше других, то только потому,
что стоял на плечах гигантов» содержала ехидный подтекст, очевидный для современников.
Правда, Ньютон несомненно прав в том, что любое творение опирается на все
предшествующие достижения разума. Гюстав Лебон писал:
«Историки, обыкновенно очень односторонние, всегда полагали, что можно написать
под каждым изобретением имя одного человека; и однако среди великих изобретений,
преобразовавших мир, каковы книгопечатание, порох, пар, электрический телеграф, нет ни
одного, относительно которого можно было бы сказать, что оно создано одной головой.
Когда изучаешь происхождение подобных открытий, то видишь, что они родились из целого
ряда подготовительных усилий: окончательное изобретение есть только венец. Наблюдение
Галилея относительно одновременности колебаний подвешенной лампады подготовило
изобретение точных хронометров, откуда должна была последовать для моряка возможность
верно находить путь в океане.
Паровая машина представляет собой сумму целого ряда изобретений, из которых
каждое требовало громадных трудов. Грек, будь он во сто раз гениальнее Архимеда, не мог
бы выдумать локомотива, ибо для выполнения такой задачи ему нужно было бы ждать,
чтобы механика осуществила успехи, которые потребовали двухтысячелетних усилий».
Искусство для творца
Для процесса творчества может быть создана разная обстановка. Понятно, что она
должна быть безопасной, удобной. Кстати, доказано: на процесс запоминания, усвоения и
анализа информации хорошо влияют различные запахи (ваниль для расслабления, мята для
обострения чувств и пр.).
Большую роль играет при формировании мыслительных навыков музыка – она
стимулирует или расслабляет слушателя, помогает индуцировать создание мозгом
различных образов, улучшающих запоминание и обучение. Музыка способна эффективно
снизить напряжение, ослабить стресс. А значит, может ускорить обучение эффективному
мышлению. Причём музыка действует и как своеобразный кодировщик информации
(мелодия или ритм помогают её структурировать и увязывать между собой её части), и как
«энергетическая подпитка» эмоций и организма. Но музыка также и специфический
инициатор мышления – источник энергии для работы мозга.
Знаете, как музыка помогает учёбе? В одном из известных американских университетов
провели эксперимент. В одной из групп третьекурсников еженедельно давали уроки пения,
слушали классическую музыку. После года таких занятий более 80 % студентов заметно
повысили уровень пространственно-временного мышления, выросли их успехи практически
по всем изучаемым предметам. Музыка, действуя некоторым раздражающим (в научном
смысле слова!) образом, побуждает к действиям, возможно бессознательным, «проясняя,
очищая психику, раскрывая и вызывая к жизни огромные и до того подавленные и
стеснённые силы… она раскрывает путь и расчищает дорогу самым глубоко лежащим
нашим силам; она действует подобно землетрясению, обнажая к жизни новые пласты» (Л.С.
Выготский). Музыка – часть искусства, которое учит и помогает творчеству, само являясь
его результатом.
Мир без искусства – это мир и без науки. Художественное творчество, неизменное и
неустанное желание человека изображать – и тем самым изучать и изменять своим
прикосновением – Мир развивает и научный взгляд, предназначенный для записи
эмпирических наблюдений за природой. Артист, музыкант или художник неоднократно
возвращается к своей пьесе, идее, картине для пересмотра и совершенствования. Так же и
учёный пересматривает свои формулы или данные экспериментов для того, чтобы заметить
изменения или проверить контрольные гипотезы.
И художник, и учёный нуждаются во вдохновении для работы. Оба они зависят от
эвристической природы исследования, приводящего к открытию. Значит, художественное
мышление не просто тесно связано с научным – они взаимоопределяют и поддерживают
друг друга.
Концепцию реальности, на которой базируется наука, можно распространить и на
искусство, если ввести (Э.М. Форстер) различие между «плоскими» и «телесными»
действующими лицами. Действия «плоского» персонажа почти полностью предсказуемы, а
вот «телесный» способен «убедительно удивить» читателя.
Искусство соединяется с наукой в жизни многих учёных.
Что касается процессов мышления и его результатов, то каждый читатель, наверняка,
уже не раз встречался с выражением «красивое решение», «красивый ответ», что также
роднит данную тему с искусством.
Вот, например, математика – язык символов, созданный человеком для отображения,
как ему казалось, объективных закономерностей природы, то музыка – как раз язык
отображения и передачи закономерностей субъективных, то есть прежде всего эмоций,
чувств человека. Древние говорили: «Музыка – искусство интонаций» – и это безусловно
так: сила воздействия музыки опирается на всю мощь нашего подсознания, на пришедшие
нам от предков инстинкты самовыражения и реакции на непроизвольные эмоции других. Не
зря церковный орган так близок по тональности к голосу человека! И хоры певчих не зря
звучат под церковными сводами. Интонации – ключ к подсознанию человека.
Современные приложения теории творчества опираются не только на логические и
обозначенные некогда принципы из области сознательного, но и наработки психологов о
бессознательном – отсюда принцип «страдания и удовольствия», «навязчивого образа»,
принцип красоты, гармонии, наконец.
К взаимопересечениям чувственного и логического ученые обращались во все времена.
«Без веры во внутреннюю гармонию нашего мира не могло быть никакой науки. Эта вера
есть и всегда останется основным мотивом всякого научного творчества», считал Альберт
Эйнштейн.
Хорошо известно, что Леонардо да Винчи был в равной мере гениальным и
художником, и учёным. Менее известно, что Галилео Галилей помимо своих весьма
продуктивных занятий естественными науками увлекался и рисованием, и музыкой (это и не
удивительно для сына создателя жанра оперы). Мало кто знает, что Исаак Ньютон, будучи
ещё ребёнком, покрывал стены комнаты своими рисунками. Когда он подрос, то построил и
смастерил множество механизмов, чтобы проверить, как действуют в реальности ранее
нарисованные им предметы. А где была бы сегодня вся наша наука, если бы скрипка не
спасала самолюбие Альберта Эйнштейна, когда учитель бранил его за неуспехи в учёбе?
Способность художественного мышления находить новые приложения возникающих
концепций подтверждает пример изобретателя парохода – инженера Роберта Фултона. Когда
он обучался живописи под руководством Бенджамина Уэста в конце XVIII века в Англии, то
изучал и прикладное искусство. В ходе изучения он узнал, как работает ткацкий станок.
Способность наблюдать и подмечать детали, развитая в ходе обучения искусству, стала
основой его работы в будущем.
«Кто в океане видит только воду – тот на земле не замечает гор» – пел Владимир
Высоцкий. Гениальный английский физик Оливер Хэвисайд играл на арфе – при всём при
том, что страдал глухотой. В 1922-м году в одной из датских газет появилось сообщение:
«Известный футболист, вратарь сборной Дании, любимец публики Нильс Бор получил
Нобелевскую премию». Чуть позже выяснилось: премию он получил по физике. С другой
стороны, Брамс сочинял самые лучшие мелодии, когда чистил обувь.
Великий физик, один из творцов квантовой электродинамики, Ричард Фейнман с
детства считался начисто лишённым художественных способностей. Тем не менее он – уже
на пятом десятке лет! – стал учиться технике рисования. И довольно скоро его картины
раскупались не хуже творений признанных мастеров. Правда, не за миллионы долларов – но
за немалые тысячи. По меркам США – более чем успешно. Подписывал он свои труды
псевдонимом Офей – не хотел, чтобы на зрителей давил его авторитет нобелевского
лауреата. Так японские мастера искусств, достигнув высшей славы, меняли псевдонимы,
чтобы проверить, ценит ли аудитория их нынешнее искусство или просто заворожена
былыми достижениями. Конечно, технические навыки, обретённые усердной тренировкой,
только помогли блистательному и разностороннему таланту Фейнмана проявиться в новой
сфере. Но вряд ли на свете найдётся человек, вовсе лишённый творческих способностей.
Тренируйтесь – и рано или поздно они тоже проявятся!
Причём тренироваться можно и нужно везде.
Бигуди № 24
Вот вы идёте принять ванну и пытаетесь её наполнить, открыв оба крана. Но вы
увлеклись размышлениями о творчестве – и забыли закрыть слив! Обычно, чтобы наполнить
ванну горячей водой, требуется 8 минут, а холодной – 10 минут. Полная ванна вытекает в
слив за пять минут. Сколько времени вам надо, чтобы заполнить ванну при открытом сливе?
Краны одинаковые, и оба работают хорошо!36
С самых древних времён (эллинская культура, Римская империя) искусство
рассматривалось как часть и средство воспитания – длительного изменения нашего
поведения и нашего организма. Так считал известнейший психолог Л.С. Выготский, и с этим
трудно спорить. Приведём ещё одну обширную цитату из его труда «Психология искусства»,
ибо трудно лучше сказать о роли искусства в развитии творческих способностей личности.
«Восприятие искусства требует творчества, для восприятия искусства недостаточно
просто искренне пережить то чувство, которое владело автором, недостаточно и просто
разобраться в структуре самого произведения – необходимо ещё творчески преодолеть своё
собственное чувство, найти его катарсис, и только тогда действие искусства скажется
сполна… Искусство… никогда прямо не порождает из себя того или иного практического
действия, оно только приуготовляет организм к этому действию. Фрейд говорил, что
испуганный человек, когда видит опасность, страшится и бежит. Но полезным является то,
что он бежит, а не то, что боится. В искусстве как раз наоборот: полезен сам по себе страх,
сам разряд человека, который создаёт возможность для правильного бегства или нападения.
…Существеннейшая особенность человека, в отличие от животного, заключается в том,
что он вносит и отделяет от своего тела и аппарат техники, и аппарат научного познания,
которые становятся как бы орудиями общества. Искусство… создаёт нарушение внутреннего
равновесия, видоизменение воли в новом смысле, оно формулирует для ума и оживляет для
чувства такие эмоции, страсти и пороки, которые без него остались бы в неподвижном и
неопределённом состоянии».
В творческом процессе сознательные и бессознательные явления взаимосвязаны – и
возможна такая организация сознательных действий, чтобы они послужили импульсом для
процессов бессознательных. Так и всякий акт искусства непременно включает как своё
обязательное условие предшествующие ему акты рационального познания, понимания,
узнавания, ассоциации и т. п. Для восприятия искусства, как говорит Л. Выготский,
необходимо созерцать сразу и истинное положение вещей, и отклонение от этого положения.
Но такой взгляд совершенно необходим и в любом виде творчества.
Следовательно, научаясь воспринимать и понимать искусство, человек одновременно
приготовляется к творческой работе, естественно и почти бессознательно добавляя к своей
способности к рациональным умозаключениям (её можно развить сознательно) почти
неуловимые элементы иррационального, интуитивного, прорывного мышления. Того,
которое после долгих лет учения и размышлений воплощается в счастье открытия –
«Эврика!» Родившаяся идея, мысль или чувство зачастую обязаны своим рождением
произведениям искусства, заключённым в книгах, картинах, музыке. Но не только в них. Как
говорил Мераб Мамардашвили, органами рождения мысли часто являются и произведения
искусства, бродящие в наших душах, и произведения искусного труда.
Три года назад от нас ушёл выдающийся отечественный математик Виктор Игоревич
Арнольд, последний гений советской эпохи. В скорбные дни 2010-го в журнале «Лехаим»
(№ 7 (219) Михаил Майков писал следующее:
«С 1993 года Арнольд преподавал в Университете Париж-Дофин, но при этом
постоянно издевался над французской системой математического образования с ее
предельно узкой специализацией, высоким уровнем абстракции, аксиоматическидедуктивным методом и принципиальным отсутствием интереса к поверке теории
практикой. Он состоял иностранным членом всех крупнейших академий мира, но, будучи
приглашен в Папскую академию наук, отверг предложение – из-за несогласия с отказом
Ватикана реабилитировать Джордано Бруно.
Однако никакие, сколь угодно эксцентрические, поступки не могли сказаться на его
репутации. Арнольд был самым цитируемым российским ученым и одним из наиболее
цитируемых математиков мира. Коллеги в один голос называли его «гением» и «самым
великим математиком современности», констатируя, что он оставался «одним из немногих
современных ученых, которые воспринимали математику в целом, не разбивая ее на
отдельные части, а видя взаимосвязи между самыми далекими областями». Математикуниверсал, он внес огромный вклад в топологию, теорию особенностей и исследование
гладких отображений. Он придал «теории катастроф» научное оформление и способствовал
выделению симплектической геометрии в отдельную дисциплину. Его имя носит множество
математических понятий: диффузия Арнольда, языки Арнольда, спектральная
последовательность Арнольда…
Но все же самый невероятный факт его и без того удивительной биографии не связан с
математикой. В 1998 году Арнольд опубликовал статью, где доказывал, что эпиграф к
«Евгению Онегину», традиционно считавшийся пушкинской мистификацией, на самом деле
восходит к «Опасным связям» Шодерло де Лакло, – и это сближение было признано
ревнивыми профессионалами «остроумным и убедительным».
Действительно, более века над проблемой бились лучшие гуманитарии – пушкинисты,
а математик, несмотря на скепсис, обнаружил эти французские мотивы, и вовсе случайно.
Дело в том, что его математический мозг обладал способностью улавливать весьма далёкие
ассоциативные связи между, казалось бы, разными вещами. Математическое творчество
спровоцировало уникальное восприятие искусства.
Мощь ассоциативного мышления является свидетельством сильного интеллекта,
вопреки шаблонам измерителей IQ, подходящим со своей короткой линейкой к одной из
граней всей умственной деятельности, и далеко не самой главной грани.
Автор полагает, что даже незаслуженно забытая старая детская игра в ассоциации, ещё
может сослужить неплохую службу молодому поколению, если его представители реально
заинтересованы в развитии ума, а не ног, лап и хвостов.
Воспитание для общества
Вернёмся, однако, к проблемам воспитания мышления, обучения интеллектуальному
творчеству, развития способностей. Почему этим вопросам необходимо уделять такое
внимание? Напомним ещё раз: потому что любая культура и любое общество есть
объединение индивидуумов. Будущая жизнь этого объединения определяется уровнем
развития каждой личности. Даже в стаях орангутангов существуют определённые основы
воспитания культуры труда и общения, методы передачи опыта и знаний о жизни и природе
подрастающему поколению. Так что, пожалуй, не только ссылки на античные времена
убеждают нас в необходимости со всей серьезностью подойти к этим проблемам.
Безусловно, различие в методиках и принципах воспитания и развития интеллекта и
личности существует и обусловлено различием в целях. Спартанское воспитание ставило
цель создать в первую очередь профессионально подготовленного «к труду и обороне» члена
общества. Так же (но по иной причине) высоко ценя и непрерывно совершенствуя
физические качества, древнеиндийская цивилизация больше апеллировала к духовным
качествам личности. Какова иерархия ценностей в обществе, такова и структура образования
и воспитания его членов.
Можно, наверное, согласиться – хотя и с оговорками – с тезисом: воспитание важнее
наследственности. Формирование личности, её души и интеллекта, определяется средой,
пожалуй, в большей степени, чем генами. Именно такого рода примеры даёт и литература
(Маугли), и жизнь (известен случай: две маленькие девочки – Амала и Камала –
«воспитывались» дикими зверями в течение нескольких лет, а после возвращения к людям
приспособиться так и не смогли и прожили очень недолго).
Возможно, такого типа пример – и судьба родившейся и выросшей вдали от общества и
цивилизации Агафьи Лыковой, последней из семьи староверов, укрывшихся от мира в
сибирской тайге. При более-менее нормальной наследственности она, похоже, не набрала
необходимый для жизни в городском обществе уровень знаний. Хотя «цивилизованный»
человек, избалованный достижениями прогресса, уже не обладает теми навыками выживания
в природной среде, каковые сохранились у обитателей этого таёжного тупика, а находится в
полнейшей зависимости от «благ…».
Хотя, разумеется, чем шире становится круг нашего знания, тем обширнее вокруг
область нашего незнания. Незнание – классический источник творчества детей. Если вам
неизвестны обычные подходы, стандартные решения, общепризнанные концепции, вы
можете подойти к решению проблемы с неожиданной стороны. Кроме того, если вы не
закрепощены знанием всех преград и ограничений, то можете найти способ сделать даже то,
чего «сделать нельзя».
Бигуди № 25
Вот как раз вполне школьная задачка. Вставьте в записанное выражение одно и то же
число два раза, чтобы равенство 6:6 = 6 стало верным. Не забывайте, что степени бывают не
только научные!37
В институте или университете происходит ориентация на взрослую личность, которая в
первую очередь получает определенную сумму профессиональных знаний и навыков. Вот
тут как раз и теряется возможность интеллектуально развить эту самую личность, дать шанс
для творческого самовыражения. Особенно в ситуации, когда каждый человек имеет
возможность пользоваться нашей общевидовой памятью – запасом знаний всей
цивилизации.
Для серьёзной творческой деятельности нужна с каждым годом всё более длительная и
дорогостоящая подготовка. Количество сведений, накопленных науками, непрерывно растёт.
Требуется и новый уровень образования, и более высокий уровень преподавания. Дело не
только в постоянном обновлении учебников – необходим иной взгляд на образование в
целом. Оно должно стать обучением не только знаниям, но и – главным образом –
творчеству.
Безусловно, обучение – процесс, который можно продолжать совершенствовать. Это и
происходит там, где осознают: имеющееся современное образование – не фабрика,
повышающая эффективность труда путём разделения времени обучения на трудовое и
полезное. Когда-то такое обучение ввёл на своих заводах Генри Форд. Но почему этот же
метод должен быть пригоден для любого обучения? Тот, кто должен в первую очередь учить
думать, не мастер на фабрике. «Фабричный» способ обучения приводит лишь к тому, что
главным критерием хорошей успеваемости становится правильный ответ. Не смысл, не
понимание, не творческий подход. Только механическое извлечение сведений из памяти.
Однако труды многочисленных исследователей – психологов, педагогов,
нейробиологов – не пропадают зря. Уже осознаётся, что учитель – помощник, проводник,
советчик, чья задача – научить использовать свои знания для совершения открытий. Такое
обучение становится исследовательским процессом, в котором суммируются достижения
различных областей знания, применяется различное вспомогательное оборудование,
создаётся необходимая для эффективного мышления обстановка, звучит музыка, радостно и
свободно развивается интеллект. Хотя… никакого специального оборудования, чтобы из 6
спичек построить 3 одинаковых квадрата, не надо. Сделайте – это тоже ваш вклад в
самосовершенствование.
Как жаль, что эти концепции пока что находят применение не в нашей стране. Как
грустно, что в нашей стране, даже несмотря на островки осознания ситуации, нет всеобщего
убеждения в том, что без серьёзной поддержки не удастся осуществить важнейший тезис:
пассивность обучения следует заменить на активное вовлечение в творчество. Наука и
искусство должны стать основой обучения открытию, где сохраняется право на ошибку.
Нам необходимы инвестиции в будущее человека, нации, страны. Развитие
мыслительных способностей, расширение интеллектуальных возможностей, осознание и
признание ценности творческого мышления и подхода к жизни – вот надёжный фундамент
будущего.
Часть 3. Как ускорять процесс и улучшать продукт
11. Как решать задачу, или Прогулки в «пространстве проблемы»
Когда путешественники вышли из лесу, перед ними открылась
крутая скалистая гора. Обойти её было нельзя – с обеих сторон
дороги были глубокие овраги.
– Трудненько карабкаться на эту гору! – сказал Страшила. – Но
гора – ведь это не ровное место, и, раз она стоит перед нами,
значит, надо через неё перелезть!
А. Волков. «Волшебник Изумрудного Города»
Вначале подведём некоторые промежуточные итоги.
В предыдущих частях книги рассказано о некоторых известных (и не очень) фактах,
явлениях и гипотезах, составляющих основу современных представлений об устройстве,
функциях и возможностях мозга и интеллекта человека. Выяснено, что понимается сейчас
под креативными – творческими – способностями, как определяется уровень этих
способностей у человека. Мы немного обсудили, как «творческий инстинкт» проявляется в
научном поиске и чем в этом случае может помочь интуиция. Ещё мы говорили о
необходимости развивать способность каждого человека мыслить творчески. Особенно
учитывая прямую связь между интеллектуальными возможностями одной личности и
процветанием всего общества. Обсуждали также, какие основания – и обоснования – для
развития этих возможностей подготовлены современной психологией и педагогикой.
Сознательно – и самостоятельно! – начать совершенствование мыслительных
возможностей своего интеллекта каждый человек может, начиная с любого момента. Точнее,
с момента осознания своей творческой недостаточности. Не стоит важнейшее дело
построения своего интеллекта откладывать до тех пор, пока в каждой школе или институте
будут введены спецкурсы по творческому мышлению.
Развивать своё мышление – для достижения высоких результатов и успеха в учёбе,
работе и просто в каждодневной жизни – нужно постоянно. Для этого можно не ожидать от
родителей, учителей, друзей или руководителей любого ранга сигнала «Думать! Всем
думать!» Просто прочтите до конца эту книгу – и приступайте.
Великий мыслитель Барух Спиноза сказал: «Того, к чему способно тело, никто ещё не
определил». Но так же никто не определил до конца, на что способны мозг, ум, интеллект
человека. Наше предыдущее рассмотрение, как кажется, указывает на это недвусмысленно.
Так что каждому из нас есть чем заняться, развивая не только мускулы, но и ум. Желательно
в первую очередь ум – ведь мускулы тоже направляет именно он.
Льюис Кэрролл изящно сравнил жизнь человеческого ума с жизнью тела.
Действительно, «последствия пренебрежительного отношения к телу нам нетрудно
представить и ощутить. Для некоторых из нас было бы неплохо, если бы наш разум стал
таким же видимым и осязаемым, как и тело, чтобы его можно было, например, показать
врачу и дать пощупать пульс». Полагая это возможным, Кэрролл попытался – и не без успеха
– переделать некоторые правила рационального питания тела в соответствующие правила
«питания» ума.
Во-первых, следует заботиться о том, чтобы наш ум получал пищу надлежащего сорта.
Мы очень скоро узнаём, какая пища полезна и какая вредна для нашего тела, и не
испытываем особых трудностей, отказываясь от куска соблазнительного пудинга или пирога
(ох, уж эти англичане!), который ассоциируется в нашей памяти с ужасным приступом
расстройства желудка… Однако, пишет Кэрролл, чтобы убедить нас в несъедобности
определённой части излюбленного нами круга чтения, необходимо несравненно больше
уроков.
Во-вторых, мы должны тщательно следить за тем, чтобы наш разум получал съедобную
пищу в надлежащих количествах: «Умственное переедание, или чтение избыточного
количества литературы, – опасное пристрастие, приводящее к ослаблению способности
усваивать пищу и в некоторых случаях к потере аппетита». Автор тоже не раз встречал
подобные, по выражению Кэрролла, «Разжиревшие Умы». Эти умы «не могли выдержать
даже лёгкой пробежки самой медленной трусцой в разговоре, были неспособны даже ради
спасения собственной жизни преодолеть логическую стенку, вечно увязали… в любом
необычном рассуждении, короче говоря, были неспособны ни на что, кроме беспомощного
блуждания по свету».
В третьих, даже если пища доброкачественна, а порции её умеренны, то всё равно не
следует поглощать слишком много сортов пищи за один раз. Умственное несварение ничуть
не лучше желудочного. А его последствия для дальнейшей работы интеллекта весьма
плачевны: вспомнить то, что нужно именно сейчас, практически невозможно – библиотекарь
памяти просто заблудился между разбросанными там и сям коробками с разнообразными
сведениями.
Кроме того, следует проследить, чтобы между последовательными приёмами пищи
соблюдались надлежащие интервалы, и, не торопясь, проглатывать пищу после того, как мы
тщательно разжуём её, чтобы она полностью усвоилась. И вот что ещё: с точки зрения
Кэрролла уму достаточно 3–4 минут для «переваривания» и «усвоения» предлагаемой ему
пищи, содержащей «не только ценный мех, но и несколько килограммов легко
усваиваемого» гарнира. То есть мозг успеет разделить кусочек информационной пищи на
фракции и продолжит обработку следующей порции.
Полезны для дальнейшего интеллектуального тренинга и такие рекомендации:
интервал между последовательными приёмами духовной пищи в действительности гораздо
короче, чем принято думать, однако в процессе размышлений нужно устраивать
кратковременные перерывы, например, раз в час. Можно оторваться от предмета
размышлений на 5 минут, но тщательно следить, чтобы в течение этих 5 минут разум был
полностью отключён от него и полностью занят размышлениями о других вещах98.
Удивительно, говорит Кэрролл, сколь сильный импульс и гибкость вновь обретает разум
после такого кратковременного отдыха. И это верно, ибо проверено на опыте много раз – и
до, и после того, как это заметил Кэрролл. В частности, выдающийся российский
нейрофизиолог Павлов показал: лучший отдых – перемена рода деятельности.
Но, честно говоря, иногда неплохо и поспать просто для отдыха. Только немного!
Замечательный совет я получил от Сальвадора Дали (увы, не лично!): для полноценного
отдыха достаточно не лечь, а сесть поспать в кресло, держа в руке тяжёлый большой ключ
(тип ключа не важен!). Как только приходит сон, ключ выпадает из ослабевшей руки и с
грохотом – для этого кресло желательно поставить на керамический пол, а ключ всё-таки
должен быть немаленьким – падает на пол. Всё, просыпайтесь! Вы уже в полном порядке!
Великий труженик Дали, отдохнув днём таким образом (на весь такой сон нужно 10–15
минут), шёл потом до позднего вечера в мастерскую – писать, делать наброски, лепить. Так
что отдыхайте на здоровье: может быть, и к вам придёт «Сон, вызванный полётом пчелы
вокруг граната за секунду до пробуждения».
Человек треть жизни проводит во сне, и ученые не престают спорить о том, для чего
это делается. Исследование, проведенное Майкеном Нидергаардом и его коллегами из
Рочестерского университета (США), добавляет свой лучик в освещение этой проблемы. Оно
показало, что что мозга во время сна освобождается от вредных веществ и токсинов,
накопившихся во время бодрствования.
Майкеном Нидергаардом обнаружил между клетками мозга тончайшие, заполненные
жидкостью каналы, которые и образуют систему самоочищения мозга от продуктов
метаболизма, наподобие того, как это делает лимфатическая система, выводя токсины из
остальных частей организма. Но её действие не распространяется на мозг: кровь,
поступающая в него, проходит особые молекулярные ворота – гематоэнцефалический
барьер. Этот барьер контролирует все, что попадает из крови в мозг. «В своем распоряжении
мозг имеет ограниченное количество энергии, и, похоже, он должен выбирать между двумя
функциональными состояниями – бодрствование и сознание или сон и очищение, – говорит
Нидергаард. – Точно так же вы выбираете, пригласить ли друзей на вечеринку. Вы либо
98 Без таких размышлений и не отключишься. Вспомните, как ходжа Насреддин пообещал богатому калеке
излечение, если тот и все его родственники пять минут подряд не будут думать об обезьяне, чью внешность
ходжа предварительно описал так красочно, что она немедленно и отчётливо предстала перед внутренним
взором всех слушателей. Естественно, калека остался при всех своих увечьях. А если бы он имел другие – не
менее интересные – предметы для размышлений и знал, что надо срочно вытащить какой-нибудь из них из
глубины памяти – тогда самому ходже пришлось бы отвечать за результат знахарства.
приглашаете друзей, либо затеваете уборку, но сделать и то, и другое одновременно не
получится».
А вот после отдыха нужно быть готовым к тому, чтобы не просто прослушивать то, о
чём говорит «преподаватель по творческому интеллекту», или просматривать книгу, видя
ковригу (или футбольную лигу, или ещё что-либо). Для обдумывания услышанного,
увиденного или прочитанного требуется гораздо большее напряжение ума, чем при
пассивном восприятии. Напряжение это столь велико, что ленивый, нетренированный,
расслабленный разум с негодованием отказывается подвергать себя такому испытанию. Ну,
вспомните, кто ни с того, ни с сего начнёт по утрам делать зарядку и обливаться холодной
водой? Напрягаться? Зачем? И вот мы хилеем и хиреем, лишь бы не заставлять своё тело
работать. То же происходит с нашим интеллектом. Ну-ка, потянемся, разомнём «мускулы»
ума!
Бигуди № 26
После смерти раввин попадает к Господу и начинает расспрашивать его о Рае и Аде.
«Идём. Я покажу тебе Ад», – и Господь вводит раввина в комнату, где вокруг большого
круглого стола в тяжёлом молчании сидят истощённые, отчаявшиеся люди. На столе, в
центре – горшок с горячим ароматным мясом, такой огромный, что с лихвой хватило бы
каждому. Мясо пахнет так вкусно, что у раввина слюнки потекли. Но к еде никто не
прикасается. У каждого из несчастных в руке ложка с очень длинной ручкой – такой
длинной, чтобы дотянуться до горшка и набрать полную ложку мяса. Но слишком длинной,
чтобы ею можно было положить мясо себе в рот! Раввин понял: страдания этих людей и в
самом деле ужасны. «Пойдём дальше. Теперь я покажу тебе Рай», – сказал Господь, и они
вошли в другую комнату. Здесь всё так же, как и в первой: такой же большой круглый стол,
на нём такой же огромный горшок с вкусной едой, в руках едоков – такие же ложки с очень
длинными ручками. Но здесь шумно и весело, все сыты, довольны, розовощёки. Раввин в
недоумении ждёт объяснений от Господа. «Ты ещё не понял?», – снисходительно
спрашивает Господь. – «А ведь требуется так немного! Им нужно было всего лишь
научиться…». Чему же, о Господи?38
В XVIII веке жил и работал известный французский естествоиспытатель Жорж Луи
Леклерк Бюффон. За свою жизнь он написал 36 томов «Естественной науки» – объёмного
научного труда, как ясно даже из названия. Обо всём. Вы, наверное, слышали об этом
учёном в связи с одним наиболее известным его опытом, который легко повторить самому
при надлежащем терпении. Много раз бросайте иглу на пол. Выражаясь точнее, на плоскую
горизонтальную поверхность с нанесёнными на ней параллельными равноотстоящими
прямыми. При этом длина иглы должна быть меньше расстояния между линиями.
Подбрасывайте иглу так, чтобы она падала, вращаясь, с высоты и при падении составляла
какой-то случайный угол с начерченными прямыми. После каждого броска отмечайте,
пересекла ли игла одну из параллельных прямых, и в каждой серии бросков находите
частоту пересечений – отношение числа бросаний, при которых пересечение линии
произошло, к общему числу бросаний. частота пересечений прямых почти во всех сериях
будет примерно одной и Если в серии много бросков, и самих серий много, то заметим – как
когда-то Бюффон: той же. Так Бюффон измерил вероятность того, что игла пересечёт одну
из линий. Дальше остались мелочи – найти эту же вероятность теоретически, что он и
сделал. Оказалось, в ответ входит число «пи». Отсюда следует: экспериментально можно
найти значение этого замечательного числа. Если у Вас терпения не хватит, то учтите:
швейцарский астроном Р. Вольф в середине XIX века осуществил 5000 падений иглы на
разграфлённую бумагу и получил для «пи» ответ 3,159. Вполне приличное согласие с более
точным значением.
Жорж Бюффон вообще много занимался теорией вероятностей. Он и монету бросал
много раз, исследуя поведение частоты событий. Так что дел у него хватало, и он, как
видите, находил время на всё. Мешало же его научным устремлениям лишь то, что он очень
любил спать. Особенно в молодости. Решив победить свою леность, он приказал слуге
Жозефу поднимать его с постели до шести часов утра, не обращая внимания на угрозы. За
это Бюффон даже повысил плату слуге. Такой, заинтересованный в результате будильник,
как понимаете, выключить уже не легко… Впоследствии Бюффон часто говорил: «Я обязан
своему Жозефу тремя, а то и четырьмя томами «Естественной науки»!» Представьте себе,
если бы Бюффон ещё и использовал совет Дали! В общем, больше думайте, меньше спите!
Естественно, чем больше усилие, тем ценнее эффект. Л. Кэрролл оценивает его так:
один час сосредоточенного размышления на какую-нибудь тему… стоит двух или трёх часов
«просто» чтения. Постарайтесь мысленно упорядочивать всё прочитанное, осознанно
помогая мозгу – Мышлению и памяти – разбивать его по рубрикам и раскладывать по
полочкам: это позволяет внутреннему «библиотекарю» при необходимости с лёгкостью
находить интересующий факт или теорему.
Вооружившись в начале пути такими полезными советами, продолжим наше движение
вглубь «пространства проблем». Уточним, однако, что наша цель – и ранее, и в дальнейшем
– не «натаскивание» читателей на методики решения различных задач – пусть и творческого
характера. Т. е. мы не собираемся просто перечислять эвристические правила действий,
обычно используемые для разрешения проблем специально перечисленных типов, и
классифицировать их до бесконечности (вспомните слова лорда Келвина о родах рычагов!).
Конкретно ориентированные стратегии движения в «пространстве проблемы»,
предназначенные для указания в нём оврагов, узких мостов, опасных поворотов и т. д.,
иногда называют дескриптивными принципами. Они напоминают книжки для абитуриентов
с названиями вроде «Характерные ошибки поступающих в вузы», где перечислены и
описаны типовые ошибки в типовых задачах. Но откуда следует, что на экзамене попадётся
именно что-то типовое? А если задача будет совершенно неизвестная? Чем учиться не
ошибаться в типичных, стандартных случаях, запомнив набор приёмов и инструкций, не
лучше ли научиться думать так хорошо, чтобы суметь самому решить любую задачу, да ещё
и, может быть, новым оригинальным методом?
Поэтому куда более важными нам представляются прескриптивные принципы: они не
только предупреждают об опасных поворотах и ненадёжных мостиках, но предписывают
кратчайший – верный и безопасный – путь к цели. Более того, по дороге иногда попутно
удаётся ещё и приобрести что-нибудь полезное. Может быть, даже такое, чего никто никогда
не видел – так появляется новое знание.
Прескриптивные принципы, в отличие от дескриптивных (М. Левин), по сути, и есть
алгоэвристические схемы или эвристики. Они показывают не то, как не надо действовать, а
именно как надо действовать при отыскании наиболее эффективного, нового, возможно,
ведущего к открытию, способа решения проблемы. Конечно, среди них не очень много
принципов абсолютно универсальных, пригодных вообще для всякой задачи. Однако для
задач из разных областей можно найти подходящие инструменты. Так сказать, способы
передвижения. Ведь там, где можно проехать на машине, не стоит надевать лыжи. Надо
лишь научиться самому выбирать – на чём и куда двигаться.
Есть ещё одна опасность в описании таких принципов. Как заметил М. Левин, иногда
прескриптивные правила действий формулируются крайне абстрактно. Например, кому
поможет при решении сложной задачи или жизненной проблемы совет типа «Нужно
составить план действий»? Какой? Как? На основании чего? Или, скажем, указание «Теперь
следует проанализировать, как совершить переход от известных фактов к обобщающему их
утверждению». А действительно, как перейти? Да, иногда достаточно некоторого общего
совета. Но иногда – и чаще – хорошо бы знать, как конкретно составлять план или выходить
на более высокий уровень абстракции, обобщения.
Мышление начинается там, где появляется задача, где возникает необходимость поиска
верного пути к решению. А это означает, что появляется и проблема выбора: либо таковы
условия задачи, либо мозг сам предлагает несколько вариантов поиска. Вот так в
«пространстве проблемы» возникает разветвлённая логическая сеть. В ней нужно идти от
узла к узлу, одновременно фиксируя и сравнивая, запоминая и вспоминая, оценивая шансы
на успех в продвижении по различным траекториям. Это не просто. Требуется достаточная
«мощность» интеллекта, чтобы найти в такой сети кратчайший путь к верному ответу.
Бигуди № 27
Томас Алва Эдисон предложил одному математику, желающему устроиться на работу
в его лабораторию, быстро определить объём колбы лампы. Более часа математик возился с
измерениями и вычислениями (колба – не шар!) и наконец гордо предъявил листок с
расчётами и ответом. Эдисон молча продемонстрировал математику и более быстрый, и
более точный способ определения объема колбы. Какой же кратчайший путь к ответу
продемонстрировал Эдисон? Не помогал ли ему мудрый Архимед?39
Оговоримся: одни люди имеют развитую склонность к постановке задачи и/или
генерации идей, другие, напротив, готовы к скрупулёзной экспертизе идей чужих. Редко
когда в одном человеке уживается способность и к мощной генерации и к грамотной
экспертизе.
Э. Резерфорда попросили рассказать о науке на страницах журнала. Он ответил:
«Тенденции современной физики? Я не могу написать об этом статью. Тут и разговоров
всего на пару минут. Всё, что я мог бы сказать, сводится к одному – физики-теоретики ходят
хвост трубой, а мы, экспериментаторы, время от времени заставляем их сызнова поджимать
хвосты».
Такова же и разница между генераторами и экспертами. Впрочем, генераторам как раз
не нужно опасаться неправильных идей и «поджимать хвост» – именно они создают
градиент движения в «пространстве проблемы».
Уточним, что называть «пространством проблемы». Определим его как множество всех
условий и формулировок данной проблемы, дополненное множеством всех мыслимых
вариантов её решения. Выбор решения – это выбор пути в таком пространстве с учётом его
«рельефа». Что предпочесть – путь напрямик через «холмы» или «овраги» (быстрое, но,
возможно, приближённое или неполное решение) или медленное движение «в обход» (с
учётом всевозможных частных случаев, с полным исследованием задачи)? А если с того
места, где мы находимся, вообще не разглядеть верного пути? Как выбрать ту точку, откуда
виден путь к решению? По сути все такие вопросы, относящиеся к данной задаче, сами
являются элементами её «пространства проблемы».
Движение в нём – это и есть процесс анализа, исследования, поиска решения.
Происходит это движение в мозгу человека: он мысленно проходит по тому или иному пути,
пытается перепрыгнуть с одного берега на другой, попадает в болото или взлетает на
крыльях озарения – и ясно видит решение за холмами Заблуждений и Ошибок! Скорость и
направление перемещения в «пространстве проблемы» зависят от стратегии мышления, его
эффективности, образности, имеющегося багажа Необходимого Знания (НЗ). Количество и
качество гипотез, мысленных образов, скорость и способы проверки предположений
характеризуют процессы обработки информации о задаче. То есть – ваше мышление.
Ещё по поводу НЗ. Безусловно, для эффективности мышления необходимо регулярное
снабжение мозга разнообразной информацией, почерпнутой из всевозможных источников:
книг, журналов, научных и популярных статей, альбомов, газет, разговоров и пр. Однако
следует поставить себе и какое-то ограничение по времени. Чтобы не стать «книжным
червём», который получает информацию ради неё самой. «Рыться в книгах» приятно, можно
это делать бесконечно долго. Но в какой-то момент пора бы и за дело приниматься: поиск
сведений, которые, может быть, окажутся полезны при решении проблемы, не следует
чрезмерно затягивать.
Избыток информации создаёт энтропию, порою ненужную. В частности, потому, что
далеко не все противоречия в накопленных данных оказываются ключами к неожиданным
решениям – куда чаще они порождаются неизбежными погрешностями и ошибками, и если
все их учитывать, упорядочить размышления не удастся.
Но как же приступать к делу, с чего начинать решать задачу? Сначала надо понять
простую вещь – вы вообще-то хотите её решать? Не боитесь неудачи? Если раньше вам уже
удавалось решать нечто подобное – конечно, будет легче. И меньше вероятность ошибки. А
если проблема совершенно нового типа, сумеете ли отыскать путь к решению? И как долго
придётся это делать?
Бигуди № 28
Перед Вами ряд чисел: 12, 1, 1, 1, 2, 1, 3…………… Напишите, пожалуйста, следующие
четыре числа. Честно говоря, автор, Чарлз Таунсенд, полагает: для этого нужно пять минут.
Но Вы-то, конечно, после прочтения всего вышеизложенного в этой книге, сумеете решить
задачу быстрее, я уверен. Помните только о времени – я хотел сказать «о часах»!40
Итак, если, несмотря ни на что, у Вас есть желание – или необходимость – решать
задачу, нужно, войдя в её пространство, немедленно его хорошенько рассмотреть. Как
говорил один мой друг, известный физик-теоретик, применим к этой трудной задаче лучший
метод решения: метод пристального рассматривания.
Прежде, чем двигаться в незнакомой «квартире», нужно сориентироваться. Слева
слышна «капель» из крана – видимо, там кухня или ванная комната. Справа, из-за двери,
явственно слышен шум проезжающих машин – наверное, открыта дверь на балкон.
Выключатель, как правило, находится либо на уровне головы (если дело происходит в
странах СНГ), либо на уровне опущенной вниз руки (в прочих странах мира). И так далее.
Примерно то же нужно сделать сразу после входа в «пространство проблемы» – в
незнакомой местности, первым делом, следует осмотреться, не жалея на это времени и
обращая внимание на все тонкости и детали. Назовём это утверждение «принципом
проникновения» (или принципом полной ясности). Иначе говоря, следует провести
первичное исследование проблемы, сориентироваться на местности. Очевидно: чем
тщательнее проведено первичное изучение «пространства проблемы», тем легче будет при
решении аналогичных задач.
Уже имеющиеся знания помогут не попасть в излюбленную в сказках ситуацию:
«Пойди туда – не знамо куда, принеси то – не знамо что». Хороший следопыт по одним лишь
следам определит, какой зверь и когда прошёл, был ли он стар, здоров, разозлён. Чтобы всё
это узнать, нужно обладать опытом и знаниями. Так что требуемое на осмотр время обратно
пропорционально опыту в решении похожих задач, количеству накопленных знаний, степени
развития интеллекта.
Как было отмечено выше, слишком большой информационный запас несколько
утяжеляет и замедляет движение мысли. Если на плечах лежит коромысло, то быстро
развернуться с ним непросто – примерно так и огромный запас знаний эрудита иногда не
позволяет быстро «развернуть» своё мышление. До какого-то момента, до некоторой
«критической массы» эрудиция действует как хорошая смазка, но в слишком большом
количестве информации мысль может просто завязнуть (вспомните, как Л. Кэрролл
описывал эрудита, отвечающего на простейший вопрос).
Вот посмотрите, как барахтаются знатоки из «Что? Где? Когда?» в не слишком
сложном вопросе. Перед этим вопросом ведущий (В. Ворошилов) отметил удивительно
удачную игру и четвёртую подряд победу шестёрки Александра Седина. Ведущий
комментирует: «Внимание на экран. Наконец-то трудный вопрос. Ждём плёнку». На
мониторе идет фрагмент художественного фильма «Свадьба» по произведению А.П. Чехова.
В эпизоде участвуют Эраст Гарин и Фаина Раневская:
– Вы мне зубов не заговаривайте… раз, два, три… Чем тревожить меня разными
словами, вы бы лучше шли танцевать.
– Я не Спиноза какой-нибудь, чтобы выделывать ногами разные кренделя. Я человек
положительный и с характером. Но дело не в танцах.
Заканчивается предложенный эпизод фильма и В. Ворошилов говорит: «Нет, как раз
дело в танцах. Почему Эпамидонд Максимович Апломбов считает, что именно Спиноза
должен выделывать ногами кренделя? Минута!»
Знатоки несколько растеряны. Речь в увиденном фрагменте явно идёт о Спинозе! Они
начинают вспоминать, что им известно об этом великом голландско-еврейском философе.
Правда, у Никиты Шангина возникает мысль, что речь может идти всё-таки не о том
Спинозе. Но остальные игроки не замечают этой реплики – и версия упущена. Сам Шангин,
переубеждённый большинством, начинает восстанавливать свои познания на сей счёт,
вспоминая, что, когда и в связи с чем мог произнести Спиноза, какие исторические сюжеты с
ним связаны.
Время истекает и сам же Шангин рискует ответить: «Cпиноза – философ XVII века,
живший в Нидерландах, был изгнан из общины за свои материалистические взгляды. Мы
считаем, что в данном случае Апломбов упоминает Спинозу в плане недостойного поведения
с точки зрения добропорядочного мещанина того времени. Плюс…»
Тут ведущий не выдерживает и перебивает его: «Ваш ответ понятен, и мы хотим
поздравить вас с двумя мыслями. Ну, во-первых, вы очень хорошо рассказали нам о Барухе
Спинозе. И, во-вторых, во время обсуждения, по-моему, вы первый подали мысль, что,
может быть, это не Спиноза, то есть не «тот Спиноза», и сами же отказались от этой мысли.
Внимание! Эпамидонд Максимович Апломбов и Антон Павлович Чехов в пьесе «Свадьба»
имели в виду, конечно, не философа Баруха Спинозу, а известного испанского танцовщика
Лиона Спинозу99, танцевавшего в то время в балетной труппе Большого театра.
Долгожданное очко телезрителей! Поздравляем автора вопроса, приз100 едет в город
Ленинград».
Далее Ворошилов прокомментировал потерю очка знатоками так: «Вот это вопрос!
Почему знатоки проиграли? Мне кажется, что они понадеялись на свои знания. Конечно, они
знают философию XVII века. Но даже Спиноза ошибался, когда говорил, что познание
добывается только разумом. А как же чувство? Знатокам сейчас как раз и не хватило чувства.
Чувства чеховского стиля, чувства юмора, если хотите. Если бы они просто сказали, что
философ Спиноза здесь ни при чём – я бы дал им очко».
Я думаю, что не только в чувстве юмора здесь было дело. Знатоков этой шестёрки, как
ни странно, подвела их огромная эрудиция и, в первую голову, определённая однобокость –
желание и умение опираться именно и только на эту эрудицию. Есть вопрос? Тут же лезем в
свои запасники и ищем там подходящий ответ. А просто подумать, побродить в
«пространстве проблемы», рассматривая на равных различные предположения? Даже
интуитивно связь книжника и мыслителя Баруха Спинозы, строго математически, с
помощью теорем и короллариев доказывавшего бытие Божие и Его совершенство, с некими
фривольными танцами кажется подозрительно нелепой. Но… срабатывает самоуверенный
всезнайка-библиотекарь, хранитель памяти: «Чего тут думать! Сейчас покопаемся в памяти и
всё найдём!»
Безусловно, интеллектуально развитому человеку необходима эрудиция в различных
областях знаний и культуры. Обычно те, кто от природы имеет хорошую память, почти
«автоматически» оснащены (и даже отяжелены) всякими познаниями. Но это не означает
специальной предрасположенности к творчеству.
99 Так произносилась в тогдашней России испанская фамилия Эспиноса.
100 Призом тогда – в 1982-м году – были избранные произведения И.С. Тургенева.
Те, кто не может (или не хочет) запоминать лишнюю информацию, далеко не всегда
лишены талантов и способностей. Однажды Эдисон пожаловался Эйнштейну, что не может
найти себе толкового помощника – никто не подходит. «А как вы определяете их
пригодность?» – поинтересовался Эйнштейн. Эдисон показал листик с вопросами: «Кто на
них ответит, тот и станет моим помощником». «Сколько миль от Нью-Йорка до Чикаго?» –
прочёл Эйнштейн и ответил: «Нужно заглянуть в железнодорожный справочник». «Из чего
делают нержавеющую сталь?» – «Об этом можно узнать из справочника по
металловедению». Просмотрев остальные вопросы, Эйнштейн заключил: «Не дожидаясь
отказа, снимаю свою кандидатуру».
Вот понимать, что нужно искать – это действительно важно. Мораль проста: указания
типа «нужно всё знать» – не более, чем весьма общие пожелания, а в каждом отдельном
случае требуется более тонкая дифференциация.
Поиск решения – не простая механистическая деятельность. В ней случаются победы
наравне с неудачами. Но даже после таких обидных, как приведенное выше, поражений,
после очередного удара «лицом об забор» психологически важно не терять желание
продолжать поиски решения, идти «до победного конца». Это желание питается не только
честолюбием («Я – да не справлюсь?»), но и любознательностью или любопытством. «Мы
любознательны, но не любопытны» – лозунг разумных собаковидных голованов в повести
братьев Стругацких «Жук в муравейнике» (для тех, кто не читал: это – продолжение суперкниги «Обитаемый остров», не путать с одноимённым фильмом!). Не обращайте внимания
на нарочитую двусмысленность этого лозунга. Именно любопытство, внимательное
«разглядывание, ощупывание и обнюхивание» всего, что может оказаться полезным или
неизвестным, способно заметно расширить круг Известного и пополнить запас
Необходимого Знания. Необходимого, поскольку это существенная часть умственных
способностей человека, которые так высоко ценились в Древней Греции, рассматриваясь,
конечно, в гармонии с физическими способностями.
Бигуди № 29
Трудно было жить в Древней Греции, особенно лентяям. Потому что с точки зрения
древних греков, человек непременно должен что-то делать, создавать, творить, овладевать
каким-то ремеслом. Именно результатами труда, достижениями в науке, искусстве и
определялась ценность его личности. Этот взгляд на смысл и радость жизни человека был
сохранён и во времена Древнего Рима. Более поздние цивилизации почти утеряли, к
сожалению, такой подход к оценке человека, но память об этом сохранилась. Какое же слово
осталось нам от древних для оценки человеческой личности, способности человека
действовать, двигаться (или – наоборот)? Подсказка: к этому слову имеет отношение и
великий Ньютон, точнее, один из его законов. Есть в этом слове и отрицание, есть и некая
энергия жизни, творчества, искусства. Думайте, вспоминайте, смотрите с разных сторон.41
12. Как решать задачу–2, или Одинокая корова не может служить
ориентиром
– Подумай только, когда тёмными осенними вечерами мне
придётся идти к своему маленькому домику, я зажгу этот фонарик.
Теперь я уже не буду блуждать в потёмках среди труб, – сказал
Карлсон и погладил фонарик.
А. Линдгрен. «Малыш и Карлсон, который живёт на крыше»
Чтобы понять, как эффективнее мыслить, неплохо бы представлять, как происходит
рождение предтечи нового знания. Перечислим основные механизмы, работающие на
рождение новой идеи: логический аппарат, определяющий основной тип деятельности
сознания, и подсознательно-интуитивный, как вспомогательный.
Впрочем, подсознание можно счесть вспомогательным лишь условно. Если логика
перебирает варианты, последовательно их просматривая и методично поднимаясь по
ступеням, то подсознание (то есть интуиция) порождает и «вываливает на стол» кучу-малу
различных идей, связанных с решаемой проблемой, зачастую, лишь весьма отдалёнными
аналогиями и ассоциациями. Подлинно креативную мысль всё равно придётся затем
отбирать и обрабатывать логическому аппарату. Иными словами, оба механизма мышления
взаимообусловлены и тесно связаны. Именно эффективность их взаимодействия, в конечном
счёте, определяет уровень творческого мышления.
Гениальный Э. Резерфорд не позволял сотрудникам работать в лаборатории после
шести часов вечера, а по выходным и вовсе запрещал появляться на работе. Молодой
советский физик – будущий академик и нобелевский лауреат – Капица101 запротестовал
против этого, но учитель сказал: «Совершенно достаточно работать до шести вечера,
остальное время надо думать. Плохи люди, которые слишком много работают и слишком
мало думают».
Причем психопатология, видимо, затрагивает и мотивационный профиль личности.
«Умственная работа, связанная с решением простых и сложных задач, не
ограничивается рамками рабочего дня, она продолжается и в свободное время. Нерешенная
задача создает доминантный очаг возбуждения в мозгу, превращаясь в цель деятельности.
Если эмоционально-волевая установка достаточно сильна, то вся интеллектуальная
деятельность подчиняется решению данной проблемы. Вероятно, у некоторых людей иногда
возникает и прочная «установка» на продуцирование острот. Формированию этой установки
способствуют похвалы и восхищение слушателей; да и сам процесс «синтезирования»
остроты приятен и тем самым упрочивает установку. Постоянная нацеленность заставляет
человека все время искать поводов сострить; он делает это порою неосознанно, почти
машинально. Чаще всего такого рода остряки создают каламбуры. По свидетельству
современников, к этой категории людей относился И. Л. Каратыгин: «Петр Андреевич
Каратыгин буквально никогда и нигде не мог обойтись без остроты или каламбура. Даже при
похоронах своего брата, знаменитого трагика В. Л. Каратыгина, несмотря на всю печальную
обстановку, он не мог удержаться от каламбура и сказал, пробираясь сквозь толпу народа:
«Позвольте, господа, добраться до братца». Если эту остроту нельзя причислить к
патологическим, то, во всяком случае, она неуместна. А граница здесь не всегда различима,»
– отмечал А.Н. Лук в работе «Юмор, остроумие, творчество».
Не может на заслуживать внимания мнение гениального сербского инженера Николы
Теслы (1856–1943) о методах работы его прежнего работодателя и идейного конкурента
Томаса Эдисона (1847–1831): «Если бы Эдисону понадобилось найти иголку в стоге сена, он
не стал бы терять времени на то, чтобы определить наиболее вероятное место её нахождения.
Он немедленно, с лихорадочным прилежанием пчелы, начал бы осматривать соломинку за
соломинкой, пока не нашёл бы предмета своих поисков. Его методы крайне неэффективны:
он может затратить огромное количество энергии и времени и не достигнуть ничего, если
только ему не поможет счастливая случайность. Вначале я с печалью наблюдал за его
деятельностью, понимая, что небольшие теоретические знания и вычисления сэкономили бы
ему тридцать процентов труда. Но он питал неподдельное презрение к книжному
образованию и математическим знаниям, доверяясь всецело своему чутью изобретателя и
здравому смыслу американца».
Эдисон не стыдился метода перебора вариантов, метода проб и ошибок: «Когда я желал
что-нибудь изобрести, я начинал с изучения всего, что сделано за прошлое время. Собираю
101 Кстати, попал он в лабораторию, доказав Резерфорду свою сообразительность. Тот первоначально не
хотел принимать молодого физика, поскольку штат был заполнен. Капица спросил, сколь точно Резерфорд
проводит свои эксперименты. Тот несколько удивился и ответил, что с точностью до 3 %. Капица заявил: «В
Вашей лаборатории как раз 30 человек, значит, при такой точности наблюдений одного лишнего сотрудника
Вы и не заметите». И был принят.
данные многих тысяч экспериментов, а затем делаю несколько тысяч новых», – писал он.
Шесть тысяч опытов с нитью для электрических лампочек ещё далеки от рекорда терпения и
упорства, поставленного Эдисоном. Изобретая щелочной аккумулятор, он произвёл
пятьдесят тысяч экспериментов!
Тесла шёл по другому пути: его изобретения были плодом большой научной,
теоретической работы, к которой вовсе не обращался Эдисон.
Но и у Эдисона были свои критерии креатива. Перед тем как принять нового
сотрудника в свою изобретательскую фирму, он сперва приглашал потенциального
работника к себе домой – отобедать. Пригоден данный специалист или нет, Эдисон
определял с помощью солонки. Как вы полагаете, в каких случаях Томас Альва Эдисон
никогда не нанимал приглашённого и почему? Так вот, если человек брался за солонку
раньше, чем пробовал на вкус предлагаемое блюдо, Эдисон никогда не предлагал ему
работу. Великому изобретателю не нужны были работники, чей образ мыслей и действий
зависел от повседневных привычек. Его интересовали люди, способные подвергать
сомнению то, что другим казалось очевидным, – испытатели.102
Вопрос в том, как заставить работать своё подсознание, расшевелить его, какой метод
пригоден для этого. Впрочем, как говаривал старый профессор математики: «В чем разница
между методом и искусственным, уникальным приёмом? Метод есть тот же искусственный
приём, которым вы пользуетесь дважды».
Создатель ТРИЗ Г. Альтшуллер утверждал: «Вся наша техническая цивилизация
держится на изобретениях, сделанных методом проб и ошибок. Работа изобретателей,
терпеливо осиливавших труднейшие задачи простым перебором вариантов, достойна
большого уважения. Но в последние десятилетия появилась теория решения
изобретательских задач. Теперь нельзя, недопустимо, непростительно тратить время,
средства, силы на «пустые» варианты! Если бы разрядник-шахматист не знал простейших
правил, приемов и годами думал над ходом е2–е4, это было бы смешно. Когда в заслугу
современному изобретателю ставят «пустые» пробы, вызванные незнанием элементарных
правил теории, это тоже смешно. Только смех этот – сквозь слезы».
Бигуди № 30
Однажды в беседе с В.А. Гиляровским один купец, гуляка и большой любитель ночных
кутежей в ресторанах, доказывал, что у него есть серьёзные основания всем другим
предпочитать французское шампанское марки «Мумм». Его доказательство было
совершенно логично, при этом купец ссылался на то, что оно неоднократно проверено на
практике. В чем же заключалось доказательство? Можно даже сказать, что для купца это уже
был надёжный метод потребления шампанского.42
Синектика
Методику эффективного стимулирования творческой активности предложил Уильям
Гордон, опубликовав в 1961-м году свой труд под названием «Синектика: развитие
творческого воображения». Речь в нём шла о создании специальных условий для работы
мозга, в которых удаётся «выдавать» нестандартные решения творческих задач. Сам автор
характеризовал синектику как теорию сознательного использования подсознательной
активности мышления. Он попытался создать условия для деятельности воображения,
интуиции, фантазии, чтобы – в жёстких рамках необходимости найти решение – Могло
возникнуть озарение.
102 Самсонова М. В., Ефимов В.В. Технология и методы коллективного решения проблем: Учебное
пособие. – Ульяновск: УлГТУ, 2003. – С.12.
Советский исследователь творческих процессов Генрих Буш нашёл Уильяму Гордону
предшественника. Им оказался Теодюль Арман Рибо (1839–1916), выдающийся французский
психолог. Основным источником изобретений Рибо называл воображение: «Он
принципиально отрицал возможность создания методики изобретательства, однако указал на
огромное значение методов объединения, разъединения и аналогии, широко применяемых
современными изобретателями. Он писал, что человек изобретает только потому, что
способе составлять новые сочетания из идей. Большое значение Т. Рибо придавал
аналогиям». Так, «метод эмпатий (по Рибо – олицетворения, одушевления технического
объекта), метод символической аналогии, метод использования метафор (по Рибо –
Мистического воображения), метод переноса (по Рибо – Метаморфозы, переноса на
основании частного сходства) и т. д.».
Участники группы – «синекторы» – должны:
– свободно погружаться в исходные данные задачи, умея в то же время
абстрагироваться от банального суждения, мысленно отвлекаться от формы, в коей
представлена вещь, нуждающаяся в совершенствовании или обследовании, выделить саму
суть задания и преодолевать ординарный ход мышления;
– иметь развитую фантазию, тягу к свободным раздумьям, аналогиям и
сопоставлениям, не замыкаться на единственной мысли, уметь переключаться с одного
предмета на другой, с одной области науки и техники на другую;
– уметь выслушивать суждения коллег, не препятствовать им в таком же творчестве,
благожелательно воспринимать идеи даже в тех случаях, когда они нечётко
сформулированы, задержать по необходимости излишнее развитие и углубление уже
найденных собственных идей и понимать, что могут появиться идеи гораздо лучшие;
– не замыкаться на единственной мысли, уметь переключаться с одного предмета на
другой, с одной области науки и техники на другую;
– преодолевать психологические барьеры, верить в конечный успех при разрешении
проблем, не сомневаться в собственных креативных качествах и наличии изобретательских
способностей у коллег;
– быть любознательным, уметь удивляться, находить особенное, отталкиваться от него
в дальнейшем, как от исходной точки в синектическом анализе.
Чем раньше в человеке пробуждается любопытство, тем больше вероятность, что он
достигнет в жизни подлинного успеха. Г.С. Альтшуллер и его последователи уже в 1980-х
годах отмечали эффект чудесного в развитии творческого человека. Чудо захватывает с
малых лет, чудо это главное событие в жизни ребёнка. Память о потрясении становится
«тем движителем, который устремит к достойной цели и сделает её единственно
приемлемой, и не позволит отступиться или сдаться ». Шотландский инженер Джеймс
Уатт, тот самый, что ввёл «лошадиную силу» в обиход и изобрёл паровую машину
«двойного действия», в детстве изумился тому, как оживает крышка чайника, когда в нём
кипит вода. Иоганн Кеплер загорелся астрономией тоже ещё в детстве, углядев как-то раз
фантастически красивую комету, а потом его поразило величественное лунное затмение.
Тихо Браге подростком наблюдал грозное затмение Солнца, это послужило толчком к тому,
что всю свою дальнейшую жизнь он посвятил изучению небесного пространства.
Маленького Генриха Шлимана изумил красивый рисунок в издании «Илиады». Александр
Македонский хотел дойти до края Ойкумены, подражая героям той же, любимой им поэмы
Гомера. В тризовской литературе чудо определяется как эмоциональное потрясение,
обладающее эвристической силой (см. список дополнительной литературы).
Цель понятна – снять тормоза с мышления, преодолеть барьер обыденного сознания,
разрушить стереотипы. Поэтому основную роль в методе играет свобода генерации
сравнений, поиска аналогий, метафорического описания условий задачи, её особенностей.
Внерациональная, образная форма поиска должна помочь найти наиболее неожиданное
рациональное решение задачи. Предлагается сначала даже не фиксировать словесное,
вербализованное представление о вариантах решения. Чувства и образы, возникающие при
движении в «пространстве проблемы», со временем найдут своё рациональное выражение в
определённой символической форме.
Вообще-то идея раскрепощения сознания при обучении не нова. Ею в совершенстве
пользовались древние философские школы – вспомните хотя бы киников Эллады или суфиев
Востока. Дошедшие до нас анекдоты о Диогене Синопском и ходже Насреддине – как раз
примеры посвящения в творчество: подготовка философом ученика к восприятию проблемы
широко открытыми глазами:
Однажды царь Александр приказал наполнить блюдо костями и послать кинику103
Диогену. Получив посылку, тот сказал: «Еда-то для псов, но дар не царёв».
Пришёл ходжа с товарищем в Ко нью для учёбы. Товарищ, впервые в жизни увидев
минареты, в удивлении спросил: «Как это их делают?» – «А ты и не знаешь? Эх, ты! –
заметил Насреддин. – Очень просто: роют глубокие колодцы и выворачивают их
внутренность наизнанку». (Вспоминаю вновь «Что? Где? Когда?»: что растёт тем более, чем
больше от него отнимают?43)
Прибегают к Насреддину и говорят: «Беда, ходжа, ваша тёща стирала белье у реки и
утонула. До сих пор не могут её найти!» Прибежал Насреддин к реке и принялся искать
выше того места, где тёща стирала. «Что вы делаете, ходжа? – спросили люди. – Ведь её
унесло вниз!» – «Э, вы не знаете мою тёщу. Она была такая упрямая, что всегда всё делала
наоборот. И под водой она поплыла, я думаю, не вниз, а вверх».
Думаю, при желании каждый может вспомнить ещё немало подобных притч.
Даже если «партия прикажет», «работать по-ленински» ли, по-ельцински ли, поинновационному, мыслить творчески по директивам невозможно. Ни раньше, ни теперь.
Мгновенно стать творческим – хоть по приказу, хоть в силу необходимости – не удастся.
Нужно создать или хотя бы указать «инструменты», позволяющие это сделать в ситуации
поиска решения. Ими могут служить основные процедуры, предлагаемые синектикой. Это
превращение незнакомого в знакомое и/или обратный переход – превращение знакомого в
незнакомое.
Бигуди № 31
Через речку на лодке нужно перевезти большую тяжёлую трубу (ремонт, что
поделаешь!). Если положить трубу в лодку, осадка её становится так велика, что сесть в
лодку человек уже не может. И все же придумать способ переправы удалось, и трубу
успешно перевезли на другой берег. Причём человек сидел в лодке и спокойно грёб веслами.
Что же он придумал, что сделал наоборот?44
На первом этапе происходит, как и следует из «принципа проникновения», вживание в
проблему, прочувствование её «изнутри». Всё незнакомое при этом постепенно становится
привычным и обычным. Иногда настолько, что пропадает ощущение всякой новизны и
необычности задачи. Но теперь, освоившись в этой обстановке, следует приступить к
обратному процессу: постараться увидеть всё окружение по-новому, найти неожиданный
ракурс. И вот тут синектика дает неплохой совет: используйте аналогии разных типов. А
именно, предлагается оперировать аналогиями:
✓ личными (попробуйте представить себя частью механизма, живой природы,
сказочным существом – драконом, гномом);
✓ прямыми (найдите явные связи и сходства между явлениями, процессами в живой
природе и своей задачей – скажем, мягкая походка льва, раскачивающиеся на лианах
103 Кинос – собака. Диоген демонстративно подражал собакам и в неприхотливости, и в бесстыдстве: мол,
если собака не стесняется своей стаи, то и человеку стесняться нечего. Отсюда и название всей философской
школы, им основанной.
обезьяны, разделение функций в муравьином сообществе, ползущая гусеница, качающаяся
ветка и т. д.);
✓ символическими (тут уж понадобится найти обобщённые образы – символы для
описания элементов или этапов решения задачи, функций основных деталей механизма;
например, итерационная процедура использована при доказательстве по аналогии со змеёй,
скользящей в свою нору, а один из фокусов с картами помогает увидеть, как собрать в одну
«колоду» металлические пластины – деталь нового механизма);
✓ фантастическими (поиск и сравнение с воображаемыми объектами и процессами;
например, перед мысленным взором возникают неземные пейзажи, существа с
удивительными функциями: способные чувствовать запах мысли, как в известном рассказе
Роберта Шекли, или обмениваться разумами, как в его же повести – как наша задача видится
в такой обстановке и может быть решена этими существами?).
«Частый прием при решении проблем – перенос.
Решение одной задачи служит подсказкой при решении другой: здесь напрашивается
аналогия с приемом сравнения по неявному признаку. (Прием переноса экспериментально
исследовал Я. А. Пономарев.)
Прием остроумия, именуемый обратным сравнением, является частным случаем
ситуаций с переменой мест или «перевернутых ситуаций».
Конструкторы и инженеры прибегают точно к такой же умственной операции при
решении своих задач. Если проблема никак не поддается решению, то её ставят «с ног на
голову», или реверсируют.
Парадокс – и прием остроумия и, пожалуй, самый признанный прием научного
мышления. «Если ни одно из возможных решений не подходит, – говорил известный
архитектор, – значит, подойдет невозможное решение». (А. Н. Лук. «Юмор, остроумие,
творчество»).
Вот точно так и художественное творчество вообще и поэтическое, в частности,
основаны на беспокойстве духа и ума. Поэтому вполне логично заключить, что, во-первых,
хороший гуманитарный уровень – это необходимый компонент творчества технического, а
во-вторых, для любого творчества, в том числе и инженерного, характерна доля «мятежности
духа». Подвергая сомнению кажущиеся неизменными правила, можно выйти на хороший
изобретательский уровень.
Соглашусь с Г. Альтовым, который отмечал великую роль фантастической литературы
и традиционной мифологии в развитии творческого мышления. В 1988-м году мне и моему
брату Нурахмеду довелось проводить Всесоюзный конкурс безумных идей. На телевидение
приходили тысячи писем со всех концов тогда ещё единой страны. Среди них запомнилось
послание от гражданина Славолюбова из Москвы. Оно моментально вызвало ассоциацию со
сказкой «Дикие лебеди». Ведь, прочитав одну эту сказку и ничего не зная об истории
крапивы, современный человек способен предложить вполне креативную идею: «Крапива –
широко распространённое неприхотливое многолетнее растение. В древности его
употребляли не только в пищу – супы и лепёшки, но и для изготовления ткани для одежды и
упаковки. Крапива обладает целебными свойствами. Предлагаю создать целенаправленные
плантации крапивы и использовать её в народном хозяйстве. Продукты продавать за рубеж –
как экзотическую русскую пищу. Если в своё время отошли от использования крапивы из-за
появления льняных и шёлковых104 тканей, то сейчас вырос спрос на экзотику, да и об
104 Строго говоря, шёлк, в отличие от льна – не альтернатива крапиве. В основе льняного волокна, как и
крапивного, лежит целлюлоза – природный полимер глюкозы. Из неё же строятся и другие растительные
волокна – конопля, хлопок, джут, сизаль… А шёлковая нить строится из серицина – полимера белковой
природы. Поэтому шёлк (как и шерсть, образуемая белком кератином) боится модных сейчас биологических
добавок к стиральным порошкам. Все эти добавки – ферменты (то есть белки с каталитическими
способностями), разрушающие белки: именно белковые примеси дают самые стойкие загрязнения. Особо
активные ферменты разрушают даже синтетические полимеры, чьи звенья связаны по схемам, похожим на
белковые: капрон, найлон… А растительные волокна к ним стойки: ферменты, разрушающие целлюлозу, в
отличительных целебных свойствах такой одежды ещё никто не задумывался…»
Согласно Крису Фриту, мозг, пользуясь ассоциативным обучением, составляет карту
окружающего мира. «По сути дела, это карта ценностей. На этой карте отмечены объекты,
обладающие высокой ценностью, сулящие награду, и объекты, обладающие низкой
ценностью, сулящие наказание. Кроме того, на ней отмечены действия, обладающие высокой
ценностью, которые сулят успех, и действия, обладающие низкой ценностью, сулящие
неуспех.
Ассоциативное обучение совершенно необходимо нам для выживания. Оно встраивает
нас в материальный мир и позволяет нам быстро и адекватно на него реагировать. Путем
ассоциативного обучения мы приобретаем важные знания об этом мире. Но мы почти не
осознаём этих знаний – наше сознание занято более возвышенными вещами. И обычно эти
более возвышенные вещи суть наши собственные личные желания и стремления».
Литературная тренировка
Впрочем, для научения творчеству полезна и литература, которую принято именовать
реалистической – то есть заботящаяся о вписывании авторских фантазий в реальные
контексты. Полезна не в последнюю очередь потому, что вдумчивый читатель может по ней
научиться и собственные фантазии увязывать с жизнью. Хотя любим мы её не только за это.
Что ни говори, а едва ли не главное в литературе – увлекательность. Хотя её, конечно,
каждый понимает по-своему. Кого-то захватывают тонкие намёки на средневековые обычаи
в «Имени розы» Умберто Эко, а кому-то хватает и цитат из русской классики в романах
Бориса Акунина. Кто-то заворожён приключениями мысли в романах английского учёного
Чарлза Сноу, а кто-то души не чает в скаковых лошадях, на чьём фоне развиваются
детективы бывшего жокея высшего класса Дика Фрэнсиса.
Кстати, не зря все приведенные примеры относятся к самому крупному из
литературных жанров. Критик Рената Гальцева пишет: «Роман – это самая «читабельная»
вещь, захватывающая вас своим течением (недаром же говорят, желая подчеркнуть
увлекательность какого-либо текста, «читается как роман»). С момента рождения жанра,
связанного, кстати, с обретением женщиной из среднего класса отдельной комнаты, где
наедине с книгой, в тишине она могла погружаться душой в сладостные грёзы наяву, роман
стал усладой и заменой жизни, – тем, что интереснее её самой. И какие бы трансформации он
ни
претерпевал,
приобретая
оттенки
социально-политического,
философского,
биографического, исторического, фантастического повествования, роман всегда оставался
экзистенциальным субститутом жизни, вовлекающим читателя в свою колею.
Интеллектуальный роман Томаса Манна или религиозно-метафизический Достоевского так
же увлекателен и жизненно насущен (для тех, кто is up to the mark 105), как и
сентиментальный. Он заставляет трепетать, погружая в трагические бездны, уноситься в
заоблачные высоты и в конце концов становиться искушённее и умудрённее».
Это, конечно, не относится к изыскам вроде формы без содержания и содержания без
формы, когда остаётся лишь гадать, что имел в виду автор и имел ли он в виду вообще хоть
что-нибудь. Гальцева отмечает: «Когда-то Розанов высказал такую возмутительную
неполиткорректность: «Свобода нужна содержанию, а пустоте свобода не нужна». И прав
был наш несравненный Василь Васильич: зачем пустоте пустота? (А свобода ведь та же всё
вмещающая пустота.)
Но, с другой стороны, она очень даже необходима – там, где нет содержания, его
можно изобразить, измыслить на просторе и затем предстать в его сверхъестественном
стиральные порошки не добавляют.
105 Соответствует уровню (англ.).
величии. Это нам, остальным, свобода самовыражения пустой бочки не нужна, а ей самой
как раз нужно много свободного пространства, чтобы заявить о себе.
Кто, если не читал, то хотя бы заглядывал в роман новейшей выделки или знакомился с
ним по диагонали, не мог не прийти к убеждению, что имеет дело с особым феноменом, с
чем-то доселе невиданным, и что все известные смены стилей и разговоры о «переломах» в
искусстве словесности совершенно не идут к данному случаю. (Мы имеем в виду,
разумеется, не массу средней литературы, а только репрезентативную для нынешнего дня)».
Подобные произведения бытуют в любую эпоху – и как это ни забавно, всегда
считаются новомодными. Похоже, их воспринимают как разрыв с традицией, но не
замечают, что они и сами образуют своеобразную традицию – имитации глубины с помощью
тонкого слоя непроглядной мути.
Впрочем, и такие творения могут быть поучительны – надо же уметь отличать
подлинники от подделок! Настоящая же литература несомненно помогает своим читателям
совершенствоваться. Причём не только логически.
Бигуди № 32
Можно сказать, что этот вопрос тоже продиктован на уроке литературы. Однажды в
царскосельском лицее ученикам было задано написать сочинение (а, может быть, стихи) на
тему «Восход Солнца». Один из лицеистов не справился с заданием, поскольку долго в
голову ничего не приходило, он успел (или сумел) написать только слова «На западе
восходит Царь природы», после чего преподаватель дал команду сдавать работы. Неумелый
сочинитель обратился к товарищам с просьбой о срочной помощи. Быстро откликнулся
лицеист по прозвищу «Обезьяна» (это ещё кто?) – прочтя единственную фразу, написанную
горе-сочинителем, он сразу дописал:
И удивлённые народы
Нe знают, что с чего начать —
…………………………..?!
Через два дня, после оглашения результатов проверки, хохотал весь лицей! Но почему?
Может быть, Вам удастся дописать (хотя бы по смыслу) последнюю строчку и тогда мы тоже
присоединимся к смеху лицеистов?
Логика в интуиции
Альберт Эйнштейн говорил о роли интуиции так: «Я верю в интуицию и
вдохновение… Иногда я чувствую, что стою на правильном пути, но не могу объяснить свою
уверенность. Когда в 1919 г. солнечное затмение подтвердило мою догадку, я не был ничуть
удивлен. Я был бы изумлен, если бы этого не случилось. Воображение важнее знания, ибо
знание ограниченно, воображение же охватывает всё на свете, стимулирует прогресс и
является источником эволюции. Строго говоря, воображение – это реальный фактор в
научном исследовании».
Другой гений – Анри Пуанкаре – писал так: «В математике логика называется
анализом, анализ же значит разделение, рассечение. Поэтому она не может иметь никакого
другого орудия, кроме скальпеля и микроскопа… логика и интуиция играют каждая свою
необходимую роль. Обе они неизбежны. Логика, которая одна может дать достоверность,
есть орудие доказательства; интуиция же есть орудие изобретательства».
«Творчество, – утверждал учёный, – состоит как раз в том, чтобы не создавать
бесполезных комбинаций, а строить такие, которые оказываются полезными; а их ничтожное
меньшинство. Творить – это отличать, выбирать».
Интуицию чаще определяют как «способность постижения истины путем прямого её
усмотрения, без обоснования с помощью доказательства». В познании мира интуитивное и
логическое не исключают, а диалектически дополняют друг друга, помогая выбрать главное,
существенное, особенное. Их синтез – это двигатель творческого мышления.
Есть мнение, что логический путь – всего лишь столбовая дорога шаблонного
мышления: когда есть логическое решение, может показаться, что не стоит и искать более
эффективное направление решения. Утверждается: интуитивный путь нестандартного,
нешаблонного мышления – альтернатива логическому пути. Недостатком же последнего
объявляется использование априорно недоказываемых положений, своеобразный
аксиоматический подход.
Но ведь эффективность того или иного стиля мышления определяется контекстом,
смыслом, условием задачи! В условиях плохо определённой задачи, когда трудно определить
исходное звено логической цепи, мышление «по шаблону», конечно, вряд ли будет
успешным. Тем не менее интуиция заработает на полную мощность лишь тогда, когда будет
сформулирован ряд гипотез – пусть неясных, расплывчатых, пусть даже не совсем
достоверных, но поддающихся дальнейшему анализу106.
Ничто не может породить нечто. Если есть только информационный хаос подсознания
(пусть даже в нём имеется некая структура, созданная аналогиями), но отсутствуют «сито»
критериев отбора и упорядочивающая система предположений, поиск решения может быть
нескончаемым. Двигаться «по спирали» вокруг логической нити – быстро и эффективно,
используя все возможности подсознания – Можно только в том случае, когда существует
одна или несколько таких логических нитей, цепочек гипотез. Сами же критерии
правильности решения могут задаваться условиями задачи или вопроса.
Приведу пример из моего опыта игры в «Что? Где? Когда?». На стол знатоков
поставили разнос. На нём – самовар, сахарница, сушки на блюдце, стакан. Из самовара
налили чаю в стакан, бросили в чай кусок сахара. После чего был задан вопрос: «Скажите,
как на космической станции «Венера-4» был устроен замок антенны?»
Понятно, что на разносе собраны все исходные данные и имеется ключ к решению. Вот
тут все эрудиты начинают извергать различные гипотезы, поворачивают так и эдак краник
самовара – ведь похоже на механизм? Предположений множество, ведь информация явно
избыточна.
Я, осмотревшись во множестве начальных данных (вот он – «принцип
проникновения»!), пытаюсь отсечь лишние сведения. Понимаю: чай налит неспроста. Да и
сахар тоже имеет отношение к делу. И вот тут включается запас НЗ: я вспомнил книгу
Валентина Пикуля «Моонзунд». Там описан эпизод минирования с кораблей акватории
Балтийского моря во время Первой мировой. Чтобы мины автоматически становились на
боевой взвод, их снабжали предохранителями, сделанными из сахара (кстати, пропитанного
хинином – чтобы морячки сахар не воровали). Мина опускается в воду, вода растворяет
сахар и у мины «ушки на макушке» – лучше не трогай!
Всё это прекрасно – но где там в космосе (точнее, на Венере) вода, чтобы растворить
сахар? И вот вновь срабатывает память, получившая от сознания бессловесно оформленный
запрос – как связаны вода и Венера? В то время, когда эта станция была в полёте (а это 1960е годы!), как раз был опубликован фантастический роман Александра Казанцева «Планета
бурь». Автор опирался на мнение учёных, в ту пору полагавших, что поверхность Венеры
покрыта океаном. Так что станция должна была проверить и эту гипотезу.
Тут меня несколько смутил тот факт, что по новым данным температура на
поверхности планеты выше 300 градусов. Поэтому при ответе я подстраховался и осторожно
сказал: замок антенны был сделан из сахара, который должен либо раствориться, либо
расплавиться в атмосфере планеты. Так был найден верный ответ – после того, как
подсознание «вытащило» в оперативную память мозга сведения о растворении в воде
сахарных предохранителей.
106 Вспомните размытые множества – ведь и в этом случае удаётся нащупывать некую оптимальную
стратегию поиска решения.
Утверждается, что к процессам, идущим в подсознании, не следует проявлять внимание
прежде, чем идея приобретёт «прочность», необходимую для дальнейшей её обработки
сознанием. Очевидно, в процессе деятельности подсознание непрерывно подключает всё
новые ассоциативные ряды и всё более удалённые информационные блоки, т. е. при его
работе «включены» обширные области мозга. Поэтому активизация какой-то одной из его
структур (при попытке однозначной фиксации мысли), несомненно, переключает на себя
основные энергетические потоки, питающие подсознание, снижая тем самым его
энергетический, а, следовательно, и креативный потенциал.
Перенос мысли из подсознания в сознание происходит тогда, когда мысль
окончательно оформлена. То есть, говоря на физиологическом уровне, образуется
устойчивый локальный очаг возбуждения одной из зон коры мозга.
Бигуди № 33
Вот задача, в которой вам тоже придётся кое-что вспомнить: например, холодные
«глечики» с молоком у бабушки в деревне. Так вот, египетские фараоны любили попить в
египетскую жару холодную воду, но сделать это было непросто – не доставлять же лёд в
бурдюках с вершины Килиманджаро? Однако фараоны и их приближённые не страдали от
невозможности пить в жару холодные освежающие напитки. Да, холодного погреба, как у
бабушки, у них не было. Зато рабов было более чем достаточно. И собственные глиняные
«глечики» у них были – только не глазурованные, к счастью. Ну и как же охлаждали воду в
Древнем Египте?45
Всесторонние мысли
Итак, тренировать, развивать, обучать нужно все системы мышления. Нужно также
знать, как использовать возможности их «взаимопомощи». Одну из самых простых и ясных
рекомендаций по развитию интеллекта можно найти у Редъярда Киплинга:
I have six honest serving men.
They taught me all I knew.
Their names are What, and Why, and When,
And How, and Where, And Who.
В дословном переводе: «У меня есть шесть честных слуг. Они обучили меня всему, что
я узнал. Их зовут: Что, Почему, Когда, Как, Где и Кто». А в качестве образца творческого
подхода приведу перевод С.Я. Маршака:
Есть у меня шестёрка слуг,
Проворных, удалых.
И всё, что вижу я вокруг, —
Всё знаю я от них.
Они по знаку моему
Являются в нужде.
Зовут их: Как и Почему,
Кто, Что, Когда и Где.
Мораль проста: инициировать своё мышление следует, задавая ему вопросы. Это ещё
один способ тормошить мышление, вполне сочетаемый с тем, что предлагает синектика.
Никаким эвристическим принципам не удастся успешно следовать, если мышление не
подготовлено к эффективной аналитической работе на самом первом этапе решения задачи.
А готов ли ваш интеллект к сложному мыслительному процессу? Можно ли его укрепить и
подготовить к любому испытанию «на прочность»? Какие есть способы и методы такой
тренировки мышления?
Однажды в чайхане пожилой человек начал давать всевозможные советы ходже
Насреддину.
– Почему я должен поступать так, как ты мне говоришь? – спросил ходжа.
– Потому что я тебя старше! – вскричал человек, оглаживая бороду.
Тогда Насреддин заметил:
– Насыщает не время, проведенное в чайхане, а количество съеденного плова.
Множество рекомендаций для расширения интеллектуальных возможностей приведено
в современной мировой психологической литературе (её можно условно назвать «Как
сделать свой мозг спортсменом»). Часть методик представлена достаточно схематично,
другие описываются подробно, обсуждаются условия и границы их применимости. Ниже мы
будем ссылаться на некоторые из известных методик – с нашими комментариями и
дополнениями. Но подобно плову, они требуют, чтобы на них не только смотрели.
Итак, что же ещё, кроме «принципа проникновения», можно использовать при решении
различного рода творческих задач?
Бигуди № 34
Попробуем подойти творчески к такой ситуации. Отец приходит домой, видит, что его
сын-школьник за столом играет с друзьями в карты! На деньги! Отец, желая быть мудрым
воспитателем и не прибегать к крайним мерам, кладёт на стол монету, скажем, новенький
десятирублёвик. Ещё одну монету – в 5 рублей – он держит в руках и обращается к сыну:
«Послушай-ка, придумай лучше, как положить монету в 5 рублей под монету в 10 рублей,
совершенно не прикасаясь к последней ничем, не дуя на неё. И желательно без телекинеза!
Справишься – забирай обе монеты». Сын думал недолго. «Ты, папа, придумывай ещё, –
сказал он, засовывая в карман деньги, – Это, пожалуй, выгоднее, чем играть в карты». А Вы
уже поняли, что сообразил сделать сын?46
Самая плохая бумажка лучше самой хорошей памяти
Вспомним: наше сознание способно при обработке информации удерживать в
«оперативной памяти» не так уж много её фрагментов – не более 7. Поскольку её емкость не
слишком велика, она быстро переполняется. Как в кинокомедиях – кто-то втискивается в
заднюю дверь битком набитого автобуса, из-за чего из открытой передней двери человек
выпадает. Так и здесь – первичные данные вытесняются последней поступившей
информацией.
Поэтому мышление не способно одновременно и воспринимать новые данные, и
извлекать что-то из памяти, и вести вычисления. Чтобы облегчить работу мышления, нужно,
как говорят, экстернализировать информацию. Собственно, это означает, что на помощь
оперативной памяти приходят другие её типы. Либо информация просто хранится на других
носителях.
Значит, нужно уметь в удобной для вас форме записывать информацию: начальные
данные, промежуточные выкладки, идеи о направлениях поиска ответа, рисовать картинки
(включаем визуальную память). Полезно не только делать записи, рисовать схемы, наброски,
но и просто разговаривать вслух, хотя бы с самим собой (включаем слуховую память). Этот
следующий шаг поисков в «пространстве проблемы» назовём «принципом бухгалтера» (в его
работе как раз требуется тщательный учёт всех передвижений капитала: в данном случае
капитал – наши наблюдения, сведения, предположения).
Заметим: вообще записывать появляющиеся мысли (и не только относящиеся к данной
проблеме) – важно и полезно. Альберт Эйнштейн, по воспоминаниям его друзей, с молодых
лет имел такую привычку. Он всегда – даже во время морской прогулки на яхте – Мог
вытащить блокнотик и записать какие-то свои соображения.
Конечно, можно сказать «Это же Эйнштейн!» Но возможно, он ещё и поэтому стал тем
Эйнштейном, на которого можно ссылаться без особых пояснений?
Сходная привычка есть у множества людей. А многие другие, думаю, нередко
сожалели о чём-то очень существенном и неповторимом, мелькнувшем и не остановленном.
Знаете, что нужно сделать? Победить свою лень. Нужно убедить себя: всё, что будет
записано, на самом деле никакие не пустяки – это часть твоей неповторимой жизни, часть
того дела, которое останется, возможно, после тебя, которым даже можно будет
гордиться107.
Известен так называемый «принцип снеговика ». Один из нас в первые школьные годы с
трудом запоминал стихотворения (в тогдашней школьной программе предусматривалось
заучивание наизусть многих десятков произведений этого жанра в самых разных стилях – и
дальнейший опыт доказал, сколь полезны оказались эти вроде бы далёкие от жизни
языковые игры). Мать подсказала: пока не запомнишь весь текст до какой-то строки, не
пытайся её учить. Совет помог: как к снеговику прилипают всё новые снежинки, так к уже
усвоенной части стихотворения легко прилипают одна за другой всё новые строки,
прикрепляемые не только памятью, но и внутренней логикой самого поэтического текста.
Тем же приёмом можно – и зачастую нужно – пользоваться не только для обучения, но и для
последовательного приближения к полному решению, рассматривая отдельные –
сравнительно простые – части задачи. Скатаем сначала отдельные маленькие «комочки» из
заранее известного. Потом каждый из них «покатаем», пока не станет ясно, как он входит в
состав всей – доселе незнакомой нам – конструкции. В конце соберём всего «снеговика». То
есть: сложную и большую задачу мы разбиваем на отдельные части, маленькие задачи,
которые легко исследовать и решать. А потом переходим на качественно иной уровень
рассуждений – и объединяем, сращиваем отдельные элементы решения.
Послушайте, как писатель Лев Разгон вспоминает о своей встрече с великим
популяризатором науки, Яковом Исидоровичем Перельманом (он не родственник Григорию
Перельману!) в 1936 году на его квартире в Ленинграде: «Пододвинув к себе стопу
очередной почты, Перельман взрезал пакеты и пробегал письма, перелистывал журналы и
газеты, советские и зарубежные – английские, американские, немецкие, французские… И
хотя все журналы и газеты, им просмотренные, были совершенно свежими и нетронутыми,
он как будто открывал их именно там, где ожидал увидеть что-то интересное. И, найдя,
обводил цветным карандашом, отмечал страницу закладкой или же брал карточку и быстро
исписывал её чётким, крупным, каким-то школьным почерком. Не оборачиваясь, он доставал
откуда-то из-за спины ящик, и карточка немедленно укладывалась на место. Смотреть на всё
это было не только интересно – увлекательно! Как будто перед тобой бесперебойно работает
хорошо налаженная, отрегулированная интеллектуальная машина, безошибочно
схватывающая, фиксирующая, выбирающая, сортирующая». Вот так работать нужно
поучиться!
Конечно, может быть, Вы и не такой известный писатель и учёный, но если Вам
нравится думать и придумывать, стоит везде и всегда иметь при себе средства для записи
мелькнувшей мысли. А если позволит Ваша лень, то можно раз в неделю или в месяц
суммировать итоги в журнале или тетради, выделяя наиболее толковые и интересные вещи.
То есть надо продолжать работать со своими идеями. И если Вам понравится записанное,
если Вы почувствуете определённое самоуважение – ай да я! – это значит, Вам нравится
ощущать себя творчески активным человеком.
107 Конечно, не переборщите. А то каждое Ваше слово будет Вам казаться «золотым» и достойным
внимания потомков.
Бигуди № 35
Замечательная цитата из записных книжек Ильи Ильфа: «Он снялся на фоне книжных
полок, причём вид у него был такой, будто…». Закончите, пожалуйста, фразу. Честное слово
– должно быть смешно!47 Чтобы продлить радость, закончите и вот это: посетителю снится
«Страшный сон. Снится Троя и на воротах надпись «… нет»». Кого же нет?48
Кстати, собственный «Журнал идей»108 можно дополнить и созданием «Журнала
принятых решений». Записывайте там, что помогло бы в реализации ваших идей, какие
стратегии решения могли бы пригодиться. Даже самые фантастические. Только не ленитесь!
Это же Ваш «золотой фонд» – Вы не только запасаетесь толковыми предложениями на
случай будущих проблем, но и тренируете своё мышление.
Да, вот ещё что: запишите для себя «Десять заповедей изобретателя», предлагаемых
американской Ассоциацией изобретателей. Подзаголовок «Заповедей»: «Как удержать в
голове перспективную идею» – именно то, что нам и надо! Итак:
1. Держи под рукой блокнот. Эдисон имел его даже в ванной.
2. Никогда не надейся, что удержишь в памяти хорошую идею.
3. Запиши идею и сделай набросок. Графический образ, как правило, запоминается
лучше.
4. Отбрось все дела и сконцентрируй внимание на идее и её развитии.
5. Новую идею легче воспроизвести в памяти, когда есть исходная база, к которой
можно вернуться.
6. Новые идеи всегда сопряжены с риском, а риск противоречит человеческой натуре.
Память сотрёт идею, если будешь невнимателен.
7. На стадии обдумывания идеи не рассуждай «почему?» и «возможно ли это?». Твори.
Твои руководители и подчинённые со временем скажут тебе, в чём твои ошибки.
8. Нацеливай идею на будущее. Записывай всё, что может пригодиться. Позже что-то
отсеется, но кое-что и останется.
9. Остынь. Вернись к своим записям на следующий день.
10. Десятая заповедь? Да, была ведь и десятая, но мы забыли её записать. Где же её
теперь искать?
Многомерный морфологический ящик
Конкретная форма записи данных (нечто вроде корабельного журнала или
штурманских записей на авторалли – для описания своих впечатлений и действий в
«пространстве проблемы»), конечно, может быть различной. Скажем, при решении задач
логического типа, где требуется проследить несколько различных логических
последовательностей («Если Ваня в красной майке, значит, не он нашёл самый большой
гриб. Следовательно, этот гриб могла найти Аня…» и т. д.), информацию удобно записывать
в виде таблицы – Матрицы. На пересечении её строк и столбцов оказываются определённые
события, которые удобно отследить и представлять далее в виде логических цепочек.
По сути дела, это самый простой и известный «метод перебора вариантов». Его удобно
применять, когда число вариантов не слишком велико. И тогда полезно использовать
определённую строгую форму записи результатов, например, в виде таблицы – чтобы не
потерять ничего важного. Но так же удобно фиксировать и свои соображения при
размышлениях над новыми, поисковыми творческими проблемами. Между прочим, это
хорошо дисциплинирует и организует мышление.
Нанося на оси воображаемого многомерного пространства в качестве координат
108 Между прочим, говорят, что такой журнал заведен сотрудниками ЦРУ и оказался очень эффективным
способом генерации новых подходов в работе «рыцарей плаща и кинжала».
характеристики предметов или процессов, находим множество «точек» – различных
сочетаний качеств, т. е. вариантов решения задачи. Помимо совершенно нереализуемых,
найдутся и весьма нестандартные и удивительные комбинации вариантов. Такой анализ
решения называют «морфологическим принципом». Его предложил швейцарский
астрофизик Ф. Цвикки. Заметьте: метод можно эффективно адаптировать для персональных
компьютеров, он очень хорошо алгоритмизуется109.
Такого рода «морфологические ящики» используются при решении изобретательских
задач. Но не только. Так можно анализировать и исторические сведения, упорядочивая их и
находя между ними скрытые взаимосвязи – то есть, по сути, новые знания.
Бигуди № 36
Постарайтесь найти, что ЭТО такое объединяет Древнюю Грецию, Египет и прочие
страны. В Древней Греции ЭТО носили и мужчины, и женщины, позаимствовав из Персии.
Для египтянам ЭТО было привычно издавна. У римлян ЭТО вошло в употребление на заре
Империи, причём имперские модницы обожали получать ЭТО из Германии. А во Франции
ЭТО появилось и стало модным благодаря Людовику XIII, в 1624 году. Что же ЭТО? Чем
таким выделялся король Людовик, что ЭТО было для него столь важно и удобно? Да, кстати,
а почему именно ЭТО из Германии так привлекало римских патрицианок? (Необходимость и
удобство ЭТОГО для Людовика были связаны с его головой…)49.
Комфорт тела и души
Мой друг, учёный-физик, никогда не садится за стол работать, пока не проверил,
хорошо ли пишет ручка и лежат ли справа от него чистые листы, а слева – уже исписанные
формулами. И он прав: работать (а думание, размышление – весьма трудная и ответственная
работа!) нужно в удобных и привычных условиях. Ничто не должно раздражать и отвлекать.
Давно замечено: стресс – сильный фактор торможения мышления. Неудобства в
процессе работы – тоже стресс. Помните, выше мы уже говорили: и обучение творческому
мышлению должно проводиться в спокойной, безопасной обстановке? Конечно, какая тут
сообразительность, какой полёт фантазии или даже методичный перебор вариантов, если
сидеть неудобно, на кухне капает из крана вода, а полка над Вами в любой момент может
рухнуть на голову! Сделайте себе красиво и удобно, чтобы ни на что не отвлекать свою
мысль – и вперёд.
Бигуди № 37
Анекдот уже из наших времён. Старенькая бабуля стоит перед витриной магазина
«Компьютеры. Оргтехника» и ругается, понося всякими словами «этих новых русских,
недобитых буржуев и жирных котов из мафии». «Народ с голоду помирает, по помойкам
шастает, – голосит бабушка, – А этим, вишь, мало им самим комфорту, так они
даже…….делают!» Что же такое, по Вашему мнению, увидела бабуля в витрине, что вызвало
такой её гнев?50
От гипотезы к индукции
Продолжение и углубление анализа задачи110 состоит в попытках проложить в
109 Весьма эффективно использование метода Цвикки в обучении в духе ландаматики.
110 Всё ещё продолжается действие всё того же принципа проникновения!
«пространстве проблемы» пути в виде гипотез. Однако каждая из них должна быть
проверена при помощи подходящих критериев. Представьте, что перед Вами некоторая
таблица чисел, где разным значениям X соответствуют некоторые значения Y. Вам нужно
всего лишь продолжить эту таблицу, найти в ней некую скрытую закономерность. Либо
продолжить числовую последовательность, обрывающуюся на некотором члене. Как
обнаружить закономерность построения таблицы либо последовательности?
Может помочь «принцип капли» – попробуем по наблюдениям за «каплей» достоверно
представить себе «океан». То есть построим по результатам небольшого числа событий (по
нескольким первым членам последовательности, по разрозненным экспериментальным
фактам и т. д.) гипотезу-функцию, некоторую гипотетическую модель ситуации, попробуем
восстановить по частному целое. Это как раз и есть приложение к решению проблем
индуктивного метода Бэкона. Не путайте с дедуктивным методом Шерлока Холмса! Там как
раз частное утверждение выводилось на основании общих знаний – например, о природе и
пороках рода человеческого.
В математике индуктивный подход к решению хорошо известен – это метод
математической индукции (во всех её разновидностях), который сменяет метод перебора
вариантов, если их слишком (в пределе – бесконечно) много. На первом его этапе некоторое
общее утверждение (вид функциональной зависимости F(k), как говорят математики)
проверяется на конкретном примере, в некоторый «начальный момент» (т. е. при
определённом значении переменной величины k). Затем выдвигается гипотеза: это
утверждение справедливо при произвольном значении переменной величины k = n. И,
наконец, исходя из этой гипотезы, это утверждение должно быть строго доказано при
значении переменной величины, увеличенном на единицу – при k = n+1. Если все три равно
важных этапа осуществлены, мы убеждаемся в справедливости общего утверждения
(гипотезы) о виде зависимости F(k) при любом значении k.
Бигуди № 38
Потренируемся в индукции. Правда, для следующей небольшой задачки нам придется
немного вернуться назад во времени – в те славные деньки, когда полным-полно было трёх–
и пятирублёвых купюр. Может быть, и не у всех граждан, но уж в некотором процветающем
банке (не будем говорить, в каком) таких купюр было неограниченное количество. И вот Вам
нужно взять кредит. Наличными. А других банков рядом нет! И оказывается, что банкиры
весьма ловко устроились и могут выдать Вам наличными любое число рублей, но не меньше
восьми. Без сдачи. Убедитесь в этом сами. Начните с самого маленького кредита: 8, 9, или 10
рублей выдаются этими купюрами без проблем, верно? Теперь остаётся понять: если вам
легко выдают N, N+1, N+2 рублей, то запросто выдадут и N+3, N+4, N+5 рублей.
Проверьте!51
Бросок через незнание
Но это лишь пример, причём работающий хорошо лишь в определённой
математической области. Для применения же «принципа капли» в иных ситуациях нужно
научить своё мышление выдвигать обобщающие идеи, видеть проблему с некоторой
вершины в «пространстве проблемы», формулировать её на другом языке. Вот тут нам
поможет «принцип паутинки». Видели, как в тёплые осенние дни маленькие паучки смело
путешествуют, перелетая с места на место на лёгких паутинках? Приземлившись, они
начинают обустраиваться, сплетая новую сеть. Так и мышление должно уметь перелетать
через целые области неизвестного, чтобы приземлиться где-то на далёком островке. А потом
соединить в прочную сеть свои рассуждения.
К этому методу близок «принцип срезания угла». Представьте, что в зимний день Вам
нужно перебраться на противоположный берег реки (или на островок на реке). Можно,
конечно, поискать мост, но вам известно, что до него довольно далеко. Попробуем поступить
проще – ведь река уже покрыта вполне надёжным прочным льдом! Правда, лёд скользкий, а
падать не хочется. И вот тогда Вы решаете не идти в обход через мост, не арендовать
вертолёт для посадки на островок и уж тем более не махнуть рукой вообще на всю эту затею
(ведь Вы уверены: ТАМ есть нечто исключительно важное!). Вы просто переобуваетесь,
снимаете тёплые ботинки, в которых удобно ходить по берегу, но скользко на льду. И
обуваете коньки. Теперь легко и быстро вы добираетесь по льду туда, куда хотели (ботинки
не забудьте – на твёрдой почве в них будет удобнее, когда окажетесь в нужном месте). Итак,
как же мы «срезали угол»? Мы перешли на другой язык, другой способ описания условия,
другой уровень абстракции!
По-настоящему эффективное мышление всегда стремится сократить путь к решению.
Даже если долгий путь тоже приводит к нему. Ну, конечно, левое ухо можно ведь почесать и
правой рукой. Правда, левой рукой удобней.
Бигуди № 39
Формулируем вопрос: имеет ли и какое отношение к палеонтологии бурное развитие
автомобильного и железнодорожного транспорта в конце XIX века? Поверьте, эта связь есть,
она совершенно объективна и может быть разумно обоснована. Можно даже подсказать, что
речь идёт о пользе для палеонтологии. Для установления искомой связи действуйте, срезая
лишние углы – отбрасывайте несущественное, смотрите на суть данной науки и потребности
развивающейся техники.52
Привычный рецепт – не лучший
Обезьяну учили доставать связку бананов, вокруг которых создали кольцевой
огненный барьер. Рядом с ней поставили ведро с водой. Когда обезьяна, бегая вокруг
огненного кольца, опрокинула ведро, вода загасила огонь. Так что обезьяна сумела сунуть
руку в центр круга и достать бананы.
Эксперимент повторили несколько раз и у обезьяны сформировалась устойчивая
логическая цепочка: «ведро» – «вода» – «путь через огонь» – «бананы». Затем эксперимент
усложнили: бананы поместили в огненном кольце на одном из двух плотов, на реке. Здесь же
есть ёмкость для воды, но пустая. На другом плоту стоит то самое ведро с водой. «Умная»
обезьяна не без труда перебирается на другой плот, с трудом перетаскивает оттуда ведро с
водой и, торжествуя, заливает огонь! А зачерпнуть подходящей посудой воды из реки не
догадалась. Вот что значит не знать «принцип срезания угла»!
Конечно, мышление обезьяны не позволяет ей правильно выделить основной элемент
решения, ключевое слово, понятие. Для неё это, видимо, «ведро с водой», а не просто «вода».
Плохо обезьяна проводит первичный анализ «пространства проблемы»…
Отсечь от глыбы мрамора всё лишнее
Ну, вот мы осмотрелись, очутившись внутри задачи. Увидев всё возможное,
обнаруживаем: информации многовато для того, чтобы всю её держать в голове и
постепенно отсеивать лишнее. Тогда мы скрупулёзно и педантично, в лучшем бухгалтерском
стиле, фиксируем сведения о задаче – и её условия, и наши мысли по этому поводу. Охватить
всю проблему целиком затруднительно или вообще немыслимо. Да и в руках у нас лишь
некоторые – на первый взгляд, даже никак между собой не связанные – фрагменты
информации. Значит, приходится пытаться представить весь океан по одной капле воды.
Если мозг выдаст интересную гипотезу, подскажет, где можно искать решение
(например, по аналогии с уже известной задачей), то, срезая углы, можно попробовать пойти
напрямик. Если удастся, оказавшись на островке Неизведанного, осмотреться и понять, что
мы попали, куда надо, значит, решение почти у нас в руках. Но ведь мы можем попасть и не
туда!
Откуда возьмётся интересная гипотеза? А вот вспомните о тех вопросах, которые, как
говорил Киплинг, помогают думать. Другое дело, что мало знать этих «слуг интеллекта» по
именам: умение ставить правильные вопросы не возникает по приказу или только из
осознания необходимости обладать этим умением. Просто же перечислять всевозможные
вопросы «обо всём» неэффективно. Требуется первоначальное препарирование проблемы, в
результате которого можно проследить или предположить историю (временную и
пространственную) каждой из частей задачи. Установление (пусть даже неверных)
причинно-следственных связей между отдельными её элементами уже позволяет
осмысленно ставить вопросы – ведь их нужно задавать по поводу чего-то конкретного. А
набор гипотез о поведении отдельных частей системы как раз и позволяет испытывать их
(гипотезы) уточняющими и дополняющими вопросами.
Давайте также вспомним один из важнейших принципов ландаматики – «принцип
снежного кома». Введём по аналогии «принцип снеговика» и используем его не для
обучения, а для того, чтобы последовательно приближаться к полному решению,
рассматривая отдельные, сравнительно простые части задачи. Скатаем сначала отдельные
маленькие «комочки», потом каждый из них «покатаем», пока не станет ясно, как он входит
в состав всей конструкции. В конце соберём всего «снеговика». То есть: сложную и большую
задачу мы разбиваем на отдельные части, маленькие задачи, которые легко исследовать и
решать. А потом переходим на иной качественно уровень рассуждений (вновь срезая угол!) –
и объединяем, сращиваем отдельные элементы решения.
Бигуди № 40
Благодаря Уотсону и Крику мы знаем, что молекула ДНК – гигантская двойная
спираль. При размножении она сначала «раскручивается» и разделяется на две отдельные
спирали. К каждой из них подсоединяются аминокислоты, и так образуются уже две
двойные спирали. Время удваивания молекулы ДНК (для бактерий кишечной палочки)
удаётся измерить с неплохой точностью. Но расчёты показывают: для объяснения
наблюдаемого времени удвоения спирали требуется фантастическая скорость
«раскручивания» спирали – почти 15 тысяч оборотов в минуту! Установить, в чём дело,
удалось не так уж и давно. Может быть, вы сами попробуете разобраться в ситуации? Ведь
перед вами явно «принцип снеговика»: смотрите не на всю спираль в целом, а на отдельные
её части! Собрать целое из частей – это тоже ведь часть нашего метода.53
А ещё потренируйтесь в отсекании лишнего на следующих словосочетаниях: патриот
Родины; продукты питания; прейскурант цен; пьяная оргия; автор произведения. Так какое
же из них лишнее и почему?54
Основы маневрирования
Итак, правила движения в «пространстве проблемы» сочетают различные принципы
эффективного мышления, логику и интуицию, догадку и медленный, но верный расчёт.
Сформулируем их ещё раз:
1) детальное изучение условия (принцип проникновения, принцип бухгалтера);
2) анализ, разбиение задачи на отдельные подзадачи, составные части, подзадачи
(принцип бухгалтера, принцип снеговика);
3) выделение нужной информации в результате тщательной сортировки (критерии, по
которым проводится сортировка, вырабатываются Вашим же мышлением на основе пп. 1 и
2; используются: принцип капли, принцип паутинки, принцип срезания угла);
4) ни в коем случае не должно быть ситуации типа «мои мысли – Мои скакуны»! –
Мышление требует последовательности, аккуратности и дисциплины;
5) и психологически важно – продолжать поиск, не сдаваться!
Между прочим, приведенные выше – и многие другие, здесь не упомянутые –
принципы ориентирования в «пространстве проблемы» заметно помогают справляться с
разными типами задач, встречающимися в тестах по проверке интеллекта. Главные этапы их
решения, оказывается, уже можно указать. Например, для логических задач это:
1) перебор возможных вариантов и запись их в виде списка или таблицы;
2) изучение каждого утверждения из списка с использованием специально
сформулированных критериев отбора.
Цель такого сравнительного анализа – поиск ключа к решению, первого и основного
звена в логической цепи. Отсекая лишние, противоречивые и тупиковые варианты111,
приходим к верному решению112. Здесь, впрочем, речь идёт о задачах с известным ответом.
Можно достаточно конкретно записать и основные приёмы решения словесных или
зрительно-пространственных задач. Все эти приёмы фактически описывают разные методы
выполнения умственной работы. Имея такой арсенал, проще подобрать и нужный
инструмент для решения задач определённого типа. Действительно, одинокая корова, как
сказано в давнем КВН, не может служить ориентиром. Даже в нашем мыслительном
пространстве. А вот принципы организации мышления могут быть прекрасными вехами при
решении задач.
13. Раз, два, три, четыре, пять, я опять иду искать…
– За мной, и, пожалуйста, не отставать! – сказала Мэри
Поппинс, оглядываясь через плечо с таким свирепым видом, словно
они спокойно шествовали по земле, а не летели по воздуху на
шариках.
П. Трэверс. «Мэри Поппинс»
В реальной жизни творческое мышление позволяет найти решение проблем, совсем не
похожих на задачи с наверняка имеющимся решением и однозначным ответом. В сложных,
поисковых задачах ответ не обязательно единственный. Ответа – по крайней мере, в
ожидаемом виде – Может и не быть вовсе.
Решение проблемы может существовать или не существовать в зависимости от вида
нескольких дополнительных – своеобразных граничных – условий. Творческие задачи, как
правило, носят такой характер, что решение их просто обязано быть нестандартным. Однако
и в поисках такого решения (или нескольких возможных решений) весьма полезны
оказываются эвристические принципы. В каждой задаче они используются в различных
комбинациях и на разных этапах решения.
Безумные полёты
Говорят: мышь, прижатая к стене, от отчаяния способна стать тигром. Так и мысль,
попавшая в тупик, может обратиться к совершенно невозможному – на первый взгляд! –
решению113. То, что казалось поначалу абсолютно безумной идеей, после «прокручивания»
111 Заметьте: это работают всё те же прескриптивные принципы.
112 Как отмечал Шерлок Холмс, когда будет отброшено всё заведомо невозможное, оставшееся будет
правильным, даже если оно совершенно неправдоподобно.
113 В фантастическом рассказе Т. Гудвина «Необходимость – Мать изобретения» космический корабль
совершает посадку на планете, вполне пригодной для жизни, но обречённой через несколько месяцев на гибель
от пролетающей поблизости соседней звезды. Изобилующей на планете алмазной пылью выведено из строя всё
в мозгу уже таковым не кажется. Что происходит? Мысль совершает отчаянный и
безрассудный «прыжок в неизвестное» и «приземляется» на другом берегу. Вот там-то,
оказывается, и удаётся найти необходимые элементы решения.
Для нахождения нестандартного и неожиданного решения этот «принцип отчаявшейся
мыши» (он близок «принципу паутинки») очень важен. Вот только нужно, чтобы мозг был
способен выдвигать такие сумасшедшие идеи! Но затем и тренируем его.
Иногда, впрочем, «безумное» решение – самое простое и практичное. Вот, скажем,
задача Короля из «Мэри Поппинс»: если двенадцать человек, работая по восемь часов в день,
должны выкопать яму глубиной в десять с половиной миль, сколько времени пройдёт –
считая и воскресные дни! – прежде чем они положат свои лопаты? Разве можно решать эту
задачу всерьёз? Приходится искать некое простое и прямолинейное решение. Кошка быстро
находит его: «Три секунды. За это время они, конечно, поймут, что им никогда не вырыть
такой ямы, да и рыть её незачем!»
Такое практичное решение – на самом деле результат иного взгляда на проблему,
умение «перепрыгнуть» через барьер условия. Чтобы увидеть возможность или даже
единственность «безумного» решения, нужно особое внимание к деталям «пространства
проблемы».
Бигуди № 41
Эта мышь не даёт вам спать уже третью ночь! На уговоры она не поддается, кота у вас
нет. Вам нужна мышеловка! Нечто простое, недорогое, но эффективное. Необязательно
супер-мышеловка, которая прослужит много лет – в конце концов, Вы не специализируетесь
на ловле мышей и не предполагаете жить на этой квартире долго, через неделю Вы
съезжаете. Но сейчас мышь нужно обезвредить! Итак, дешевая и эффективная
недолговечность взамен дорогой долговечности. Нужна мышеловка одноразового действия.
Вопрос: как сделать её, если у Вас есть только упругая пластмассовая трубка? Как уговорить
мышь войти туда так, чтобы назад она не смогла выйти? Положить приманку? Верно! Но как
сделать так, чтобы мышь, войдя в трубку за приманкой, не смогла оттуда выйти? Что-то
должно её не пустить обратно, но что? Сама трубка? Для этого нужно…55
Мелочи жизни
Один из важнейших советов по прикладной психологии некоторые авторы называют
«Крохотные истины». Суть этого специального упражнения для тренировки внимания в том,
чтобы научиться в окружающей действительности выделять и фиксировать не только то, что
сразу бросается в глаза, но и различные мелочи. Впрочем, при ближайшем рассмотрении они
могут оказаться и не такими уж и несущественными.
Например, выглянув из окна гостиницы в новом, ранее неизвестном городе, можно
заметить сначала лишь общие архитектурные особенности, какой-то памятник в сквере,
вывеску магазина в доме напротив. Но хорошо бы обратить внимание и на расположение
остановок городского транспорта, и на то, что прохожие легко одеты и не держат в руках
зонты. Иначе придётся на всякий случай быть готовым к дождю, таскать с собой зонт целый
день, ибо, как говорил Козьма Прутков, неизвестно, какие сюрпризы готовит нам атмосфера.
Внимание к мелочам полезно не только в этой простой ситуации. Оно тренирует
кратковременную, оперативную, память. Учит человека не только фиксировать мелочи, но и
расставлять в определённом – диктуемом их внутренними связями – порядке. То есть заодно
движущееся оборудование корабля – приходится взамен сооружать конструкции, где движутся только газы,
жидкости да электроны. Двигатели корабля взорвались от попадания алмазов в реактор – и экипаж вынужден
изобрести антигравитационный привод. А когда обнаруживается, что и обшивку залатать не удастся –
Мощность нового привода наращивают настолько, чтобы сделать кораблём всю планету.
и анализировать. Требуется лишь научиться тщательно «сканировать» взглядом обстановку,
запоминая всё, что встречает глаз – какие предметы находятся рядом с Вами или друг с
другом, какие неподвижны, что движется и в каком направлении. Затем уже можно
включить анализ и объяснить себе, что почему и куда движется, попробовать найти логику в
расположении предметов, мысленно их переставить и перечислить.
Первый этап фактически представляет собой глубокое погружение в окружающий мир,
некое созерцание, при котором мозг сосредоточенно перечисляет и запоминает предметы и
явления.
Этот метод созерцания обстановки напоминает одну из разновидностей упражнения в
концентрации мысли, издавна применяемую тибетскими монахами. Суть их техники – в
выборе какого-нибудь пейзажа, например, сада. Его следует созерцать во всех подробностях,
запоминая растущие в нём разнообразные цветы, их группы, деревья, для каждого из них –
высоту, форму ветвей, различия в листве, короче говоря – все детали, которые только можно
заметить. Создав для себя отчётливую картину сада, видимую с закрытыми глазами так же
ясно, как и с открытыми, начинают из сочетания признаков, составляющего сад, мысленно
удалять одну за одной различные детали. Цветы постепенно теряют окраску и форму,
рассыпаются. Рассеивается даже оставшаяся на них пыль. Деревья лишаются листвы, их
ветви сжимаются, уходят в ствол. Ствол утончается, превращаясь просто в прямую линию.
Эта линия становится всё тоньше и наконец исчезает. В результате остаётся только голая
земля. У земли последовательно отнимаются цветы, камни. Сама земля тоже постепенно
исчезает.
Такой метод концентрации используется в Тибете также для медитации.
Интересно бы использовать аналогичный метод для «загрузки» некоторой задачи в
подсознание. Представьте пространство вашей проблемы, задачи, ситуации в виде сада, по
которому вы идёте. По дороге встречаются известные частные факты и детали – представьте
их в виде цветов, растений, кустов, обойдите вокруг каждого, рассмотрите подробно.
Пройдите сквозь весь сад, постойте у каждого куста или дерева. Затем попробуйте
«пересадить» кусты и деревья в другом порядке. Рассмотрите всякие варианты устройства
сада, выйдите, вспомните ещё раз, как выглядел сад при разных способах рассадить в нём
кусты и деревья. И уходите, оставляя в себе воспоминание о нём. Спорим, вы уже не
забудете «прогулок по саду»114? Вот так можно применить принцип проникновения в
«художественной форме». И будет ваше сознание или подсознание «гулять по саду», пока
задача не будет решена. Такой метод работает весьма эффективно.
А при надлежащей внимательности к условию (хорошо ориентируйтесь в
«пространстве проблемы»!) легко находится ответ и для такой шутливой и, на первый
взгляд, бессмысленной задачи. Вы капитан корабля, идущего в Cингапур (Либерию,
Марокко….) с грузом… (следует долгое перечисление характера и количества груза).
Помимо груза, на корабле в качестве пассажиров находятся… (следует список пассажиров с
указанием их возраста, веса, пола и т. д.). В конце концов условие заканчивается вопросом:
сколько лет (или какой цвет волос, или глаз или ещё что-либо) капитану?
Абсурдность вопроса кажущаяся: внимательный анализ сразу же показывает – смысл
вопроса определяется лишь первой фразой условия! Нестандартная, неожиданная постановка
задачи требует и нестандартного же решения. «Проникновенный» анализ проблемы
показывает несвязанность между собой всех её данных, так что не запутается в них только
нешаблонное мышление.
Бигуди № 42
114 Подобным приёмом пользовался легендарный мнемонист Шерешевский. Он мысленно расставлял слова
или числа, которые ему произвольно называли, вдоль знакомых улиц или коридоров. И запоминал таким
способом сотни случайных – никак друг с другом не связанных – объектов.
Опишем фокус, проведенный в домашних условиях школьником, неплохо знающим
физику. Он написал на листке печатными буквами слова «КОФЕ» и «ЧАЙ», наполнил водой
пробирку и предложил родителям сквозь неё посмотреть на каждое из этих слов. Одно из
слов осталось неизменным, а второе – перевернулось. В чём здесь дело? Как только станет
ясно, при чём тут пробирка с водой, Вы сразу сообразите, какое слово не изменилось. Или
наоборот – проще сначала понять, почему не меняется слово, а потом уж разберемся с
пробиркой? В общем, немного физики, немного симметрии… Будьте внимательны! Кстати,
можно ли обнаружить тот же эффект, глядя на эти слова через, например, аквариум –
параллелепипед?56
Свести задачу к предыдущей
Один из важнейших, на мой взгляд, принципов, работающий практически в любой
сложной задаче – «принцип сведения». Речь идёт всего лишь о том, чтобы упростить
сложную и запутанную задачу, свести её к некоторой другой, намного более простой задаче
(в математике этот метод известен под названием метода рекурсии – «возвращения»).
Вспомним: М.А. Розов определял новое знание как нечто неизвестное, сведенное к чему-то
ранее известному. Словом, голова удава, укусив за кончик собственного хвоста, может
обнаружить ответ там, где «весь опыт».
Заметьте – для работы этого принципа нам, возможно, понадобится использовать и
прочие вышеописанные принципы действий в «пространстве проблемы».
Принцип сведения легко понять с точки зрения действий мозга: ему проще работать в
ситуации, когда информации меньше. Если мы упрощаем условие, закапываем овраги и
срываем холмы в «пространстве проблемы» – путь к решению становится хорошо виден!
Кстати, если при решении сложной задачи Вам вспоминается похожая задача, но с уже
известным решением, значит, Вы уже свели Вашу задачу к более простой 115! Этот принцип
близок к известному среди изобретателей «принципу матрёшки»: решив «внешнюю» задачу,
переходим к решению подобной, но уже «внутренней», более простой.
Правда, есть и пределы упрощения. Если «переборщить» – получим совсем другую
задачу. И тогда даже после её решения придётся всё равно проникать в суть дела.
Упрощение, разбиение большой проблемы на ряд относительно простых подзадач
(«принцип снеговика») задаёт новые направления движения в «пространстве проблемы». Эти
направления особенно нужны, когдa первичный анализ показывает: мы в тупике! Нужен
отчаянный скачок! Нужны упрощающие предположения! Нужны подсказки! Откуда же им
взяться? Если надеяться не на кого, будем думать сами. Будем искать какие-то странные,
удивительные, неожиданные особенности, проявляющиеся в самой задаче.
Принцип сведения к известному в естествознании восходит ещё к Аристотелю: он
объяснял падение тел «понятным» желанием всех тел стремиться к центру Вселенной (по
тогдашним представлениям – к центру Земли). В XIX веке Джеймс Клерк Максвелл пытался
объяснять свои уравнения электромагнитного поля, сводя его к «понятным» шестерёнкам,
заполняющим всё пространство «понятного» упругого эфира. Майкл Фарадей был убеждён,
что силовые линии электрического или магнитного полей – «понятные» реальные упругие
струны. Исааку Ньютону была совершенно понятна корпускулярная природа света. И это его
убеждение в простоте и понятности такой механистической картины, затормозило развитие
волновой оптики почти на два века! Вот так – действительно, перебарщивать в упрощении
опасно.
115 Впрочем, иной раз старая задача оказывается сложнее новой – например, потому, что со времён её
решения мы обогатились искусством и познанием. Вспоминается старый анекдот о разнице математического и
физического мышления. Перед вами чайник, водопроводный кран, газовая плита и спички. Надо вскипятить
воду. Естественно, вы наполняете чайник, разжигаете плиту, ставите чайник на неё и ждёте, пока он закипит.
Теперь предположим, что чайник уже наполнен и газ горит. Физик поставит чайник на плиту и будет ждать.
Математик погасит плиту, выльет воду и тем самым сведёт задачу к предыдущей.
Бигуди № 43
Когда в Париже появилась знаменитая впоследствии башня инженера Эйфеля, у нее
было много противников. Ги де Мопассан был одним из наиболее известных её критиков
(среди них были также известный композитор Шарль Гуно, Александр Дюма-сын и многие
представители интеллигенции): он считал, что Эйфелева башня – бесполезная и чудовищная
конструкция, оскорбляющая вид любимого города. Если во время прогулки взгляд писателя
случайно падал на ажурные очертания башни, которую его друзья сравнивали с гигантской
фабричной дымовой трубой, настроение его немедленно портилось. Поэтому он всё время
искал место, откуда не мог бы видеть это невыносимое сооружение. Где найти такое место в
Париже, не слишком удаляясь от красивейшего района Парижа – Марсова поля, где и
установлена башня? Задачу знаменитый писатель решил просто – он нашёл, как сам
выражался, «… единственное место во всём огромном Париже, откуда её не видно». Там он
регулярно обедал. Где же это место? Как называется оно теперь (это уже вопрос на
эрудицию)? Не кажется ли Вам, что Мопассан действовал, может быть и неосознанно, но в
полном соответствии с «принципом матрёшки»?57
Подключение переменных
Понятно, что в условии задачи много различных неизвестных, переменных величин
(какие-нибудь X, Y, Z….). Сложность задачи в том и проявляется, что: а) этих неизвестных
слишком много; б) непонятно, независимы ли они или как-то связаны между собой; в) что
происходит, когда они меняют свои значения; г) в каких пределах они могут меняться.
Вот этот последний пункт имеет особое значение: если нам удаётся узнать, каких
предельных значений достигают переменные величины, а затем увидеть, как меняется
задача, переформулируется проблема, когда Х становится равен 0 (или когда часть
механизма вообще удалена, или когда некий человек не то что опоздает на 5 минут, но не
придёт совсем, или ещё что-либо) – тогда мы свели задачу к другой, родственной, но более
простой задаче.
Постепенно «включая» переменные величины, возвращая их от экстремумов, мы
находим, как они влияют на ход решения полной задачи. И является ли зависимость условия
от этих параметров непрерывной, линейной (когда малое изменение параметра способно
лишь слабо изменить ответ задачи), или «пороговой» – в этом случае от какого-нибудь
незначительного на первый взгляд сдвига резко меняется условие, смысл и ход решения.
Например, при X > 0 математическая задача зачастую требует принципиально иного
решения, чем при X = 0 (или: добавление ещё одной шестерёнки позволяет получить иное
значение скорости, или: появление, даже с запозданием, некоего человека совершенно
меняет ситуацию или даже всю жизнь…)
Вот ещё один пример из моей игровой практики в «Что? Где? Когда?». Нам
продемонстрировали музыкальные духовые инструменты – валторну и трубу – и прозвучал
вопрос: с какой целью валторна «скручена» в несколько раз?
Я к музыке имею весьма отдалённое отношение, но физическое образование у меня
хорошее. Как физик, я представляю: издаваемый инструментом звук зависит не от формы, но
от длины инструмента. Это – из НЗ, из моего запаса. Но ведь больше ничего мне «принцип
проникновения» не подсказывает!
Поскольку надвигается тупик, ищу возможность для мысленного прыжка. Эту
возможность подсказывает «принцип сведения» – сведём задачу к другой. Но как?
Видоизменив условие. Какой параметр задачи можно изменить? Форму трубы – вряд ли:
слишком уж она проста. А вот валторну можно в мысленном эксперименте «раскрутить»,
развернуть – и получится длинная труба!
Я родом из Средней Азии и часто видел там длинные трубы – карнаи, издающие
низкий, гулкий звук116. Ещё из детства помню, как эти карнаи после выступления разбирали
на несколько частей и складывали. Ясно – длинные трубы функционально неудобны.
Особенно в оркестровой практике. Следовательно, их могли сворачивать для удобства.
Но будет ли такой ответ полон117? Зачем тогда на валторне различные клавиши –
регистры? Следовательно, я ещё не рассмотрел полностью «пространство проблемы», не все
неизвестные параметры задачи проанализировал. Для чего служат переключатели регистров?
Они изменяют тон звучания. Но ведь разный тон имеют трубы разной длины. Значит, в одну
свёрнутую плотно – для удобства пользования – валторну «впихнули» сразу несколько труб,
разной длины. Регистры – просто удобный механизм переключения тона, т. е. перехода с
трубы одной длины к трубе другой длины. Так появляется полный ответ.
Обратите внимание: сведение задачи к иной, которую удобнее и проще исследовать, не
происходит механически, по инструкции. Требуется определённый запас Необходимых
Знаний, умение быстро его использовать, всё то же «срезание угла».
Что важно: принцип сведения или принцип снеговика в применении к некоторым
экстремальным значениям параметров (вот мы развернули трубу полностью и для начала
вообще выбросили переключатели регистров – так и получили нечто вроде среднеазиатской
трубы) совершенно алгоритмизуем. Конечно, некоторая инструкция нужна. Но сам принцип
не требует вспышки, озарения, прыжка интуиции.
Мы «медленно спускаемся с холма», методично осматриваем границы «пространства
проблем» (именно там концентрируются, собираются все случаи предельных значений
параметров задачи). Лишь потом, выделив особенности задачи и обобщающие гипотезы,
мышление готовится к полёту вместе с «паутинкой» – набором сведений и критериев.
Направление прыжка зависит от предлагаемых гипотез. Или от ранее незамеченной детали в
условии. Или от подсказки.
Бигуди № 44
Маленький Вовочка, твёрдый «хорошист», возвращается домой из школы. Мама
спрашивает:
– Тройки есть?
– Нет.
– Двойки есть?
– Нет.
– Замечания есть?
– Нет.
– Дай посмотреть дневник!
Просмотрев дневник, рассерженная мама отвешивает Вовочке славный подзатыльник.
За что?! Что привело маму в ярость (заметим: на все вопросы Вовочка ответил чистую
правду)? Приглядитесь к «области изменения переменных» этой простой задачи. Или
предложите свои варианты!58
Внешний толчок
Подсказкой, резко меняющей направление поиска решения, может послужить почти
любой информационный импульс. Вот любопытный пример. Проводится психологический
эксперимент. К потолку подвешиваются две верёвки, причём точки подвеса находятся на
116 Примерно так же выглядят и звучат карпатские трембиты (от латинского tromba – труба). Наверное, както похоже выглядели и библейские «иерихонские трубы».
117 Мы ведь должны использовать и весь доступный набор критериев для проверки правильности ответа.
определённом расстоянии друг от друга. Человек, стоящий на полу и держащий за конец
одну из верёвок, не может дотянуться до другой. Ухватившись за вторую верёвку, он
вынужден будет выпустить из рук первую. Задача же: двумя руками ухватиться за обе
верёвки.
После долго наблюдения за тщетными попытками поймать (медленно подтягиваясь, с
разбега, ногой) конец второй верёвки, психолог (экспериментатор, холодный наблюдатель
или сжалившийся приятель), проходя мимо озадаченного ловца верёвок, задевает
«нечаянно» за свободный болтающийся конец второй верёвки. Она начинает раскачиваться.
Вот это и есть прекрасная подсказка!
Чем сильнее качнёшь, тем больше амплитуда колебаний. Если привязать к концу этой
верёвки что-то тяжёлое (например, какие-нибудь плоскогубцы, которые тоже «случайно
завалялись» неподалёку), то можно вполне прилично её раскачать. Пока она будет
раскачиваться, можно успеть подбежать, схватить конец неподвижной веревки одной рукой
и – остаётся дождаться, пока качающаяся верёвка не подлетит поближе. Задача решена.
Но можно и не ждать подсказок «от Природы». Почему бы не поискать в уже
известном, увиденном, услышанном что-то похожее, аналогичное?
Аналогии – не только приём, облегчающий понимание или описание явления. Они
дают возможность развернуть своё мышление к другим областям знания, другим явлениям и
обнаружить в них – пусть не полное! – сходство с нашей задачей. Только прежде чем
использовать или искать эти сходства и различия, нужно понять, с чем именно мы ищем
аналогии, в чём особенность изучаемых явлений и процессов. Сначала нужно выявить
специфические черты наблюдаемых явлений или предметов, а только потом рассматривать
иные информационные области в поисках аналогий с ними.
Бигуди № 45
Инженеру Самюэлю Брауну поручили придумать новую конструкцию моста: он
должен быть лёгким, экономичным в проектировании и строительстве, желательно –
оригинальной конструкции! Лежа под деревом, инженер перебирал в голове различные
известные конструкции, одновременно пытаясь придумать, как их модернизировать,
улучшить. Но ничего заслуживающего внимания не приходило в голову. И вдруг он увидел
на ветке… Вот он – эскиз будущего моста! Что же такое увидел инженер Браун на дереве?
Какой новый тип мостов он ввёл в практику строительства мостов?59
Переключение
Наверняка каждому из нас в жизни приходилось говорить с сожалением что-то вроде:
«Сколько сидел, ничего не получается. И так пробовал, и так. Голова уже пухнет». Так вот,
над одной задачей не имеет смысла сидеть слишком долго – когда она не выходит, мозг
устаёт, его способность выдавать новые варианты решения заметно снижается. Ходить
одними и теми же путями не просто надоедает – крайне утомительно! Необходимо
переключение. И сразу обнаруживается: голова опять на что-то способна – подключаются
дополнительные нейронные сети, открываются другие хранилища памяти, строятся новые
логические пути.
Мозг активно изучает новую проблему – уж здесь-то всё получится! Но и о прежней,
нерешённой, не забывает: «озадаченный» мозг – Может быть, даже из самолюбия: я, да не
могу?! – не смирится с отсутствием решения и передаст проблему «на рассмотрение»
подсознания118. Причём решение может появиться неожиданно для Вас, но не для мозга –
он-то знает, что и не прекращал работать над задачей: сравнивал данные, отыскивал
118 М. Левин называет его «инкубатором идей».
аналогии, перебирал варианты.
Ключевым, решающим вкладом в победу над задачей может стать совершенно
случайная, на первый взгляд, информация, приходящая с изменением даже просто условий, в
которых Вы работаете. Любой новый элемент обстановки, другое освещение, по-другому
записанное или сформулированное условие – все эти детали способствуют появлению
ассоциативных рядов или аналогий, ведущих к цели
Извилины безусловно надо напрягать, но не случайно озарения наступают в моменты,
когда казалось бы ты о проблеме не думаешь, расслабляешься. И само сознание
расслабляется, а где-то там, внутри, дозревает и крисаллизуется мысль, и вот в такой-то
момент и начинает действовать вулкан информации из подсознания. Известные
натурфилософы современности, например, Ганс Селье, полагали, что озарение наступает в
пограничном состоянии, «где-то» между сознательным и бессознательным. Именно Селье в
книге «От мечты к открытию» заметил, что переходы от сна к бодрствованию и обратно,
недомогание и выздоровление меняют работу мозга, переключают его. Извилины,
безусловно надо напрягать, но чтобы мысль закрутилась, чтобы она выкристаллизовалась, а
это происходит в подсознании, необходимо дать расслабиться сознанию!
Бигуди № 46
Существует древняя легенда об основании города Карфагена. Дочь тирского царя,
Дидона, потеряв мужа, убитого её братом, бежала в Африку. Там она купила у нумидийского
царя столько земли, «сколько охватит воловья шкура». Когда сделка состоялась, на глазах у
царя Дидона выхватила острый нож и… Что же такое сделала Дидона, что сумела построить
крепость Карфаген на своем участке земли (впоследствии там возник великий город,
Карфаген). Обращайте внимание на мелочи! Неужели Дидона пригрозила царю
нумидийцев?60
Между прочим, в разрушении Карфагена решающую роль сыграли факторы
субъективные. И главным было вовсе не упрямство Марка Порция Катона Старшего,
прославившегося повторением в конце каждой речи: «Кроме того, полагаю, что Карфаген
должен быть разрушен». Всё было решено гораздо раньше – причём в самом Новом Городе
(именно так переводится финикийское Карт Хадашт).
Когда Ганнибал буквально выпрашивал у родной торговой республики подкрепления,
чтобы добить ослабленную, но ещё боеспособную армию противника, почтенные купцы
сомневались. Род Барка (молния), к коему принадлежал знаменитый полководец, и так
богатейший в Карфагене – всей Испанией владеет119. Если победит – ещё и Италию
присоединит. Тогда с ним вовсе конкурировать невозможно будет. Пусть уж лучше
разорится или вовсе погибнет – а Рим и без него найдётся кому разбить. Не нашлось…
Похожее поведение и мотивацию мы наблюдаем и в наши дни… и совсем рядом.
Поговори со мной
Между прочим, каждый на своём опыте знает: когда задача «не выходит», так и тянет
обсудить с кем-нибудь свой вариант решения, посоветоваться с приятелем, учителем или
просто умным человеком. Если даже никто не сможет указать точно верный путь к ответу –
всё равно в разговоре могут всплыть такие – ранее незамеченные – детали, которые помогут
в дальнейших поисках.
Понятно, такое общение – причём не только со специалистами именно в таких задачах
– подразумевает интенсивный обмен информацией. Каждая новая мысль, высказанная
119 Нынешняя Барселона основана отцом Ганнибала и названа в честь всего рода.
собеседником, может стать «камнем, брошенным в пруд» мышления (или подсознания), и
вызвать на нём «рябь» ассоциаций.
Конечно, умение и возможность не только общаться на тему конкретной задачи, но и
вообще вести обоюдоинтересный и полезный разговор, обмениваться мыслями (разумеется –
нетривиальными, нестандартными, требующими определённого напряжения ума), с одной
стороны, предполагает наличие уже достаточно развитого интеллекта, с другой стороны,
создаёт условия для интенсивного обмена информацией. А, значит, повышает вероятность
получить нужный толчок в правильном направлении.
Между прочим, отмечено: навыки решения различных творческих задач применимы и
при разрешении многих жизненных проблем. Конечно, жизнь ставит задачи, где ответ либо
не единственный (тогда требуется какой-либо критерий оптимальности – для выбора из
нескольких почти равноценных ответов), либо возможен при некоторых дополнительных
условиях (их придётся домыслить самому).
Задача может быть вообще внутренне противоречива: ответа по сути дела нет, но
находить какой-то выход нужно! И мозг ищет оптимальную, с наименьшими потерями,
«стратегию выхода из ситуации». Или же решается переформулированная задача: ищется
такое – как можно меньшее! – изменение её условий, чтобы доминанта задачи оставалась
неизменной, но решение уже существовало.
Есть, вообще-то, ещё один удобный подход для нахождения решения вашей жизненной
проблемы: нужно отстраниться от неё, отсечь лишние эмоции, а сделать это можно,
сформулировав эту проблему в виде «задачи из учебника». Тогда её можно записать,
подобрать подходящий для её анализа мыслительный инструментарий, задать недостающие
начальные условия, выдвинуть рабочую гипотезу и т. д. Как видите, методы практически те
же, что и выше – т. е. наши принципы, вооружившись которыми, мы совершали прогулки в
«пространстве проблем», универсальны.
Бигуди № 47
Может быть, вам неизвестна эта замечательная задача выдающегося математика,
академика В.И. Арнольда, ушедшего от нас в 2010-ом году: на книжной полке рядом стоят
два тома Пушкина, первый и второй. Страницы каждого тома вместе имеют толщину 2 см, а
каждая из обложек имеет толщину 2 мм. Неграмотный червь прогрыз (перпендикулярно
страницам) от первой страницы первого тома до последней страницы второго тома. Какой
длины путь прогрыз голодный червь? Как утверждает Арнольд, эта задача совершенно
недоступна академикам, но с ней неплохо справляются дошкольники. А к какой категории
вы отнесете себя? В качестве подсказки: сам Арнольд называет эту задачу топологической.
Добавлю ещё интересное и, возможно, полезное для решения примечание Арнольда: «…
редакторы (журнала «Успехи физических наук», где эта задача была приведена автором в
тексте статьи, чтобы пояснить различие в подходах к делу математиков и физиков) в отличие
от дошкольников, на опыте с которыми я основывал свои планы, решить задачу не смогли,
поэтому изменили условие, чтобы подогнать его под указанный мной ответ…, так: вместо
«от первой страницы первого тома до последней второго» набрали «от последней страницы
первого тома до первой страницы второго»».61
14. Мысленный эксперимент
Я уже говорил: значительная – а порою и решающая – часть научного поиска проходит
в форме мысленного эксперимента. Здесь же надо подчеркнуть: такой эксперимент полезен
не только в науке. Он и в повседневной жизни позволяет свести неизвестное к известному. И
тем самым понять – а то и создать – нечто новое.
По большому счёту любая задача, в которую человек вторгается прежде, чем поставит
мысленный эксперимент, – это чёрный ящик. И, казалось бы, если не знаешь устройства, что
у него внутри, начни копаться – по информации на выходе сообразишь о содержимом
«ларчика». Так думает подавляющее большинство, и действие у них опережает
мыслительный эксперимент. Так живут и работают годами многие.
Космическая стрельба
Когда я учился то ли в 7-м, то ли в 8-м классе, на республиканской физической
олимпиаде предложили одну из легендарных задач академика Капицы. Была даже такая
брошюра: «Задачи академика Капицы. Сотня задач и все без решений». Великий физик
ценил не столько конкретные знания, сколько умение проникнуть в суть проблемы, а уже
оттуда определять, какие именно законы, таблицы и константы нужны для её решения.
Потому и придумал задачи, проверяющие и тренирующие это умение.
Итак, нам предложили представить себе космонавта в свободном пространстве – и с
двумя пистолетами в руках. И спросили, в каком случае он улетит дальше: если выстрелит из
обоих пистолетов одновременно – или сначала из одного, а затем из другого.
Я горжусь тем, что – единственный из участников олимпиады – в своём решении
прежде всего оговорил, что задача не имеет однозначного решения в общем случае.
Мысленно представив себе всевозможные картины развития событий, я понял: в
зависимости от положения пистолетов возможны бесчисленные варианты движения.
Наиболее прост анализ случая, когда космонавт держит пистолеты у пояса – на уровне
центра масс – стволами вперёд, как ковбой при стрельбе «от бедра». Если в таком положении
выстрелить из одного пистолета, значительная часть усилия отдачи уйдёт на закручивание
тела вдоль продольной оси. Соответственно второй выстрел должен будет погасить это
вращение. На перемещение же тела как целого уйдёт лишь часть малая импульса отдачи.
Если же стрелять из обоих пистолетов сразу, то вращательные моменты взаимно погасятся, и
весь импульс уйдёт в продольное движение. Значит, перемещение будет больше.
При других положениях пистолетов можно получить иной результат. Например, если
оба раза вытягивать руку так, чтобы ось ствола прошла через центр масс, импульс тоже
будет целиком уходить в продольное движение, и последовательные выстрелы дадут тот же
результат, что и залп от пояса. Но тогда поза от выстрела к выстрелу будет меняться – а
изначальная формулировка неявно подразумевает постоянство позы.
Полив при заморозках
К сожалению, далеко не всегда мысленный эксперимент выстраивается быстро и
просто. И тогда новая ситуация надолго остаётся оторванной от уже известных знаний.
Помню одну прогулку с отцом – известным в Узбекистане физиком, заслуженным
учителем. Мы проходили мимо яблоневого сада. Стоял мороз: в континентальном климате
Узбекистана зимы бывают очень холодны. И я поразился, увидев, что поливальщики
пускают по каналам сада воду – таким же щедрым потоком, как летом.
«Что за нелепица?» – спросил я у отца. – «Что, воду девать некуда?»
Он усмехнулся: «Ты же прошёл университетский курс физики. Почему не применяешь
знания, полученные на занятиях?» Я сперва даже не понял, какие знания нужны. А отец
объяснил: «В данном случае можно обойтись даже школьным курсом. Видишь, какой мороз?
Ночью, наверное, будет ещё сильнее. У яблонь корни близки к поверхности. Если они
замёрзнут – весь сад погибнет. А вода, замерзая, выделяет очень много тепла. Пока она вся
не замёрзнет – температура около неё ниже нуля не опустится. Поливальщики согревают
корни».
Я тогда уже окончил физический и биологический факультеты Ростовского
университета. Успешно сдал тяжелейшие курсы – вроде термодинамики и статистической
физики. Позднейшая работа показала: уровень полученного мною образования был вполне
серьёзен. Но в тот момент знания ещё не устоялись в голове, не сложились в цельную
систему. Потому и не стыковались с простейшим жизненным опытом.
Тогда я начал понимать, что такое знание, начал задумываться о том, чем знание
отличается от информации. И только информация примененная, для сопоставления с
реальной действительностью, в виде задачи, того или иного жизненного вывода – и есть
знание, как и утверждал профессор Крушинский. Нужно быть как можно ближе к практике.
Учитель преподает физику и тут же приводит пример, волнующий школьников
(современные автомобили, электромобили и т. д.). Он должен перескакивать на подобные
«подтемы». Вот я сегодня, двигаясь на велосипеде с большой скоростью, должен был
совершить прямоугольный поворот. Сегодня был дождь, и если вчера я практически
тормозов не нажимал, а просто сделал сильный наклон, и это позволило сделать крутой
поворот – то сегодня я затормозил. Потому что, как человек понимающий физику в
практике, знаю: если сегодня сделать наклон – при очень маленьком коэффициенте трения,
то велосипед не удержится, а если я не сделаю наклон, то я не впишусь в поворот.
Соответственно мне приходиться гасить скорость. Это самый настоящий жизненный пример,
демонстрирующий связь с законами физики. Другое дело с математикой. её сложнее все
время соразмерять с практикой. Но математика – это несколько иная наука, которая
позволяет человеку развивать мощь абстракции. И это надо объяснить: абстракция позволяет
человеку обойти преграды, непреодолимые экспериментально. Не понимая в чем дело, но
через поле математики обойти и найти выход. Но в этом-то и есть связь с практикой.
Поэтому можно утверждать, что математика нужна каждому, и она примет даже того, кто,
казалось бы, находится в совершенно другой области. Неслучайно Ломоносов говорил:
«Только для того стоит её изучать, что она голову в порядок приводит».
Еще один пример, того как изобретательский прием работает в повседневной жизни.
Сам я родом из Узбекистана. Страны, традиционно бедной дровами, живущей на
привозном угле. И тем не менее, прежде мы пользовались печным отоплением. Топить печь
не только трудоемкое, но финансово емкое дело На учительскую зарплату нельзя и по сей
день купить столько высоко качественного угля, чтобы хватило на весь отопительный сезон.
И с решением этой проблемы моим отцом связано у меня одно из ярких детских
впечатлений. Дело в том, что раньше в Центральной Азии традиционным топливом был так
называемый кизяк. В России это добро именуют коровяком. Короче, сушенными коровьими
лепешками, если не оскорбить слух иного телезрителя более крепким словом. Их
заготавливали в большом количестве пастухи. Для своих очагов. Отец же почти задаром
приобрел тонну бросового угольного порошка, заплатил разве только за перевозку. Угольная
пыль считалась только отходом, который применяли в специально оборудованных
котельных.
Из всего того добра, что накапливалось под нашей коровой, замешав его на угольном
порошке, отец делал густую смесь. А мы, дети, помогали ему, специальными формочками
делая из неё брикеты. Под жарким южным солнцем брикеты быстро высыхали, так что их
можно было складировать в штабеля в сарае. Зимой же я увидел, как они ярко горят, и печь
гудела посильнее, чем если бы мы топили дровами или углём. Фактически мы использовали
попутное бросовое сырьё. Пыль позволяет осуществить более интенсивную газификацию
угля, способствует его полному выгоранию, а органический горючий наполнитель как бы
склеивал пыль воедино.
В этом новаторстве действовал принцип объединения в пространстве двух разных,
казалось бы, вещей, что и породило новое качество. Точнее, общее, что было у угля и кизяка
– это их горючесть. И она была многократно усилена за счёт синергетики – сложения заранее
во времени и в пространстве двух способностей к горению. Восстановленная целостность
угля позволяла обойтись без того, чтобы распылять уголь каким-то хитрым образом.
Устранение мелкодисперсности приводило к тому, что и сгорание дармового «композита»
происходило постепенно.
Ещё одно детское впечатление: когда я учился в шестом классе, до нас, наконец, дошел
газ. Дома провели паровое отопление. Прораб, руководивший проводкой, сказал, что «за
вами остаётся изоляция труб на чердаке и тех, что снаружи дома». Он при этом назвал какойто дефицитный и дорогой по тем временам изоляционный материал. Отец размышлял где-то
около суток, потом он собрал нас и объявил, что можно гораздо лучше изолировать трубы,
используя подручный материал. Обратим внимание, что изобретатель, вероятно, задумался и
о тяжести конструкции, ведь трубы были на чердаке, то есть решал сразу несколько
технических противоречий.
Мы тогда выписывали огромное количество газет, которые копились по прочтении
пачками. Отец показал нам, как надо сворачивать трубки во всю ширину газетного листа, а
чтобы они не разворачивались, поверх наматывали капроновую нить из распущенных старых
чулок. Изготовив не одну сотню трубок, мы с братьями стали с интересом ждать, что же
будет дальше, и зачем это всё отцу понадобилось. А он обложил трубы парового отопления
со всех сторон трубками газетными и спаял сверху изолентой. И так по всей длине. Газетная
бумага и капроновая нить мне представлялись такими непрочными материалами, но так или
иначе температура на выходе держалась градусов на десять выше, чем предсказал тот мастер
парового отопления. Да и когда мы через двадцать лет переехали из того дома, а покупатель
осматривал свою будущую собственность, вся изоляция оказалась на месте и исправно
сохраняла тепло.
Так что знания – не просто накопленная информация. Это ещё и умение применять её.
Искусство выходить в то информационное пространство, где удобнее всего преобразовать
новые сведения, сочетать их с уже известными. И, конечно, искусство вернуться из
воображаемого пространства в реальность, применить новую мысленную структуру к
практике.
Людоедка на экране
Конечно, нужен определенный компромисс: если известно слишком мало, нет важной
информации, то и неизвестное не к чему сводить. Помню, мой трёхлетний сын смотрел
известный американский сериал «Скорая помощь». В очередном эпизоде шло кесарево
сечение. Тяжёлая операция, на экране потоки крови – и вдруг раздаётся детский крик.
Сын спросил: «Почему резали тётю, а появился ребёнок?» Я ответил: «Его достали у
тёти из животика». Сын задумался и через несколько минут спросил: «А почему тётя съела
ребёнка?»
Согласитесь, это типичный пример общеизвестной детской гениальности. Ребёнок
всегда пытается свести новое к тому, что уже знает. Трёхлетний мальчик не удовлетворился
полученным от меня ответом – это само по себе похвально. Он поставил себе новый вопрос:
если ребёнка достали из живота, то как он туда попал? Это неизвестное для него он стал
сводить к тому немногому, что было ему уже известно. Что он знал про живот? Туда
попадает всё, что человек съест. Значит, если ребёнок оказался в животике, то всё наличное
знание мальчика показывает: его не иначе как съели.
Почему я так подробно останавливаюсь на таком простом примере? Потому что
мальчик мыслил методологически правильно. Именно такой методологии мы пытаемся
учить. Но если ребёнка не портить неправильным образованием, у него проявится именно
эта – на мой взгляд, генетически заложенная в человека120 – правильная методика
получения нового знания.
Чуковский в знаменитой книге «От двух до пяти» приводит замечательные
лингвистические примеры. Скажем, ребёнок произносит «колоток» вместо «молоток»,
потому что слово «колотить» ему уже знакомо. Или просит «отскорлупать» орехи, по
120 Механизмы мышления, не приспособленные к постижению окружающего мира, вымирают. К
сожалению, вместе со своими носителями.
обычным законам языка сводя воедино несколько слов. Только играя в такой языковый
конструктор, можно за 2–3 года овладеть всем богатством лингвистических законов,
созданных за десятки тысяч лет эволюции человеческой культуры.
Для ребёнка, писал К.И. Чуковский в памятной книге «От двух до пяти», выдержавшей
два десятка одних только прижизненных изданий, «невыносимо сознание, что он не
способен к тем действиям, которые у него на глазах совершают другие. Что бы кто ни делал
на глазах у двухлетнего мальчика, он в каждом видит соперника, которого ему надлежит
превзойти. Он не может допустить и мысли, что кто-нибудь другой, а не он будет
действующим, а стало быть, и познающим лицом в этом мире.
Дети только потому не пугаются собственной своей неумелости, что не подозревают об
истинных размерах её. Но всякий раз, как по какому-нибудь случайному поводу они
почувствуют, до чего они слабы, это огорчает их до слёз.
Ребёнок хочет быть Колумбом всех Америк и каждую заново открыть для себя. Всё
руками, всё в рот, – поскорее бы познакомиться с этим неведомым миром, научиться его
делам и обычаям, ибо всякое непонимание, неумение, незнание мучает ребёнка, как боль.
Мы все к двадцатилетнему возрасту были бы великими химиками, математиками,
ботаниками, зоологами, если бы это жгучее любопытство ко всему окружающему не
ослабевало в нас по мере накопления первоначальных, необходимейших для нашего
существования знаний.
К счастью, ребёнок не представляет себе всех колоссальных размеров того
непонятного, которое окружает его: он вечно во власти сладчайших иллюзий, и кто из нас не
видел детей, которые простодушно уверены, что они отлично умеют охотиться за львами,
управлять оркестром, переплывать океаны и т. д.
Великую радость должен почувствовать этот пытливый и честолюбивый исследователь
мира сего, когда ему становится ясно, что обширные области знания уже прочно завоёваны
им, что ошибаются другие, а не он.
Другие не знают, что лёд бывает только зимой, что холодной кашей невозможно
обжечься, что кошка не боится мышей, что немые не способны кричать «караул» ит. д. А он
настолько утвердился в этих истинах, что вот – Может даже играть ими.
Когда мы замечаем, что ребёнок начал играть каким-нибудь новым комплексом
понятий, мы можем наверняка заключить, что он стал полным хозяином этих понятий;
игрушками становятся для него только те идеи, которые уже крепко скоординированы между
собой.
Не нужно забывать, что именно координация, систематизация знаний и является
важнейшей, хотя и не осознанной, заботой ребёнка.
Умственная анархия невыносима для детского разума. Ребёнок верит, что всюду
должны быть законы и правила, страстно жаждет усваивать их и огорчается, если заметит в
усвоенном какой-нибудь нечаянный изъян».
Журналист из Ульяновска Анастасия Чеховская в статье «Главная детская тайна»
(электронная газета «Вести образования», 19.12.2011) затрагивает феномен, знакомый
каждому взрослому, но порядком подзабытый за давностью лет: «Тайный детский язык – это
не только элемент детской игровой культуры, детского фольклора, но и творческая,
развивающая игра, в которой есть место и ребёнку, и взрослому. К сожалению, сейчас они не
так популярны, как сто лет назад: другое время, другие социальные реалии, но фольклористы
полагают, что создавать секретные языки – это естественная потребность детей шести-девяти
лет. Секретный язык – это обучение через игру.
Прагматика секретных языков заключается в том, что дети ограждают себя от
насмешек взрослых, которых веселят все эти дома на деревьях, тайные убежища, вигвамы,
пароли и пещеры. Период секретов такой же естественный, как период вранья в три года
(умные люди не дадут соврать, что это показатель растущего интеллекта и включившегося
образного мышления). Это значит, что мозг развивается – и нужно направлять интерес
ребёнка в сторону чтения и самообразования.
Какая польза может быть от тайных детских языков? Тема секретного языка выводит
на тему головоломок, шифровок, ребусов, а позже – языков программирования и
криптографии. Для ребёнка с аналитическим складом ума польза будет заключаться в том,
что он довольно рано поймёт, что язык – это конструктор, из которого можно собрать всё что
угодно. Он научится чувствовать элементы языка: его фонетику, морфологию, синтаксис.
Для детей творческого, фантазийного склада секретный язык – это как банка варенья
для сладкоежки. Язык позволяет конструировать реальность. И законы, по которым
создаются секретные детские языки, прекрасно работают для языков сказочных. Как говорят
феи? Звёзды? Лошади, когда люди их не слышат? А как выглядят существа, которые говорят
тем или иным образом?
Следом за существами рождаются их обычаи, одежда, города и деревни, конфликты и
герои. Рождается эпос и целая смысловая вселенная. А значит, ребёнок получает стимул для
творчества словесного: рассказов, стихов, картин о фантазийных мирах».
Условные, «институтские», секретные, «тарабарские», «балабурные» языки относятся к
искусственным. Они были выделены филологами и этнографами ещё в XIX веке,
зафиксированы в произведениях Н.В. Гоголя, А.Н. Энгельгардта, В.И. Даля, Н.Г.
Помяловского, В.Г. Белинского и других. Научное освещение искусственные языки
школьников (заумный язык, языки с меной окончания основы, оборотные, или обратные,
языки) получили ещё в 1920-х годах. Кстати, презрительное наименование «тарабарский» «в
отдельных случаях было подсказано и закрепилось в детской среде насмешками взрослых,
осудительно относящихся к детским разговорам на искусственных языках, как к пустой
болтовне наподобие лепета «Тара-бара / Вчера была, / Сёдни не пришла».
Другое дело, что законы – только механизм переработки реальной информации. Если
её нет – результат переработки может оказаться совершенно нелепым. Математики и
программисты говорят: «Garbage in – garbage out» (мусор на входе даёт мусор на выходе).
К счастью, информацию можно накопить довольно быстро. Даже если со школой не
повезёт – открыты библиотеки, доступен Интернет… А вот умение перерабатывать эту
информацию нужно тренировать упорно и повседневно. Ставьте мысленные эксперименты
на любом доступном материале, ищите неизвестное на каждом шагу – все усилия окупятся.
Бигуди № 48
Еще задачка о детях. В одной детской больнице маленькие пациенты любили играть с
очаровательными плюшевыми мишками, которые были в палате. Малышам так сильно
мишки, что они стали исчезать: выздоравливая, маленькие пациенты уносили их домой. Что
придумали врачи, чтобы игрушки оставались в больнице? Ну, подумайте, что бы сделали
Вы, если бы Вам стало жалко больного мишку?62
И вот другой аналог этой задачи: один богатый и хлебосольный человек Х любил
приглашать многочисленных друзей и знакомых к себе на обед. Однако после нескольких
таких пиршеств прислуга стала докладывать о пропаже дорогих серебряных вилок – друзья и
приятели разбирали их, видимо, как сувениры, на память о прекрасном обеде, что ли.
Немного подумав, хозяин велел… После этого пропажи прекратились. Что же Х сделал с
этими вилками?63
15. Просто игра?
– Я не играю, – ответил Иа.
– А что же ты там делаешь? – спросил Кролик.
– Можешь отгадывать до трёх раз, Кролик. Рою землю?
Неправильно. Прыгаю по веткам молодого дуба? Нет, неправильно.
Жду, чтобы мне кто-нибудь помог выбраться из реки? Теперь
правильно! Дайте Кролику время подумать, и он всегда всё
отгадает!
А.А. Милн. «Винни-Пух и все-все-все»
Пожалуй, самый простой и приятный способ развития своего мышления – играть.
Можно, конечно, понимать игру крайне серьёзно, как, например, Франсуа Шатобриан: он
говорил, что игра в игру есть жизнь121. А можно относиться к игре просто как к тренировке
творческих способностей. Великий математик и философ Готфрид Лейбниц считал, что
наиболее изобретательно человек проявил себя в создании разнообразных игр.
Играми для ума психологи часто называют совокупность различных приёмов,
призванных «растормошить» мозг, поставить процесс генерирования творческих идей в
большом количестве на научную основу. Практически то же самое имеется в виду, когда
говорят о тренировке мышления, интеллекта с использованием игр (здесь под играми можно
понимать всё, что угодно).
Игровая интуиция и логика
В нашей коллективной монографии «Турбулентное мышление. Зарядка для
интеллекта» мы уже обращали внимание читателя на такой факт. В 2011 году во всемирную
электронную паутину была вброшена информация о том, что «развивающие игры являются
пустой тратой времени, поскольку совершенно не повышают IQ (intelligence quotient)
взрослых людей. Авторы ссылались на «крупное исследование с участием 11 000 человек,
проведенное в Великобритании», проведенное сообществом «ученых мужей», что цепляло за
оставшиеся в наличии извилины доверчивых пользователей Интернета, как это всегда делает
негатив и «фэйк», и как вообще вчерашние советские люди могут быть традиционно
доверчивы к слову прессы.
Если же внимательно присмотреться к «вбросу», то становится ясно, что там речь там
идет не о всех развивающих играх, а только о компьютерных, и даже не о всех
компьютерных, а лишь определенных, специально разработанных:
«За последние пять лет появилось обилие «развивающих» компьютерных игр, в
аннотации к которым говорится, что они «повышают интеллект» и позволяют «сохранять
мозг молодым». В этой связи руководство научно-популярной программы Bang Goes The
Theory, выходящей на телеканале BBC One, предложило ученым провести исследование,
которое должно было определить, насколько эти игры действительно развивают мозг и
улучшают такие способности человека, как память, логика, пространственное воображение.
Исследование возглавил Эдриан Оуэн, ведущий специалист по исследованию мозговой
деятельности Кембриджа. В эксперименте, который был широко разрекламирован в
телепередаче, приняли участие 11430 человек в возрасте от 18 до 60 лет. Результаты работы
опубликованы в журнале Nature. Данное исследование показало, что никакого повышения
своего IQ участники эксперимента не получили. Перед началом эксперимента каждый
участник прошел тест на оценку интеллектуальной деятельности мозга. Тесты были
разработаны при участии британских ученых из Медицинского исследовательского совета и
Общества борьбы с болезнью Альцгеймера. Затем испытуемые в течение шести недель
каждый хотя бы по десять минут «развивали мозг», играя в специально разработанные игры
на специальной страничке на сайте BBC. Всего участники эксперимента могли играть в
двенадцать игр, половина из которых требуют таких способностей мозга, как планирование,
решение задач и анализ, а другие шесть кратковременной памяти, внимания, знания
математики и интерпретации визуальной информации».
Из этого следует лишь тот печальный факт, что разработанные игры были вовсе не
121 Немного раньше мы с вами уже вошли ещё в одно пространство – пространство игры «Что? Где?
Когда?». Слова Шатобриана как нельзя лучше описывают характерный для такой игры способ жизни.
столь развивающими, как того хотелось. Но совершенно не следует, что традиционные
действительно интеллектуальные игры, известные веками, не способны существенно
повлиять на развитие. lQ, хоть и признанный, но весьма условный критерий для оценки
таланта и степени развития. Надо играть в реально развивающие игры, и начинать это делать
в весьма юные годы, для достижения лучшего эффекта. И не надо верить всему тому, что
пишут на разных интернетовских заборах (например, в блогах)!
Действительно, в седьмой главе работы «Использование латерального мышления»
Эдвард де Боно рассматривает роль случайности в истории научных открытий и технических
изобретений. Анализируя её, он приходит к выводу о высокой значимости игровой
деятельности для развития латеральных способностей мышления:
«Случайность может предоставить возможность увидеть нечто такое, чего никогда не
стали бы искать намеренно.
Этот фактор и определяет роль случайности при выработке новых идей. А коль скоро
это так, то могут существовать методы ускорения этого процесса. По-видимому, идеальным
методом ускорения является игра. Однако последняя должна быть абсолютно бесцельной,
без какого бы то ни было плана и направления. Так же как тщательная организация
эксперимента является попыткой подтолкнуть природу на путь логического исследования,
так и игра представляет собой попытку ускорить процесс случайного возникновения какогото явления, которое мы бы никогда иначе не нашли. Забавляться игрой – своего рода
эксперимент со случайностью. Такого рода игра далеко не лёгкое дело, ибо малейшие
сознательные усилия во время игры разрушают её назначение быть свободной игрой.
Ценность игры в данном аспекте не вызывает сомнения. Именно свобода от планов и
целей позволяет случайности столкнуть друг с другом такие явления, которые иначе никогда
бы не соединились, позволяет создать такую цепь событий, которая иначе никогда бы не
создалась. Кажущаяся бесполезность игры, как правило, отталкивает людей от подобного
занятия. Шаблонно мыслящие люди стыдятся играть, в то время как если чего и следует
стыдиться, так это неумения играть.
Джеймс Клерк Максвелл, один из величайших научных и математических гениев,
любил играть. Он мог в разгаре званого обеда, забыв об окружающих, заняться игрой со
столовыми приборами, лучом света, отраженным от стакана или капли воды. Максвелл знал
цену игре; ещё будучи подростком, он услышал лекцию одного художника, которыйдобился
успеха именно с помощью игры. С этого и началась его научная карьера. Играя булавками и
нитками, Максвелл установил, как с их помощью можно начертить овал (тут ошибка автора
или переводчика: этими средствами проще всего начертить эллипс, сходный с овалом
внешне, но существенно иной по математическим свойствам – авт. ), а отсюда пришёл к
объяснению законов отражения света. В те времена он был ещё настолько молод, что его
доклад в Эдинбургском королевском научном общество вынужден был читать кто-то другой,
поскольку человек в коротких штанишках ещё не мог выступать с кафедры.
Почему дети перестают играть? Возможно, потому, что таинственный мир, в котором
происходят чудеса, превращается в обыденный мир, где каждая вещь имеет объяснение.
Скучая от поверхностной осведомлённости, дети прекращают игру. Если же ребёнок не
довольствуется поверхностными объяснениями, то вещи никогда не станут настолько
известными, чтобы игра с ними стала скучной. Возможно, что игра активно не одобряется
логически мыслящими взрослыми, которые уверены в её бесполезности и определяют время
возмужания как обязанность заниматься полезной деятельностью.
Во время игры идеи возникают сами собой и, возникнув, порождают новые. Идеи не
следуют одна за другой в логической последовательности, однако если разум не
предпринимает никаких попыток управлять ими и в тоже время достаточно любопытен,
чтобы следовать за ними, то идей будет более чем достаточно. Полезность идеи может
проявиться не сразу; как правило, это происходит спустя некоторое время. Даже если
никакой конкретной идеи в голову не приходит, тем не менее общее знакомство с ситуацией,
возникающей в процессе игры, может оказаться весьма ценной предпосылкой для
дальнейшего развития идей».
Далее по тексту Боно оговаривает, впрочем, существенный психологический момент:
«трудность игры без видимой цели состоит в том, что она нередко воспринимается как
пустая трата времени; и никто не утешается тем, что впоследствии она может принести
дивиденды».
Конечно, как гласит пословица, «без труда не выловить и рыбки из пруда»! Игра
должна совершить множество циклов, прежде чем на её основе произойдёт прорыв в новое
измерение проблемы, родится новая идея.
«Деловая игра – это создание ситуации выбора и принятия решения, в которой
воспроизводятся условия, близкие к реальным. В ней предполагаются такие роли
участников, которые позволяют им осмыслить, пережить и освоить новые функции. В игре
содержится конкретное событие или явление, подлежащее моделированию, и допускается
отнесение игрового времени к любому периоду (настоящему, прошедшему, будущему). Как
правило, деловая игра – это модель отрезка будущей профессиональной деятельности
обучающихся. Это имитация управленческой, исследовательской, педагогической реальной
деятельности учителя, руководителя учебного заведения.
Отличительными признаками деловой игры можно назвать: имитацию в игре реального
процесса с помощью модели; распределение ролей между участниками игры, их
взаимодействие друг с другом; различие интересов у участников игры и появление
конфликтных ситуаций; наличие общей игровой цели у всего коллектива, которая
достигается в процессе взаимодействия игроков и объединяет всех её участников; учёт
результатов деятельности; реализацию в игре цепочки решений, каждое из которых зависит
от предыдущего, а также от решений, принимаемых другими участниками игры.
Несомненным достоинством деловых игр является то, что они соединяют теорию и
практику, способствуя формированию в том числе и профессиональных знаний, и
практических умений. Игры повышают интерес к изучаемому предмету, так как они
сопровождаются положительными эмоциями.
Деловые игры можно сгруппировать следующим образом:
1. «Разминочные» игры типа «мозговой атаки», «клуба знатоков», тематические
развлекательные игры. Их задача заключается в том, чтобы раскрепостить интересы и
воображение участников, активизировать игровую и коллективистическую мотивацию,
ориентировать на нестандартный подход к изучаемому материалу.
2. Ситуативно-ролевые игры. Включают в себя анализ конкретных ситуаций и их
ролевое проигрывание.
3. Конструктивно-ролевые, проблемно-ролевые, дискуссионные игры. Целью их
использования является формирование навыков принятия и эффективного исполнения
деловых ролей, обучение взаимодействию и сплочённости, продуктивному сотрудничеству,
участие в выработке коллективных решений.
4. Творческие игры. Это коллективное творчество по созданию технических,
художественных, изыскательских и т. п. проектов. Включение учащихся в эти игры
способствует развитию творческого потенциала, воспитанию инициативности, смелости,
настойчивости, ответственности.
Имеются и другие классификации деловых игр: управленческие, исследовательские,
учебные и т. д.
Игры (или методики развития интеллекта – по сути, это одно и то же) можно разделить
на два типа: линейные и интуитивные.
Первые удобны для развития возможностей оперировать с массивами информации,
создавать их, классифицировать, комбинировать информацию из разных массивов, т. е.
перегруппировывать данные, извлекать сведения по «ключевым словам» и т. д. Такие игры
развивают левое полушарие.
Интуитивные методики основаны на тренировке воображения, умения пользоваться
ассоциациями, включать подсознание (для чего нужно предварительно загрузить его
информацией), «ловить» возникающие идеи и пр. В этом случае активизируется работа
правого полушария мозга.
Реальный процесс творческого мышления должен сочетать, использовать в
необходимой степени обе стороны активности мозга – и умение работать с
информационными структурами, и способность порождать новую информацию на основе
известной. Психологи советуют работать с линейными и интуитивными играми для ума
«вперемешку», наугад задавая мозгу задачи из разных областей так, чтобы включать в
работу разные полушария мозга.
К числу линейных игр можно отнести так называемые метаграммы – составление
цепочек слов, в которых можно, заменяя по одной букве (удлинять или укорачивать слова
нельзя), перейти от одного слова к другому. Например, как сделать из «мухи» «паука»? Вот:
муха – Мука – лука – лупа – купа – кура – кара – пара – парк – паук. А из 5 сделать 7 можно
так: пять – путь – суть – сеть – семь. Кстати, сделать из «мухи» «слона» уже сложнее.
Проводятся даже конкурсы любителей метаграмм: побеждает тот, кто составит наиболее
короткую цепочку.
Слова, отличающиеся только порядком входящих в них букв, называются
анаграммами. Скажем: фарш – шарф, или апельсин – спаниель. А попробуйте сами
придумать анаграммы к словам атлант, лепесток или кулон.
Попытайтесь составить самостоятельно палиндром – фразу, одинаково читаемую слева
направо и справа налево. Например: Аргентина манит негра; я не реву – уверен я.
В 1817-м году француз Жан Франсуа Сюдр придумал свой вариант международного
языка – сольресоль. Он предложил составлять слова по определённым правилам из семи
музыкальных нот (то ли был большим любителем музыки, то ли подумал, что названия нот
уж точно знакомы всем народам мира122). Можете для тренировки сами попытаться
придумать такие правила и даже написать что-нибудь на «музыкальном» языке.
Сомнительно, впрочем, что вы ещё кого-то уговорите пользоваться этим языком123.
Кто рассказал, где брод, тот перешёл реку дважды
Редко кто задумывается над тем, что многие известные крылатые фразы и пословицы,
сродни парадоксальному «детскому» мышлению, имеют продолжения, которые в корне
меняют привычный смысл:
«Собаку съели, хвостом подавились»;
«Кто старое помянет – тому глаз вон, а кто забудет – тому оба»;
«Старый конь борозды не испортит, да и глубоко не вспашет»;
«Ума палата, да ключ потерян»;
«О мёртвых либо хорошо, либо ничего, кроме правды» и т. д.
Это свидетельство того, что справедливость любого выражения (тезиса, аксиомы и
т. п.) не является абсолютной. Только мы, в отличие от детей, скованные своим опытом, не
хотим в том себе признаться.
122 В этом случае он, кстати, был неправ. Например, восточная музыка строится не на семи, а на пяти
основных нотах. А почти через век после творения Сюдра Арнольд Шёнберг выстроил додекафоническую –
двенадцатитоновую – систему композиции, где все 12 нот, отчётливо различимых нормальным слухом в
пределах одной октавы (двукратного изменения частоты колебаний), равноправны. Строго говоря,
человеческий слух различает в октаве 24 частоты, но сочетания частот, отличающихся столь мало, заведомо
неблагозвучны, а потому для музыки нужно использовать звуки, отстоящие друг от друга вдвое дальше
различимого минимума.
123 Сюдр, несмотря на множество похвальных отзывов из разных академий и даже премию лондонской
всемирной выставки 1851-го года, не добился, чтобы на сольресоле хоть кто-то заговорил в повседневной
жизни.
Попробуйте выбрать одно или несколько понравившихся выражений и вывернуть их
смысл, доказать их ошибочность.
А.Н. Лук продолжает череду подобных фраз: «Люди часто пользуются привычными
положениями, установившимися формулировками, которые отражают их коллективный
опыт. Иногда эти привычные выражения подвергаются как будто незначительной
перефразировке – в результате их смысл утрачивается, извращается или меняется на
противоположный. При этом может получиться бессмыслица, но порою в кажущейся
бессмыслице открывается новый, более глубокий смысл.
Вот образцы парадоксов Оскара Уайльда:
Ненужные вещи в наш век единственно нам нужны.
Холостяки ведут семейную жизнь, а женатые – холостую.
Ничего не делать – самый тяжкий труд.
Если скажешь правду, все равно рано или поздно попадешься.
Естественность – это поза.
Время – потеря денег.
Своя рубашка дальше от тела.
Я интересуюсь лишь тем, что меня совсем не касается.
Я могу поверить лишь невероятному.
Эта женщина и в старости сохранила следы изумительного своего безобразия.
Когда люди соглашаются со мною – я вижу, что я неправ.
У него было одно из тех, типично британских, лиц, которые стоит увидеть однажды,
чтобы уже не вспоминать никогда».
«Метафоры в языке обладают выраженной эвристической функцией, имеющей
значение не только в поэзии, но и во всех областях научной и практической деятельности. Их
главная функция заключается в генерировании идей», – считал известный советский
разработчик разных аспектов теории творчества Г.Я. Буш и цитировал при этом крупного
англо-американского философа и математика Макса Блэка (Максим Чёрный родился в Баку,
но уже в 1912 году вывезен родителями в Лондон, 1909–1988): «Каждая наука должна
начинаться с метафор и кончаться алгеброй, возможно, что без метафор никогда не было бы
никакой алгебры». «Похвалить философа за метафору – всё равно, что похвалить логика за
красивый почерк», – остроумно замечал Блэк, но сам считал метафору начальным актом
творчества, совместимым с серьёзным размышлением.
Одним из способов метафоротворчества, отмеченным многими известными умами,
выступает сравнение по неявному признаку. «Нетрудно убедиться, что внезапность,
присущая истинному остроумию, проявляется и в сравнениях по неявному или случайному
признаку, когда в непохожих и несравнимых предметах выделяют неожиданное свойство,
которое позволяет провести сопоставление:
Закон, что столб: преступить нельзя, а обойти можно.
Совет подобен касторке: его легко подать, но чертовски трудно принимать.
Девицы вообще подобны шашкам: не всякой удается, но каждой желается попасть в
дамки.
Специалист подобен флюсу: полнота его односторонняя
Многие люди подобны колбасам: чем их начинят, то и носят в себе.
Человек с мелкой душой подобен бутылке с узким горлышком: чем беднее его
содержимое, тем больше шуму, когда оно изливается.
Писатели как монеты – чем старее, тем дороже; видно, мы ценим ржавчину больше
золота.
Жизнь подобна сказке – не важно, насколько длинна, а важно – насколько хороша.
Дж. Свифт сравнил обещания с коркой от пирога: их для того и пекут, чтобы потом
сломать.
«Какое сходство Клит с календарем имеет?
Он лжет и не краснеет». (П. Сумароков – по кн. «Юмор, остроумие, творчество», А.Н.
Лук).
Бигуди № 49
Еще один тип игры со словами – омонимы (от homos – одинаковый, onyma – имя),
загадки, в которых задумывают слова, одинаковые на вид и по звучанию, но имеющие
разный смысл. Их использовали во Франции XVIII века при составлении каламбуров, в
Германии они считались стихотворениями. Итак, разгадаем омоним:
Первую в школе все изучают.
Ну, а второй из двустволки стреляют
Третью исполнят нам два барабана,
Иль каблуки отобьют её рьяно.
Ответом будет лишь одно слово!64
Между прочим, потренируйте своё мышление и на таких обычных загадках. Например:
где вода столбом стоит?65 И ещё: мету, мету – не вымету, несу, несу – не вынесу, стемнеет –
сама уйдёт.66 Тут вам и геометрия, и физика… А вот здесь зашифровано то, ради чего мы с
вами и стараемся: не на меру, не на вес, а у всех людей он есть.67
Об операционном методе языка Диал
Мои соавторы по ряду изданий – Сергей Ёлкин и Дмитрий Гаврилов – были
свидетелями и непосредственными участниками дерзкого эксперимента, продолжающегося с
переменным успехом и по сей день. Речь идёт о создании принципиально нового
универсального языка на стыке двух ведущих теорий философии и фундаментальной физики
– диалектику Гегеля и Теории симметрии.
Сам я по одному из высших образований – философ, а кроме того имел длительный
опыт сотрудничества с физиками-теоретиками Виталием Бейлиным и Григорием
Верешковым – Мы работали на книгой «Вакуум, элементарные частицы и Вселенная».
Поэтому, наверное, мне близок и понятен такой синтез.
Чтобы не напутать в фактологии, воспользуюсь текстом одной из лекций, читаемых
моими коллегами – С.В. Ёлкиным и Д.А. Гавриловым – молодым специалистам одной
крупной инжиниринговой компании124 в рамках курса «Инженерно-техническое
творчество»:
«Серьёзной проблемой точных наук является письменный характер универсального
языка – Математики, затрудняющий любое усвоение научных обоснований во множестве
сфер деятельности. Решение же любой серьезной научно-исследовательской задачи на стыке
различных областей упирается, прежде всего, в проблему невозможности представить знание
универсальным образом. Мы часто не задумываемся над сущностью того или иного понятия,
вещи, объекта, берем его как данность, не пытаясь вникнуть в суть. За частоколом
математических формул, описывающих, например, поведение технической системы, легко
утрачивается физический смысл.
Хотелось бы за каждым математическим высказыванием видеть его физический смысл
или геометрический образ. Формальный характер математики входит в противоречие с
неформальным характером творческого мышления.
124 «ЛУКОЙЛ-Инжиниринг» уже несколько лет реализует проект «Академия молодого инноватора»,
направленный на повышение творческого потенциала работников, а, значит, интеллектуализацию
производственного процесса и увеличение нематериальных активов.
В этой связи в 1986 году советским учёным, канд. физ. мат наук Валентином
Куликовым, на тот момент научным сотрудником Института земного магнетизма,
ионосферы и распространения радиоволн им. Н. В. Пушкова Российской академии наук
(ИЗМИРАН) были заложены основы нового универсального языка, которые он изначально
воплотил в форме научно-фантастического романа. Предполагалось, что язык послужит
инструментальным средством для решения накопившихся проблем теоретической физики и
квантовой механики. В качестве основы для его построения была взята диалектическая
логика125, применённая к абстрактной теории симметрии.126 Язык получил название Диал.
В отличие от математики он по первоначальному замыслу должен был иметь не только
письменный, но речевой характер. В. В. Куликов набрал в технических вузах Москвы первые
экспериментальные группы добровольцев для разработки языковой среды, в составе которых
оказались и нынешние авторы этих лекций по «Инженерно-техническому творчеству».
С 1986 года осуществлялась проверка свойств языка в различных областях знания и
составление базового словаря. Мы работали и с последующими группами: научных
работников, аспирантов, студентов и, конечно, школьников.
В ходе этой экспериментальной деятельности выяснилось, что Диал является в
сущности языком-транслятором знаний из одной области в другую, эффективным средством
междисциплинарного обмена, развития творческого воображения, производства
неординарных идей и генерирования изобретательских решений, подчас парадоксальных».
Разработчики Диала отмечают: «Междисциплинарный характер современного
познания во многом обусловлен тем, что наука из «дисциплинарной» сферы деятельности
превращается в «проблемно ориентированную.
А развитый язык позволяет решать самые сложные задачи на стыке дисциплин. Но
только диалектически выстроенный язык-транслятор (по Куликову) способен реально
перевернуть мышление и сделать его подлинно сильным. Только так можно экстремально
развить интеллект!»
Главная функция языка – отражение всех без исключения сторон вещей, свойств и
отношений мира природы и мира человека. Разработчики Диала основывались на идее, что в
своём развитии все языки проходят, в общих чертах, те же этапы, что не только сознание, но
и сама Вселенная. Если предположить, что в разных частях Вселенной действуют некоторые
универсальные законы – должен быть всеобщий язык для их отражения, для трансляции
одной формы знания в другую [17].
Первые результаты эксперимента были доложены научному сообществу ещё в 1990
году на Циолковских чтениях, о развитии методологического подхода сообщалось и в
последующие годы – на чтениях Ильенковских [15] и в других четырёх десятках публикаций
– от строго научных до популярных изложений. Заинтересованный читатель может легко
найти сетевую группу социальной сети «ВКонтакте» под названием «Интеллектуальная
атака» и ознакомиться там с полной библиографией метода.
Ещё раз следует отметить, что сам Диал – не простой алгоритмический или
математический язык. Каждый может, усвоив простейшие базовые принципы, говорить на
нём с друзьями, петь, учить языку других.
С другой стороны математические свойства языка позволяют делать более
эффективными профессиональные исследования и интенсифицировать изобретательскую
125 Собственно к диалектике как системному видению мира, хоть и в неявном виде, призывает максима,
которую приписывают Клоду Гельвецию: «Знание некоторых принципов легко возмещает незнание многих
фактов».
126 Так рассуждение по аналогии дало науке и технике многие блестящие результаты, нередко совершенно
неожиданные, но вряд ли удивительные с точки зрения фундаментальной физики и философии. Аналогии
являются отражением универсальных законов сохранения и преобразования симметрии вещей на всех уровнях
бытия.
деяте5льность.
Мои коллеги приводят такой пример:
«В широкоизвестной ТРИЗ эмпирически выявлено 50 типовых приёмов разрешения
технических противоречий кроме того – 76 стандартов на решение изобретательских задач.
Между тем, совершенно очевидно, что эмпирические приёмы являются частными случаями,
отражениями глобальных принципов. И знание как раз этих некоторых фундаментальных
принципов замещает необходимость запоминания множества эмпирических сведений,
фактов и приёмов.
В Диале впервые использованы фундаментальные принципы преобразования
симметрии для решения изобретательских, в том числе инженерно-технических, задач. Диал
весь пронизан универсалиями: законами развития систем и приемами разрешения
противоречий. С помощью супероператоров Диала можно логически получать,
конструировать любые необходимые новые «стандарты»».
Разработчики выделяют следующие главные принципы языка Диал:
✓ Симметрич ность. Каждый знак, звук Диала является абстрактным оператором,
преобразованием симметрии, обозначающим некоторую вещь, процесс перехода чего-то
одного во что-то другое, их отношение. Например, законы сохранения импульса и энергии
могут быть получены из преобразований симметрии.127
✓ Самоприменимость. Каждый оператор воздействует сам на себя, порождая свою
противоположность.
✓ Иерархичность. Каждый оператор действует над «полем» других, «низших»,
операторов. На каждом уровне иерархии языка можно общаться с той или иной долей
абстрактности. Основание иерархической системы – оператор, выражающий полное
отсутствие всякой информации (ноль, ничто, незнаемое, молчание).
✓ Конструктивность . Все формы языка собираются и разбираются, словно в детском
конструкторе. Например, оператор рождения, взаимодействуя с оператором смерти, дает
оператор жизни или времени, обратный ему – оператор пространства или памяти. Тот же
оператор
пространства
мы
можем
получить,
обернув
порядок
исходных,
взаимодействующих операторов, действуя на оператор смерти оператором рождения. Чем
сложнее языковая конструкция – тем конкретнее смысл.
✓ Произвольность обозначений. Имена «низших» операторов произвольны. Здесь
характерна полная условность обозначений и возможность присвоить событию, процессу,
вещи любое знаковое имя, однако затем это имя должно быть зафиксировано. «Высшие»
операторы составлены из «низших» и являются формулами или словами Диала. Правила
составления формул (или слов) также являются операторами. Этот принцип позволяет нам
решать изобретательские задачки, как математические уравнения, где за икс обозначен
идеальный конечный результат. Тем самым Диал адаптирует эвристические и формальнологические (алгоритмические) методы друг к другу.
✓ Динамичность . Развитие есть переход от одной симметрии языка к другой. Он
описывается как рекурсия – «возвращение к себе», т. е. происходит за счет воздействия
операторов (процессов) самих на себя. Преобразования симметрии понимаются двойственно,
как того, что с одной стороны преобразовывает – изменяет, а с другой стороны оставляет
неизменным – сохраняет.
✓ Контекстная зависимость . Знаки Диала отражают, на самом деле, симметрии
природы, вещи со всеми их связями. Идентификация любого знака осуществляется через
контекст. Общаясь на Диале, изобретатели могут не опасаться за свои ноу-хау, даже
рассказывая о них – в эти тайны никто не проникнет, а секреты никто не украдет.
127 Трансляционная симметрия пространства и времени.
Наиболее полным изданным описанием операторного метода Диала на сегодня
являются избранные разделы из наших совестных книг «Инженерная эвристика» или
«Самоучитель игры на извилинах», и «Турбулентное мышл.
Разработчики, насколько мне известно, после нескольких лет перерыва, вернулись к
регулярному преподаванию Диала в качестве метода развития «сильного мышления». И
поскольку «лучше один раз увидеть, чем сто раз услышать», а моих коллег не составляет
труда найти в Интернете, здесь мы не будем углубляться в дальнейшие детали.
Язык и мышление
Я думаю, что программа постижения языка существует в мозге не на уровне инстинкта,
а гораздо глубже. Но эту программу портят, она с возрастом слабеет в процессе
«самопознания» и подверженности воздействию «дебелизора».
Мышление формируется в общении. И разработчики языка Диал справедливо
отметили, что знаковые системы или языки, которыми пользуются при общении,
симметричны, т. е. существуют такие преобразования, благодаря которым семантические
структуры одного субъекта общения могут быть сопоставлены семантическим структурам
другого субъекта общения
Раз овладение языком в совершенстве невозможно без языковой среды – тем самым
развитие языков имеет для прогресса общества не меньшее значение, чем развитие
производительных сил благодаря совершенствованию орудий труда. Их преобразование и
является источником движения цивилизации от эпохи разобщенного мира до
корпоративного (информационного) общества.
«Границы моего языка означают границы моего мира», – отметил Людвиг
Витгенштейн. Ему вторит и Мирча Элиаде: «всегда брался за новый язык, чтобы овладеть
новым рабочим орудием».
Крис Фрит утверждает, что обучение – это не только демонстрация и подражание:
«Моделирование сознания человека, с которым мы разговариваем, позволяет нам изменить
способ общения с этим человеком. Мы можем принять во внимание, что этот человек знает и
что он способен понять. Люди обладают разными знаниями и способностями, поэтому мы
общаемся со всеми по-разному. Это может показаться очевидным, но есть ряд удивительных
и тонких отличий, которые мы вносим в характер своего общения, сами того не осознавая.
Когда мать говорит что-то своему младенцу, она делает это особенным голосом. Она
пользуется так называемым «сюсюканьем» (baby talk) или «материнским языком»
(motherese)128. Та же самая мать особым голосом будет говорить и со своей кошкой. Но
между этими типами голоса есть тонкая разница. И с кошкой, и со своим ребенком мать
говорит более высоким голосом. Этот особый голос больше похож на голос ребенка и
кошки, потому что они меньше, чем она, а у живых существ меньшего размера обычно более
высокий голос. Но только разговаривая со своим ребенком, мать преувеличивает разницу
между гласными звуками. Она произносит звуки «и», «у» и «а» так, что они сильнее
отличаются друг от друга. Это «растягивание пространства гласных» дает карикатуру на
нормальную речь, с преувеличенными отличительными чертами звуков языка, на котором
говорит мать.
Ребенок выучивает звуки родного языка, подражая своей матери. Пользуясь этой
карикатурой нормальной речи, мать облегчает своему ребенку изучение родного языка.
Когда она говорит с кошкой, она не прибегает к такой карикатурной речи. Она знает, что
кошка всё равно не научится говорить.
В тот момент, когда моя модель вашей мысли совпадает с моей собственной мыслью и
128 Подразумевается, что «отцовский язык» (fatherese) отличается от материнского.
у меня больше нет нужды показывать вам, что возникла проблема, можно считать, что
сообщение прошло успешно. При этом принципиально, что в этот самый момент между
вашей моделью моей мысли и вашей собственной мыслью тоже не остается отличий. Этот
момент взаимного согласия завершает успешную передачу сообщения. Строя модели
внутреннего мира других людей, наш мозг решает задачу, требующую проникновения в
чужое сознание. И именно эта способность строить модели чужого сознания и создала
пропасть между людьми и всеми другими видами живых организмов. Если бы мы не могли
строить мысленные модели окружающего мира и делиться ими, у нас бы не было ни языка,
ни культуры». (Крис Фрит. «Мозг и душа»).
Обучение языку есть обучение некоему типу мышления (и наоборот): насколько
универсален язык, настолько продуктивно и мышление. Математика – первый почти
универсальный язык интеллекта – эволюционируя, породила свою противоположность, а
именно феномен «математического мышления».
Такой проблемы не существовало перед всесторонне развитым Ломоносовым, когда он
произносил классическое: «Математику уже затем учить следует, что она ум в порядок
приводит». Увы, ныне приводит она его в порядок настолько своеобразный, что собственные
достоинства обращает в недостатки. И более прав Бернард Шоу, отметивший: «Человечество
околдовано неудержимым движением науки, и только искусство способно вернуть его к
реальности».
Тщетные попытки привить ребёнку этот кастовый язык прежними графическими
методами обречены. У подавляющего большинства учеников процесс усвоения математики
проходит психически мучительно, крайне болезненно, поскольку, согласно тому же Э.В.
Ильенкову [18], символика математики – с точки зрения обучаемого – «оторвана от
реального, конкретного смысла». «Боль» же, как известно, формирует барьер,
отчуждённость, сохраняющуюся чаще всего на всю жизнь.
Воистину был прав Сократ, когда молвил: «Заговори, чтоб я тебя увидел».
Другое серьёзное препятствие на пути триумфального шествия «языка науки», как
считают создатели Диала (см. выше), – отсутствие сколько-нибудь строгого математического
описания мира окружающей нас бытовой реальности и мира эмоций. Однако если в
разрешении наших психологических, эмоциональных, бытовых проблем математические
методы вдруг окажутся столь же эффективными, сколь и в естественных науках – значит, их
стоит использовать.
Математика возникла в античном мире как способ упорядочения всех накопленных к
тому времени приёмов размышлений. Она даёт каждому, кто удосужится погрузиться в её
основы, громадный набор не только готовых способов думания, но и приёмов дальнейшего
упорядочения всех собственных находок. То есть именно математика – главный инструмент
борьбы с энтропией (мерой хаоса) сознания.
Выдающийся отечественный философ Эвальд Васильевич Ильенков установил: в мозгу
человека практически нет встроенных структур с конкретными рефлексами и навыками
поведения. Зато необычайно – куда лучше, чем у большинства прочих животных – развита
способность к установлению взаимосвязей между малейшими крупицами накапливаемого
опыта. С твёрдой уверенностью можно заявить, что математика – дизайнер мысли.
Рассказывают, что великий физик Гиббс был весьма замкнутым человеком и на
заседаниях ученого совета университета, в котором он преподавал, пребывал в молчании. Но
когда решался вопрос о том, чему уделять в новых учебных программах больше места –
Математике или изучению иностранных языков, он не выдержал и произнес речь:
«Математика – это язык!» – сказал он.
А пожалуй, первым из европейцев, в полной мере оценившим универсализм
математики, был гениальный английский естествоиспытатель, францисканский монах
Роджер Бэкон (около 1214 – после 1292). В том, что он был гением, нет никаких сомнений.
Например, в работе «Epistola fratris Rogerii Baconis de secretis operibus artis et naturae, et de
nullitate magiae » – Бэкон уже рассуждает о способах технического использования различных
явлений природы для создания в будущем полезных человеку механизмов и
приспособлений. Он предсказывает создание домкратов, подводных лодок, летательных
аппаратов, механических «колесниц», безопорных мостов, телескопа и микроскопа,
астролябии, а также лазерного оружия, пользуется разработанными собственноручно
очками. Зашифровывает в виде анаграммы состав независимо открытого им пороха,
указывая на разные аспекты его военного применения. И это XIII век!
Бигуди № 50
Кстати, о языке. Однажды к Аристотелю пришел юноша, который после долгой
витиеватой и цветистой речи сформулировал, наконец, свою просьбу. Он хотел, чтобы
мудрец взял его к себе в обучение и научил, как правильно и логично излагать свои мысли,
чтобы юноша впоследствии мог стать известным оратором. После своей речи юноша
спросил, какую плату возьмет с него Аристотель. Аристотель, выслушав юношу, сказал:
«Хорошо. Но с тебя плата будет вдвое больше, чем с остальных учеников». Увидев
изумление юноши, он пояснил: «С тобой работы вдвое больше – прежде чем научить тебя
говорить, мне придётся…». Что же сказал юноше Аристотель?68
Игра как метафора
Можно привести ещё множество интереснейших примеров игр с буквенными
(графемными и фонемными) и словесными множествами – скажем, поиски закономерности в
последовательности слов, букв, рисунков. Во всех случаях анализируются структура и
характерные признаки некоторого массива информации. Даже известная школьникам игра в
«виселицу» или «балбеса» относится к линейным играм, развивающим – хотя и несколько
односторонне – интеллект. Однако не стоит увлекаться задачами лишь одного типа –
интеллект не должен быть «хромым» на какую-то одну из извилин. Гибкость мысли
тренируется на задачах разных видов.
К простейшим задачам на развитие интуитивного мышления можно отнести, например,
решение ребусов129 – занимательных задач, где нужно расшифровать текст, записанный с
помощью рисунков. Вот тут уже понадобится не только способность быстро
структурировать по определённым признакам – в соответствии с условием задачи –
Множества слов. Теперь нужно расшифровывать графический, рисуночный код, для чего
недостаточно только перебирать в памяти слова и их комбинации: нужно включить и
пространственное мышление, и воображение, и смекалку.
Множество интереснейших примеров можно найти в литературе. Не будем перечислять
здесь наиболее известные книги о различных интеллектуальных (в разной степени) играх
(см. список литературы). Наиболее эффективными для развития логики и аналитических
способностей нам представляются книги типа «Ну-ка, догадайся!». Конкретная
формулировка заданий (задачек, вопросов, головоломок, шарад, ребусов и пр.) в них может
быть разной, однако цель одна: не только получить результат, но и научиться максимально
использовать все уровни своего мышления.
Игры со словами и фразами – хороший способ делать знакомое незнакомым. Именно
эта процедура лежит в основе поиска новых идей, когда давно известный предмет нужно
увидеть как бы в первый раз, по-новому. Иначе говоря, такая игра – обновление старых и
создание новых метафор. Метафора здесь – способ описания задачи, «вскрывающий» её,
предлагающий аналогии, указывающий на характерные признаки. И в этом случае мы
129 Латинское rebus – творительный падеж от «res» – вещь, дело (от того же корня происходит и res publica
– общее дело). Так что название этого рода головоломок переводится как «вещами» – в смысле «написанное
вещами вместо слов».
близки к принципам синектики, где метафоры используются для построения системы
аналогий.
Заметим: в процессе создания метафор используются различные принципы движения в
«пространстве проблемы». В частности, переход к предельным значениям параметров
задачи.
Например, как создать идеальный открыватель банки? Идеально было бы, чтобы она
открывалась сама130. Например, «лопнула от гнева». «Рассердилась, даже покраснела».
Значит, нагрелась? А от нагревания расширяется полоска специально подобранного металла
и… Вот уже идея.
Игры можно классифицировать и по другим признакам. В частности, по типу исходной
информации: игры с полной информацией (шашки, шахматы и др.) и с неполной
информацией, когда цель игры достигается одновременно с добыванием необходимых
данных. Пример игры с неполной информацией – «морской бой», где нет сведений о
расположении кораблей противника. Но там эти корабли хотя бы расставляет сам противник,
о чьих предпочтениях можно и догадаться. А в карточных играх особую роль играет и
фактор случайного выбора.
Крис Фрит: «В 1956 году наука о создании устройств, способных делать разные
хитроумные вещи, получила название «искусственный интеллект». Исследовательская
программа этой науки, как и любой другой, предполагала, что начать нужно с решения
самых легких проблем. Восприятие окружающего мира казалось сравнительно легким делом.
Почти все люди умеют с легкостью читать рукописный текст и узнавать лица, и поначалу
казалось, что создать машину, способную читать рукописный текст и узнавать лица, должно
быть тоже не особенно сложно. Игра в шахматы – напротив, очень сложное дело. Очень
немногие люди способны играть в шахматы на уровне гроссмейстера. Создание машин,
умеющих играть в шахматы, лучше было отложить на потом.
Прошло пятьдесят лет, и компьютер, предназначенный для игры в шахматы, выиграл у
чемпиона мира.131 Проблема научить машину восприятию, напротив, оказалась очень
сложной. Люди по-прежнему умеют узнавать лица и читать рукописный текст намного
лучше, чем машины. Почему же эта проблема оказалась такой сложной? Оказывается, даже
моей способностью видеть, что сад у меня за окном полон разных объектов, очень сложно
наделить машину. Тому есть много причин. Например, видимые объекты перекрывают друг
друга, а некоторые из них ещё и движутся.
Откуда я знаю, что это за коричневое пятно – часть забора, или дерева, или птицы?
Мой мозг решает все эти удивительно сложные задачи и заставляет меня думать, что я
воспринимаю мир, не прилагая никаких усилий. Как же он это делает?».
Поиск выигрышной стратегии в играх с участием нескольких человек – это зачастую
весьма сложная, нетривиальная и творческая задача. Она породила целую область
современной математики – теорию игр. Так же, как классическая теория вероятностей
выросла из наблюдений одного из заядлых игроков в кости, шевалье де Мере. Эти
наблюдения, между прочим, он удосужился провести, зафиксировать характерные
особенности, расклассифицировать, а потом уже поделился своими соображениями с
профессиональными математиками. Т. е. шевалье благодаря своей наблюдательности –
одному из элементов творчества – сумел в обычной для того времени игре разглядеть
130 Принцип идеального конечного результата – один из ключевых в ТРИЗ. Не представляя себе идеала, не
поймёшь, чем отличается от него реальное положение дел. А не зная различий, искать способы их устранения
можно лишь наугад.
131 В 1997 году сконструированный корпорацией IBM суперкомпьютер Deep Blue выиграл в шахматы у
Гарри Каспарова, которого многие считают одним из величайших шахматистов всех времен. Компьютер
победил во многом благодаря своей способности проводить сложные математические расчеты. Он мог
анализировать 200 иллионов ходов в секунду. Люди так в шахматы не играют. – Крис Фрит.
проявление неизвестных, необычных, нестандартных закономерностей.
Есть немало интеллектуальных игр с одним участником. В частности, это уже
упоминавшиеся различные головоломки, математические задачи, кроссворды.
Между прочим, сказано новое слово и в области кроссвордов. В «Науке и жизни» за
декабрь 2002-го г. приводится описание новой игры для эрудитов – кроссенс. Это табличка
из девяти картинок на совершенно разные темы. Задача играющего – установить
однозначные ассоциативные связи между соседними картинками. Например, на одной
картине изображен Геракл, на другой – овёс. Ну, тут цепочка ассоциаций проста. От
греческого Геракла (собственно, в греческом произношении – Гераклес) к римскому
произношению Геркулес, а затем к овсяной каше и, следовательно, к растению овёс.
В том же журнале можно найти и более сложную ассоциативную связь: репродукция
«Красные виноградники в Арле» – Ван Гог – картина Ван Гога «Едоки картофеля» – просто
«едоки» картофеля – колорадский жук. Забавно? Не только. Ведь это ещё и хорошая
тренировка памяти, увеличение «мощности» интеллектуально-логического аппарата.
Бигуди № 51
Есть обоснованное мнение, товарищи: удачливый игрок может выиграть ровно столько,
сколько проиграют другие игроки. Очевидный закон сохранения «денежной массы». И всё
же кое-кто, например, Сэм Ллойд, великий изобретатель головоломок, утверждает: есть игры
с более выгодными условиями для игроков. Послушайте его рассказ: «Четыре весельчака
сели играть и играли всю ночь до рассвета. Причём они играли за деньги, а не просто для
забавы. Как и полагается, у каждого был свой счёт. Когда стали подсчитывать выигрыш,
оказалось, что у всех он одинаков!» Как это понимать? Если никто не проиграл, как же они
все выиграли? Да, не забудьте – они, конечно, играли в одном месте, одновременно!69
От головоломки к науке
Эту важнейшую особенность занятий различными головоломками – они зачастую
оказываются яркими, удивительными и изысканными математическими «изюминками» –
заметили на самой заре цивилизации. Первый учебник математики, дошедший до нас из
древности – «папирус Райнда» (по имени нашедшего его англичанина, подарившего его
Британскому музею) или «папирус Ахмеса» по имени писца, жившего в XIX веке до н. э. Это
кусок папируса длиной около 5 метров. В нём 84 задачи, которые решали ученики школы
писцов. Но чтобы занятия были интересны и увлекательны, часть задач напоминает
головоломки. Вот самая известная из них: «в 7 домах живет по 7 кошек, каждая из них съела
по 7 мышей, каждая мышь съела по 7 колосьев, из каждого колоса могло получиться по 7
мер хлеба – сколько всего предметов перечислено?»132. Математические пособия в древней
Индии и Китае тоже были сдобрены россыпями головоломок.
В петровской Руси в 1703-м году типографским способом издана «Арифметика»
Магницкого. Один из разделов этого учебника, в течение полувека бывшего основным
руководством по математике в стране, назывался: «Об утешных некиих действах чрез
арифметику употребляемых». Так что ясное понимание роли математики – и в особенности
её «головоломной» части – для развития мыслительных действий существовало издавна. Да
и многие знаменитые впоследствии учёные – причём не только математики – наших дней
тоже начинали свою «жизнь в науке» с решения разных забавных задачек-головоломок из
книжек Ллойда, Перельмана, Кордемского, Маковецкого, Гарднера и многих других.
Математические игры и фокусы, угадывание чисел, задачи на переливания, смеси,
132 Задача заодно напоминает, почему в древнеегипетском пантеоне была богиня-кошка Баст. Кто ещё мог
защитить от грызунов щедрые урожаи, собранные на плодородных нильских илистых наносах!
взвешивания, разделение на части и другие забавные истории с людьми и числами не только
укрепляют интерес к знаниям, научают конкретным вычислениям, но и успешно укрепляют
логическую ветвь интеллекта. Иначе говоря, задачи учат искать заранее не очевидные
ответы. Недаром замечено, что склонность к играм – одна их характерных черт творчески
одарённых людей. Давайте попробуем решить такую задачу: найти число, которое равно
сумме своих делителей. Упростим ситуацию: пусть это число меньше 10. Тогда ответ
находится быстро: это число 6, которое равно и произведению 1х2х3, и сумме 1+2+3. Но
если попытаться обнаружить общую закономерность появления таких чисел – называемых
совершенными – в ряду натуральных, то придётся стать профессиональным математиком.
Что, наверное, не так уж и плохо.
А вот ещё задача: разбить число 10 на сумму двух чисел, дающих в произведении 40.
Это замечательный пример того, как из решения занимательных задач вырастает серьёзная
новая область математики – нам придётся для удовлетворения условиям задачи расширить
привычную область арифметических действий и выйти в поле комплексных чисел! Заодно
наше мышление учится строить обобщения, выходить на следующий уровень абстракции.
Обобщение, расширение области действий известной операции – не единственный
приём. Можно использовать в качестве своеобразной игры приём инверсии. Т. е. найти
возможность существования «мира наизнанку», наоборот. В математике это зачастую
означает просто отказ от одного из «столпов» известной теории. Как отказ от Пятого
постулата Евклида, приведший к открытию Яношем Больяи и Николаем Лобачевским
неэвклидовой геометрии.
Обратите внимание, насколько интереснее и быстрее можно получить решение
математической задачи или головоломки, если идти нестандартным путём. Известный
математик Роберт Смаллиан говорит: «Решение, подсказанное здравым смыслом…гораздо
интереснее и уж, конечно, более творческое, а также содержит больше информации, чем
сугубо математическое».
Нью-йоркский математик Джо Бирман сказал, что для него, как для американца,
очевидно, каково правильное решение задач из американских тестов по математике: «Дело в
том, что я точно представляю себе степень идиотизма составителей этих задач».
А вот изящный пример – задача из книжки Смаллиана: 10 кошек и собак съедают
вместе 56 галет, собакам полагается по 6 штук, кошкам – по 5. Сколько же собак и сколько
кошек? Нетрудно решить задачу стандартным методом составления уравнений, считая «х
штук кошек» и «(10 – х) штук собак». Однако можно поступить гораздо проще. Сначала
скормим всем животным по 5 галет. Теперь все кошки сыты. Но остаётся ещё 6 галет,
предназначенных, следовательно, уже только для собак. Дав каждой собаке по одной
дополнительной галете, мы накормили всех и узнали, сколько было собак и кошек.
Вспомним ещё и известный рассказ А. Чехова «Репетитор», где ученик старшего класса
решает со своим подопечным – сыном старого купца – задачу: «Купец купил 138 аршин
чёрного и синего сукна за 540 рублей. Спрашивается, сколько аршин купил он того и
другого, если синее стоило 5 руб. за аршин, а чёрное 3 руб.?» Попав впросак, «репетитор»
конфузится, его ученик ехидно улыбается. Ситуация крайне неловкая. Тем более, что старый
купец утверждает, протягивая руку к счётам: «И без алгебры решить можно…, вот-с, понашему, по-неучёному». И, щёлкая костяшками счётов, быстро получает правильный ответ.
Дело не в том, что юный «репетитор» тоже должен был уметь считать по старинке, на
счётах. Нет, скорее он должен был бы – вместо того, чтобы вспоминать лихорадочно, как
решать задачу стандартно, «с иксом и игреком» – включить мышление, перейти от
конкретных смыслов к формальной логике их бытия, а затем обратно133. И тогда
имеющееся НЗ тоже включилось бы в работу.
Конечно, стандартные рецепты тоже неплохи – они потому и стали стандартами, что
133 Вспомним: как раз наши школьники на олимпиадах не умеют видеть за частностями общее.
чаще всего помогают справиться с задачей быстро. Но на переход к стандарту тоже уходят
силы и время – и иной раз больше, чем нужно для поиска нестандартного решения.
Кстати говоря, выполнение интеллектуальных тестов в духе Айзенка тоже можно
рассматривать как игру. Просто у неё есть ещё одна цель – определение уровня своих
способностей. Многие психологи (например, Л. Брайт) не только предлагают сами наборы
интеллектуальных тестов, но и формулируют «учебные пособия» к ним, т. е. анализируют
способы решения тестовых заданий, выделяют отдельные их типы, предлагают придумывать
собственные задачи и головоломки. Так формируются навыки мышления, усваиваются
основные его принципы, которые можно будет применять в реальной жизни. Понятно, зачем
мы учимся решать задачи – не для того только, чтобы уметь решать именно такие (и только
такие) задачи, но для того, чтобы развить, активизировать и сделать более креативным,
творческим своё мышление.
То есть суть дела в утверждении: не всякая игра творческая, но любое творчество
содержит элемент игры.
Бигуди № 52
Интеллектуальные игры – не только шахматы, го, маджонг, судоку. Или домино. Есть,
например, интеллектуальная игра NEYRON. Как говорят основатели, это «игра в стиле ЧГК
или «Кто хочет стать миллионером»». В игре более 10 000 участников, играют целыми
вузовскими командами. Уже есть её вариант для мобильных телефонов: так сказать,
интеллект – в массы! Я уж не говорю о великих родоначальниках, популярнейших ЧГК,
Брейн-ринге и «Своей игре»! Цель интеллектуальных игр практически всегда одна: набирать
очки (рейтинг, призы…) за верные ответы на интеллектуальные вопросы из различных
областей знания. Тут для нас с вами огромное поле деятельности! Например: Пётр I очень
любил… и не расставался с… даже во время военных походов. Непременно брал с собой…,
выезжая за пределы России по государственным делам или на отдых. Частенько…
изготавливал сам. А один из голландцев, описывая пребывание этого уникального человека в
их стране, подчеркивал, что «не предавался любимейшему из времяпрепровождений наших
вод – карточной игре. В свободное время он…». Что же такое очень любил делать царь в
свободное время и даже во время военных походов? Добавим и то, что император Николай
II, хотя и признавал за Петром I много заслуг, тем не менее, не любил его за увлечение
западной культурой и попрание всех чисто русских обычаев.70
16. «Необходимость – Мать изобретательности»
– Вот, играй, – сказал Малыш. – Из этого набора можно
сделать и автомобиль, и подъёмный кран, и всё, что хочешь.
А. Линдгрен. «Малыш и Карлсон, который живёт на крыше»
Давайте ещё раз – ненадолго! – вернёмся к определению интеллекта.
Ещё совсем недавно интеллектуалом у нас считали любого владеющего иностранным
языком. Это и понятно: преподавали языки в наших школах настолько, мягко говоря,
неважно, что свободно говорящий, например, на английском заслуживал уважения – хотя бы
за то, что не поддался разрушительному обучению.
Чаще всего основным признаком интеллекта, не мудрствуя лукаво, называют
эрудицию. Об этом мы уже говорили. Эрудиция – дело, конечно, полезное. Проблемы наши
повторяются часто, и для их решения зачастую хватает стандартных приёмов или
принципов. Но ведь многознание не научает разуму. Зная – точнее, «помня» – даже тысячи
решений стандартных проблем, можно не найти выхода из положения, отличного от
стандартов лишь в мельчайших деталях.
По мнению известного футуролога Рэя Курцвейла, скоро будет создан искусственный
неокортекс, расширяющий возможности нашего интеллекта за счет того, что мозги людей и
компьютеров свяжет нейронет – сеть, которая станет следующим шагом в развитии
нейроинтерфейсов, уже сейчас позволяющих управлять внешними объектами с помощью
мысли. И это будет время симбиоза биологической и искусственной жизни, означающее, что
история людей, как биологического вида закончена. Начнется история постчеловечества –
вида, который будет создавать сам себя (Андрей Константинов, «Мозг из машины» //
«Русский репортер», 20 марта 2013, № 11).
С моей точки зрения, интеллект – прежде всего изобретательность, умение находить
нестандартные ходы. Классический пример – лечение инфаркта шунтированием: операцией
по созданию обходных кровотоков взамен закупоренных.
Об оценке интеллекта, хоть и в узкоспециализированном его варианте, рассказывается
в одном анекдоте. Некий инженер умер и попал в ад за всякие земные прегрешения. В
скором времени его перестал устраивать уровень комфорта и он принялся за
усовершенствования своего нового места пребывания. Установил там кондиционеры,
освежители воздуха, эскалаторы с круга на круг, ввел барботаж и кипящий слой для
грешников и т. п.
Звонит Бог Сатане: «Ну, как там у вас дела?»
Сатана ему в ответ: «Дела идут как никогда! У нас теперь есть кондиционер,
освежители воздуха, эскалаторы и «черт знает что» ещё это инженер установит в самое
ближайшее время!»
Господь (возмущенно и удивленно): «ЧТО?» У вас есть ИНЖЕНЕР!?! Слушай, тут
какая-то ошибка. Инженеры не должны попадать в ад! Сейчас же пришли мне его наверх!»
Сатана: «Да ни за что! Он теперь у меня в команде!»
«Немедленно пришли его ко мне наверх, или я буду с тобой судиться!» – негодует Бог.
Сатана: «Ага, и где ты у себя найдешь адвоката?!».
Изобретения эпох
Изобретательность – качество мышления, которое трудно переоценить. И нужно
обладать гибким, творческим, изобретательным умом не только для того, чтобы создать
проекты подводной лодки и вертолета в XV веке, как Леонардо, или придумать способ
записи человеческого голоса, как Эдисон.
А вот Эдисон изобретал лишь то, для чего назрело время. Например, Шарль Кро
впервые продемонстрировал механический способ звукозаписи за полгода до Эдисона – но
знаменитый в ту пору поэт просто не удосужился обратиться к столь низменной материи, как
патентование. Впрочем, самоограничение Эдисона проистекает из печального опыта. Его
первую разработку – прибор для голосования – конгресс США отверг: ручное голосование
оставляло весьма выгодный простор для закулисных манипуляций. С тех пор великий
изобретатель занимался только заведомо востребованными идеями. Потому, собственно, и
стал великим: на средства от удачно реализованных разработок он создал в Менло-парке
один из первых в истории прикладных исследовательских центров, где любой его замысел
подхватывали десятки сотрудников.
Изобретательность и глубокая интеллектуальная проработка особенно важны в той
сфере, где от каждого решения зависят жизни не единиц, а тысяч – да-да, именно в политике!
Вот тут, проигрывая в уме, можно проиграть во всём.
Те, кто обладает хотя бы эрудицией, в ситуациях, давно и многократно пройденных, не
станут заново изобретать велосипеды. Если есть информация о проблеме и выучены (или
могут быть где-то прочитаны) стандартные приёмы и способы (см. выше) действий в
«пространстве проблемы», то есть надежда на решение задачи с «наименьшими потерями».
Если есть «мудрый визирь» и понимание собственной неизбежной интеллектуальной
ограниченности, тоже есть шанс избежать полной неудачи.
Но если вместо всего этого имеется только негибкий «квазиинтеллект», как у
современных промышленных роботов среднего класса, то надеяться на эффективное
решение общественных и политических проблем не приходится. Тем более на решение
умное и изобретательное. И тогда тем, кто способен к творческим действиям и креативной
проработке информации, приходится искать оптимальные и нестандартные варианты
компенсации чужих разрушительных действий в пространстве любой проблемы.
Крис Фрит: «Мы воспринимаем не мир, а его модель, создаваемую мозгом. Наше
воображение совершенно некреативно. Оно не делает предсказаний и не исправляет ошибок.
Мы ничего не творим у себя в голове. Мы творим, облекая наши мысли в форму набросков,
штрихов и черновиков, позволяющих нам извлечь пользу из неожиданностей, которыми
полна действительность. Именно благодаря этим неиссякаемым неожиданностям
взаимодействие с окружающим миром и приносит нам столько радости».
Если уж человек имеет высокоразвитый интеллект, то успеха он добьётся в любом
деле. И наука, и техника, и политика – такие, казалось бы, разные сферы деятельности – в
равной мере доступны гибкому и творческому мышлению. Результаты его деятельности
поражают своим разнообразием, эффективностью и полезностью. Вот пример.
Вопреки распространённому мнению, далеко не все американские купюры украшены
портретами президентов. Например, на $10000 нарисован Саймон Чейз – не только министр
финансов в правительстве президента Линкольна, но и основатель Chase Manhattan Bank. А
на крупнейшей банкноте, которую разрешено вывозить за пределы США – $100 – изображён
один из виднейших американских учёных, Бенджамен Франклин. Правда, на купюру он
попал за достижения не столько научные, сколько политические.
Первый посол США во Франции в совершенстве использовал тогдашние противоречия
крупнейших держав Старого Света. Ради традиционного противостояния с Англией
французский король не только признал заокеанскую революцию, но и всесторонне
поддержал её. Повстанческую армию водил в бой не только плантатор Вашингтон, но и
маркиз де лаФайетт. Производство пороха для привезенных из Франции ружей наладил
любимый ученик великого химика Антуана Лорана де лаВуазье, Элетер Ирене дюПон де
Немур, чьи потомки до сих пор владеют одним из крупнейших химических концернов мира.
Придворный часовщик и великий драматург Пьер Огюстен Карон де Бомарше по приказу
короля маскировал поставки военных грузов под коммерцию (и даже разорился на этом
прикрытии).
Конечно, всё это было не только следствием дипломатических талантов старого
учёного. Но без его творческого подхода к международным взаимоотношениям любовь
блистательного дворянства к буржуазным выскочкам вряд ли была бы столь пламенной.
Авторитет Франклина в Париже в свою очередь покоился на его научном и
общекультурном творчестве. Издатель «Альманаха бедного Ричарда» обогатил мир не
только изречением «Время – деньги». Ему принадлежат бифокальные очки, дающие равный
комфорт и при чтении, и при взгляде вдаль. Лёгкие и устанавливаемые где угодно печки,
известные у нас под оставшимся от гражданской войны названием «буржуйка» – тоже его
изобретение. А самое прославленное творение Франклина – громоотвод, спасший несметные
имущества и бесчисленные жизни – следствие его исследований электричества. Заодно
Франклин первым установил, что существует два вида электрических зарядов.
Изобретения зачастую внедряются в жизнь весьма не гладко.
Хотя физик Фаренгейт изобрёл новый инструмент, названный затем ртутным
термометром, идея использовать его для измерения температуры человеческого тела пришла
к нему не сразу. Это произошло, когда Фаренгейт докладывал в Академии наук в пользе
своего изобретения, и в Академии ему сказали, что пусть он лучше этот прибор засунет себе
в з… С тех пор это, в самом деле, стало способом измерения температуры тела, в том числе и
ряда домашних животных…
Например, тот же громоотвод был предметом множества судебных процессов. Так, во
Франции будущий прославленный революционер врач Жан-Поль Марат доказывал в суде:
громоотвод на одном доме повышает вероятность поражения молнией соседних зданий и
поэтому должен быть запрещён как источник повышенной опасности для окружающих.
Опасность от изобретения отвёл оппонент Марата на процессе – в ту пору начинающий
адвокат, а впоследствии соратник Марата и столь же пламенный революционер –
Максимиллиан Робеспьер. Он показал суду труды Франклина, откуда было видно:
утверждение Марата физически совершенно неверно. Громоотвод не перенацеливает
электрический разряд в сторону, а вызывает его на себя и проводит в землю безопасным
путём. Поэтому установка громоотвода полезна не только для самого домовладельца, но в
какой-то мере и для него не столь рачительных соседей.
Отметим, что последующие американские политики не были столь многогранными
изобретателями. В частности, только один из президентов получил официальный патент на
изобретение. Но это был великий освободитель рабов и победитель в гражданской войне
Авраам Линкольн.
Переходя к временам более близким, мы обнаружим на верху политической лестницы,
к сожалению, лишь немного столь многогранных в своей интеллектуальной деятельности
людей. Кстати, к ним, безусловно, можно отнести мэра Москвы Ю.М. Лужкова – инженерахимика, автора многочисленных изобретений в самых разных сферах.
Бигуди № 53
Поучительная история об изобретениях. Ещё в XVII веке Блез Паскаль и независимо
Готфрид Лейбниц (но и не только они) придумали и сконструировали различные типы
арифмометров. Причём Лейбниц утверждал: машины такого типа будут способны работать
не только с группами цифр, изображающими числа, но и с группами символов,
изображающими формулы, тексты и т. д. Он был абсолютно уверен: механические
устройства смогут правильно выполнять логические операции – подобно тому, как
арифмометры правильно выполняют арифметические действия. Эта идея большинству
современников Лейбница казалось полным абсурдом! В начале XVIII века взгляды Лейбница
были осмеяны в известном романе. Кто же автор этого романа, любимого и читаемого до сих
пор?71
От идеи до практики
Конечно, интеллект проявляется не только в изобретениях. Изобрести мало – надо ещё
воплотить придуманное в жизнь. А с этим бывают проблемы – зачастую не только
технические. Когда-то знакомые попросили академика Ландау помочь устроиться на работу
их родственнику – весьма, по их словам, способному молодому человеку. На что великий
физик заметил: умение найти работу тоже входит в число способностей. Это к тому, что
внедрение открытий и изобретений – особое занятие, в котором тоже не обойтись без умения
креативно и изобретательно мыслить.
Российское общественное мнение относится к изобретателям двояко. Те, кто, найдя
одну – две удачные идеи, тратит потом все силы на их проталкивание и даже не
задумывается, достойны ли находки подобных усилий, получили нелестную кличку
«чайник» – за чрезмерно бурное кипение возмущённого разума. Зато подлинные творцы,
щедро разбрасывающиеся десятками и сотнями новшеств, пользуются бесспорным и
массовым уважением. Только давайте не вносить в список подлинных изобретателей
некоторых полуобразованных функционеров, которых уже расплодилось несчётное
количество по ходу так называемых либеральных реформ. Вот уж кто не остаётся в тени,
раздавая обещания за наш с вами счёт: дескать, и напоим, и оздоровим. Счастье для всех и
«даром», так сказать. Ну нет, совсем не даром – сколько там запрошсил «великий Петрик
Первый» на программу «Чистая вода»?
К сожалению, и в наши дни слишком многие изобретатели не дожидаются
прижизненного воплощения своих находок. А ведь это не личное дело изобретателя – его
творение может иной раз осчастливить многие миллионы. Да и денег изобретение требует,
как правило, несравненно меньше, нежели потом экономит. Поэтому упущенные
возможности приносят ущерб не только изобретателю, но и всему обществу. Так что
прижизненное восстановление справедливости – в общих интересах.
Иногда технические трудности внедрения изобретения следуют из факторов
общекультурных. Например, застёжку «молния» Уиткомб Джадсон из США запатентовал
ещё в 1893-м. Но добыть средства на устранение неизбежных мелких шероховатостей
новинки так и не смог до самой смерти в 1909-м: никто не считал новое устройство полезнее
предыдущих. Поэтому проработка конструкции растянулась на десятилетия. Лишь в 1937-м
французский модельер Жан Клод понял: на основе новой застёжки можно создать более
изящные фасоны одежды. Только это сделало «молнию» действительно популярной и
привлекло к её совершенствованию деньги, достаточные для создания всего нынешнего
многообразия.
А вот какую историю одного изобретения рассказывает великий К.Э. Циолковский в
предисловии к своей книге «История моего дирижабля».
Роберт Фултон, изобретатель парохода, предлагает Директории (французскому
правительству, сменившему Робеспьера и потом сменённому Бонапартом), а затем и самому
великому полководцу своё изобретение, однако учёные советники Наполеона не желают его
слушать. Удивительно, но именно такие знаменитости и выдающиеся умы, как Лаплас,
Монж и Вольней, ставят над идеями Фултона могильный крест. Понятно, что после этого и
сам Бонапарт лишает великого изобретателя своей протекции.
Фултон представил императору Франции чертежи не только пароходов, но и подводной
лодки. Однако Наполеон, смахнув чертежи на пол, гневно сказал: «Ты просто хочешь, чтобы
мы потратили деньги на пустые и невозможные проекты, ловким манёвром хочешь
обескровить казну Франции? Нет, я не поверю шарлатану!» Понял бывший повелитель
империи свой роковой просчёт – недооценку идей изобретателя – лишь когда его уже везли в
ссылку на остров Святой Елены. Стоя на палубе парусника, Наполеон увидел идущий
встречным курсом пароход, созданный по чертежам Фултона. И тогда Наполеон, проводив
корабль взглядом, задумчиво проговорил: «Вот она, моя ошибка».
Увы, в истории великих изобретений, да и вообще в истории науки немало ошибок и
заблуждений. Причём это относится в первую очередь к тем, кому по должности, или по
нраву, или по недоразумению (в буквальном смысле) выпало «тащить и не пущать». Разве
выдающийся астроном Араго не выступал крайне резко против создания железных дорог,
уверяя всех в их практической нереализуемости 134? А сколько времени был вынужден
потратить на хождения по кабинетам чиновников почти голодающий вместе со своей семьёй
Самюэль Морзе, чтобы создать первую в мире телеграфную линию? А «водоход» Кулибина,
проданный на слом, потому что бурлаки были дешевле? И ещё многие и многие творения
мысли и рук показались ненужными или оказались несвоевременными, как паровая «пушка»
Архимеда, оптический телеграф Кулибина (через 35 лет такой же телеграф русское
правительство купит у французов за 120 тысяч рублей – в ту пору корова обычно и рубля не
стоила), махолёты да Винчи. Только время расставляет всё по своим местам…
Может быть, причина такого недоверия к новым изобретениям – не только
подсознательная нелюбовь и неосознанный страх перед всем новым, свойственные
среднестатистическому чиновнику. Мешает и огромное количество псевдо-изобретателей,
заваливающих серьёзных учёных горами своей «продукции» вроде всё тех же вечных
двигателей. Или непроверенных фильтров.
Изобилие продуктов такого творчества, в котором нелегко оказывается найти
134 Тогдашние оппоненты железных дорог ориентировались не только на дороговизну металла. Куда
важнее было простое рассуждение: трение колёс локомотива о рельсы не сможет преодолеть трение колёс
вагонов. Разница между трениями качения и скольжения ещё не была в ту пору толком изучена.
«жемчужное зерно», заставляет вспомнить высказывание Чарлза Кеттеринга: «Изобретатель
– тот, кто не относится слишком серьёзно к своему образованию». Но тем не менее это не
оправдывает тот факт, что по-настоящему важные и необходимые открытия и изобретения
могут годами и десятилетиями оставаться неизвестными и невостребованными. Вспомните
Наполеона: история, как всегда, способна повториться – и не только как фарс…
Бигуди № 54
Впрочем, неплохим изобретателем может оказаться и лицо весьма высокого ранга,
например, царствующая особа. Исторический пример: во времена Екатерины II молодые
столичные щёголи завели новую моду. Они стали разгуливать по Петербургу в белых
перчатках, с непременными лорнетами, дабы благосклонно рассматривать хорошеньких
горожанок. Мода сия зело не понравилась императрице. Однако попросту запретить новую
моду, издав указ, было для Екатерины дурным тоном – как же демократию потом строить в
этой стране? Хоть какой-нибудь Думы, которая взяла бы на себя быстрое обсуждение сразу в
трёх чтениях и единогласное голосование за новый неумный законопроект с последующим
его обоснованием, ещё не было, слава Богу. Пришлось Екатерине Великой выкручиваться из
ситуации самой. То есть указ царский, конечно, издавать пришлось, однако решение
императрицы было весьма остроумным. После сего указа никто из модников и не подумал
бы щеголять в белых перчатках с лорнетом в руках. Неприлично-с… Что же за указ издала
Екатерина? Почему вдруг стало неприличным для молодых дворян использовать эти детали
одежды?72
Концепция как изобретение
Рассуждая о теории красоты и красоте теории Эдвард де Боно в девятой главе книги
«Использование латерального мышления» отмечал:
«В известном смысле наука является высшей формой искусства, поскольку здесь
совершенство новой идеи не является делом вкуса или моды. И хотя науке явно недостаёт
эмоциональной насыщенности и всеобщей притягательной силы, тем не менее ей внутренне
присуща строгость. Различие между требованиями искусства и науки особенно наглядно
представлено в творчестве Леонардо да Винчи. Искусство Леонардо прекрасно – это
несомненно. Однако и его научные идеи подчас определялись единственным критерием –
красотой. Так, в набросках предложенного им летательного аппарата Леонардо больше
внимания уделил оформлению приспособлений, помогающих воздухоплавателям сойти с
аппарата на землю, чем самой летательной способности аппарата. Великого художника
больше занимала завершённость того, что было доступно восприятию, нежели реализация
того, что может понять только посвящённый».
Вообще говоря, изобретателем можно назвать каждого, кто создаёт нечто новое в
какой-либо сфере жизни общества. Вот примеры.
Конечно, великим изобретателем был Леонардо. Об этом свидетельствует множество
сделанных им чертежей «крылатых машин», движущихся под водой кораблей, создание им
камеры-обскуры. Но разве не была изобретением созданная им и – увы! – не дожившая до
наших дней конная статуя миланского герцога Сфорца? Неужели не художественное
изобретение – воздушная перспектива в «Джоконде» или потрясающая энергия знаменитых
картонов, изображающих битву при Ангиари? В этих двух последних примерах некоторые
художественные приёмы, позволившие создать выдающиеся произведения искусства,
использованы впервые.
«Архимедов винт» – выдающееся изобретение древнего гения. Как и его способ
фокусирования солнечной энергии с помощью системы зеркал или невероятной мощи
камнеметательные машины. Но разве не было изобретением открытие Архимедом
математического «метода исчерпывания» – прообраза исчисления бесконечно малых,
созданного Ньютоном и Лейбницем лишь спустя почти 2 тысячи лет? А изобретение
Альбрехтом Дюрером гравюры – это именно техническое продвижение? Или всё же великое
достижение в художественном творчестве? А жанр (впоследствии выросший в творческий
метод) научной фантастики, созданный Жюлем Верном, неужели не великое изобретение?
Или новая музыкальная форма – симфония, придуманная Йозефом Гайдном?
А чем считать обратную перспективу классических икон – Божьим указанием или
следствием бесталанности первых иконописцев и незнания ими законов живописи? Или
открытием безвестного художественного гения? Как показал знаменитый разработчик
систем управления ракетной и космической техникой академик Борис Раушенбах, такая
перспектива (всмотритесь в рублёвскую «Троицу»!) – не просто выбор из множества
психофизиологически равноправных систем отображения трёхмерного мира в двухмерный.
Это выбор именно идеологический, утверждающий примат взгляда на мир извне, с точки
зрения его творца. То есть икона – наглядная демонстрация божественного восприятия
божьего творения.
Жаль, что я не могу себе позволить уделить слишком много внимания этим
волнующим рассказам о создании великих произведений искусства и технических
изобретений – всё же не это главный предмет нашей книги. Обратим внимание, однако, ещё
хотя бы на взаимовлияние открытий и изобретений в различных сферах жизни и творчества,
взаимопомощь рук человеческих и мысли, тела и духа. Вот скрипки Антонио Страдивари –
конечно, плоды изобретательского гения, изменившего форму инструмента, создавшего
особый состав лака и грунтовки скрипок. Рядом с его инструментами, секрет звучания
которых так до конца и не разгадан, таинственно поющие скрипки Амати, инструменты
Гварнери. Великие изобретения технических гениев и только? Но если бы не эти скрипки
великих итальянских мастеров, разве написал бы Никколо Паганини «Вариации на одной
струне» и «Кампанеллу», а Джузеппе Тартини свои «Дьявольские трели»? На каком
инструменте можно было бы всё это сыграть?
Антон ван Левенгук, создатель уникальных микроскопов (после его смерти осталось
273 микроскопа и 172 линзы, собственноручно изготовленные), великий экспериментатор,
регулярно, каждые два месяца направлявший сообщения о новых открытиях – он мог
рассматривать предметы размером в 1.5 микрона! – в Лондонское Королевское общество, он
кто – естествоиспытатель, учёный, техник? Стараясь, как он сам писал, «сделать очевидной
истину…насытить свою страсть проникнуть… в начало вещей», разве не внёс он, пусть и
косвенно, выдающийся вклад в мировую художественную культуру? Ведь именно после
того, как Джонатан Свифт посетил его мастерскую – лабораторию в Дельфте, написаны две
новые части удивительных «Путешествий Гулливера».
Между прочим, все линзы Левенгука не шлифованные. Гениальный мастер капал
расплавленное стекло в воду и затем из бесчисленных застывших капелек отбирал дающие
наиболее отчётливое изображение. Технология по тому времени революционная.
Бигуди № 55
В передаче «Что? Где? Когда?» показали изображение старомодной – примерно
середины XIX века – высокой башни без окон. Вопрос, если отбросить литературные
красоты, выглядел просто: какую продукцию выдавало металлургическое предприятие, где
стояла башня?73
Не зря Левенгук держал свою технологию в секрете, и лишь последующие
исследователи, сравнив сотни его изделий, разобрались в методе их создания.
Уровень фантазии
Можно даже расклассифицировать и технические, и художественные открытия –
изобретения. Скажем, отнести к высшему уровню создание новых жанров или видов
искусства. В этом смысле создатель симфоний Гайдн, изобретатели первых опер Галилей
(старший) и Скарлатти, как и Дюрер с Верном, могут быть отнесены к изобретателям
«высшего уровня». Чуть пониже, где изобретения осуществляются в рамках уже
существующего жанра, расположим творцов следующего уровня. Например, Бетховен ввёл в
Девятую симфонию хор, Гойя усовершенствовал графическую технику создания офортов,
Герберт Уэллс создал социально направленную фантастику. Далее можно
усовершенствовать или найти новое необычное применение для инструментов
художественного творчества. Вот, скажем, «светящиеся» портреты работы Решетова, или
музыкальные иллюстрации Мусоргского «Картинки с выставки», или лингвистические
сказки Петрушевской, или… Понятно, что примерам несть числа.
А.Н. Лук приводит примеры фантазии, замешанной на несуразностях: «Одно время в
Вене пользовались успехом анекдоты о графе Артуре, построенные на «остроумии
нелепости».
«Какая разница между крокодилом? – спрашивает граф. – Между крокодилом и чем? –
Не крокодилом и чем, а между крокодилом? – Не знаю. – Крокодил в воде плавать умеет, а
по суше – нет».
Остроумие нелепости широко используется в жанре литературной пародии. Отличие
этого приема от доведения до абсурда состоит в том, что абсурдность высказывания
достигается путем преувеличения или преуменьшения, а остроумие нелепости заложено в
самой ситуации, противоречащей повседневному опыту и здравому смыслу.
Читатели знаменитой сказки Льюиса Кэролла «Алиса в стране чудес» много раз
смеются над остроумием нелепости. Возьмем, к примеру, рассказ о чеширском коте, на лице
которого почти всегда была улыбка. Иногда улыбка исчезала, и оставалось только лицо; но
случалось, что исчезало лицо, и оставалась лишь улыбка». К подобным актам можно отнести
и случай с Марком Твеном: когда печать распространила ложные слухи о его смерти, он
выступил с опровержением: «Слухи о моей смерти сильно преувеличены». Нелепость
формулировки сделала её остроумной.
Если же говорить только о фантастике, то можно использовать для оценки
произведений этого жанра шкалу «Фантазия», предложенную Генрихом Альтовым и Павлом
Амнуэлем. По этой шкале научно-фантастическое произведение оценивается по пяти
показателям:
✓ новизне идеи,
✓ убедительности её обоснования,
✓ человековедческой ценности (естественно, ибо хорошая фантастика – это хорошая
литература),
✓ художественному уровню,
✓ по чисто субъективному принципу: «нравится – не нравится».
Каждый показатель оценивается в баллах от одного до четырёх. Например, если в
рассказе используется уже известная идея, взятая без каких-то изменений у другого
писателя, оценка его по новизне – один балл. Если идея известна, но уже изменена и
позволяет видеть в старой ситуации нечто новое – два балла. Совершенно новая идея «стоит»
четыре балла.
В художественном творчестве классификация по уровням достижений всё же хотя и
возможна, но далеко не столь выразительна, как в техническом. Здесь можно выделить пять
уровней творчества. На первом, низшем, можно расположить мельчайшие изобретения, для
которых вполне достаточно знаний и технических средств в рамках одной профессии.
Второй уровень мелких изобретений требует уже знаний и технических решений в пределах
одной отрасли. Уровень средних изобретений возможен, если вы свободно ориентируетесь и
оперируете средствами в пределах одной науки.
Для крупных изобретений – создания новой технической системы – понадобится иной
уровень мышления. Придётся использовать знания разных наук, найти возможность по-
новому их синтезировать. Однако здесь ещё не требуется выйти за пределы существующего
знания.
Такой выход нужен только для крупнейших изобретений. Чтобы была создана
принципиально новая техническая система, могут оказаться необходимы такие технические
средства и научные сведения, которых ещё просто нет. Совершить открытие, выйти за
пределы известных знаний – необходимое здесь условие. Только в этом случае удастся
реализовать замысел. Тот самый замысел, который мог появиться либо как следствие
планомерной творческой работы, когда после долгих размышлений понимаешь логику
движения мысли, либо в результате интуитивного интеллектуального прыжка – озарения, с
которым, по словам Ницше, встречается далеко не каждый, кто готов к такой встрече.
Можно привести не один пример, подтверждающий разумность приведенной
классификации уровней изобретения. Но не это важно. Главное, что изобретательство в
любом деле опирается на интеллектуальную смелость, на умение находить неожиданные,
неизвестные ранее приёмы, методы, решения. Конечно, способности необходимы, чтобы
там, где обычно человек видит просто летящую птицу, увидеть летящего на механических
крыльях человека. Увидеть мыслью – и суметь эти крылья создать. Так что здесь не только
идея важна, но и способность её претворить в жизнь.
«Конечно, вдохновение – праздник мастера, но ремесло… оно тоже необходимо
художнику. Ремесло поставил я подножием искусству» – так говорил композитор Роберт
Шуман. И это относится ко всем видам творчества: нужно подготовить своё мышление к
творческой работе, а ещё нужно быть готовым не только генерировать идеи, но и уметь их
подпирать, закладывать под них фундамент.
Бигуди № 56
На этой известной работе изображены следующие предметы:
1) средневековые «лекарства» от сложных болезней и сумасшествия;
2) атрибуты Сатурна-покровителя;
3) символическое изображение геометрии, как искусства;
4) единица, как символ художественного начала;
5) намёк на возможность астрологической интерпретации;
6) символ бренности;
7) символ правосудия;
8) алхимический процесс;
9) символ устремлений человека.
Что это за работа? Кто её автор? Почему автор так её назвал? Последний вопрос,
конечно, носит субъективный характер, просто интересно, что вы сами об этом думаете.74
Ни дня без мысли
Благодаря книгам и идеям Г.С. Альтшуллера можно смело утверждать, что научиться
изобретать можно. Ну, конечно, без нестандартных решений, новых идей это вряд ли будут
изобретения очень высокого уровня. И всё же нужно заставлять свой мозг работать
постоянно. Вот тогда, может быть, и удастся повстречать то самое озарение, которое
посещает не всех. Великий Томас Алва Эдисон, автор более тысячи изобретений, установил
для себя минимальную квоту изобретений. Полагалось выдавать хотя бы одно маленькое
изобретение каждые 10 дней, а каждые полгода что-то покрупнее.
Почему бы и для себя не установить такие «социалистические обязательства»: каждый
день – хотя бы одна интересная и неожиданная мысль, идея, рассуждение? Каждую неделю –
идея посолиднее, каждый месяц… И пусть даже эти идеи покажутся нелепыми, смешными
или никому не интересными. А вдруг Ваша мысль на пути повстречает достаточно
безумную, чтобы быть почти гениальной, идею? Но для этого мысль должна двигаться!
Все сформулированные выше принципы творческого мышления можно дополнить
некоторыми приёмами, эффективно работающими в чисто изобретательской ситуации. Когда
требуется найти нестандартное техническое решение, соединив в одном механизме функции,
казалось бы, несоединимые. Когда нужно заставить силы природы действовать
неожиданным образом. Но это лишь на первый взгляд. Знаменитый «водоход» Кулибина
использовал собственную энергию течения реки, чтобы двигаться… против течения. Канат
заносили далеко вперёд и укрепляли на берегу. Затем он наматывался на барабан лебёдки,
установленной на корабле. Вращало же этот барабан течение реки, ударяя по специальному
колесу с лопатками.
В изобретательстве – как «отрасли» высокоинтеллектуального труда – изучение
проблемы при выходе на границы её «пространства» может заметно помочь продвинуться в
решении. Нужно рассмотреть предельную ситуацию, допуская, чтобы параметры задачи
менялись в очень широких пределах, приобретали экстремальные значения. Попросту
говоря, выходили даже за рамки реальности.
Например, чтобы взглянуть на проблему с иной стороны, или выделить основную
«загвоздку», или переформулировать задачу по-другому, можно попробовать «выключить»
силы гравитации и представить себе движение в таком мире. А потом уже подумать, какие
силы помогут произвести такое «выключение». Или представить себе, что изменились
свойства пространства: оно перестало быть однородным и изотропным, его свойства
меняются от точки к точке и от направления к направлению – Может быть, поток «чегонибудь» может как раз и обеспечить такие изменения? А если нужно получить
«пространство» с требуемыми свойствами, наверное, стоит попробовать пропустить через
него поток с определёнными характеристиками?
А можно ввести и самого себя в «пространство задачи», вообразив, что это ты сам
двигаешься внутри механизма и можешь разглядеть, как зацепляются шестерни, включается
под действием ручейка электронов процессор или происходит квантовый переход. Или
вообще сам всё это производишь… Значит, нужно стать очень маленьким, или наоборот,
огромным. Т. е. надо и самого себя – как параметр задачи – сделать экстремально
выделенным. Можно и наоборот, вместо себя ввести в задачу «маленького человечка»:
например, Джеймс Клерк Максвелл придумал маленького демона, сортирующего молекулы
по скоростям, отделяя быстрые от медленных. А потом, когда станет ясно, как эти человечки
осуществляют именно нужную операцию, можно подумать и о том, чем их заменить. Кому
передать их функции – потоку жидкости, воздуха, нескольким дополнительным механизмам
или ещё чему-либо.
Бигуди № 57
Почти у каждого на кухне сейчас имеется устройство, в котором в качестве «маленьких
человечков» работают молекулы воды, содержащиеся практически во всех продуктах.
Заставьте этих «человечков» быстрее двигаться – и Вы добились того, чего хотели!
Интересно, чего же Вы добивались, и как Вы сумели заставить быстрее двигаться
«маленьких человечков»? Кстати, первое такое устройство было установлено в ресторане
Бостона и весило оно тогда почти 400 кг. Размеры тоже впечатляли – высота устройства
была почти с рост взрослого человека! Изобретено же оно сразу после Второй мировой
войны, в 1946-м году.75
Теория фантазии
Изобретательских задач великое множество. Как и прекрасной литературы на эту тему.
Начиная с книг замечательного писателя, изобретателя и учёного Г.С. Альтшуллера
(Альтова). Это именно он разработал Теорию Решения Изобретательских Задач (ТРИЗ),
придумал и создал Регистр Фантастических Идей. Сюда заносились новые идеи,
обнаруженные писателем и учёным в различных фантастических произведениях. Изучение
их показало: большинство фантастических объектов и идей представляют собой результат
некоторых преобразований совершенно реальных предметов или процессов.
Более того, оказалось: приёмы, с помощью которых эти преобразования
осуществляются, практически совпадают с изобретательскими приёмами. Что, вообще
говоря, неудивительно – это тот же ряд операций, с помощью которых творческая личность
преобразует действительность. Мысленно или реально, при создании нового механизма или
в работе над фантастическим романом – это всё работает, генерируя идеи, одно и то же
мышление.
Анатолий Вассерман отмечает: «Знания, не относящиеся к теме основной работы,
позволяют взглянуть на нее в неожиданном ракурсе – подметить что-то качественно новое…
Творческий метод фантастики – искусственное конструирование обстоятельств,
позволяющих автору ярче выразить ключевые черты интересующих его явлений. В наших
быстро меняющихся обстоятельствах фантастика, на мой взгляд, куда эффективнее реализма
постигает реальность. Потому, что позволяет сосредоточиться на ключевых чертах явления,
выделить их и таким образом облегчить их анализ. Примерно таким же образом эксперимент
в контролируемых условиях значительно облегчает анализ явления по сравнению с его
наблюдением в природе… Фантастика – нечто вроде лаборатории, позволяющей
контролируемо изучать нужное явление».
Г. Альтов свёл типы преобразований в одну таблицу, названную фантограммой. Даже
одни названия этих приёмов отражают их универсальный характер: «Сделай наоборот»,
«Увеличение или Уменьшение», «Ускорение или Замедление», «Динамизация или
Статичность», «Универсализация или Ограничение», «Дробление и Изменение»,
«Квантование или Непрерывность», «Внесение или Вынесение», «Сдвиг во времени»,
«Оживление», «Изменение связей (взаимодействий)», «Изменение законов природы».
По сути дела Г. Альтов сконструировал «морфологический ящик» – Матрицу, чьи
ячейки заполняются новыми комбинациями известных понятий, объектов или процессов. В
данном случае по одной «оси» откладываются характеристики совершенствуемой вещи
(процесса), а на другой – основные эвристические приёмы их изменения. Рассматривая
элементы «ящика – Матрицы», можно выделить наиболее перспективные пути движения к
решению проблемы.
Большую и важную работу по ТРИЗ и Регистру Фантастических Идей Альтов вёл,
начиная с середины 1950-х годов. Возможно, часть этих наработок уже опубликована в
собрании сочинений Генриха Сауловича. Знаю, что в Интернете под редакцией писателя и
астрофизика Павла Амнуэля появился регистр идей научно-фантастических.
Группа энтузиастов (в основном – Молодых литераторов московского Клуба
Любителей Фантастики, созданного писателем-фантастом Юрием Никитиным) в конце 1990х заново составляла Регистр, независимо от имеющегося. В Клубе посчитали: любой авторсоставитель нового Регистра выигрывает в плане тренинга интеллекта больше, чем обычный
читатель имеющегося. Может, это и так.
Способы анализа и решения разнообразных задач, приёмы изобретательской
деятельности – все они имеют одну природу, всё это – различные проявления универсальных
принципов творческого мышления. Овладев ими, каждый сможет почувствовать себя
творцом, радостно встречающим неожиданные, необычные, удивительные мысли.
Приведу обширный отрывок из нашей коллективной монографии «Турбулентное
мышление. Зарядка для интеллекта»:
«Эффективным способом развития творческого мышления является выдумывание
артефактов. Примерно 180 лет назад Оноре де Бальзак описал шагреневую кожу,
исполняющую желания, но теряющую в размерах по мере исполнения этих желаний
(соответственно источалась и человеческая жизнь). Фёдор Сологуб в «Мелком бесе» вывел
среди важных, но бессловесных персонажей серую недотыкомку, вьющуюся по углам, не
имеющую ни определённой формы, ни содержания. Розовые очки, глядя через которые
человек испытывает переполняющее его чувство радости, напрочь забывает о повседневных
проблемах и заботах, «разработал» Клиффорд Саймак в «Пыльной зебре».
Карел Чапек представил такой эликсир бессмертия («Средство Макропулоса»),
который герои книги (олицетворяющие человечество со всеми его недостатками и пороками)
всё-таки отказываются принимать. Потому как никто из них не захотел бессмертия за счёт
выхолащивания смысла этой смертной жизни. Братья Стругацкие затем развили эту идею в
«Пяти ложках эликсира», обогатив исходник философскими и психологическими нюансами.
Трансляция идей – неизбежный и необходимый, равно как и их генерация, процесс, в
том числе и среди современников. Бывает, что идея просто носится в воздухе. Вспомним
хотя бы роман Сергея Снегова «Посол без верительных грамот», опубликованный в 1977
году, в котором сновидения зарегистрированы как средство общественной информации, и
каждый может транслировать и опубликовывать собственные сновидения. Уже через
несколько лет в произведении Кира Булычёва «Глубокоуважаемый микроб» описывается
«инопланетная коробочка», осуществляющая перевод человеческих воспоминаний прямо из
мозга в визуальную форму с последующей их продажей. Житель Великого Гусляра, города,
рождённого фантазией Кира Булычёва, изобретательный профессор Иван Христофорович
Минц разработал вакцину от тунеядства, заставляющую человека периодически и
трудолюбиво работать во благо общества. Стоит капнуть всего лишь две капли в стакан – и
самый отъявленный лентяй уже моет полы. Но, увы – прав был Парацельс, всё решает
чувство меры.
Альтернативная разработка принадлежит Станиславу Лему, его «Альтруизин» можно
описать так: «Психотрансмиссионный препарат, предназначенный для любых белковцев.
Вызывает распространение всех чувств, эмоций и ощущений того, кто непосредственно их
переживает, среди других существ, находящихся на расстоянии не более 500 локтей.
Основан на принципе телепатии. Непередача мыслей гарантируется. На роботов и растения
не действует. В соответствии с идеей изобретения альтруизин должен внедрить в каждое
общество дух братства, дружбы и глубочайшей симпатии, поскольку соседи каждой
счастливой особи тоже испытывают счастье». Вот только беда, оказалось, что боль тоже
передаётся, теми же путями, как и счастье, как и в реальной жизни «в нагрузку».
Тот же Лем в «Звёздных дневниках Йона Тихого» описал незагорающиеся спички для
детей, устройство для утилизации бесполезно растрачиваемой детьми энергии, жидкость для
выведения неприятных воспоминаний и тормоз для предотвращения вращения планет вокруг
оси.
Из артефактов за авторством Кэрролла вспомним навскидку некоторые: улыбка без
Чеширского кота; веер, помахиванием которого можно уменьшить себя до лилипутских
размеров; море из собственных слёз, которые надо сперва наплакать, а потом в них плавать;
гриб, откусив с одной стороны которого – уменьшишься, а с другой – вырастешь.
Дмитрий Сорокин в романе «Оберег» о былинных временах сластолюбивого князя
Владимира описывает простынь-самостилку. Полотно наподобие скатерти-самобранки, в
развёрнутом виде предлагает вниманию воспользовавшегося ею голую деву. Была, по словам
автора, изобретена восточными магами в одной стране, где князь извёл всех гулящих.
Наверняка знаком нашим читателям и неразменный пятак братьев Стругацкий из
«Понедельника, начинающегося в субботу». Пятак возвращается, если им платить. Если его
бросить, обронить, потерять, он остаётся там, где упал. Пятак возвращается в тот момент,
когда сдача из рук продавца переходит в руки покупателя. Если при этом держать руку в
одном кармане, пятак появится в другом кармане. Фантаст Сергей Лукьяненко придумал
почтовый конверт, который можно отправить в прошлое. А Рэй Брэдбери в произведении
«Чепушинки» вывел целый класс вещиц артефактного толка. Штуковины, финтифлюшки,
пустяковинки, барахлинки, штучки-дрючки, ерундовины и т. п. – предметы, являющиеся
образом собирательным (как внешне, так и функционально). Данными предметами торгует
прилетевший на Землю инопланетянин, изготовляющий их по одному ему известной
технологии. Все предметы имеют разные модели, в зависимости от года выпуска – в разные
года, люди по-разному представляли себе те или иные предметы.
Обозначить пока ещё неизвестное, но необходимое для выполнения уже сегодняшней
задачи, как «штуковину» или «фактор икс» – распространённый изобретательский приём»
[26, C.76–78].
Бигуди № 58
Вопрос мой связан и с «маленькими человечками», и с фантастическими
изобретениями. Думаю, многие читали в детстве замечательную книгу «Незнайка в
Солнечном городе» и помнят, какое количество интереснейших изобретений там описано. Я,
когда читал эту книгу, ещё не слишком разбирался в физике и технике. Поэтому почти все
изобретения казались мне вполне осуществимыми в нашей жизни. Ну, вот разве что в
машину на газированной воде я не слишком верил. Не напомните ли, что это за машины
такие были: Циркулина, Планетарка, Радиолярия?76 Может быть, у нас сейчас уже есть
такие же реальные устройства? Или всё это – чистейшая фантастика? Как, например,
труболёты, на которых зимой ездить просто замечательно, а летом – очень неудобно, потому
что… А почему?77
17. Проблема та же, только вид сбоку…
Ведь если вы Медведь с опилками в голове и думаете о делах, вы
иногда с огорчением обнаруживаете, что мысль, которая казалась
вам очень дельной, пока она была у вас в голове, оказывается совсем
не такой, когда она выходит наружу и на неё смотрят другие.
А.А. Милн. «Винни-Пух и все-все-все»
Позвольте вначале рассказать одну замечательную историю, связанную с именем
императора Наполеона Бонапарта.
Через считанные недели после того, как император вернулся с острова Эльба и на
плечах восторженных солдат был внесён в свой дворец, вновь разразилась война. Она была
мало кому выгодна, но тем не менее стала неизбежной.
Вернувшись во Францию, Наполеон Бонапарт честно попытался договориться с
коалицией враждебных держав о взаимном ненападении. Страна устала от непрерывных
войн с 1792-го по 1814-й год, обескровилась, была готова ещё много лет зализывать раны в
мире с соседями. Так что император, успевший, помимо военных подвигов, дать стране
новое – чисто буржуазное – законодательство, был склонен не столько творить новые чудеса,
сколько пожинать плоды былых посевов.
Но поверить скоропостижному миролюбию былой грозы Европы было практически
невозможно. Да и французская промышленность, взращённая в тепличных условиях
континентальной блокады, могла зачахнуть под вольным ветром торговли с Британией –
следовательно, могла в любой момент потребовать новой конфронтации. Независимо от воли
крестьянского большинства французов самые богатые и влиятельные из них объективно
нуждались если не в войне, то по меньшей мере во вражде с богатым островным соседом.
Как известно, поиск врага и жизнь в постоянном напряжении весьма выгодны всякой
власти. Британские же промышленники, давно уже занявшие ключевые позиции в
структурах власти, намеревались воспользоваться всеми властными возможностями, чтобы
предотвратить появление на континенте серьёзного конкурента. Так что, как видите, все
экономические предпосылки для войны были. Ружье, висевшее на стене, не могло не
выстрелить…
Довольно быстро выяснилось: за год изгнания «маленький капрал» не растерял
военные навыки. Прежде всего, разумеется, тот навык, который полководцы революционной
Франции успели за десятилетия противостояния всей Европе отточить до совершенства.
Немыслимому численному превосходству противника можно было противопоставить только
умение разделять его – а добившись хотя бы временного перевеса, бить по частям135.
Вот и теперь Наполеон блистал тем же искусством. По непролазной осенней
нидерландской грязи его армии носились, как по учебному плацу. Войска вражеской
коалиции не успевали соединиться и разлетались от французских ударов во все стороны.
Наконец, изгнав с поля боя пруссаков под командованием фельдмаршала Блюхера,
император отрядил маршала Груши преследовать отступающих и отправился на охоту за
последним в этой кампании противником – британцами, которыми руководил герцог
Веллингтон.
Встреча самой блистательной армии с самой упрямой разразилась близ нидерландского
(ныне бельгийского) городка Ватерлоо. Веллингтон, как всегда, выбрал самую удобную для
обороны позицию. В осенней грязи его войска, как всегда, держались до последней
возможности – даже когда сопротивление выглядело совершенно безнадёжным. Чтобы
прорвать британскую оборону, Наполеон бросил в бой последние резервы. Гром пушек даже
сквозь туман разносился на десятки новомодных в ту пору – именно революцией введенных
– километров.
Эту канонаду слышали Блюхер и Груши. Но выводы сделали разные.
Маршал Груши славился прежде всего дисциплиной. За всю свою военную карьеру он
ни разу не оставил боевой приказ неисполненным хотя бы в самой мельчайшей подробности.
Наполеон поручал ему труднейшие задания – и был уверен в конечных результатах. За это и
поднимал его по служебной лестнице. Высшее воинское звание Груши заслужил честно.
Фельдмаршал Блюхер же не зря слыл старым лисом. Приказы он, конечно, тоже
исполнял – но так, что авторы приказов только диву давались. Вот и сейчас он исхитрился
ночью оставить перед носом у Груши скромный арьергард – основные же силы увёл на
выручку союзнику.
В авангарде Груши быстро почувствовали неладное. Слишком уж мало следов
оставляла армия, за которой они гнались. А главное – слишком долго слышалась пальба у
Ватерлоо. Если Веллингтон всё ещё не сломлен – не лучше ли прекратить преследование и
помочь главным силам?
В конце концов генералы не выдержали – обратились к маршалу напрямую. Они не
только сомневались в успехе своей погони. Они ещё и сознавали: если британцы пересилят,
бить пруссаков будет незачем.
Понимал это и Груши. Наполеон имел из кого выбирать, и его маршалы действительно
были цветом армии. Но за всю свою карьеру Груши ни разу не ослушался приказа. А тут
приказ был прям и недвусмыслен: гнаться за пруссаками, не дать им оправиться и
перегруппироваться.
Военный совет, проведенный на марше, не изменил ничего. Армия Груши продолжала
погоню за призраком Блюхера. И с каждым шагом удалялась от Ватерлоо. Так что помощи
Наполеону не предвиделось.
Правда, в обычных обстоятельствах помощь бы и не потребовалась. Последние
французские резервы уже почти прорвали британскую оборону. Наполеон был в двух шагах
от победы. И в этот самый момент войска Блюхера вырвались на поле боя – и упали на
антинаполеоновскую чашу весов.
Из всех маршалов Наполеон выбрал для погони того, который не мог нарушить приказ
ни при каких обстоятельствах. Нерушимая исполнительность и безынициативность Груши
оказывалась необходимой десятки раз. Но рано или поздно не могла не обернуться
катастрофой. Опираться можно только на то, что оказывает сопротивление. Великая империя
рухнула, когда главной её опорой оказался человек без собственных решений, принимающий
135 Сам генерал Бонапарт потому и стал императором, что применил ту же технологию во
внутриполитических битвах. В 1799-м он расколол лидеров республики так, что без особого труда преодолел
сопротивление каждого.
их по инерции, по давно известному и вроде бы апробированному образцу.
В этой ситуации сыграла роковую роль не только невозможность для маршала – как
для человека военного – на свой страх и риск нарушить данный ему приказ. Конечно, с точки
зрения военного и это в самом деле безумная идея. И, увы, она была достаточно безумна,
чтобы оказаться верной.
Да, следовать букве инструкции, не вдумываясь в её дух, иногда очень соблазнительно.
В самом деле, особых духовных усилий для этого не требуется. А если случится что-нибудь
непредвиденное (а тем более нежелательное), всегда можно свалить вину на тех, кто не
предусмотрел соответствующий случай в инструкции. Хотя заранее ясно, что никакая
инструкция не может предусмотреть всего. Да и то, что в неё включено, ещё надо уметь
искать.
Вот ещё пример, который приводит уже упомянутый выше выдающийся физиолог,
первооткрыватель стрессовой реакции организма, лауреат Нобелевской премии Ганс Селье.
При изучении одной болезни (столь редкой, что во всей его родной Канаде в тот момент
никто ею не болел – пациенты нашлись только в США) ему понадобилось исследовать
образцы мочи. Чтобы она осталась достаточно свежей для исследования, её везли самолётом
(в запечатанных контейнерах). Но в аэропорту работники Королевской таможни не
пропустили контейнеры, поскольку данный предмет не упомянут в Королевской таможенной
книге ни в рубрике «Предметы, освобождаемые от пошлины», ни в рубрике «Предметы,
облагаемые пошлиной», а следовательно, пропущен быть не может.
Селье пишет: «Раздражённый этим глупейшим препятствием, я попросил нашего
декана написать официальное письмо самому высокому таможенному начальству и
объяснить, что для фундаментальных исследований иногда бывают необходимы предметы,
способные производить на людей непосвящённых несколько странное впечатление, но без
которых всё-таки не обойтись. Письмо было написано в достаточно сильных выражениях и
возымело действие. Всего несколько дней спустя мы получили ответ, из которого
явствовало, что, конечно же, федеральные власти предусмотрели такие случаи, просто
неопытный таможенник не смог найти в книге соответствующее место. Между тем объект
пересылки вполне чётко упомянут среди «освобождаемых от пошлины предметов», ибо он
со всей очевидностью подпадает под категорию «бывшие в употреблении личные
принадлежности». Теперь всё было в порядке, но, поскольку я знал, что к этому времени
содержимое посылки всё равно испортилось, я не стал её забирать. В последующие дни нас
одолевали настойчивые телефонные звонки из аэропорта, умоляющие избавить их от
«товара», который, по-видимому, стал вести себя угрожающим образом. Я же и пальцем не
пошевелил, утратив ко всему происходящему какой бы то ни было интерес».
Далее Селье с юмором заканчивает эту историю: «Наконец почтальон принёс
напечатанное типографским способом извещение – очевидно, и подобные случаи оказались
предусмотренными, иначе не нашлось бы подходящего бланка. От руки был вписан только
номер посылки, печатный же текст гласил: «Если в течение пяти (5) дней с момента
получения настоящего извещения Вы не востребуете вышеупомянутый товар, то посылка
будет вскрыта и её содержимое распродано с аукциона». Признаюсь, я не проследил за
дальнейшими событиями и по сей день не знаю, кто оказался счастливым обладателем
«вышеупомянутого товара».
Двигаться в сторону от всех
Помните старый анекдот о двух обезьянах, для испытания интеллекта которых высоковысоко подвесили связку бананов?
Одна обезьяна немедленно принялась прыгать. Сначала с места. Потом с разбега.
Известно ведь: чтобы съесть – надо достать, чтобы достать – надо дотянуться, то есть
допрыгнуть. Так что дело лишь в том, чтобы хорошенько разбежаться, сильно оттолкнуться,
и…
Правда, другая обезьяна почему-то прыгать не стала. Как-то ей лениво, что ли, было.
Нашла какие-то палки, верёвки, и стала мастерить длинный шест. Ну, тут конечно, первая
обезьяна ей всё и высказала: «Ты что, парень? Отлыниваешь?». А вторая в ответ: «Да вот
сейчас подумаю, как бы сделать палку подлинее – и всё, сразу банан достанем».
Такой ответ кого хочешь разозлит! Так что первая ей по-доброму объяснила: «Ты тут
не умничай. Чего тут думать – прыгать надо!»
Разве не так происходит часто и с нами, когда нам встречается незнакомая ранее
задача? Ведь и правда, сразу же хочется «прыгать» – это знакомое дело, это мы умеем, чего
тут думать – «прыгать надо»! Так мы ограничиваем своё мышление, прерываем поиск нового
решения.
Кстати, такая функциональная ограниченность категорически противопоказано
изобретателям. Ведь в этом деле важнейшее качество – увидеть новое, необычное
применение известного объекта, предмета, процесса. Но это и не удастся сделать, если
упрямо признавать за каждым предметом только одну функцию.
Знаете задачку: как измерить высоту здания, если у Вас в руках только часы-будильник
и термометр? Если Ваш интеллект ещё не свободен от функциональной слепоты, то проще
вообще отказаться от поиска решения.
А ведь имеется даже не один вариант решения! Скажем, можно с крыши бросить
термометр, засекая на часах начало падения и момент удара о землю. Зная время падения и
ускорение свободного падения, легко вычислить высоту здания. Можно, кстати, обойтись и
одними часами – засеките определённый момент времени и бросайте часы вниз. Потом
спуститесь и посмотрите на остановившиеся в момент удара о землю часы. Так определится
время падения.
Или такой экзотический способ: измерить высоту, просто последовательно
прикладывая термометр. Длину же термометра можно определить, используя для этого свои
руки – помните, что такое дюйм? Это длина первой фаланги большого пальца руки. В
британской системе мер дюйм принят равным 2.54 сантиметра. Вот видите, а ведь казалось,
что не получится. Может быть, придумаете и ещё какой-то способ?
Не жалеть умственных сил
Самое главное в борьбе за свои интеллектуальные достижения – понимать, что
мышление стремится двигаться с «наименьшими затратами». Принцип наименьшего
действия, похоже, действует и при движении в пространстве любой проблемы. Работая над
любой задачей, нужно объяснять самому себе: решение нужно искать даже там, где не
хочется. Где его, как поначалу представляется, и не может быть. Но ведь это лишь на первый
взгляд. Нужно заставить свою мысль отойти от фонаря и вести поиск не только там, где
светло!
Вновь приведу пример из своего архива телепередачи «Что? Где? Когда?». После того,
как знатоки быстро ответили на несколько сравнительно простых вопросов, ведущий
говорит: «… Ну, что ж, такая быстрая, молниеносная игра. Сейчас последняя надежда у нас
на Галину Лукьянову, оператора из города Лыткарино. Внимание! Слушаем вопрос».
Вопрос звучал так: «Этого актёра знают все. Видели этого актёра многие. Псевдоним
его знает кое-кто, а имя настоящее никто не знает. Вопрос, кто он, этот актёр?»
После минутного размышления и обсуждения шестерка Седина не находит ответа.
Единственная возможность – просить помощи у Клуба. За игру это можно сделать лишь
один раз. Пошло обсуждение. Однако Клуб тоже растерян. Есть только версии, но ответа
нет:
– Это Мухтар. Из фильма «Ко мне, Мухтар»
– Это Петрушка.
– Пётр Иванович Уксусов.
– Кукла.
Но снова звучит таймер. Минута использована. Ведущий требует немедленного ответа.
Отвечает Александр Друзь: «По нашему мнению, которое в общем-то у нас зародилось за
столом и нам любезно подсказал наш Клуб… мы думаем, что это артист Петрушка.
Настоящей фамилии не знает никто его, а псевдоним его Пётр Иванович Уксусов».
Комментарий В. Ворошилова: «Один ответ – Петрушка. Напомню вопрос. Этого
Актёра знают все (именно Актёра!), видели этого Актёра многие, псевдоним знает кое-кто, а
имя настоящее никто не знает.
Имени у этого Актёра нет, автор ему не дал имени, поэтому его и знать никто не может.
Псевдоним этого Актёра – Сверчков-Заволжский. Ну, может быть, кое-кто из вас и
вспомнит. Внимание! Персонаж из пьесы Горького «На дне» так и называется – Актёр, и
этого Актёра знали все из вас».
Нужно сказать, что во время ответа Александра Друзя был показан фрагмент из
спектакля «На дне» театра «Современник». В роли Актёра – Валентин Никулин. Да, это был
тест не на эрудицию, это было испытанием сообразительности, гибкости и рисковости
мышления.
Увы, объекты чаще всего и охотнее всего воспринимаются мышлением в привычном
образе и в связи с их обычным употреблением. Поэтому так трудно бывает переключиться на
иное, неожиданное их применение.
Бигуди № 59
Анекдот от физтехов. Студент физтеха приходит на экзамен, кладёт ногу на стол,
тушит сигарету о ведомость, бросает на стол пачку банкнот и говорит преподавателю: «Здесь
5 штук баксов – по штуке баксов за каждый балл». Сможете ответить этому студенту за
преподавателя, учитывая, что он всё же человек «раньшего времени», как говорил
Паниковский, и вряд ли поставит самоуверенному студенту желаемую отметку?78
От новых деталей к новой картине
Чтобы повернуть объект другой стороной, нужно найти в нём какие-то новые свойства
и качества. Значит, нужно получить новые знания о характеристиках предметов и процессов.
Это вовсе не хождение по замкнутому кругу: для творческого решения задачи – что
подразумевает получение синтетического нового знания – следует вначале получить новое
знание о составляющих элементах задачи. Это – частное новое знание, относящееся к
отдельным подзадачам. В конце все детали будут собраны в единую картину.
Как работает приём «сделай наоборот», «переверни проблему»? Читатель слышит
каждый день: «погоду передают»! А ведь мало кто задумывается, что честь
изобретениястоградусной шкалы термометра принадлежит шведскому геофизику и
астроному Андреасу Цельсию – профессору Упсальского университета. Правда, Цельсий
принял за ноль градусов температуру кипения воды при нормальном атмосферном давлении,
а за сто градусов – температуру таяния льда. Когда же стали использовать термометры,
шкалу оказалось удобнее перевернуть, поменять местами 0 и 100 градусов, принять таяние
льда за ноль. Эту идею выдвинул великий шведский натуралист Карл Линней. Он в те же
годы – первая половина XVIII века – преподавал в Упсальском университете медицину и
естествознание.
Между прочим, даже простая осведомлённость о какой-то конкретной идее может
помешать формированию Вашей личной оригинальной идеи. Некоторые специалисты
советуют на начальном этапе знакомства с проблемой исключить чтение книг и статей по
этой теме, стараться самому поставить задачу, как будто до сих пор она вообще никем и
никогда не решалась. Так – будто бы – Мышление не загоняется в рамки чужих
представлений.
Это предложение мне кажется спорным. По крайней мере, оно не универсально: то, что
поможет одному, сразу же поставит в тупик другого, и он вообще не сможет сформулировать
проблему. Нужно подходить индивидуально – ведь расшевелить своё мышление можно и
пользуясь чужими идеями. Тут всё зависит от способности их анализировать,
перерабатывать, относиться к ним критически.
Бигуди № 60
В одном журнале за 1901-й год опубликованы два любопытных сообщения
относительно одного и того же факта, интересного и важного для современников. Первое
сообщение было посвящено выступлению на медицинском Конгрессе в Риме доктора
Казагранди, в котором докладчик наглядно продемонстрировал его вред. Второе сообщение
гласило: «Большая часть населения Дрездена подала в городской совет петицию,
ходатайствуя о запрещении дамам…». Внимание, вопрос: о чём шла речь в этих
сообщениях? И что же такое обнаружил учёный доктор, доказывая вред этого? Видимо, он,
внимательно изучив некий объект, увидел в нём новое качество, не правда ли?79
Идти во все стороны
Но только ли функциональная «зашоренность» на вариантах использования предмета
или процесса мешает эффективному мышлению? Увы, не только. Вот пример. Вы решаете
трудную задачу, попали в тупик и наконец сформулировали несколько приемлемых
вариантов дальнейшего движения. Один из этих вариантов знаком Вам более других – нечто
похожее уже когда-то встречалось в задачах и там было успешно применено. И теперь Вы
подсознательно отдаёте предпочтение этому варианту. Хотя решаете уже совсем иную
проблему, нужны тут совершенно другие методы. Но память о прошлой удаче повышает
уровень ожиданий – кажется, что «пробиваться» нужно именно в этом направлении. Ведь
когда-то это же сработало! Нужно много усилий приложить, чтобы переубедить себя и
продолжить поиски. А для этого следует ещё раз детально сравнить ту, успешно решённую в
прошлом, задачу, и новую – возможно, принципиально отличающуюся.
В процессе пересмотра вариантов следует периодически ревизовать имеющиеся
методы и направления движения. Гипотезы, предположения, идеи – всё нужно
«перетряхивать», по мере необходимости удалять залежавшиеся и потерявшие ценность. Не
должно быть идей, от которых никогда, ни при каких условиях нельзя отказываться.
Представление о такой незыблемой и окончательной истине не даёт мысли свободно
двигаться, создаёт в пространстве проблемы своеобразный «центр притяжения». К нему, как
мужик в корчму, будут невольно заворачивать в своём движении логические цепочки. И это
неравноправие идей может стать губительным. Ведь истина капризна. Она может ожидать
Вас не там, где Вы раньше с ней встречались. Да и выглядеть будет не так. Нужно
внимательно всматриваться в «лица» встречных идей, чтобы не пропустить её,
Единственную.
И не давать предвзятости одерживать над собой верх! Потому что ум может… все
ухудшить. Публикации в Journal of Personality and Social Psychology информируют, что
«более изощренные в познавательном смысле участники исследований продемонстрировали
большую предвзятость. Исследование проводилось на предмет различных предрассудков, и
те, кто размышлял дольше, оказались более склонными к совершению ошибок. Почему же
умные люди иногда выглядят такими тупыми? Авторы не дают ответа на этот вопрос. Они
просто предполагают, что живущий в мозгу способ обработки информации позволяет
гораздо легче замечать предрассудки других людей, чем обращать внимание на собственные
ошибки такого же рода. «Неясно, как справиться с этой проблемой. Можно выпить, чтобы
ослабить остроту своего ума», – говорят исследователи, но не гарантируют, что станет чуть
лучше. (Why Smart People Are Actually Dumb, 14/06/2012 19:05).
Всё может быть
В процессе выработки синтетически полного решения проблемы наверняка придётся не
один раз использовать ранее сформулированные принципы эффективного мышления,
обобщая задачу или рассматривая её отдельные аспекты. «Прохаживаясь» таким образом в
пространстве проблемы, рассматривая пограничные (с предельными значениями параметров)
ситуации, удаётся постепенно отойти от стандартных устойчивых способов решения.
К вопросам Киплинга автор метода мозгового штурма А. Осборн предлагал добавить
вопрос «Какие другие использования данного предмета могут быть?».
Это ещё один вариант заставить себя выбраться из проторённой колеи мышления и
найти что-то новое.
Примеры бытового уровня – шариковая ручка с несколькими стержнями, запасными
или разных цветов, набор отвёрток при одном держателе, или свёрл для одной и той же
дрели, сборная и разборная гантель… Что касается операций, то в том же токарновинторезном станке с тех пор, как его придумал в конце XVIII века английский механик
Генри Модсли, основной принцип – согласование вращения шпинделя станка и движения
подачи резца. Резцы могут крепиться к единому механизму в пространстве, хотя и
использоваться последовательно и попеременно – во времени. На том же принципе
параллельного или последовательного выполнения нескольких операций, объединения
потоков и частей или дробления на части и потоки, основано искусство монтажа, и не только
кино– и фото-, а и любых коммуникаций и линий, от информационных до трубопроводных.
Телефон-факс и ксерокс (сканер+принтер) объединяют в себе несколько функций, хотя
они и проистекают во времени последовательно, но зато осуществляются в едином
устройстве вместо двух разных. И музыкальные центры, и кухонные комбайны – соединение
нескольких устройств с разными функциями в одно.
Тренировка мышления есть фактически развитие гибкости своего мышления. Приведем
её определение, данное У. Гордоном: это характеристика мышления, которая «предполагает
выделение существенных сторон изменений, возможность отхода от привычных действий,
от стереотипа, нахождение новых путей решения, комбинации элементов прошлого опыта».
Креативность изобретателя гибнет там, где произносится «это невозможно», «этого не
может быть». Но именно там, где Вы услышите эти слова, следует ожидать остроумных
перпендикулярных решений существующей проблемы.
Например, блестящий – заслуженно прозванный «генералом» – специалист по
военному делу (по совместительству фабрикант пряжи, один из основателей теории
коммунизма и верный спонсор своего соратника Карла Маркса) Фридрих Энгельс в главе
«Теория насилия» своего знаменитого полемического труда «Анти-Дюринг» писал: «…
оружие теперь так усовершенствовано, что новый прогресс, который имел бы значение
какого-либо переворота, больше невозможен. Когда есть пушки, из которых можно попадать
в батальон, насколько глаз различает его, когда есть ружья, из которых с таким же успехом
можно целить и попадать в отдельного человека, причём для заряда требуется меньше
времени, чем для прицела, – то все дальнейшие усовершенствования для полевой войны
более или менее безразличны».
В 1877-м это утверждение было не слишком далеко от истины. Точность изготовления
оружия достигла предела возможностей тогдашних станков. А главное – дымный порох
радикально сдерживал совершенствование конструкций и ограничивал как дальнобойность,
так и скорострельность: скажем, появившиеся незадолго до того картечницы
(многоствольные – поэтому тяжёлые и крепящиеся на станках – винтовки с механическим
приводом системы перезаряжания) считались оружием для стрельбы вслепую – первый же
десяток выстрелов перекрывал дымом поле зрения стрелка. Но как мог выдающийся
диалектик забыть: любое препятствие, противоречащее развитию техники, рано или поздно
будет преодолено!
Впрочем, Энгельс всё-таки в основном теоретик – вся его практика сводилась к
командованию добровольческим батальоном в Германии во время революции 1848–9-го
годов. Но вот сходное мнение выдающегося практика. «Отец» тяжёлой германской
артиллерии фельдмаршал фон Шлиффен в 1909-м году заявил: «Представляется уже
бесполезным добиваться дальнейших усовершенствований и ставить перед изобретателями
новые задачи. Всё мыслимое уже достигнуто». Это высказано как раз накануне появления
радикальных нововведений в этой области – Миномётов, сверхдальнобойных (до 150 км!)
пушек, автоматических орудий…
Выступая в 1933-м году на заседании Британской ассоциации содействия развитию
науки, Э. Резерфорд заявил: «всякий, кто ожидает получения энергии в результате
трансформации атомов, говорит вздор». И это сказал тот, кто открыл структуру атома!
Великий физик был не так уж неправ. Энергия, нужная для преодоления
электрического отталкивания между ядром и заряжённой частицей (или же для получения
нейтрона, не отталкивающегося от ядра), сопоставима с энергией, выделяющейся при
преобразовании. Самопроизвольное выделение нескольких нейтронов при распаде очень
тяжёлых ядер – вроде урана – оказалось неожиданным на заре ядерной эры подарком
природы. Рассчитывать на такие подарки нельзя. Правда, надеяться можно. Но
первооткрыватель атомного ядра не хотел рисковать своей репутацией, ставя надежду выше
точного знания.
Опыт высказываний, приведенных здесь, давно уже обобщён в формуле: если
выдающийся специалист говорит, что некое дело возможно – он скорее всего прав; если же
он утверждает, что оно невозможно – скорее всего ошибается!
Бигуди № 61
Как Вы считаете, подтверждает или противоречит вышеприведенной обобщающей
формуле знаменитое утверждение Пьера Ферма, сформулированное и записанное им на
полях книги и впоследствии названное Большой теоремой Ферма? И вообще – доказана ли
она наконец (это уж, скорее, проверка эрудиции)? Ну, да если великий математик и ошибся,
не будем принимать этого близко к сердцу: как говорил Дизраэли, ошибки великих людей –
всего лишь утешение для тупиц…80
На крайний случай
Можно выделить два типа интеллектуальной гибкости: спонтанную и образную.
Безусловно, это разделение носит индивидуальный характер. Однако развить в себе
образную гибкость мысли можно. Применение методов синектики – поиск метафор и
сравнений при формулировке разных типов аналогий – развивает эту характеристику
мышления. Значит, «расшевелить» свои ленивые мысли вполне возможно? И есть способ
взглянуть под иным углом на ту же проблему? Конечно.
Ранее мы предложили некоторые принципы, которыми стоит руководствоваться при
ознакомлении с «пространством проблемы» и движением в нём. Так вот, ещё раз скажем:
очень полезен принцип экстремумов – изучение задачи в том виде, который она приобретает,
когда её переменные параметры достигают своих предельных значений. Скажем, равны
нулю или стремятся к бесконечности. Например, классическая механика получается из
квантовой, если приравнять нулю постоянную Планка, или из теории относительности, если
счесть скорость света бесконечной. Предельный переход заметно упрощает задачу. Впрочем,
иногда и усложняет: некоторые классические задачи удобнее решать с помощью
математического аппарата современной физики.
Заметьте: в основе изобретательской деятельности часто лежит именно такой подход.
Допустим, что сила тяготения равна нулю, и… Или: представим, что трение отсутствует,
тогда… В своём предельном варианте задача может легко решаться. И это эффективная
подсказка. Ведь так удаётся выделить основную загвоздку, уцепиться за тот самый «гвоздь в
подошве».
Вот ещё один пример широко используемого в изобретательстве «принципа перехода в
другое измерение или по ту сторону проблемы».
Традиционные шахматы – игра на плоскости. Столбовые шахматы [19] выходят и на
вертикаль. Конструктивно фигура выполнена в форме шашки с выпуклостью сверху и
вогнутостью внутри, а на боковую поверхность нанесён соответствующий шахматный
символ. «Съедая» фигуру противника, игрок не снимает её с игрового поля, а накрывает
своей фигурой, захватывает. Образующийся столб из шашек-фигур ходит по правилам,
предусмотренным для верхней фигуры, при этом он может распадаться на части (ходит либо
весь столб, либо его верхняя часть, оставляя на клетке нижнюю часть столба). А если
«съедается» вражеская фигура, уже находящаяся на столбе, то её в таком случае можно
снять, открыв (освободив) свою фигуру, «съеденную» противником ранее, и таким способом
вновь нарастить свои силы.
Кроме того, шахматы (и шашки) отличает ярко выраженная детерминированность. И
можно внести в игру вероятностный фактор посредством игральной кости, на стороны
которой нанесены попарно символы каждого из трёх ортогональных измерений игрового
пространства: вверх – вниз (вертикальное), вправо – влево (горизонтальное), вперёд – назад
(ортогональное) (или символы шахматных фигур – в случае столбовых шахмат).
Перемещение осуществляется в соответствии с тем, какой стороной повернётся игральная
кость. Ещё одна модификация данного кубика может быть использована в настольных играх,
непосредственно связанных с перемещением какого-либо знака по игровому пространству и
использующих в качестве игрового элемента различия во времени жизни. Так предлагается
игральная кость, на стороны которой нанесены попарно символы каждого из трёх измерений
(периодов) времени: прошлое – вчера, настоящее – сегодня, будущее – завтра. Попарность
достигается тем, что вводится различие времени по дополнительному фактору – полу.
Например: на сторонах игрального кубика попарно изображены – Мальчик – девочка,
мужчина – женщина, старик – старуха…
«Проблема принятия решений в стрессовых ситуациях издавна привлекала людей,
вынужденных по роду своих занятий часто действовать в состоянии неопределённости,
когда выбор одной из нескольких приемлемых альтернатив (такой несложный во время
теоретических дискуссий!), оказывается, заставляет проявлять силу характера – ведь выбор
делается по мотивам, которые самому до конца осознать трудно. Для тех, кто в своей
повседневной работе сталкивается с необходимостью мыслить быстро, решительно, порой на
грани риска, не всегда имея возможность на ходу исправить все минусы принятого решения
(а действовать надо было непременно!), для этих людей шахматы представляют отличную
игровую модель для развития способности ориентироваться в потоке самой разнообразной
информации.
К сожалению, традиция рассматривать шахматы только как игру видимыми пешками и
фигурами всё ещё сильна в сознании не только любителей, но и шахматных теоретиков,
которые совершенно игнорируют их философское содержание» – отмечал выдающийся
советский гроссмейстер Давид Бронштейн в «Самоучителе шахматной игры».
Что тут скажешь? Приходит на память знаменитый фильм-притча Ингмара Бергмана
«Седьмая печать», где рыцарь Антониус Блок играет партию в шахматы со Смертью на
протяжении всего повествования. «Играет» в буквальном и переносном смыслах.
Вспоминается и рассказ из серии «Хроники Амбера» – «Синий конь и танцующие горы» –
Роджера Желязны, где представитель Порядка и представитель Хаоса ведут между собой
игру, напоминающую трёхмерные шахматы. При этом фигуры олицетворяют ключевые
фигуры в судьбе Вселенной, а ходы – процессы, происходящие с ними или при их участии.
Зоолог Томас Генри Хаксли (1825–1895) – один из ярких сторонников теории
эволюции Дарвина и увлечённый шахматист – писал: «Шахматная доска – это мир, фигуры –
явления Вселенной, а правила игры – то, что мы с вами называем законами природы!»
Такое мнение, однако, оспорил Норберт Винер, но именно шахматы и навели его на
нижеследующие размышления ещё в 1950-е годы (Wiener N. The Human Use of Human
Beings. Cybernetics and Society. London: Eyre and Spotswood, 1954). Этот математический
гений сформулировал, на наш взгляд, одну из основных причин разрыва между учёным,
изобретателем и прочим обществом. И в ней раскрыта загадка, почему большинство из
наших коллег в области науки и на поприще изобретательства – «не от мира сего»:
«Что касается природы дьявола, то известен афоризм Эйнштейна, представляющий
собой больше, чем афоризм и действительно являющийся положением, выражающим основы
научного метода: «Бог коварен, но он не злонамерен». Здесь слово «бог» употреблено для
обозначения тех сил природы, которым присущи свойства, приписываемые нами его очень
смиренному слуге – дьяволу, и Эйнштейн имеет в виду, что эти силы не обманывают. Повидимому, этот дьявол по своему характеру близок Мефистофелю. Когда Фауст спросил
Мефистофеля, что он такое, Мефистофель ответил: «Часть силы той, что без числа творит
добро, всему желая зла». Иначе говоря, дьявол не безграничен в своей способности
обманывать, и учёный, который в исследуемой им Вселенной ищет стремящуюся запутать
нас позитивную силу, напрасно теряет время. Природа оказывает сопротивление стремлению
раскрыть её тайны, но она не проявляет изобретательности в нахождении новых и не
подлежащих расшифровке методов, с тем чтобы затруднить нашу связь с внешним миром.
Это различие между пассивным сопротивлением природы и активным сопротивлением
противника наводит на мысль о различии между учёным-исследователем и воином или
участником состязаний. Учёный-исследователь должен всегда проводить свои
эксперименты, не боясь, что природа со временем раскроет его приёмы и методы и изменит
свою линию поведения. Следовательно, его работа направляется его лучшими намерениями,
тогда как игроку в шахматы нельзя сделать ни одной ошибки, не обнаружив, что бдительный
соперник готов извлечь из этого все выгоды, чтобы нанести ему поражение. Таким образом,
шахматный игрок руководствуется скорее худшими, чем лучшими намерениями. Возможно,
это утверждение представляет собой результат личного предубеждения, так как я нашёл
возможным для себя эффективно работать в науке, в то время как игрока в шахматы из меня
не вышло вследствие моей постоянной небрежности, проявляемой в критические моменты
игры.
Учёный склонен, следовательно, рассматривать своего противника как благородного
врага. Такая точка зрения необходима для его деятельности как учёного, но она может
превратить его в игрушку в руках беспринципной военщины и политиканов. Следствием
этой позиции является трудность понимания учёного толпой, ибо для толпы больший
интерес представляют индивидуальные противники, чем такой противник, как природа.
Мы погружены в жизнь, где мир в целом подчиняется второму закону термодинамики:
беспорядок увеличивается, а порядок уменьшается. Всё же, как мы видели, второй закон
термодинамики, хотя и может быть обоснован в отношении всей замкнутой системы,
определённо не имеет силы в отношении её неизолированных частей. В мире, где энтропия в
целом стремится к возрастанию, существуют местные и временные островки
уменьшающейся энтропии, и наличие этих островков даёт возможность некоторым из нас
доказывать наличие прогресса». 136
Изобретённая моим братом Нурахмедом игра «RussPack», получившая ещё в начале
1990-х компьютерное воплощение и входящая в комплект поставки игр Microsoft, по форме
родственна головоломке «15». Но эти «пятнашки» необычные. В них применён
изобретательский приём, который я бы назвал инверсией137. В классической головоломке
136 Винер Н. Человеческое использование человеческих существ: кибернетика и общество / Винер Н.
Человек управляющий. – СПб.: Питер, 2001. – С. 30–31.
137 У Альтшуллера он фигурирует ещё среди операторов АРИЗ – алгоритма решения изобретательских
задач, впервые сформулированного в начале 1960-х – под названием «сделай наоборот», а впоследствии
«выверни наизнанку». Любое решение изобретательской задачи – это в первую очередь выворачивание её
условий и факторов наизнанку, не считаясь с безумностью результата.
Сэмюэла Лойда одна фишка отсутствует, и на пустое место можно продвинуть любую
соседнюю. Здесь же одна фишка лишняя. Она торчит за пределами виртуальной игровой
коробки. Её можно двигать вокруг массива всех остальных, а можно вдвинуть внутрь. Тогда
она продвигает весь ряд, в который попала, и с другого конца ряда выдвигается новый
управляющий элемент. Таким образом, мы меняем топологию пространства, выворачиваем
его наизнанку.
Раз уж промелькнуло слово «виртуальный», упомяну и ещё одно изобретение моих
братьев, сделанное в середине 1990-х годов. Обычно по компьютерному миру ходят, двигая
мышку рукой – но это противоестественно для человека. Нурахмед и Нурулла Латыповы
применили к компьютерной мышке всё тот же приём инверсии. Они впервые предложили
способ погружения человека в виртуальную реальность с возможностью свободного
перемещения на собственных ногах в любую сторону на любые расстояния.
Для этого человек надевает виртуальный шлем, подключённый к компьютеру, и
помещается в тренажёр в виде пустотелой сферы – как бы внутрь шарика радио-мыши.
Сфера установлена на шаровых опорах и может вращаться вокруг любой своей оси. Здесь
уместна аналогия с белкой в колесе: обычно мышь перемещается по коврику (или сфера – по
полу), а мы оставляем её на месте. В отличие от беличьего колеса и стандартных тренажёров
«беговая дорожка» человек в сфере может перемещаться в любую сторону. На стандартной
спортивной беговой дорожке скорость бега регулируется скоростью проворачивания ленты,
в сферическом тренажёре – наоборот: скорость проворачивания сферы естественно
регулируется скоростью перемещения человека «на своих двоих». Моделируемое
компьютерное пространство при этом изменяется в соответствии с перемещением человека,
его действиями, направлениями его взгляда.
Предлагаемая система позволяет пользователю отрабатывать свои действия в любой
сымитированной обстановке. Все действия протоколирует компьютер: в дальнейшем можно
провести их разбор, вынести оценки, выработать рекомендации по улучшению действий в
подобных обстоятельствах. Программно можно синтезировать абсолютно любую
трёхмерную обстановку с соответствующими звуками, с любыми объектами, имеющими
свои независимые действия и свои правила поведения. И человек чувствует себя в ней
естественно, ведь задействовано всё тело, вся моторика.
Думаю, это изобретение – следующий шаг после того как тоже братья (Люмьер)
придумали кинематограф. И если в классном кино мы втягиваемся в действо, происходящее
на экране психологически, то через виртуальную сферу мы реально становимся участниками
происходящего, артистами и зрителями в одном лице.
От финиша к старту
Иногда весьма полезным оказывается «принцип обратного движения». Представьте,
что Вы уже у цели: известно уже практически всё, что нужно для получения окончательного
ответа. Остался последний шаг. То есть Вам понятно, что нужно для получения
окончательного решения, вы «забыли» на минуточку, что до этого последнего этапа ещё не
дошли. Но теперь Вы точно знаете, чего Вам не хватает – а где же взять вот это…, откуда мы
знаем, что…, с чего мы взяли тот факт, что… Вот на эти-то вопросы мы теперь и будем
искать ответы, двигаясь назад практически уже от «финишной ленточки».
Так удаётся вернуться к исходной точке. К тому самому месту в начале пути, где Вы
ещё недавно стояли, озираясь в недоумении. Скажем, в доказательстве теоремы Вы
возвращаетесь к началу, по пути доказывая все те леммы, без которых не верен
окончательный вывод. Если удалось пройти от сердцевины лабиринта к его началу, разве
теперь Вы не сможете совершить обратный путь?
Бигуди № 62
Строго говоря, это задача по математике. Однако идея её решения весьма изящна. Мне
хотелось бы, что вы почувствовали красоту идеи. Итак, однажды утром, как только взошло
солнце, буддистский монах начал восхождение по узкой тропе к храму на вершине горы.
Монах шёл по дорожке с разной скоростью, иногда останавливался, чтобы отдохнуть,
закусить сушеными фруктами, попить воды. К храму он подошёл незадолго до захода
солнца. После нескольких дней размышлений, молитв и поста монах пустился в обратный
путь. Он вышел на рассвете, как только взошло солнце, и спускался тоже с неодинаковой
скоростью, отдыхая по дороге. Средняя скорость спуска, конечно, превышала среднюю
скорость подъёма. Нужно доказать: на тропе есть такая точка, которую монах проходил во
время спуска и подъёма в одно и то же время суток (хотя и в разные дни). Так вот идея:
представьте, что на рассвете, как только восходит солнце, два буддистских монаха
одновременно пускаются в путь. Один поднимается вверх по узкой тропе, к храму на
вершине горы. Второй начинает спуск с этой вершины, от храма. Они идут навстречу друг
другу и…. Вот перед вами почти решение задачи. Ещё чуть-чуть…81
Мозговой штурм
Один из способов найти нестандартные пути к решению – Метод мозгового штурма.
Правда, штурмовать проблему нужно не в одиночестве. То есть творить новое, высказывать
смелые идеи, проходить сквозь «Великую китайскую стену» трудной задачи придётся
сообща138.
Конечно, не всякое творчество возможно при работе группой. Скажем, писать стихи –
по крайней мере, хорошие – «бригадным методом» невозможно. А вот решать научную или
техническую проблему – и интересно, и полезно.
В научно-фантастическом рассказе Раймонда Ф. Джоунса «Уровень шума» высокие
военные чины – как всегда, с соблюдением максимальной секретности – сообщают группе
учёных: некий исследователь-одиночка сумел построить антигравитатор – прибор,
уничтожающий гравитационное поле.
Среди учёных – представители очень разных специальностей: физики, математики,
психологи и др. Все они крайне скептически относятся к этому сообщению, однако им
демонстрируют случайно сделанную видеозапись трагически закончившегося полета
гениального изобретателя.
Учёные недоумевают, сомневаются, однако возникает и мысль: «А вдруг…?».
Начинаются споры, выдвигаются и рушатся гипотезы. После нескольких недель совместной
работы выкристаллизовывается странная, почти невозможная идея. Но ещё через месяц её
уже можно попытаться проверить экспериментально. Через полгода работы создан прибор, в
поле которого небольшой предмет «взлетает» на полметра, теряя вес! Всего на полметра! А
как же поднимался на десятки метров над землей погибший исследователь?
И вдруг тот самый неизвестный гений, разбившийся при испытаниях своего прибора,
входит в лабораторию! Так раскрывается секрет: всю эту инсценировку задумал некий
психолог. Цель – использовать метод «мозгового штурма» для решения важнейшей научной
проблемы. Но малейшая предвзятость по отношению к труднейшей задаче науки и техники,
известная уверенность в абсолютной невозможности её решения (вот вам и функциональная
ограниченность!), опирающаяся на современные знания – и решение никогда не было бы
найдено. Чтобы не было и тени сомнения в возможности её решения, поставлен весь этот
138 Дэвид Копперфилд, кстати, делал это и один – если не считать десятков ассистентов, готовящих трюк. В
их толпе он и укрывается, сменив в нужный момент одежду. Так что сообща работать всё равно легче. А
зачастую и полезно, и интересно.
спектакль, закончившийся столь успешно. Вот красивое, художественное и наглядное
описание метода «мозгового штурма».
Впервые этот метод применил А. Осборн в США в 1939-м году. Известно: любое новое
предложение может быть подвергнуто немедленной и жёсткой критике. Поэтому не очень
приятно эти предложения высказывать. Процесс генерации идей, их обсуждения и «доводки»
тормозится, боязнь критики мешает творческому мышлению. Креативная идея Осборна:
создать для получения новых идей тепличные условия, вообще запретив критику.
Так что исходной идеей мозгового штурма было отделение процедуры генерирования
идей замкнутой группой специалистов от процесса анализа и оценки высказанных идей.
Известно: некоторые люди по природе являются «генераторами» идей, но не очень хорошо
их анализируют. А другие, напротив, предпочитают детальный критический разбор.
Если же говорить строже, то «мозговой штурм» определяется как процесс, в котором
отдельные личности стараются стимулировать и вдохновить друг друга для выработки
новых идей и подходов. Как пишет У. Петерсон, цель и смысл этого процесса определяются
возможностью «достучаться до подсознания каждого участника группы и создать условия
для совместного активного участия отдельных ментальностей».
Таким образом, мозговой штурм – своеобразное и сознательное превращение каждого
отдельного и независимого интеллекта в часть общего творчески активного и творящего
механизма. Это создание сплава интеллектов, каждый из которых, взаимодействуя с
другими, получает дополнительное пространство для развития своего потенциала. Это
некоторое «совместное мышление», усиливающее силу творческого воображения.
Как правило, штурм проходит недолго, около 40 минут – ведь напряжение мысли очень
велико и в какой-то момент приходит утомление. Каждому «генератору» на выступление
отводится не более двух минут. Благодаря механизму ассоциаций идеи порождают одна
другую. В конце концов возникает некая «цепная реакция», лавина идей. В этот момент
интенсивность их генерации достигает пика, хотя затем и спадает. Отдельные разумы
объединяются на основе общего, по выражению Юнга, «коллективного бессознательного».
Важное место занимает ведущий штурма. Он должен не просто «следить за порядком»
выступлений, но своими вопросами и замечаниями непрерывно расшевеливать участников,
направлять их мысль, выделяя узловые моменты в рассуждениях и фиксируя на них
внимание.
Итоги штурма подводит отдельная группа аналитиков и экспертов. Практика
показывает, что в результате «усушки и утруски», а также отсева и отбора остаётся около
10–15 % нетривиальных и интересных идей. Очень и очень немало – если учесть, что бывает
достаточно одной идеи, чтобы изменить ход событий, процессов или действий139.
Итак, сформулируем основные правила проведения мозгового штурма (МШ):
1) цель МШ – генерация большого числа идей, прямо или косвенно связанных с
исходно поставленной проблемой; использование МШ может стать первым шагом в поиске
творческого решения;
2) все участники МШ абсолютно равноправны, каждый участник имеет возможность
свободно выразить любую свою мысль (при этом совершенно не обязательно, чтобы она
имела непосредственное отношение к проблеме – управлять ассоциациями мы не умеем, так
что решение может прийти благодаря чьей-то фразе о невыносимой духоте, или о новом
фильме); все мысли, в том числе самые бредовые или шутливые, высказываются без
опасения подвергнуться критике;
3) равное уважение к участникам означает и равное уважение ко всем высказываемым
139 Во время Второй мировой войны флоты антигитлеровской коалиции страдали не только от германской
и японской авиации, но и от торпед вражеских – лучших в ту пору – подводных лодок. Во время мозгового
штурма кто-то предложил выстроить вдоль борта матросов, свободных от вахты, и по команде дуть на торпеду.
Так и сделали: торпеды сбивала с курса мощная струя из водоотливных насосов, способных откачать всё, что
вливается в отсек из довольно крупной пробоины.
суждениям, предложениям, гипотезам, какими бы абсурдными или не имеющими отношения
к делу они ни представлялись на первый взгляд;
4) нужно уметь «прицеплять», увязывать свои мысли и идеи с другими
высказываемыми идеями; каждый может использовать чужую мысль, чтобы её
видоизменять, комбинировать с другими и т. д.; так образуется «клубок» взаимосвязанных и
взаимовлияющих мыслей (в английской терминологии это называют piggybacking – «кучамала»);
5) чем больше высказывается идей, тем лучше, остаётся только ждать, когда их
количество перейдёт в новое качество – вот тогда возникнет мысль, венчающая весь процесс
решения, мысль-синтез, мысль-решение, мысль – как меч Александра, разрубающего узел,
затянутый Гордием.
Известны и правила подведения итогов мозгового штурма. Чтобы он прошёл
плодотворно, нужно:
✓ все высказанные идеи зафиксировать, сохранив их в оригинальной форме;
✓ для оценки, проверки и анализа идей создать – также после обсуждения –
определённый шаблон, набор критериев, которые формулируются в связи с исходно
поставленной задачей;
✓ имея в руках «решето» из критериев проверки, пропустить через него все
высказанные предложения и идеи, отобрать все пригодные – удовлетворяющие критериям;
✓ обсудить пути и методы развития и/или реализации каждой из этих идей.
Как и в иных интеллектуальных действиях, при генерации идей во время мозгового
штурма использование аналогий весьма плодотворно. Автор синектики У. Гордон для этого
случая предложил использовать те же четыре вида аналогий, о которых мы уже говорили
выше. Упомянем их кратко ещё раз. Это:
✓ прямая, когда рассматриваемый объект сравнивается с более или менее сходным
объектом из другой отрасли техники или из живой природы;
✓ личная, когда используется известное в психологии понятие эмпатии, мысленного
соединения с рассматриваемым объектом; решающий задачу человек старается
почувствовать себя не просто внутри объекта, но стать им самим: я – катящееся колесо, я –
башня, на которую падает дождь, и т. д.;
✓ символическая (обобщённая абстрактная): в этом случае предмет ассоциируется с
самым главным признаком, остальные «мелкие нюансы и детали» отбрасываются;
✓ фантастическая – в решение задачи вводятся сказочные элементы: маленькие
человечки, «живая» вода, демоны (как у Максвелла); потом эти элементы будут
переименованы и займут своё место в реальности.
Чтобы проиллюстрировать работу метода, приведём одну из наших любимых
изобретательских задач.
В 218-м году до нашей эры во время похода в Италию карфагенский полководец
Ганнибал подошёл к реке Родан – такой широкой, что мост через неё не построишь. Более
сорока слонов, что придавали армии мощь, никак не хотели переправляться на италийский
берег. Однако решение было найдено.
Если представить себя слоном и пофантазировать за него! «Почему же я не желаю
пойти в воду»? А ещё лучше – представить себя рекой и поразмышлять: «Почему слон
боится в меня ступить»! И тогда для вас станет очевидным: либо слон не подходит по своему
эмоциональному состоянию для реки, либо река – для слона.
Выходит, надо привести слона в такое состояние, чтобы пересилить его водобоязнь!
Ганнибал собрал стадо на берегу. Самого злого слона удалось привести в ярость.
Испуганные животные сами140 устремились в реку прочь от своего взбешённого сородича.
140 ТРИЗ рекомендует добиваться, чтобы нужное действие исполнялось само, без прямого воздействия со
стороны. В данном случае эта рекомендация выполнена точно.
Когда слоны теряли брод, течение всё равно выносило их на противоположный берег.
Что же может быть иным состоянием для воды? Твердь. И коли мост построить нельзя,
то надо создать у слона иллюзию, что вода – вовсе не вода.
По другому свидетельству слонов перевезли на плотах. У берега пунийцы прикрепили
канатами к деревьям покрытый дёрном широкий помост, куда погонщики загоняли своих
слонов, и уже оттуда животных заставляли переходить на плоты, также покрытые дёрном и
по внешнему виду не отличающиеся от суши. Окружённые со всех сторон водой, слоны
волновались, а некоторые даже падали в реку, однако и они сумели выбраться на берег. Всё
закончилось благополучно. Ганнибал, поставив себя на место животных, создал у них вместо
испуга иллюзию покоя.
Эмпатия, театр эмоций – традиционные методы креативного мышления. Кстати,
многие полководцы были неплохими психологами. Александр Македонский, скажем,
одержал победу над персами при Тире, когда создал у них иллюзию численного перевеса. Он
приказал ночью развести множество костров вокруг лагеря врага, чем и напугал его…
Вернёмся к Ганнибалу. Его приключения на том не окончились, и уже через несколько
суток полководцу ещё раз пришлось блеснуть изобретательским талантом.
Войско пунийцев совершало беспримерный по дерзости переход через заснеженные
вершины. Воины спускались по узкой и крутой скользкой тропе. Одно неосторожное
движение – и человек летел в пропасть. Воины подошли к огромной скале. Обойти её было
невозможно. Тем не менее на следующий день армия Ганнибала, включая слонов,
спустилась с гор в долины Италии.
Представьте себя скалой, войдите в её положение и останьтесь в её положении! Что
способно вас напугать настолько, чтобы вы сами убрались с пути грозного войска?
Ганнибал буквально прошёл сквозь гору: он изменил её состояние, приказав разлить по
скале вино и уксус, а потом нагревать породу. Чередуя операции нагрева и охлаждения,
воины раздробили скалу в щебень и построили первую в мире высокогорную дорогу. Кстати,
Ганнибалова дорога просуществовала несколько сотен лет.
Красивое решение. Физики вообще говорят: истинное должно быть красивым.
Использование различного вида аналогий способствует нахождению новых
понятийных характеристик, необходимых для нестандартного хода мысли.
Утверждают (и не без оснований): метод мозгового штурма могут использовать как
члены семьи для достижения семейной гармонии, так и члены правительств разных стран
для установления мира на земле. Однако для этого нужно договориться о том, чтобы все
придерживались определённых, заранее и твёрдо установленных правил141.
Очень важно, чтобы участники были согласны не только выслушать, но и поучиться
друг у друга. Чем дружественней и теплее атмосфера, чем больше взаимного доверия и
согласия, тем интересней сам процесс мозгового штурма и тем значительней получаемый
результат.
Мозговой штурм, особенно если он проходит в дружеской, доверительной и
уважительной обстановке, весьма эффективный творческий метод для выработки новых
идей. По сути дела, эта одна из разновидностей игр для ума, позволяющая дружными
усилиями выбраться из накатанной колеи и сформулировать нечто поистине новое. Или
найти ответ на трудный вопрос.
Вот ещё один поучительный пример из антологии финальных игр «Что? Где? Когда?»
(1982-й год). Ведущий (В. Ворошилов) сообщает знатокам, что их соперником будет
Прасковья Игнатьевна Соломейчук из села Рожнов Ивано-Франковской области. И она будет
вести раунд вместе с Вильямом Шекспиром. На мониторах появляется телезрительница и
141 В семье этого, наверное, не так сложно добиться. А вот правительственные чиновники, да ещё из
разных стран, решение этих процедурных вопросов наверняка затянут очень надолго. Может быть, в этом-то и
причина мировых проблем?
произносит: «Здравствуйте, уважаемые знатоки… Я вам сейчас прочту несколько строк из
Шекспира:
Вы, низкорослый народец карликов,
Что на лугу при свете луны
Вычерчиваете эти волшебные круги
Ярче зелени лугов.
Вы, шаловливые бездельники,
Радостно пробуждающиеся вечером,
Когда на кухне гасит свет огонь,
Чтобы в полночь раскрывать круги».
После этого следует вопрос: «Уважаемые знатоки! Что это за волшебные круги, и кто
они, эти шаловливые бездельники, о которых пишет Шекспир. Подумайте минуту и
ответьте».
Знатоки плохо поняли вопрос и пытаются «разрыхлить его», уточняя каждое слово. Но
так как в их распоряжении всего одна минута, обсуждение состоит из коротких реплик
членов команды. Мозговой штурм начался:
– Ребята, это «Сон в летнюю ночь»!
– Карлики? А что там за кольца, там какая-то периодичность проглядывает. Вот что.
– Бездельники, но почему они рано пробуждаются? А что это за карлики?
– Что это за круги? Радостно пробуждающиеся…
– Слушайте, что же это за круги?
– А почему бездельники? Может и правда карлики какие?
– Это явление природы! Природы! Совершенно точно.
– А может – смена времён года?
– А круги, круги вместе – не круг.
– О чём идет речь… Лунная радуга?
– Лунная радуга?
– Нет, по-моему, это грибница. В которой гриб растёт.
Наконец ведущий требует ответа – Минута прошла. Отвечает С. Пестов, первым
высказавший версию: «Шаловливые бездельники – это, скорее всего, грибы. Они растут из
грибницы, с каждым годом всё больше распространяясь по радиусу, и растут по окружности
определённой. Вот низкорослый народец, и как раз он эти круги и делает. Грибы ищешь –
точно в прятки. Когда находишь один гриб – надо искать по кругу».
Невозмутимый В. Ворошилов констатирует: «Да, в последний момент вы всё-таки
решили, что это грибы. Ну, что же, читаем ответ телезрителя. «Издавна все народы называли
эти круги волшебными или даже «ведьмиными». Эти шаловливые бездельники, как называет
их Шекспир, этот низкорослый народец-карлик – обычные грибы!»» Так успешно
заканчивается короткий мозговой штурм трудного вопроса.
Трудный вопрос
А кстати, какой вопрос считать трудным? Особенно если учесть все различия знаний и
вкусов игроков. Как привести их к единой мере?
В 1989-м мы с моим братом Нурахмедом задумались над формализацией оценки
вопросов. И довольно скоро появилось решение: самый надёжный критерий – статистика.
Зададим один и тот же вопрос сразу многим командам – и посмотрим, сколько из них
справятся.
Уже летом 1989-го В. Ворошилов провёл экспериментальные игры по новой системе.
Сразу десять команд одновременно записывали (чтобы не помогать и не мешать друг другу)
свои ответы. За каждый правильный ответ команда получала столько очков, сколько команд
ошиблось. Чем сложнее вопрос, тем больше ошибок – и тем больше получат те, кто
справился.
С сентября по декабрь 1989-го прошёл цикл телевизионных игр по новым правилам. К
сожалению, найти формат передачи, передающий весь накал соревнования, сразу не удалось,
и телегруппа вернулась к привычной игре: одна команда против авторов вопросов. Но уже
16-го декабря всё того же 1989-го состоялся первый тур первого официального турнира по
спортивной версии «Что? Где? Когда?», где команды состязаются друг с другом. Сейчас в
этих соревнованиях участвует по всему миру – по разным оценкам – 10–15 тысяч команд.
Конечно, правила с тех пор изрядно поменялись – но в основе всё та же идея игры на одних и
тех же вопросах.
А для телевидения мне с тех пор удалось разработать новые форматы передач, где
отвечать – и тем самым заодно участвовать в оценке вопросов – Могут не только игроки в
студии, но и миллионы телезрителей. Когда такая игра будет запущена – сколько людей
сможет оперативно сравнивать свои способности с опытом ветеранов интеллектуальных игр!
От штурма к планомерной осаде
Если есть с кем обсуждать варианты решения, вплоть до самых сумасшедших и
фантастических, не ожидая насмешек, а надеясь на совместное выделение всего
конструктивного, имеет смысл ввести проведение мозгового штурма в повседневную
практику творческой работы. Правда, правильнее будет именовать его теперь мозговой
осадой или конференцией идей – этот термин предложен ещё в середине 1970-х немецкими
креативщиками.
Мозговая осада предназначена для сбора идей и их первичной обработки. Хорошо
подготовить осаду крепости-проблемы – означает, прежде всего, способствовать
раскрепощению участников от оков привычек, а вовсе не предусматривать каждую мелочь.
✓ Необходимо правильно и заранее сформулировать задачу.
✓ Участники осады должны быть знакомы с проблемой в общих чертах, т. е. не
обязательно быть узкими специалистами.
✓ Состав «осаждающих» от раза к разу меняется. Неизменным остаётся наиболее
активное ядро предыдущих участников.
✓ Присутствие женщин стимулирует мужчин, женская логика дополняет мужскую
прагматичность (этому мы посвятим особый раздел).
✓ Желательно участие в «осаде» людей разного возраста. Дети высказываются
первыми, чтобы на них не давил авторитет старших участников142.
✓ Для участия в «осаде» приглашают за два – три дня до её начала.
✓ Кто замыслил «осаду», тот и становится верховодой, он же формулирует проблему.
Он выполняет роль арбитра – смотрит за соблюдением правил общения между участниками,
но не участвует в критике их друг другом. Заполняет возникшие паузы, прерывает
словоизвержения.
✓ Мозговая осада – коллективный труд. Не оговаривают, не записывают и не
запоминают, кто какие идеи высказал. Опорочить любую идею может всякий, в этом нет
никакой чести. Критика высказанных идей разрешена лишь экспертам – второму эшелону
осаждающих, которые анализируют «безумство» авангарда – генераторов.
Но даже и с самим собой можно проводить некий индивидуальный мозговой штурм.
Составляйте список вариантов решения, перечень побочных идей, возможные ответвления,
рассматривайте все пути в «пространстве проблемы». Подбирайте информацию сами в
Интернете, читайте произведения великих учёных, старайтесь посетить лекции, дискуссии и
142 Лучшие философы на свете – Мальчишки, у которых пробивается борода (Платон).
конференции, где можно услышать специалистов по интересующим вас вопросам.
Попробуйте установить мысленную связь между решаемой проблемой и высказываниями
великих людей – выбирайте из известных вам цитат наиболее подходящую по духу и теме и
поработайте с ней. Покрутите заложенную в ней мысль, поищите ассоциации со своей
задачей.
Вообще при такой атаке на проблему фиксируйте – письменно или на диктофоне – всё,
что Вам приходит в голову, превращайте Вашу проблему в подобные, но упрощённые,
вводите для помощи «маленьких человечков», ищите аналогии. В общем, не давайте
отдохнуть вашему «фонтану» идей!
Времени придётся потратить побольше, чем при мозговом штурме с участием
талантливых друзей. Но лучше ведь продвигаться, пусть даже медленнее, чем стоять на
месте. Так Вы тренируете свою способность расширять интеллектуальный кругозор, свой
умственный арсенал. Ну, а если Вы творец-одиночка, такой вариант творчества наиболее
эффективен – никто не помешает, не собьёт с мысли, не поведёт всех по неверному пути.
Бигуди № 63
В давние времена, когда Англия ещё владела своими индийскими колониями, один
английский чиновник занимался выплатой жалования индийским солдатам. Чиновника,
кстати, звали Уильям Гершель, и был он 2-й баронет Гершель, внук и сын (соответственно)
знаменитых астрономов и математиков Уильяма и Джона Гершелей. С 1857-го года он
работал в британской администрации в Бенгалии. В течение 15 лет Гершель выплачивал
жалование большому количеству индийских солдат. Все они были для него, европейца, на
одно лицо, имена их часто повторялись. Получив жалование, солдаты нередко приходили и
утверждали, что денег ещё не получали. Иногда посылали друзей или родственников, чтобы
те ещё раз получили их жалование. Чтобы избежать этого, Гершель вначале обратил
внимание на одну мистическую идею, распространённую как в Индии, так и в Китае. Он
сумел извлечь пользу из этой идеи, обнаружив важное явление, которое затем использовал в
своей практике работы с неграмотными солдатами. Внимание, вопрос: какую мистическую
идею использовал Гершель, и основы какой науки заложил этот представитель столь
прославленного рода? Как бы Вы сами стали решать такую проблему, с которой столкнулся
Гершель?82
18. Методы, методики и методички…
– Вот вам пример того, что можно сделать, если не
полениться, – сказал Иа. – Тебе понятно, Пух? Тебе понятно,
Пятачок? Во-первых, Смекалка, а во-вторых, Добросовестная
Работа. Ясно?
А.А. Милн. «Винни-Пух и все-все-все»
Выше немало сказано об общих принципах решения проблем. Всем и каждому уже
понятно: гибкость ума – качество исключительно важное и поддающееся развитию. Полезно
даже просто знать: сидеть в тупике (или как там при игре в шашки называется положение,
когда шашка заперта и не имеет хода?) неплодотворно. Не «вырисовывается» мелодия, не
«звучат» краски на картине, не сходится интеграл – переключись на другое дело! Конечно,
лозунг «не откладывай на завтра то, что можно отложить на послезавтра» здесь ни при чём.
Отложи на время в подсознание ту проблему, которая пока не поддаётся решению.
Прокручивая варианты, мозг что-нибудь толковое придумает. Только подкидывай ему
«пищу» время от времени.
Опять-таки о необходимости питания своего мышления уже сказано в связи с цитатами
из Кэрролла. Добавлю только: хорошо бы сам процесс получения информации сделать
насыщеннее. Почему бы не делать при чтении заметки «на полях» – а вдруг в голову придёт
что-то вроде Большой теоремы Ферма? У великого математика не хватило места, чтобы
изложить полностью возможное доказательство гипотезы, и над доказательством бились
учёные (и не только) всего мира в течение нескольких столетий. А вы держите сразу под
рукой блокнот, ручку и фиксируйте те мысли, которые покажутся дельными. Или, как уже
говорилось, носите с собой диктофон. Короче, не упустите мелькнувшую идею! Вот так
постепенно и накопится множество мыслей, предложений, идей – ваша личная
интеллектуальная сокровищница.
Теперь сформулируем набор конкретных методик, которые разрабатывались и
апробировались долгие годы. Я, конечно, использовал в работе по интеллектуальному
тренингу различные предложения и методы других специалистов, изобретателей,
психологов. Использовал, естественно, творчески – видоизменяя, перерабатывая, дополняя.
Предлагаемые ниже методы и методики – по сути, перегруппированные принципы
мышления и решения задач, так что каждый из этих методов – некоторая комбинация
принципов ориентирования и движения в «пространстве проблемы».
Например, методика «Разделяй и властвуй». Речь идёт о том, чтобы, применяя принцип
проникновения (осмотра и анализа условий задачи), выделить и зафиксировать – составляя,
скажем, список – основные характерные признаки задачи. Т. е. разделить большую проблему
на ряд маленьких. Конструируя большой и сложный агрегат, приходится ведь иметь дело с
каждым отдельным его узлом. Такая сравнительно небольшая часть всего комплекса имеет
самостоятельные функции, рабочие параметры. И следовательно, есть возможность
совершенствования этой выделенной части. А затем уже и всей установки в целом.
Для каждой «подзадачи» есть своё «подпространство». Вот в нём и нужно сначала
двигаться. Оно и поменьше, и попроще. И на вопросы типа «А нельзя ли убрать…?»,
«Можно ли обойтись без…?» ответить легче – ведь они относятся лишь к подзадаче,
некоторому частному случаю. Только нужно не забывать о цели – всё же мы решаем лишь
часть задачи и нужно, получая промежуточные ответы, сверять их с общими требованиями.
Конструируя новый тип электромясорубки, не обязательно создавать к ней малогабаритный
суперэлектродвигатель, делающий до 10 тысяч оборотов в минуту143. Характеристики
задачи в целом всё же ограничивают «подпространство подзадачи». Любая новая идея,
относящаяся к этому «подпространству», должна быть спроецирована на всё пространство и
там уточнена.
Известно полезное обобщение этой методики. Его можно назвать «Атомы и
молекулы». Речь идёт о том, чтобы продолжать дробить задачу на части до тех пор, пока
каждая из частей представляет собой осмысленную и самостоятельную подзадачу. Иногда
разумно разбивать задачу пополам, отдельно анализируя каждую из них таким же образом.
Т. е. каждая часть вновь разбивается на две подзадачи и т. д. При каждом разбиении следует
для вновь получающихся частей задачи вновь найти характерные особенности, признаки.
Изучая всё сужающиеся подпространства подзадач, в каждом из них находим такие признаки
и – комбинируем их разными способами. Составляем разные «молекулы» из «атомов» –
характерных признаков отдельных частей задачи. Новые сочетания признаков могут помочь
сформулировать новый подход, необычную гипотезу. А если повезёт, то и безумную идею.
Но вот что важно: в каждом из этих методов опасна всё та же ограниченность
мышления. Поэтому фиксируя все «дороги и тропинки» в пространствах задач и подзадач,
143 Правда, иногда и подобные фантазии дают полезный побочный эффект. Больше века назад шведский
инженер Густав Лаваль, проектируя новый сепаратор – отделитель сливок – для фермеров, решил отказаться от
самой сложной и дорогой по тому времени части конструкции – зубчатого редуктора, превращавшего
медлительное вращение ручки в стремительное вращение сосуда с молоком. Вместо этого он придумал
высокооборотную паровую турбину. Правда, паровой котёл с турбиной оказался немногим дешевле – да в
крестьянском хозяйстве и не удобнее – редуктора. Но на транспорте и в энергетике турбины Лаваля популярны
до сих пор.
нужно немедленно, как только какое-то предположение или предложение возникло,
отыскать и сформулировать для него альтернативный вариант. Заставить себя посмотреть на
проблему с другой стороны. Если удастся, то надо найти несколько дополнительных,
совершенно отличных от исходного, вариантов решения. Эти ответвления тоже нужно чётко
фиксировать и анализировать их «подпространства задач». Долго? Но что же делать – ведь
вы ищете НОВОЕ, делаете ОТКРЫТИЕ! Такой метод общего характера можно назвать
«Новое старое пальто» – ведь похоже на перелицовку?
Но как конкретно «перелицевать» проблему, посмотреть на неё по-новому? Как, в
поиске альтернативных подходов для подзадач, находить в их подпространствах иные
системы отсчёта? Как разворачивать задачу, чтобы увидеть в ином ракурсе? Опыт – и мой в
том числе – показывает: среди удачных методик – известная схема постановки вопросов
«Скампер». Это аббревиатура из нескольких английских слов. SCAMPER – это: Substitute?
(подставить, заменить), Combine? (скомбинировать, соединить в другой комбинации), Adapt?
(подстроиться, адаптировать), Modify? (модифицировать, видоизменить), Put to others?
(предложить другое применение), Eliminаte or minify? (убрать, устранить или свести
действие до минимума), Reverse or rearrange? (перевернуть, обратить, изменить порядок).
План действий таков: после того, как изучены «пространство» и «подпространства
проблемы», в каждом из них, по отношению к каждому варианту движения в них, по
отношению к каждому характерному признаку или параметру задаются все вопросы из этого
набора. Ответы фиксируются. Из них отбираются те, что содержат перспективные и
нестандартные идеи. Далее к каждой новой идее можно вновь применить методику
разделения на части, нахождение альтернативных вариантов и т. д.
Можно даже «механизировать» метод генерации новых идей. Когда-то Раймонд
Луллий изобрёл машину, позволяющую путём комбинирования понятий, записанных на
вращающихся независимо друг от друга дисках, получать «новое знание». Примерно так
можем действовать и мы. Кратко описанный выше морфологический «ящик» – та же машина
получения знаний. Это удобный способ анализа пространства параметров задачи. Записывая
в виде матрицы известные нам связи и соотношения между параметрами, можно выявить
ранее незаметные зависимости. Затем, для отбора доминирующих характеристик, можно
убирать их поочерёдно и анализировать, как реагирует задача, что меняется, какие
параметры становятся главными и т. д.
Есть известный анекдот: некий учёный утверждает, что таракан слышит лапками.
Доказательство: поставим таракана на стол и постучим о стол – таракан бежит. Теперь
оторвём ему две лапки и вновь постучим – бежит, но уже хуже! Оторвём все лапки – на стук
не реагирует, следовательно, слышать перестал. В этой притче верно то, что решаемую
проблему надо изучать последовательными шагами. Конечно, выводы из меняющейся
ситуации следует делать поумнее.
Нужно составить список допустимых и недопустимых вариаций для всех параметров
задачи и, комбинируя столбцы и строки матрицы параметров, искать их новые сочетания.
Это позволит увидеть новые возможности в пространстве проблемы.
Наверняка Вам в детстве доверяли раскладывать разрозненные пуговицы. Множество
всевозможных пуговиц, больших и маленьких, разноцветных, гладких и шершавых,
пластмассовых и металлических – какие возможности для игры! Но их надо разложить по
кучкам. И Вы находите разные варианты классификации – по цвету, по размеру, по
принадлежности (от пальто, от платья…). Вот так же следует поступать и с мыслями по
поводу задачи, с вариантами решения, с новыми идеями: их нужно «раскладывать по
кучкам», группировать, находить родственные, близкие, взаимозависимые. При этом тоже
разумно пользоваться графическими методами: рисовать схемы, матрицы, деревья.
Но вот ещё какое «дерево» неплохо бы посадить в своем пространстве проблемы. Его
иногда называют «Деревом головной боли», потому что оно представляет собой схему или
диаграмму всевозможных препятствий, затруднений и опасностей. Расположите все эти
источники лишней головной боли по степени возрастания сложности, нарисуйте ствол
дерева, на нём запишите условие задачи, а затем пририсовывайте к стволу ветви –
горизонтальные линии, на которых фиксируются препятствия для решения (на нижних
ветвях более лёгкие; постепенно, с движением к вершине дерева, опасность возрастает – тут
самые сложные проблемы, которые следует преодолеть на пути к ответу). Самое главное
потом – надо стать «лесорубом» и начинать придумывать способ, как избавиться от ветвей,
т. е. как их рубить – начиная, естественно, снизу.
Уже говорилось, какую важную роль играют аналогии и ассоциации. Так вот, искать их
можно крайне примитивным образом. М. Левин называет этот способ «Методом случайного
поиска». Предлагается всего лишь открывать совершенно случайным образом какой-нибудь
словарь (кстати, выбор подходящего к задаче словаря или просто книги сам по себе может
быть не случаен) и, взяв первое попавшееся слово, искать связь между ним и решаемой
задачей. Искать возможные ассоциации. Пытаться увидеть аналогии между смыслом этого
слова и сутью задачи. Потом вновь открыть книгу и ткнуть пальцем в другое слово. И опять
устанавливать связь между своей задачей и понятиями, приходящими откуда-то «со
стороны». Так вполне можно набрести на нетривиальную мысль, аналогию, новый класс
решений задачи.
Кстати, в поисках нового может помочь даже такой простой «грамматический» приём:
меняйте местами слова, превращая существительные в прилагательные или наоборот – там,
где это возможно. Например, вместо «окрашенная поверхность» получится «поверхностное
окрашивание» – и от некоторого специфического предмета вы перешли к процессу!
Такой способ выхода за рамки функциональных ограничений близок предлагаемому в
синектике методу фокальных плоскостей. Суть в том, чтобы какие-то признаки отобранных
(неважно каким образом) объектов или процессов ассоциировать с предметом или
процессом, находящимся в центре внимания – в фокусе. Приписывая ему признаки других
объектов (т. е. рассматривая его в различных плоскостях), можно получить новые сочетания
признаков или характеристик. Скажем, вы хотите усовершенствовать обычный кирпич.
Давайте используем признаки другого объекта, например, мочалки. Получаем:
«разноцветный кирпич», «мягкий кирпич», «впитывающий жидкость кирпич»144. А если
взять признаки дерева? Получится «гибкий кирпич», «срастающийся после излома кирпич»,
«кирпич, имеющий живую внутреннюю структуру». Отсюда уже недалеко до единой,
объединяемой некими внутренними связями, системы кирпичей, обладающей уникальными
свойствами гибкости и прочности. Обратите внимание: это просто конструирование всё
новых морфологических матриц. Ключевым здесь является удачный выбор вспомогательных
слов, порождающих цепи ассоциаций.
Очень удобная методика поиска новых решений – использование разных «картинок».
Назовем эту методику «Весёлые картинки»145. Это методика визуализации мышления: мы
выделяем в задаче отдельные характерные признаки, каждый признак изображаем на
отдельной карточке с помощью рисунка или графического символа. Удобно (и красиво)
использовать разные цвета, на обороте указываем, какой признак «закодирован» рисунком.
Теперь разложим все карточки картинками вверх, перемешаем или перегруппируем их.
Отделим какую-то группу и поглядим на сочетания карточек – Может быть, мы уже пришли
к новой идее? Если заходим в тупик, добавим ещё картинок или создадим их новый набор.
Это всё тот же метод шаблонов, который используется в архитектуре. Его предложили
архитекторы Александер, Ишикава и Силверстайн, когда исследовали возможности создания
нового подхода к дизайну зданий.
А можно использовать и вероятностный подход к задаче. Например, так. Выберите
144 Кирпич, пропитанный бензином – очень удобные дрова для походного костра в безлесных местах.
Главное – хорошенько охладить прогоревший кирпич, прежде чем заново его пропитывать.
145 До сих пор помню этот замечательный детский журнал и его героев.
несколько характерных признаков вашей задачи, пронумеруйте их, а потом – используйте
какой-то метод выбора случайных чисел (например, бросайте кости), чтобы случайным
образом выбрать первый признак. Потом – второй. Вот уже можно рассмотреть разные
сочетания этих двух характеристик. Затем добавим к ним третий, выбранный таким же
способом, и вновь проанализируем сочетания признаков. Такому методу можно дать
название «Счастливый случай».
А нельзя ли использовать при штурме проблемы «помощь Клуба»? Точнее, действовать
по поговорке «Одна голова хорошо, а две – лучше». Для этого способа расширения
пространства проблемы есть название «Страна Советов». Всё очень просто и давно каждому
известно: не знаешь сам – спроси у товарища. Или у незнакомца. Правда, решая
математическую задачу, не стоит ожидать дельной мысли от соседа – филолога. Хотя, если
проблема носит изобретательский характер, или имеет отношение к таким современным
вещам, как бизнес, реклама, управление, то полезно обсудить её с разными людьми,
поговорить с представителем иной сферы деятельности.
Вообще, если вы начальник (неважно чего), руководитель или даже президент, имеет
смысл окружить себя людьми, легко воспринимающими всё новое, способными быстро
перерабатывать информацию и предлагать нестандартные идеи. Правда, окружить себя мало
– нужно уметь прислушиваться к их словам! «Хочешь счастливо править – найди умного
визиря» – эту истину знал почти каждый султан. Но слушать полезные советы умел и хотел
уже далеко не каждый. А при этом ещё и самому думать, чтобы извлекать из чужих слов
неожиданно важную мысль – это уже искусство. Философ Мераб Мамардашвили считал
очень важным уметь «… выразить лишь нечто мелькнувшее на какое-то мгновение, несомое
атмосферной волной беседы в диалоге». Ещё великий Платон говорил: только в беседе
может «что-то быть», что-то возникнуть.
В общем, просеивайте через сито собственного мышления намёки, сравнения,
аналогии, подаренные вам другими. И, глядите – среди отработанной породы блеснул
самородок! Ведь это и есть талант – способность оптимально воспринимать и отбирать
информацию. Из любого источника.
Например, уже упомянутый 2-й баронет Гершель сначала просто узнал, что индусы
искренне считают, что отпечаток своего пальца – гораздо более обязывает и сдерживает их,
чем подпись. Тем более, что практически все бенгальские солдаты были неграмотны. Раз так,
Гершель стал заставлять солдат ставить отпечатки двух пальцев как на списки с именами (на
договора, заключаемые со служащими), так и на квитанции. Жульничеству был положен
конец. Однако среди использованной породы, как я уже сказал выше, можно найти
«жемчужное зерно». Гершель обнаружил: отпечатки пальцев одного человека никогда не
идентичны отпечаткам другого. Затем он научился различать их по рисунку и узнавал
многих людей по «картинке их отпечатков пальцев», названия которых узнал из учебника
анатомии (это папиллярные линии). Не останавливаясь в изучении своего открытия, в
последующие годы Гершель пришёл к выводу: рисунок линий на пальцах не меняется,
сколько бы лет ни прошло. Вывод очевиден: человек может постареть, заболеть, сильно
измениться внешне, но уникальный рисунок папиллярных линий остается неизменным. Даже
после смерти. Пока остановимся на этом, ибо история дактилоскопии обширна и, на самом
деле, должна бы начинаться ещё с Китая VII века н. э.
Процесс творчества иногда сравнивают с настройкой собственного мозга на одну
частоту. Предполагается: как только настройка произошла, мышление, находясь на «волне
открытий», генерирует (или ловит?) целый поток неожиданно новых мыслей. Выше мы
привели различные способы настройки своего мышления на креативную волну. И
обсуждение (с самим собой, друзьями, специалистами, знакомыми), и изображение связей
между явлениями и вещами в виде рисунков, схем, любых картинок, и запись как вариантов
решения, так и всевозможных помех – всё это способы задержаться на нужной «частоте»,
подольше поработать в разных краях «пространства проблемы». А ещё меняйте свои
привычки и маршруты движения – я имею в виду движения в обычном 3-мерном
пространстве! Создайте вокруг себя необычную обстановку, новый мир, странное и
непривычное окружение – и ваши мысли тоже изменят свой маршрут, станут неожиданно
свободными.
Итак, перед Вами множество советов, предложений, рецептов. Вы действительно
хотите сделать что-то новое? Тогда проанализируйте всё, что Вам предлагается, отберите то,
что более всего подходит для решения Вашей задачи – и используйте.
Будьте терпеливы и настойчивы, меняйте приёмы мышления, когда чувствуете, что,
старый метод себя исчерпал. Никогда не старайтесь работать только с давно известными и
привычными методиками – их эффективность могла определяться лишь тем, что вы были в
похожих ситуациях и решали близкие по духу проблемы. Поэтому для творческих задач
подыскивайте те приёмы мышления, которые для Вас новы, неожиданны, непривычны.
Удачи! Не забывайте лишь слова великого математика Георга Кантора,
основоположника теории множеств. Он, описывая взаимоотношения своей науки с
реальностью, говорил: «Голова может уходить глубоко в облака – но ноги должны
оставаться на земле».
Бигуди № 64
А пока что отправимся далеко в пространстве и во времени. Мы – внутри реальности
рассказа Роберта Хайнлайна. Ситуация: на частично колонизированной Луне в результате
аварии потерялась слепая девочка. Она вундеркинд-музыкант, с абсолютным слухом. На
Луну прилетела на гастроли (!) – с концертами в лунных поселениях, в освоенной части
Луны. С ней установлена радиосвязь, но пеленгатор не в состоянии определить место, откуда
ведётся передача. Поскольку воздуха в скафандре хватит лишь на несколько часов, найти
девочку нужно быстро. Как? Площадь той части лунной поверхности, где может находиться
девочка, разделяют на 82 квадрата (почему на 82?) и каждый из них облучают… Каким
лучом? Здесь важна Ваша идея. Какой принцип обнаружения девочки?83
19. Цветочки и ягодки или… «Сделайте себе красиво»
– У вас теперь необычайно острый ум, друг мой, – сказал
Гудвин, – только научитесь им пользоваться!
А. Волков. «Волшебник Изумрудного города»
Грустный факт: утверждают, что уже к семи годам только 10 % населения имеют
признаки высокой креативности, а при достижении зрелого возраста их доля снижается до
2 %. Все дети гениальны – а куда же деваются способности 98 человек из каждых ста, когда
дети вырастают?
«До восьмидесятых годов нейрофизиологов учили, что после того, как мы достигаем
возраста около шестнадцати лет, наступает зрелость мозга и рост его полностью
прекращается. Если разрушатся волокна, связывающие какие-то нейроны, эти нейроны
навсегда останутся разъединенными. Если потерять нейрон, он никогда не восстановится.
Теперь мы знаем, что это не так. Наш мозг очень пластичен, особенно в молодости, и
сохраняет свою пластичность на всю жизнь. Связи между нейронами постоянно возникают и
разрушаются в ответ на перемены в окружающей среде.146
Мышцы, которыми мы не пользуемся, постепенно усыхают, но наш мозг реагирует на
неиспользуемые части тела иначе. Если у человека ампутирована рука, небольшой участок
его мозга перестает получать какие-либо сигналы от нервных окончаний, находившихся в
146 У певчих птиц участок мозга, задействованный при пении, разрастается на время сезона размножения, а
затем снова уменьшается в размерах. При этом не только образуются новые связи между нейронами, но и
новые нейроны возникают, а затем опять исчезают. – Примеч. авт.
этой руке. Но нейроны этого участка не погибают. Они начинают использоваться для других
целей. К этому участку непосредственно примыкает другой, который получает сигналы от
нервных окончаний лица.
Наш мозг постоянно изучает окружающий мир. Ему постоянно приходится
распознавать встречающиеся предметы, чтобы понять, стоит ли приближаться к ним или
избегать их. Ему нужно учиться срывать плоды и не хватать рукой осу, чтобы она нас не
ужалила. Более того, это обучение происходит без помощи учителя. Рядом с нами нет
никого, кто мог бы всегда подсказать нам, правильно или нет то, что мы делаем». (Крис
Фрит. «Мозг и душа»).
Почему так мало людей, способных производить новый интеллектуальный продукт?
Неважно, о какой области творчества идёт речь – ведь жизнь непрестанно требует принятия
решений, совершения поступков, выбора варианта действий. Может быть, потому что мало
кто задумывается о необходимости выхода из наезженной колеи мышления. Решая только
стандартные, однотипные задачи, мозг чувствует себя вполне комфортно (как тот самый
Разжиревший Ум, о котором писал Кэрролл) – и так идут дни за днями.
Между тем, по меткому выражению Бернарда Шоу: «Способность мыслить – одна из
страстей, и она может давать более длительное наслаждение, чем любая другая страсть». Как
я отметил выше, о наслаждении творчеством ещё до Шоу говорил Чехов.
Мир не стоит на месте: то, что не сделали Вы, отложив размышления на завтра (как
героиня «Унесённых ветром»), окажется сделанным не Вами. Вы теряете шанс не только на
«цветочки» – получить удовольствие от размышления над новыми проблемами, ощутить
счастье, когда удается достичь нового результата, наградить себя орденом «Ай да я!» за то,
что смогли сделать нечто недоступное другим. Но Вы теряете и возможность собрать
«ягодки». Ведь предложенная Вами новая идея (механизм, книга, музыкальная пьеса,
технологический процесс, вид или способ рекламы, математическая теорема и т. д. и т. п.)
может принести не только субъективную радость свершения. Но и вполне объективные блага
разного характера (каждый может себе представить желаемое по своему вкусу).
Давайте ещё раз совершим экскурс в историю. И вновь во времена великих людей и
деяний – в XIX век, в эпоху Наполеона.
За перипетиями нидерландской кампании императора Франции внимательно следила
вся Европа. Все державы устали от войны не меньше Франции. Если бы Наполеону удалось
продержаться до зимы (тогда в снегу воевали разве что русские), его противники скорее
всего согласились бы на достаточно затяжное перемирие. Упрочив за зиму собственную
экономику, наладив традиционно сильную сеть дипломатических интриг, Франция могла к
весне добиться распада враждебной коалиции.
Больше всего от такого сценария пострадала бы, конечно, Британия. Правительство,
долгие годы бросавшее в пламя войны и собственные ресурсы, и всех союзников, в
одночасье оказалось бы банкротом. И не только политическим: несметные миллионы
золотых соверенов, вложенные в истребление единственного реального конкурента,
оказались бы выброшены безвозвратно. Следовательно, государственные долговые
обязательства скорее всего были бы в лучшем случае заморожены на долгие годы, а то и
вовсе обесценены.
Естественно, Лондонская биржа ждала известий из Нидерландов, как манны небесной.
Малейший слух оборачивался тысячами распоряжений о покупке или продаже. Увы, кроме
слухов, не было ничего: мир ещё не опутала телеграфная паутина.
Один из крупнейших лондонских финансистов – глава британской ветки банкирского
дома Ротшильдов – озаботился получением надёжной информации заблаговременно. По всей
Европе стояли его кареты с лучшими лошадьми, каких только можно было тогда купить за
деньги. Благо требования к упряжным и верховым лошадям разные, так что при всех
несметных потребностях вооружённых сил кое-что оставалось и на долю частных
владельцев. А уж среди них не было равного Ротшильду по готовности вкладывать деньги в
действительно необходимое.
Конная эстафета в считанные часы домчала пакет с известиями о разгроме французов
до берега Ла Манша. Там уже ждали корабли, также купленные Ротшильдом. От меловых
скал Дувра новые кони понеслись в Лондон. И Ротшильд получил заветные известия прямо
посреди главного торгового зала биржи. Было ясно: ещё через несколько часов весть о
британско-прусской победе дойдёт до всех. И тогда государственные обязательства, доселе
обесцененные политической неопределённостью, вновь станут надёжными – следовательно,
дорогими. Скупив их немедленно, можно было через пару дней изрядно обогатиться.
Именно так поступил бы в ту пору любой биржевой игрок, получивший эксклюзивную
информацию. Но не Ротшильд. За ним следили тысячи глаз. Вслед за ним скупать бумаги
немедленно бросилась бы вся биржа. И ему удалось бы обогатиться лишь на той
незначительной доле обязательств, которые он добыл бы первым приказом на покупку.
Ротшильд представлял себе ход мыслей коллег по бирже куда отчётливее, нежели они –
его. Разница в уровне рефлексии – Мышления о самих мыслях – позволила ему принять
творческое решение, по тем временам совершенно неожиданное.
Принесённую депешу он прочёл в центре зала. И, лихорадочно сунув её в карман,
бросился продавать все правительственные ценные бумаги, которые у него – как и у любого
биржевика – к тому времени были.
Надёжность и оперативность ротшильдовой информационной службы к тому времени
была общеизвестна и многократно подтверждена. Если великий банкир, повинуясь
полученным сведениям, продаёт британские обязательства – значит, Британия проиграла.
Избавляться от облигаций побеждённого бросились все. За считанные минуты бумаги стали
ненамного ценнее той самой бумаги, на которой были напечатаны.
И тогда, повинуясь незаметному знаку банкира, десятки его тайных агентов занялись
скупкой. Вскоре Ротшильд покинул биржевой зал обладателем чуть ли не всех обязательств
британского правительства. А ещё через несколько часов биржевики рвали на себе волосы,
узнав, от каких сокровищ только что избавлялись за бесценок.
Впрочем, это был не последний творческий манёвр легендарного финансиста. Чегочего, а денег ему хватало и до Ватерлоо. А вот чего не хватало предельно остро – так это
общественного признания. Еврей со множеством родственников на континенте не
пользовался особым уважением замкнутой британской дворянской касты. Даже несмотря на
то, что его род ещё в Средние века обзавёлся титулом баронов Священной Римской империи
германской нации. Сама эта империя, только что исчезнувшая под натиском всё того же
Наполеона, уже не освящала своим авторитетом красный щит (rot schield!) на баронском
гербе.
Ротшильд ещё раз использовал изобретательский приём инверсии. С биржи он
отправился в министерство финансов. И попросил оплатить ему государственные
обязательства не по номиналу, а всего лишь по той бросовой цене, по которой он их скупил.
Да и то лишь потому, что правительство Британии не вправе что бы то ни было принимать
вовсе безвозмездно.
Но разницу между номиналом и ценой скупки Ротшильд всё же получил. Разумеется,
не деньгами. А новым – уже чисто британским – титулом, доступом в высшие круги
общества и власти, возможностью в дальнейшем заключать контракты с правительством в
приоритетном порядке… Да мало ли выгод может получить человек, столь творчески
распорядившийся зависимостью государства от себя!
Банк Ротшильдов процветает до сих пор. Достаточно напомнить, что он входит в
пятёрку финансовых организаций, ежеутренне определяющих цену золота на лондонском –
и, следовательно, на мировом – рынке. Основы могущества этого банка заложены ещё в
глубоком средневековье тысячами сделок с мелкими германскими князьками. Но
непревзойдённым его сделал именно тот из Ротшильдов, который оперативной информацией
и собственными творческими ходами подчинил себе на несколько часов усилия двух
могущественнейших империй.
Надеюсь, этот исторический пример – один из великого множества – вполне
убедительно показывает: креативный способ действий оправдывает себя всегда и везде. В
любой сфере человеческой деятельности, в различных странах, в разные времена.
Помните, мы говорили о прогулках по ранее неизвестным местам, когда непривычные
пейзажи, невиданные улицы, лица помогают увидеть привычный мир с другой стороны,
стимулируют мысль? Добавим лишь: не кажется ли вам, что прогулки внутри самого себя
тоже стоило бы сделать интересными? Создадим в своём мышлении новый «интерьер»,
переключим его на нечто небывалое. Как? Очень просто.
Наше мышление следует за нашей жизнью, поведением. Давайте изменим свой
повседневный образ жизни, внесём новизну в устоявшийся распорядок. Приготовим на
завтрак не стандартную яичницу, а – закажем пиццу из соседнего кафе (сегодня это
запросто). Пойдём на работу другой дорогой и увидим много интересного: здесь что-то
строят, здесь открылся магазинчик, а тут вообще разбит чудный цветник! Переоборудуем
своё рабочее место – станки, конечно, переставлять не будем, но что-то перекрасить
несложно. Можно переставить мебель, купить себе новую чашку (или сервиз). Можно даже
начать делать зарядку по утрам или бегать! Можно и нужно делать всё, способное изменить
вашу жизнь, сделать её ярче, убрать из неё набор стандартов и штампов. Это немедленно
скажется и на вашем мышлении.
Согласно А. Н. Луку, «мозг облекает любую мысль в ту или иную конкретную кодовую
форму, причем разные люди обладают неодинаковой способностью пользоваться зрительнопространственным кодом, словесным, акустически-образным, буквенным, цифровыми т. д.
Способность манипулировать с данным типом символов можно совершенствовать, но не
беспредельно. Врожденные особенности мозга и условия развития в первые годы жизни
предопределяют преимущественную склонность к использованию тех или иных кодов
информации. Задача развития творческих способностей не в том, чтобы у человека,
склонного
к
зрительно-пространственному
мышлению,
вырабатывать
навыки
манипулирования математическими символами. Нужно помочь человеку «найти себя», то
есть понять, какие символы, какой код информации для него доступен и приемлем. Тогда
мышление будет максимально продуктивным и доставит ему высшее удовлетворение.
Способ кодирования информации должен гармонически соответствовать содержанию и
структуре отображаемых явлений. Дифференциальные уравнения – наиболее адекватный
метод описания движений планет. Тензорное исчисление хорошо описывает явления в
упругих телах, а электрические цепи удобнее описывать с помощью функций комплексного
переменного.
Мысленные операции над явлениями и объектами внешнего и внутреннего миров
осуществляются мозгом в конкретных кодах. Если человек склонен к использованию
зрительно-образных представлений, говорят о «зрительном воображении». Преобладание
акустически-образных представлений свидетельствует о «музыкальной фантазии».
Склонность к освоению действительности в словесно-образной форме характеризует
«поэтическую фантазию» и т. д. (Само по себе это ещё не делает художника, композитора
или поэта. Эти профессии требуют целого комплекса способностей, навыков, а также
личностных характеристик, которые не являются собственно творческими способностями,
но способствуют их реализации.)
Фундаментальные законы переработки информации мозгом неизменны, но способ
кодирования накладывает печать и на форму внешнего выражения результатов, и на выбор
объекта, а если смотреть шире – на выбор содержательной области мышления.
Счастливое совпадение индивидуальных особенностей мышления со структурой
проблем, стоящих перед данной наукой в данный период времени, – по-видимому, одно из
необходимых условий проявления научного гения».
Американский психолог Лесли Уилсон говорит: возможность быть креативным
неразрывно связана с умением превращать свою жизнь в праздник. Креативность –
повседневное творчество. Чем более наполнена наша жизнь маленькими праздниками,
радостью от самой жизни и связанных с ней успехов, тем чаще мысль оказывается способна
к полёту. Творчество невозможно, если нет свободы мысли и духа, если нет вдохновения.
Неожиданное и необычное приходит в наши мысли, когда мы сами стараемся открыть ему
дорогу, находя источники для размышлений, ассоциаций, творческого вдохновения везде – в
природе, музыке, поэзии, истории, книгах, во встречах с новыми людьми. И в новом
собственном «Я».
Бигуди № 65
Поучимся у богов мудрости. И креативности. Давным-давно, сын Геры, титан
Прометей (это имя означает «предвидящий»), по одной из версий мифов, самолично вылепил
из глины людей, а Афина вдохнула в них жизнь. Может быть, поэтому Прометей так любил
людей (как свои творения), что похитил с Олимпа частицу священного огня и передал его
людям. Заодно он научил людей сохранять огонь, присыпая его золой. За похищение огня
Зевс-Громовержец приказал Гефесту приковать Прометея к скале и обречь его на вечные
муки: прилетавший каждый день (по другой версии – каждый третий день) орёл расклёвывал
у Прометея печень, которая затем снова вырастала. Муки Прометея длились то ли несколько
столетий, то ли 30 тысяч лет (в разных изложениях мифов). Но потом Прометей помог
Гераклу, указав ему дорогу к Гесперидам (это славные девицы, нимфы, дочери Геспера –
Вечерней Звезды и Никты – Ночи; они охраняли яблоки, естественно, золотые, пока их –
девиц – не похитили). За это указание могучий освободитель нимф, Геракл, убил стрелой
орла и освободил Прометея. Но главное – он уговорил Зевса помиловать Прометея.
Помиловать-то можно, но слово Зевса должно быть нерушимо! Сказал «навечно» прикован к
скале, значит навечно! И как же поступил мудрый Зевс? Смог ли он разрешить
противоречие: Прометей должен быть прикован к скале, чтобы не нарушалось слово Зевса, и
не должен быть прикован к скале, чтобы быть свободным? Кстати, люди с тех пор носят коечто, может быть, в память о Прометее.84
Часть 4. Мысли мужские и женские
Мужчины, я хочу напомнить вам,
Что первым экземпляром был Адам.
На нём трудился бог по мере сил,
На нём сначала опыт накопил.
Из кинофильма «Труба»
20. Формальное равенство
«Самая глупая женщина легко справится с самым умным мужчиной. Но сладить с
дураком сумеет лишь самая умная» – сейчас трудно установить авторство этой мысли, но
обсуждая субъективные процессы в нашем сознании, необходимо помнить о
многочисленных объективных различиях. В том числе и половых.
Уже второй век подряд женщина большей части мира борется за равноправие с
мужчиной. А уж если женщина того захочет, она забирает у мужчины всё. Даже фамилию.
Успехов Женщина достигла немалых – и персональных (так, Софья Ковалевская и
Мария Склодовская прославились в науке исключительно собственными усилиями, без
политического шума), и коллективных (например, оговаривать в законе ограничения допуска
женщин куда бы то ни было – даже на объективно вредные работы – давно уже считается
неприличным).
Но даже самые рьяные борцы (или борицы?) за равные права не пытаются утверждать,
что женщины и мужчины равны не только юридически, но и физически. Или хотя бы
интеллектуально.
Женщины и мужчины соревнуются раздельно не только в лёгкой – а с недавних пор и
тяжёлой – атлетике, где очевидные анатомические различия затрудняют достижение
сопоставимых результатов. В интеллектуальных видах спорта соревнования также
разделены. Даже чемпионки мира по шахматам лишь изредка участвуют в мужских турнирах
– и обычно занимают места во второй половине итоговой таблицы. В динамичной игре
«Jeopardy!» (её российская версия – «Своя игра» – бытует на нашем телевидении уже лет 10)
победы женщин над мужчинами отмечаются особенно бурными овациями зрителей в студии
– просто потому, что очень уж редки. Разве что в «Что? Где? Когда?» смешанные команды
чаще всего успешнее чисто мужских или чисто женских. Как раз потому, что командное,
коллективное мышление требует гармоничного сочетания принципиально разных типов
интеллектуальной деятельности.
Кстати, наблюдая за командами знатоков, я пришёл к выводу: ценность творческих
способностей ощущается даже на подсознательном уровне. Парадоксально, но именно
поэтому женщины ценят мужчин с буйной шевелюрой больше, нежели лысых. Генетическая
программа закрепила отбор на интеллект. А кудрявая голова кажется больше.
21. Различное комбинируется
Но отчего же вообще женское и мужское мышление – да и вся прочая биология обоих
полов – настолько различна? Да и к чему вообще природе потребовалось создавать два пола?
Тем более столь различных?
Зачем два пола существуют, уже довольно давно объяснили генетики. Обмен
фрагментами генетического материала многократно увеличивает число вариантов,
сопоставляемых в ходе естественного отбора, и тем самым заметно ускоряет эволюцию.
Причём для этого ускорения достаточно двух полов. Рост числа вариантов в случае
комбинирования трёх и более полов не компенсирует резкого снижения вероятности встречи
нужного числа партнёров.
А почему двое участников единой эволюции так непохожи, объяснил ещё в 1960-х
годах доктор биологических наук Виген Артаваздович Геодакян.
22. Пробные версии
Прежде всего заметим: для сохранения и наращивания численности большинства видов
живых существ требуется гораздо больше женщин, чем мужчин. Просто потому, что
оплодотворение – самая лёгкая и скорая часть многоступенчатого и долгого процесса
размножения. Скажем, курица способна снести одно – два яйца в день. А со сколькими
курицами за этот же день готов пообщаться один петух?
В то же время генетический механизм определения пола чаще всего выдаёт примерно
равное количество мужских и женских особей. Исключения немногочисленны – так, пчёлы
способны порождать заметно меньше трутней, чем рабочих особей – и скорее подтверждают
общее правило – скажем, мотыльки (не слишком далёкие эволюционно от тех же пчёл) обоих
полов рождаются практически поровну. И это при том, что у насекомых хромосомное
определение пола в принципе допускает сравнительно несложное управление. А у высших
животных генетика пола такова, что управлять полом на стадии оплодотворения вовсе
невозможно.
Таким образом, в большинстве биологических видов мужчины явно избыточны. Они
оказываются расходным материалом. Вид может тратить по меньшей мере 9/10 запаса
особей мужского пола на цели, никак не связанные с размножением – и при этом его
численность будет расти ничуть не медленнее.
Для большинства видов живых существ стратегически необходима непрерывная
эволюция. Живя в меняющемся мире, рискованно самому оставаться неизменным.
Динозавры блестяще вписались во все экологические ниши тогдашней Земли. Но первого же
заметного изменения природных условий147 хватило, чтобы от всего грандиозного
великолепия разнообразных гигантов остались ископаемые кости да немногочисленные
юркие ящерицы.
Поскольку направление изменений мира заранее неведомо 148, эффективная эволюция
требует достаточного запаса разнообразия, чтобы любой срочно понадобившийся вариант
уже был под рукой. Вот мужчины и обеспечивают самим своим существованием это
необходимое разнообразие.
И разнообразие это тем больше, чем сильнее оторваны мужчины от всей
технологической цепочки размножения. Скажем, у пингвинов самка только сносит яйцо, а
высиживает149 его самец – и отличить самца от самки может, по известному анекдоту,
только другой пингвин. Кукушки вовсе не заботятся о своих яйцах – и тоже почти
неотличимы на глаз. Бык остаётся поблизости от коровы с телёнком, чтобы защищать их от
волчьих стай, а олени достаточно быстроходны, чтобы чаще всего просто убегать от волков.
В результате отличить оленя от важенки куда легче, чем быка от коровы.
23. Направление и зигзаги
Сравнив средние по мужчинам и по женщинам значения какого-нибудь показателя,
можно предсказать направление его эволюции. Например, паучихи в среднем заметно
больше своих супругов – и большинство видов пауков постепенно мельчает. У людей
мужчины в среднем выше женщин – и средний рост человечества уже несколько
тысячелетий150 увеличивается.
В пещерах израильской горы Кармел обнаружены скелеты древних людей. Все
женщины – типичные неандертальцы: без подбородка, с низким лбом… Все мужчины –
кроманьонцы: люди вполне современного типа. Антропологи полагают, что кроманьонское
племя захватило женщин в соседнем племени ещё не вымерших неандертальцев. Геодакян
же ехидно интересуется: даже если предположить, что неандертальских мужчин
кроманьонцы перебили – куда они девали собственных женщин? И утверждает: в ту эпоху
женщины ещё не успели эволюционировать от неандертальского облика к кроманьонскому.
Конечно, эволюционируют и показатели, по которым мужчины и женщины вовсе не
отличаются. Например, число зубов у нас одинаково – но специалисты по эволюции
полагают, что в последующих поколениях зубы мудрости скорее всего вовсе исчезнут.
Впрочем, некоторое различие наблюдается и здесь: 8-е зубы у мужчин в среднем выходят из
строя, требуют ремонта или даже удаления ощутимо чаще и раньше, чем у женщин.
Но эффективная эволюция требует большого выбора вариантов. Поэтому значения
практически любого показателя у мужчин различаются значительно больше, чем у женщин.
Например, женские спортивные рекорды ощутимо ниже мужских – но большинство женщин
147 По самой популярной сегодня гипотезе, угодивший в Землю крупный метеорит мощным ударом поднял
в воздух столько пыли, что несколько лет подряд по всей планете держалась пасмурная и холодная погода.
148 Слишком уж многие факторы могут повлиять на состояние окружающей среды. А безудержное
размножение биосферы способно радикально усилить даже самые малозаметные перемены.
149 Точнее, выстаивает. Яйцо, уложенное на антарктический лёд, замёрзнет в считанные секунды. Поэтому
пингвин кладёт его на собственные лапы и накрывает сверху животом. В такой позе приходится стоять
несколько недель почти неподвижно.
150 Более ранних данных просто не хватает для надёжных утверждений.
может подойти к ним заметно ближе, чем большинство мужчин к своим (и даже к женским).
Бесспорных гениев среди мужчин несравненно больше, чем среди женщин – зато в любой
психиатрической лечебнице мужчины составляют бесспорное большинство пациентов.
Вообще женщины стабилизируют род, накапливая в себе только те изменения, которые
уже проверены эволюцией. Они прочерчивают плавную траекторию. А мужчины опробуют
новое в бесчисленных зигзагах вокруг неё.
24. Кипит наш разум
Человек отличается от большинства видов земной живности способностью
приспосабливаться к изменениям в мире без изменений собственного тела. Вместо этого он,
пользуясь собственным разумом, создаёт разнообразные приспособления, согласующие его
возможности с окружающими условиями. То есть главное, что должно эволюционировать у
человека – разум.
Именно поэтому мужской интеллект заметно разнообразнее женского. И поэтому же он
в среднем несколько выше. Ведь человечество в целом эволюционирует в направлении роста
интеллектуальности. А мужчины движутся по любому эволюционному пути впереди
женщин. Чтобы своими судьбами проверить все зигзаги этого пути, своими телами засыпать
все ухабы и овраги. Воистину, если женщина и называет мужчину самым умным, значит, она
понимает, что второго такого дурака ей не найти.
Ярче всего выражены различия по новейшим эволюционным приобретениям
человечества: абстрактному мышлению, пространственному воображению, юмору,
творческим способностям. Женщины чаще бывают прекрасными исполнителями, но среди
выдающихся учёных, архитекторов, художников, режиссёров мужчин явно больше.
По этой же причине в мужском организме активнее, чем в женском, нервная регуляция
биологических процессов. У женщин относительно выше активность более древней
химической регуляции.
Гуморальные – то есть передающиеся через жидкости тела – стимулы действуют
заметно медленнее скоростных нервных импульсов. И главное – они несравненно менее
прицельны. Даже гормоны – природные катализаторы многоступенчатых изменений –
действуют не только на те ткани, где имеются их специализированные рецепторы. А уж на
изменение концентрации соли, глюкозы или жира реагирует практически каждая клетка.
Поэтому мужчина способен реагировать на любые изменения значительно скорее и
точнее, чем женщина. Зато женские реакции куда гармоничнее: на гуморальные воздействия
откликается весь организм, а за прицельным нервным сигналом прочие структуры тела могут
не поспеть.
Чтобы обеспечить согласование действий всего организма, мужская нервная система
вынуждена одновременно распространять множество различных сигналов. Её мощности на
это хватает не всегда151. Поэтому сбоев в работе мужского организма в целом и нервной
системы в частности заметно больше.
Функции человеческого мозга довольно чётко распределены между полушариями. У
других животных эта асимметрия выражена не так явно. То есть человек эволюционирует в
151 Нечто подобное наблюдается и в экономике. Ещё в 1970-х годах советские математики – академики
Глушков и Канторович – установили: вычислительной мощи всего мира не хватает – и ни в каком обозримом
будущем не будет хватать! – для оптимального управления всей экономикой страны из единого центра, даже
если этот центр каким-то чудом сможет получать всю информацию, необходимую для определения оптимума.
Зато делегирование права решения на самый низкий из уровней, способных это решение принять, в сочетании с
косвенными воздействиями на спектр допустимого выбора – то есть вся совокупность рыночных механизмов
управления – позволяет вывести экономику на уровень несравненно высший, нежели реально достижимый при
централизации. Хотя, конечно, ошибки и сбои зачастую случаются и в рынке – но в среднем (и по времени, и по
всему спектру возможных направлений развития) искусно контролируемый рынок опережает чистый строго
централизованный план.
сторону роста дифференциации полушарий. Соответственно мужской мозг в среднем
асимметричнее женского. И это сказывается на стиле мышления. В женской логике заметно
больше эмоций, тогда как мужчины стараются даже откровенно эмоциональные шаги
обосновать логически.
Конечно, это различие не достигает пределов, создающих подлинные трудности для
взаимопонимания. Но люди зачастую стараются подчеркнуть его, чтобы точнее
соответствовать половым ролям. Потому что человек – животное общественное. И
ориентируется на то, чего общество от него ожидает.
25. Пол политика
Мужчина в среднем нацелен на изменение всего мира. Женщина чаще сама
приспосабливается к этим изменениям. А если что-то меняет – то чаще всего ближайшее
окружение. Страсть женщин к смене платьев, перекраске волос или перестановке мебели –
предмет множества анекдотов, сочинённых мужчинами. Они чаще всего просто не
понимают, что в таких скромных формах женщина согласует общечеловеческую жажду
изменений со специфически женским стремлением к стабильности.
Такое гармоническое согласование стабильности с изменениями необходимо,
разумеется, не только в повседневном быту. Знаменитый английский исследователь
иерархических структур Лоуренс Питер делит корпорации на мужские и женские. Мужские
нацелены на бурную экспансию. Женские стремятся получше обустроиться в уже освоенных
экономических нишах.
«Принцип Питера», конечно, воспринят во всём мире как юмористическое
произведение. На самом же деле специфическим английским юмором окрашено очень
серьёзное содержание, способное прояснить множество сложных общественных явлений.
Подобным же образом можно выделить половой диморфизм в политических
стратегиях. Специфически мужская политика: непрерывные резкие изменения, размашистая
– во весь опор – реакция даже на самые мелкие поводы. Специфически женская: плавная
подстройка, согласование и гармонизация, деликатная компенсация внешних изменений.
Формула Михаила Евграфовича Салтыкова (Щедрина) «Свирепость законов российских
умягчается единственно необязательностью соблюдения оных» весьма наглядно
иллюстрирует женскую политику во взаимоотношениях с мужской.
Примеры мужской политики памятны каждому. Генерал Шарль де Голль и «железная
леди» Маргарет Тэтчер, столь же железный канцлер Отто Бисмарк («Величайшие вопросы
истории решаются не голосованием в парламентах, но железом и кровью») и «единственный
мужчина в правительстве Израиля» премьер-министр Голда Меир изменили свои страны – а
то и весь мир – настолько, что результаты их деятельности памятны нам по сей день. А
последствия гипертрофированно мужской власти Бориса Ельцина Россия будет
расхлёбывать, наверное, ещё несколько десятилетий.
Политика женская куда менее заметна. Не только потому, что не сопровождается
яркими скорыми переменами. Но и потому, что окружает нас постоянно. Именно женская
политическая стратегия даёт каждому из нас хотя бы тот минимум стабильности, без
которого вовсе невозможна сколько-нибудь осмысленная деятельность – в том числе и по
мужской стратегии.
Мужская и женская стратегия в политике взаимодополняются – в идеале так же
гармонично, как и в биологии. Более того, возможно – и, по-видимому, оптимально –
сочетание обеих стратегий в деятельности одного лидера.
Например, мэр Москвы в своё время по-мужски встряхнул столицу, заставил всё её
хозяйство стремительно меняться в соответствии с новой экономической ситуацией – и
постоянно по-женски поощряет естественную эволюцию. В самой же экономике он
гармонично балансирует мужскую и женскую стратегию: поощряет самостоятельное
развитие бизнеса (прежде всего малого и среднего – то есть соответствующего женской
стратегии малых непрерывных изменений), но удерживает за городом возможности
регулирования принципиальных аспектов. Правда, в тех случаях, когда столичные решения
расходятся с решениями федерального законодательного блока, мэр зачастую реагирует
слишком уж резко, по-мужски. Это похвально психологически, но далеко не оптимально
политически: противостояние двух сопоставимых сил заметно тормозит общее развитие.
Конечно, прогибаться под мир на каждом шагу столь серьёзный лидер не может и не должен.
Но некоторая пластичность в политике – по образу и подобию его же собственной гибкости
в экономике – ему не повредила бы.
Бигуди № 66
Нет, это задача не о мужчинах и не о политиках. Хотя… Итак, представьте, на узком
мосту встретились два водителя – шотландца (а они считаются в Великобритании скупцами
и упрямцами). Разминуться на мосту невозможно. Тратить бензин, чтобы дать задний ход, не
хочет никто (да и уступать другому – выше всяких моральных сил). Один из них, надеясь,
что другому надоест ждать, не спеша раскрывает газету и спокойно начинает читать. На Ваш
взгляд (политика, мужчины, шотландца и т. п.), что должен сказать или сделать другой?85
Креативные решения – это всегда парадоксальные, неочевидные ходы. Золотое правило
механики («правило рычага») в одной из формулировок гласит – проигрывая в пути,
выигрывай в силе. Похожий принцип уже упомянутой ТРИЗ говорит: чтобы достигнуть
идеального конечного результата – измени состояние системы во времени и/или в
пространстве. Эти же принципы, очевидно, работают во всех областях жизни. А политика –
одна из важнейших составляющих жизни. Хоть в семье, хоть на международной арене.
Вспомним, например, А.М. Горчакова – русского дипломата, министра иностранных
дел и государственного канцлера России. Придя на вершину политической власти в 1856-м
году, после поражения в Крымской войне, он решил выдержать паузу для реванша, хотя
противники только и ждали от России – как от обиженной стороны – активных действий.
Широко известной стала фраза Горчакова: «Говорят, что Россия сердится. Нет, Россия
не сердится. Россия сосредоточивается». Нейтралитет России в войнах Пруссии с Данией
(1864), Австрией (1866), Францией (1870–1871) позволил германским армиям одержать
действительные победы. Между тем Австрия, угрожая России в 1853-м ударом с фланга, не
позволила разбить Турцию до подхода войск западной коалиции, а Франция была одной из
главных участниц этой коалиции. После разгрома Австрии и Франции стало возможно
отменить постыдный для нашей страны договор, практически отнявший у России
возможность самостоятельной политики на Чёрном море.
Циркуляр Горчакова 1870-го года извещал правительства других стран, что Россия не
считает связанной себя постановлениями, ограничивающими её суверенные права.
Недовольство многих европейских стран, обессиленных в войнах, не привело к каким-то
санкциям против Российской империи, и вскоре их представители признали за Россией право
держать военный флот на Чёрном море и строить укрепления на его побережье.
Дипломатический триумф «выжидающего и сосредоточивающегося» Горчакова сделал его
одной из центральных фигур мировой политики.
Бигуди № 67
Эта история о том, что в каждой задаче, в каждой ситуации есть второстепенные
элементы, а есть главный элемент, «гвоздь на котором всё висит». Так вот, представьте, двое
джентльменов гуляют вдоль Темзы. Вдруг раздаётся крик: «Спасите!» Один из
джентльменов бесстрашно бросается в реку, вытаскивает на берег ребёнка и передаёт его
своему приятелю для оказания первой помощи. Но не успел он отдышаться, снова слышится
крик. Всё тот же джентльмен вновь бросается в воду и спасает второго ребёнка. Но дело на
этом не закончилось! Когда достойный джентльмен вытаскивает из воды третьего ребёнка,
то видит, что его приятель куда-то уходит. Пораженный джентльмен спрашивает: «Куда же
Вы, дружище?» Приятель на ходу отвечает: «Не беспокойтесь. Вы тут пока спасайте детей, а
я выясню, кто…». Что же хочет выяснить второй достойный джентльмен?86
Послесловие
Каждая мысль – событие. И возникающее не по собственному желанию.
Мысль неустойчива и непредсказуема. Поток ассоциаций, сравнений, воспоминаний
кажется неуправляемым. Невозможно им руководить, невероятно дойти до желанной цели,
используя дисциплинированное и послушное (пусть и эффективное!) мышление. Впрочем,
как говорил философ М. Мамардашвили, «с невозможностями необходимо иметь дело хотя
бы потому, что по дороге к невозможному только и можно что-то иметь, что-то разрешить».
Идеал сверхчеловека, созданный Фридрихом Ницше, невозможен и утопичен. Ницше,
пожалуй, знал, что сверхчеловеками мы не станем, но по дороге к идеалу, «стремясь быть
сверхчеловеками, мы станем людьми». По дороге к совершенному, эффективному и
потрясающе творческому мышлению мы становимся – как минимум! – Homo Sapiens,
Человеками Мыслящими.
Открытия, догадки, изобретения кажутся почти невозможными, но ведь совершаются!
Это удивительно, но – неоспоримо. Удивляясь самому факту мышления, человек начинает
мыслить.
Один из создателей самого представления об эволюции Жан Батист Ламарк полагал:
разнообразие видов живого создано их собственными желаниями. Кто-то вытягивал изо всех
сил шею, чтобы дотянуться до нежных листьев на верхушках деревьев, кто-то упражнялся в
беге за добычей, кто-то нырял поглубже за рыбой и водорослями – так и возникли жирафы,
гепарды, киты…
Правда, впоследствии Чарльз Дарвин показал: желания тут ни при чём. Неумолимое
давление условий жизни позволяет размножаться лишь тем, у кого случайно оказались
признаки, более подходящие для этих условий.
Но в области интеллекта прав Ламарк! Мы действительно можем развивать свой разум
по собственной воле. И развивать в самых разных направлениях – тому порукой все
перечисленные в этой книге творческие приёмы.
Ваши способности – в Ваших руках, Ваши интеллектуальные мощь и красота зависят
от Вас. Откройте в себе скрытые таланты, переделайте свой духовный интерьер так, чтобы
находить опору для творческого вдохновения, освободите своё мышление для собственных –
пусть маленьких – открытий и побед.
Осваивайте какие-то из несметных экологических ниш творческой деятельности или
создавайте новые – как Дюрер создал гравюру, как на заре эволюции фотосинтезирующие
организмы создали на Земле кислородную атмосферу, где расцвели бессчётные иные виды.
Не стесняйтесь сделать свою жизнь более творческой – значит, ярче и насыщеннее: лучше от
этого будет не Вам одному.
Замечательный английский поэт Джон Донн, не потерявшийся даже на фоне своего
современника Вильяма Шекспира, писал: «Ни один человек не есть Остров, целостный сам
по себе; каждый человек есть часть Континента, часть Материка; если Море смоет Комок
земли – это потеря Европы, такая же, как в случае, если бы был смыт Мыс, как если бы это
было Имение твоих друзей или твое собственное; смерть любого человека уменьшает меня,
потому что я часть Человечества; и поэтому никогда не спрашивай, по ком звонит колокол, –
он звонит по тебе». Мы обычно повторяем эти слова в горе. Но они столь же верны в
радости. Потому что при каждом достижении любого из нас растёт и совершенствуется наша
общая ноосфера – Вселенная разума.
Крис Фрит утверждает, что мы встроены во внутренний мир других людей точно так
же, как мы встроены в окружающий материальный мир. «Все, что мы делаем и думаем в
настоящий момент, во многом определяется людьми, с которыми мы взаимодействуем. Но
мы воспринимаем самих себя иначе. Мы воспринимаем себя как деятелей, обладающих
независимым сознанием. В этом и состоит последняя иллюзия, создаваемая нашим мозгом.
По сравнению с другими животными люди делают много странных вещей. Мы
разговариваем. Мы пользуемся орудиями труда. Нам иногда свойственно альтруистическое
поведение. И, что самое странное, наше альтруистическое поведение иногда проявляется
даже по отношению к посторонним.152 Экономисты изучают это поведение на примере
несложных игр с деньгами».
Развитием памяти, мыслительных и творческих способностей, воображения нужно
руководить осторожно и умело – во избежание не только сбоев, но и разочарований. Ведь не
все станут гениями – да и не всякий захочет. Но вообразите общество, где каждый обладает
живым творческим воображением, хорошей памятью, умением быстро и не шаблонно
мыслить – и готов каждый день применять все это в работе. Как пойдет развитие всей такой
страны!
Главное богатство каждой страны – это люди, её населяющие. Со своими желаниями,
потребностями, переживаниями. Каждый со своим интеллектом.
А интеллект должен работать и развиваться всегда – ни послесловие книги, ни
последняя перевернутая страница не могут стать шлагбаумом для этого процесса.
А поэтому:
Бигуди № 68
Пожалуйста, закончите мысль, которую лаконично сформулировал Станислав Ежи
Лец: если голова сталкивается с книгой и раздаётся глухой пустой звук, то не всегда
виновата…. Кто там виноват?!87
И, кто бы это ни был, будем совершенствоваться вместе! Позаботимся о своих
извилинах, обратившись к россыпи «бигудей», выбирая их не только из приведенных далее
избранных задач и упражнений на сообразительность, но и за пределами книги – в самой
жизни.
Избранные задачи и упражнения на сообразительность
Уже больше века кур выращивают в промышленных масштабах в инкубаторах. Учёные
обнаружили, что у птиц, выращенных таким образом, отсутствует рефлекс на поиск
местоположения как воды, так и корма. На вторые сутки после вылупления многие цыплята
гибнут от голода. Как быть?88
В химической лаборатории обнаружился «генеральский эффект». Одна заковыристая
реакция протекала лишь в том случае, если её проводил именно лаборант Н. А ни у кого
иного ничего похожего на выходе не получалось У нашего же Н. результат был уникальный.
Опыт ставили много раз. Колба, в которой проходила реакция, была во всех случаях закрыта.
Заметили только, что когда в лаборатории находился ещё кто-нибудь, кроме Н., результат
реакции был самый банальный. Как это объяснить?89
В детандере – части аппарата для производства жидкого гелия – эдакой вертикальной
152 Объяснение истоков альтруизма – одна из важнейших проблем эволюционной биологии. Естественный
отбор заставляет нас ожидать, что животные будут вести себя так, чтобы увеличить собственные шансы, а не
шансы кого-нибудь другого на выживание и размножение. Объяснение истоков альтруизма через групповой
отбор было важным достижением биологии XX века. Если мы будем заботиться о своих родственниках, наши
гены могут сохраниться, даже если мы сами погибнем. Холдейн сформулировал это так: «Я готов отдать жизнь
за двух родных братьев или восемь двоюродных». Но зачем помогать посторонним? – Крис Фрит.
трёхметровой трубе (диаметр которой 10 сантиметров) случилась неприятность. В него
уронили железный гвоздь, мячик из резины и большую гайку из меди. А как доставали все
эти предметы, если в трубе пока ещё нет жидкого гелия? Кстати, достали одновременно, и
аппарат при этом не разбирали!90
Разложите по трем стаканчикам одинакового размера десять одинаковых шариков так,
чтобы число шариков в каждом стаканчике было нечётным.91
Двое краснокожих сидели на брёвнышке, один повыше ростом, другой пониже. Тот,
кто пониже ростом, доводится сыном тому, кто повыше ростом, хотя тот, кто повыше
ростом, – не его отец. Как Вы это объясните?92
Когда можно носить воду в решете?93
Кусок сливочного масла весит 1 кг. Как без разновесов взвесить 375 г масла?94
Сколько раз из чёртовой дюжины можно вычесть число три?95
Сидит человек, а Вы не можете сесть на его место, даже если он встанет и уйдёт. Где он
сидит?96
В старинной задаче говорится: два отца и два сына нашли на дороге, ведущей в Бомбей,
три рупии (серебряные монеты) и быстро поделили их между собой, причём каждому
досталось по монете. Как им это удалось?97
Один король хотел сместить своего премьер-министра, но при этом не хотел его
слишком обидеть. Он позвал премьер-министра к себе, положил при нём два листка бумаги в
портфель и сказал: «На одном листке я написал «Уходите», а на втором – «Останьтесь».
Листок, который Вы вытащите, решит Вашу судьбу». Премьер-министр догадался: на обоих
листках написано «Уходите». Как же, однако, умудрился он при этих условиях сохранить
своё место?98
Люди, приезжавшие в одну деревушку, часто удивлялись местному дурачку. Когда ему
предлагали выбор между блестящей 50-центовой монетой и мятой пятидолларовой купюрой,
он всегда выбирал монету, хотя она стоит вдесятеро меньше купюры. Почему он никогда не
выбирал купюру?99
Предчувствуя скорую кончину, старый шейх велел позвать двух своих сыновей и
сообщил им, что в расположенном неподалеку оазисе он закопал несметные сокровища.
Шейх повелел сыновьям отправиться на поиски клада, завещав его целиком тому, чей
верблюд достигнет оазиса вторым. Сыновья шейха призадумались. Если каждый из них
будет стремиться отстать от другого, то они никогда не доберутся до оазиса и не смогут
выкопать сокровища. Изрядно поломав головы, но так ничего и не придумав, они
отправились за советом к кадию. Кадий велел сыновьям шейха спешиться и, подозвав их к
себе, прошептал что-то каждому на ухо. Выслушав совет кадия, наследники шейха
торопливо вскочили на поджидавших их верблюдов и что было духу помчались к оазису.
Какой совет дал кадий сыновьям шейха?100
Представьте себе, что Вы сидите за рулём автомобиля и хотите узнать скорость
машины, идущей впереди Вас. Как это сделать проще всего?101
Геометр берётся начертить четыре окружности различной длины, сохраняя
неизменным раствор циркуля. Каким образом?102
Можно ли растворить в стакане неограниченное количество сахара?103
На привале туристы нечаянно рассыпали пачку соли, да так, что она смешалась с
песком. Как быть?104
Почему блестят начищенные ботинки?105
Прежде на улицах городов часто висели громкоговорители. Их старались поднять
повыше, но почти никогда не устанавливали выше десятого этажа. Почему?106
Представьте себе бумагоизготовительный механизм длиною в несколько сотен метров.
На входе в машину загружается сырьё, на выходе – уже готовый рулон бумаги. Предложите
способ, как дистанционно обнаружить, нет ли где на линии производства обрыва бумаги.107
Собака была привязана к десятиметровой верёвке, а прошла 300 метров. Как ей это
удалось?108
Мэри уронила серьгу в кофе, но хотя чашка была полна до краёв, Мэри сумела достать
серьгу, даже не намочив пальцев. Как это могло быть?109
У профессора есть домашние животные. Все они, кроме двух, собаки, все они, кроме
двух, кошки, и все они, кроме двух, попугаи. Сколько всего домашних животных у
профессора?110
Если в 12 часов ночи идёт дождь, то можно ли утверждать, что через 72 часа в том же
месте будет солнечная погода?111
Что бросают, когда нуждаются в этом, и поднимают, когда в этом нет нужды?112
Если бы сейчас было на два часа позже, то до полуночи оставалось бы в два раза
меньше времени, чем если бы сейчас было на час позже. Сколько сейчас времени?113
В 1920-е годы сотрудники уголовного розыска при оперативной работе использовали
вместо удостоверений так называемые «знаки скрытого ношения». Знак состоял из
служебного и маскировочного жетонов. Служебный жетон был скрыт под лацканом
пиджака. На маскировочном жетоне, привинченном с лицевой стороны лацкана, могло быть
три различных рисунка: охотник, стреляющий в утку, всадник, преодолевающий
препятствие, или… Назовите третий, самый распространённый вид рисунка.114
Знаменитая «чёрный пенни», первая в мире почтовая марка, выпущена в Англии в
1840-м году. Принцип оплаты пересылки почты марками имел огромный успех и
впоследствии принят во всем мире. Однако всего через год «чёрный пенни» заменили
«красным пенни». Почему?115
Однажды английский король Георг II лёг спать вечером 2-го сентября и проснулся
только утром 14-го сентября. Его врачей и никого из его окружения это не обеспокоило,
возможно потому, что всем был известен подобный же случай с французским королем
Генрихом III: тот заснул 9-го декабря, а проснулся 20-го декабря. Что же случилось с
ними?116
В Библии содержится множество историй о болезнях, войнах и разрушениях. Кому,
согласно Библии, единственному из всех, удалось уничтожить четверть населения Земли?117
В начале Первой мировой войны в униформу британских солдат входила коричневая
матерчатая фуражка. Металлических касок у них не было. Через некоторое время
командование армии обеспокоилось большим количеством ранений в голову. Решили
заменить фуражку металлической каской. Но вскоре командование удивилось, узнав, что
количество ранений в голову увеличилось. Необходимо заметить: интенсивность сражений
была примерно одинаковой до и после введения касок. Так почему же число ранений в
голову увеличилось, когда солдаты стали надевать каски, а не фуражки?118
Чтобы стать поваром у знатного вельможи, претенденту предложили приготовить
блюдо из равных количеств, жидкости, продукта, соли. Он ответил: «Нет проблем!», и
принялся за приготовление блюда. Что это было за блюдо?119
В некоторых штатах США наказуемо одно действие. Людей, которые пытаются его
совершить, сажают в тюрьму, но людей, которые успешно совершили его, никогда не
наказывают. Что это за преступление?120
Первый такой механизм изготовлен испанскими механиками по заданию святейшей
инквизиции, и не один грешник испытывал на себе его действие. Дожил он почти в
неизменённом виде до наших дней, и все вы наверняка испытывали его действие и на себе.
Что же это за механизм?121
«Его величество Георг под номером Четвёртый…» родился в 1763-м году. Он правил
Англией с 1820-го года до своей смерти в 1830-м году. Он был не сильно знаменит, а если и
знаменит, то покровительством всевозможным искусствам, но именно ему принадлежит
нововведение в обуви. Его сапоги отличались от других. Мы пользуемся его изобретением и
сейчас, но в то время оно было очень необычным. Что он изобрёл?122
Одного человека спросили: «Сколько вам лет?» Он ответил: «Порядочно. Я старше
некоторых своих родственников почти в шестьсот раз». Может ли такое быть?123
Считается, что есть веская природная причина, по которой у птичьих яиц один конец
тупее другого. Что это за причина?124
Принцип неоднородности. Довольно часто при изготовлении гири в основной металл
намеренно вкрапляют кусочек свинца или меди. Для чего это делается?125
Мужчину находят убитым в его кабинете. Тело мужчины наклонено над письменным
столом, в руке зажат револьвер, на столе лежит диктофон. Полицейские включают диктофон
покойного и сразу слышат записанное на пленку сообщение: «Я не могу больше жить. Жизнь
больше не имеет для меня никакого смысла…». После этого раздаётся выстрел. Как Вы
считаете, почему полицейские сразу поняли, что мужчина был убит?126
Чтобы люди могли попасть в канализационные коллекторы или добраться до других
подземных коммуникаций, используются люки. В подавляющем большинстве случаев
крышки люков имеют круглую форму, а не квадратную или прямоугольную. Почему?127
Петя и Миша играли на грязном и тёмном чердаке дома. Потом они спустились вниз. У
Пети всё лицо было грязным, а лицо Миши чудом осталось чистым. Несмотря на это, только
Миша отправился умываться. Почему?128
Может ли человек, находясь на Земле, увидеть Солнце восходящим с запада?129
Я однажды наблюдал такую картину. В поезде стояли отец и сын, одетые потуристически. Мальчику очень хотелось пить. Он долго пытался открутить пробку у
пластиковой бутылки с газированной водой. Мальчик то сильно взбалтывал эту закрытую
бутылку, то ударял по ней ладонью, но это не помогало: пробка не откручивалась. Отец не
мешал сыну, но когда у того иссяк запал, сказал ему: «Оставь бутылку в покое, и ты
увидишь, как через какое-то время пробка легко открутится».130
Один рыбак купил себе новую удочку длиной 5 футов. Домой ему приходится
добираться общественным транспортом, где правилами запрещено перевозить предметы
длиной более 4 футов. Как упаковать удочку, чтобы проехать в общественном транспорте не
нарушая правил?131
Отец с двумя сыновьями отправился в поход. На их пути встретилась река, у берега
которой находится плот. Он выдерживает на воде или отца, или двух сыновей. Как
переправиться на другой берег отцу и сыновьям?132
Имеются трёхлитровая банка сока и две пустые банки: одна – литровая, другая –
двухлитровая. Как разлить сок так, чтобы во всех трёх банках было по одному литру?133
В некоторой стране есть два города. В одном из них живут только люди, которые
всегда говорят правду, в другом – только те, кто всегда лжёт. Все они ходят друг к другу в
гости, т. е. в любом из этих двух городов можно встретить как честного человека, так и
лгуна. Предположим, Вы оказались в одном из этих городов. Как, задав один-единственный
вопрос первому встречному, определить, в какой город Вы попали – в город честных или в
город лгунов?134
Французский королевский устав предписывал: при встрече двух офицеров первым
честь отдаёт тот, чьё звание ниже. А если встречаются два офицера с равными воинскими
званиями? Кто по уставу должен отдать честь первым?135
После смерти великого русского химика Д.И. Менделеева в его халате нашли записку,
где было всего четыре буквы: «э», «м», «м», «н». Вначале предположили, что это
зашифрованное имя тайной возлюбленной, но всё оказалось гораздо прозаичнее. Что же
стоит за этими буквами, если первая «м» означает можно, а «н» – нельзя?136
Снежная лавина накрыла, разметала лагерь альпинистов. Выкарабкались, вытащили,
что могли, собрались. Слава Богу, все живы. Один, правда, здорово повредился. Лихорадит.
Доктор умудрился спасти аптечку! Вколол обезболивающее, достал новое чудо
фармацевтики. В ампуле порошок и написано «Растворить в столовой ложке кипящей воды».
И ни у кого ни спичек, ни зажигалки, ни лупы. Но доктор физику знает. Через минуту
порошок начал растворяться в кипящей воде. Как он это сделал?137
Один богач завещал всё своё богатство тому монастырю, который отслужит по нему
количество обеден, равное половине количества оставшихся дней существования этого
монастыря. Много монастырей хотело получить это богатство, но не знало, как выполнить
условие завещания. Наконец, настоятель одного монастыря сказал, что он знает, как
выполнить условие завещания. Как же он собирался выполнить его?138
Вход в парк некоего могущественного правителя был запрещён. Если нарушитель
попадался, его ожидала смерть, но ему предоставлялось право выбирать между виселицей и
обезглавливанием. Он должен был что-то заявить, и если его утверждение было верно, его
обезглавливали, а если ложно, то его вешали. Что нужно заявить нарушителю, чтобы
избежать установленного правила и остаться живым?139
Во время второй мировой войны над пропастью между Германией и Швейцарией был
мост. Немецкому часовому было приказано стрелять в каждого, кто попытается бежать из
Германии через мост в Швейцарию, и возвращать назад тех, у кого нет разрешения на вход в
Германию. Часовой был на немецкой стороне моста. Он сидел в будке и каждые три минуты
выходил наружу, чтобы осмотреть мост. Одной женщине было необходимо срочно попасть в
Швейцарию. Она не могла получить разрешения на это. Она знала, что может проникнуть на
мост, когда часовой сидит в будке, но чтобы пройти весь мост, требовалось пять – шесть
минут. На мосту негде прятаться, и часовой мог с легкостью застрелить её, если бы увидел,
что она направляется в Швейцарию. Как она смогла перебраться через мост?140
Этот безымянный полковник английской армии значился под номером 888. За большие
заслуги в Первой Мировой войне на поприще связи легендарный полковник похоронен со
всеми воинскими почестями. Кто он?141
На Фиджи долгое время имели хождение только бронзовые монеты. Но появившиеся
там в сравнительно недалёком прошлом медно-никелевые монеты гораздо больше пришлись
по вкусу жителям острова. Чем же они им так понравились?142
Древние скандинавские саги рассказывают: норвежцы, отправляясь в морские плавания
на поиски новых земель, брали с собой свой житейский скарб и деревянные столбы с резным
изображением первопредка (родовой столб). Для чего, приближаясь к новым берегам,
переселенцы бросали в волны эти столбы?143
Однажды ходжа Насреддин попал в гости к деревенскому имаму. Так в исламе
именуется глава религиозной общины. Чаще всего это опытный проповедник, не только
руководящий общей молитвой, но и дающий рядовым верующим наставления в вопросах
веры и поведения. Имам спросил Насреддина: «Чего ты хочешь – спать или пить?» Видя, что
про еду имам и не заикается, ходжа сказал… что же именно?144
Мой брат как-то раз взял в буфете два стакана чая и два бутерброда с сыром. «С вас 87
копеек», – сказала буфетчица. Брат моментально сказал: «Проверьте! Вы ошиблись!» Как он
сообразил, что продавец ошибся?145
Один из понравившихся мне вопросов телевизионного «Что? Где? Когда?» довольно
краток. В 1923-м году в Канаде в связи с принятием этого правила резко упали цены на
говядину. О каком правиле идёт речь?146
Мой дед жил в сельской местности и частенько ездил на телеге. Он заметил, что
переднее колесо горит – в буквальном смысле – чаще, чем заднее. Из-за чего это
происходит?147
Последний раз по ходу игры «Что? Где? Когда?», где я участвовал в составе команды
ветеранов, нам показали четыре разные по архитектуре, времени и месту строительства, но
высокие башни. Ведущий спросил: «В кармане у нашего ассистента, стоящего у игрового
стола, есть щепотка того, что изготавливали в этих башнях. Что это?»148
Рядом с железной дорогой, с одной и той же стороны от полотна, есть цех для
производства кокса и цех, где обогащается руда. Где должен быть расположен завод, на
который доставляют и руду и кокс, чтобы от того и от другого цеха до завода было бы
кратчайшее расстояние?149
Найдите произведение 26 различных множителей, где вычитаемый знак – всякий раз
очередная буква латинского алфавита: (х – a)(х – b)… (x – y)(x – z)150
Задач о поиске фальшивой монеты среди настоящих – очень много (ведь и сама
история подделки монет насчитывает не одно тысячелетие). Вот сравнительно простая. Есть
девять монет. Одна из них – фальшивая – легче остальных. В вашем распоряжении
уравновешенные аптекарские весы без разновесов. По условию максимум за два
взвешивания необходимо определить, какая из девяти монет фальшивая.151
А вот задача куда сложнее. Есть девять монет. Одна из них фальшивая. В вашем
распоряжении уравновешенные аптекарские весы без разновесов. По условию максимум за
три взвешивания необходимо определить, какая из девяти монет фальшивая.152
Более того, оказывается, что иной раз в большем числе монет проще разобраться.
Имеется 13 монет, из них ровно одна фальшивая, причём неизвестно, легче она настоящих
или тяжелее. Требуется найти эту монету за три взвешивания. Весы – стандартные для задач
этого типа: две чашечки без гирь.153
И закончу эту серию сравнительно короткой задачей. Имеется 10 монет. Одна из них
фальшивая и легче настоящей монеты. Как с помощью чашечных весов без гирь определить
за три взвешивания максимум какая из монет фальшивая?154
Один из этапов этого поиска в своё время бродил по свету как самостоятельная задача.
Известно, что из пяти одинаковых по виду колец одно несколько легче каждого из
остальных. Разница в весе небольшая, поэтому без использования весов лёгкое кольцо не
обнаружить. Как найти это кольцо, если, не пользуясь гирями, произвести взвешивание не
более двух раз?155
Задача Льюиса Кэрролла про обезьяну и груз: «Через блок, прикреплённый к потолку,
переброшен канат. На одном конце каната висит обезьяна, к другому прикреплён груз, вес
которого в точности равен весу обезьяны. Предположим, что обезьяна начала взбираться
вверх по канату. Что произойдёт при этом с грузом?» Математики и физики, которым он
показывал эту задачу, приходили к разным выводам. А что произойдёт по вашему
мнению?156
Одному человеку очень хотелось попасть в театр. Билет стоит 1 шиллинг 6 пенсов, а
денег у человека было всего лишь 1 шиллинг. Подумав, человек решил заложить свой
шиллинг у ростовщика. Ростовщик внимательно осмотрел монету и, убедившись, что она не
фальшивая, дал человеку под залог 9 пенсов. С 9 пенсами и квитанцией на 1 шиллинг в
кармане человек вышел от ростовщика и повстречал на улице приятеля, которому предложил
купить квитанцию. Приятель, решив, что сделка выгодна, купил квитанцию за 9 пенсов.
Теперь у человека было 9 пенсов, полученных от ростовщика, и 9 пенсов, вырученных от
продажи квитанции. Этой суммы ему как раз хватило, чтобы купить себе билет в театр. Кто
и сколько потерял в результате всех операций?157
Классическая тризовская задача, чей анализ с позиций этой теории приведен в книге
«Месяц под звёздами фантазии». В конце прошлого века великий русский учёный-химик Д.
И. Менделеев обратил внимание на чрезвычайно опасную операцию – сушку пороха горячим
воздухом. При ней часто происходили взрывы, поскольку порох электризовался при трении,
и в результате проскакивала искра. Как Менделеев решил эту проблему.158
Кастрюля наполнена до краёв водой. Как отлить воды ровно половину кастрюли, не
пользуясь никакими вспомогательными устройствами – ни другой посудой, ни
измерительными приборами?159
Квадратное поле окружено рвом шириной 3 м. Ров наполнен водой. Как перейти на
четырёхугольное поле, если имеются две толстые доски длиной тоже ровно по 3 м. Ни
гвоздей, ни молотка, вообще ничего под руками больше нет, кроме этих двух досок.160
В двух ящиках для уроков труда хранятся ножницы – по 20 штук в каждом. Перед
уроком труда учительница взяла несколько ножниц из одного ящика, а затем из второго
взяла столько, сколько осталось в первом. Сколько ножниц осталось в обоих ящиках?161
Представьте себе квадратный пруд, в углах которого растут вековые дубы.
Потребовалось увеличить площадь поверхности пруда вдвое, но при этом нельзя губить
деревья. И ещё хотелось бы, чтобы пруд оставался бы квадратным. Как это сделать?162
Выдающийся русский актёр Александр Остужев по нескольку часов в день тренировал
голос, изучал мимику своих партнёров. Он работал не только над своей ролью, но и
выучивал всю пьесу от первой до последней реплики. Назовите причину такого упорного
труда актёра.163
Лоси, как известно, любят полакомиться листьями деревьев и тонкими ветками,
которые расположены так высоко, что лоси не могут до них дотянуться. Внимание! Вопрос!
Кто же помогает лосям полакомиться любимой едой? Назовите этих животных!164
По легенде, при начале строительства храма Соломона каждому каменщику
сообщались тайные слова. Мастерам – одни, подмастерьям – другие, ученикам – третьи. Для
чего это делалось? И когда эти слова становились нужны?165
Внимание! Перед вами чёрный ящик! В чёрном ящике находится то, с чем древние
мудрецы сравнивали Вселенную. Они говорили, что у нынешнего момента может быть
бесконечное количество прошлых, которые привели к «сейчас». И такое же количество
будущих, проистекающих из «сейчас». Что в чёрном ящике?166
Несколько дачников пользуются одной лодкой. Они привязывают её цепью,
закреплённой одним концом на причале. У каждого дачника – свой замок для цепи. Каждый
хочет, имея ключ только от одного замка, в любой момент отвязать лодку и поехать кататься,
не дожидаясь своих товарищей. Как замыкать лодку?167
В чешском городе Дечин, на левом берегу реки Эльба, лежит так называемый
«Голодный камень». На этом камне высечена надпись: «Если увидишь меня – заплачешь».
Внимание! Вопрос! Когда можно увидеть эту надпись?168
Назовите то, что, по мнению французского искусствоведа Андре Базена, с момента
создания и по сей день служит для того, чтобы мумифицировать время.169
В одном из маленьких городов Италии ежегодно проводится всеитальянский конкурс
лжецов. Как-то раз победила маленькая девочка, приславшая на конкурс такую небылицу: «В
нашем городе была такая суровая зима, такая суровая зима, что даже пламя от свечи…»
(завершите высказывание)170
И последняя задачка, как говорится, на преодоление инерции мышления и снятие
чувства собственной значимости. Её приводят в авторском курсе «Инженерно-техническое
творчество…» мои коллеги и соавторы по другим изданиям Сергей Ёлкин и Дмитрий
Гаврилов. Я же воздержусь от подсказки. Решите-ка задачку сами: «Все с детских лет знают,
что именно в большинстве случаев даже взрослому не поднять за хвост с пола. Но,
повзрослев и поумнев, как кажется на первый взгляд, узнав контрольный ответ,
придумывают массу других вариантов, словно бы оправдываясь. О чём речь?»
Литература
Cписок использованной литературы:
1. Полани М. Личностное знание: На пути к посткритической философии. М., 1985.
2. Эфроимсон В.П. Предпосылки гениальности (Биосоциальные факторы повышенной
умственной активности) // журнал «Человек», № 2–6, 1997; № 1, 1998.
3. Лебон Г. Психология народов и масс. – СПб.: Макет, 1995.
4. Сухарев В.А., Сухарев М.В. Психология народов и наций. – Донецк: Сталкер, 1997.
5. Юнг К.Г. Душа и миф. Шесть архетипов. – Мн.: Харвест, 2004.
6. Радин Пол. Трикстер. Исследование мифов североамериканских индейцев с
комментариями К. Г. Юнга и К. К. Кереньи. – СПб.: Евразия, 1999.
7. Селье Г. От мечты к открытию. Екатеринбург, 1999.
8. Taylor С W. (ed.). Creativity. Progress and potential. N.Y., 1964.
9. Taylor С W., Barron F. (ed.). Scientific Creativity: Its Recognition and Developpment.
N.Y., 1963.
10. Лауэ М. фон. Статьи и речи. М., 1969.
11. Альтов Г.С. Вектор фантазии / Фантастика 73–74, – М.: Молодая Гвардия, 1975, – C.
265–278.
12. Гегель Г.В.Ф. Наука Логики, – СПб.: Наука, 1997.
13. Куликов В.В. Узник бессмертия, – М.: CИНТЕГ, 1998.
14. Куликов В.В., Ёлкин С.В., Гаврилов Д.А. Универсальный межзвёздный язык Диал
как средство научного общения и производства открытий // Труды XXV Чтений,
посвященных разработке научного наследия и развитию идей К.Э. Циолковского. Секция
«К.Э. Циолковский и философские проблемы освоения космоса». Симпозиум «Проблемы
поиска жизни во Вселенной». Калуга, 11–14 сентября 1990. – М.: ИИЕиТ АН СССР. 1991.
15. Гаврилов Д.А., Ёлкин С.В. Язык Диал как средство междисциплинарного общения
и интенсификации изобретательской деятельности // Ильенковские чтения. Международная
научная конференция 18–20 февраля 1999. Тезисы докладов под ред. докт. филос. наук.
проф. Лобастова Г.В. – М. – Зеленоград: Московский институт делового администрирования,
Московский комитет образования, 1999. – С. 303–307.
16. Латыпов Н.Н., Бейлин В.А., Верешков Г.М. Вакуум, элементарные частицы и
Вселенная: В поисках физических и философских концепций XXI в. – М.: «Издательство
МГУ», 2001.
17. Куликов В.В., Гаврилов Д.А., Елкин С.В. Универсальный искусственный язык –
«hOOM-Диал». Методические указания для изучающих язык. – М.: Гэлэкси Нэйшн, 1994.
18. Ильенков Э.В., Философия и культура., – М.: Изд-во полит. лит-ры, 1991.
19. Хайдаров Ж.С., Латыпов Н.Н., Гаврилов Д.А. Св. RU № 6337 UI, «Настольная
игра», 6 A 63 F 3/00, заявка № 95113117/20 от 18.07.95, бюл. № 4 от 16.04.98.
20. Гаврилов Д.А., Ёлкин С.В., Латыпов Н.Н., Фрумин С.Н. Старые и новые столбовые
шахматные игры, – М.: CИНТЕГ, 1998. – 99 с.
21. Латыпов Н.Н. Патент № 2109336 на изобретение «Способ погружения пользователя
в виртуальную реальность и устройство для его реализации», С1, 6 G 06 F 19/00, G 06 F
161:00, A 63 G 31/02, A 63 F 9/22, заявка № 95113085/09 от 14 июля 1995 г.
22. Латыпов Н.Н. Латыпов Н.Н. Патент № 2106695 на изобретение «Способ
представления виртуального пространства пользователю и система для осуществления
способа», С1, 6 G 09 B 19/10, заявка № 96116415/12 от 14 августа 1996 г.
23. Латыпов Н.Н. Патент № 2109337 на изобретение «Способ погружения пользователя
в виртуальную реальность и устройство для его реализации», С1, 6 G 06 F 19/00, G 06 F
161:00, A 63 G 31/02, A 63 F 9/22, заявка № 96109689/09 от 06 мая 1996 г.
24. Лук А. Н. Юмор, остроумие, творчество. – М.: Искусство, 1977.
25. Латыпов Н.Н. Путеводитель по извилинам. Тренинг интеллекта. – М.: Вече, 2010.
26. Латыпов Н.Н., Гаврилов Д.А., Ёлкин С.В. Турбулентное мышление. Зарядка для
интеллекта. – М.: АСТ, 2013.
27. Латыпов Н.Н., Ёлкин С.В., Гаврилов Д.А. Самоучитель игры на извилинах. – М.:
АСТ, 2012.
28. Латыпов Н.Н., Ёлкин С.В., Гаврилов Д.А. Инженерная эвристика. – М.: Астрель,
2012.
29. Латыпов Н.Н. Основы интеллектуального тренинга. Минута на размышление. –
СПб: Питер, 2005.
30. Фритт Крис. Мозг и душа. – М.: Астрель, 2010.
Источники задач, приводимых в книге:
1. Афонькин С.Ю. Учимся мыслить логически: Увлекательные задачи для развития
логического мышления. – СПб.: Литера, 2002.
2. Береславский Л.Я. Азбука логики: Как помочь ребёнку учиться легко и с
удовольствием. – М.: Астрель, АСТ, 2001.
3. Варга Б., Димень Ю., Лопариц Э. Язык, музыка, математика. – М.: Мир, 1981.
4. Гетманова А.Д. Занимательная логика для школьников: Ч.1. – М.: Владос, 1998.
5. Ёлкин С.В., Гаврилов Д.А. Инженерно-техническое творчество в нефтегазовой
отрасли. Избранные лекции курса и сборник задач. – М.: Центр стратегической
конъюнктуры, 2014.
6. Злотин Б. Л., Зусман А. В. Месяц под звёздами фантазии: школа развития
творческого воображения. – Кишинёв: Лумина, 1988.
7. Копытов Н.А. Задачи на развитие логики: Введение в язык математики. – М.: АСТПРЕСС, 1998.
8. Кордемский Б.А. Математическая смекалка. – М.: ГИ Физ-м литер. 1958.
9. Кордемский Б.А. Увлечь школьников математикой. – М.: Просвещение, 1981.
10. Кралина М.В. Логика. – Екатеринбург: У-Фактория, 2002.
11. Кэрролл Л. Логическая игра. – М.: Наука, 1991.
12. Лихтарников Л.М. Занимательные логические задачи. – СПб.: Лань, МИК, 1996.
13. Олехник С.Н. Нестеренко Ю.В. Потапов М.К. Старинные занимательные задачи, –
М.: Главная ред. Физ. – Мат. лит., 1988. – 160 с.
14. Сергеев И.Н., Олехник С.Н., Гашков С.Б. Примени математику. – М.: Наука, 1989.
15. Синицына Е.И. Логические игры и загадки. – М.: ЮНВЕС, 2000.
16. Смаллиан Р.М. Принцесса или тигр? – М.: Мир, 1985.
17. Смаллиан Р.М. Алиса в Стране Смекалки. – М.: Мир, 1987.
18. Сухин И.Г. 800 новых логических и математических головоломок. – СПб.: Союз,
2001.
19. Тихомирова Л.Ф., Басов А.В. Развитие логического мышления детей. – Ярославль:
Академия развития, 1996.
20. Чистяков В.Д. Сборник старинных задач по элементарной математике с
историческими экскурсами и подробными решениями. Минск, изд-во Минобр БССР, 1962.
Рекомендуемая дополнительно литература:
1. Альтов Г.С. И тут появился изобретатель, – М.: Детская литература, 1984.
2. Альтшуллер Г.С. Найти идею. Введение в теорию решения изобретательских задач. –
Петрозаводск: Скандинавия, 2003.
3. Альтшуллер Г.С. Алгоритм изобретения, – М.: Московский рабочий, 1973.
4. Альтшуллер Г.С. Творчество как точная наука, – М.: Советское радио, 1979.
5. Альтшуллер, Вёрткин, 1994 – Альтшуллер Г.С., Верткин И.М. Как стать гением:
Жизненная стратегия творческой личности. – Мн.: Беларусь, 1994.
6. Альтшуллер Г.С. Селюцкий Б.С. Крылья для Икара, Петрозаводск: Карелия, 1980.
7. Боно Э. Серьёзное творческое мышление. Пер с англ. Д.Я. Онацкая. – Мн.: Попурри,
2005.
8. Вертгеймер М. Продуктивное мышление, – М.: Прогресс, 1987.
9. Гаврилов Д.А. Трюкач, Лицедей, Игрок. Образ Трикстера в евроазиатском
фольклоре. – М.: Ганга, ИЦ «Слава!», 2009.
10. Гаврилов. Д.А. Трикстер. Лицедей в евроазиатском фольклоре. – М.: Социальнополитическая мысль, 2006.
11. Губерман И.М. Чудеса и трагедии чёрного ящика. – М.: Детская литература, 1969.
12. Ёлкин С.В., Гаврилов Д.А. Инженерно-техническое творчество в нефтегазовой
отрасли. Избранные лекции курса и сборник задач. – М.: Центр стратегической
конъюнктуры, 2014.
13. Иванов И.С. Инвентология. Краткий авторский курс. – М.: Галлея-Принт, 2010.
14. Ивин А.А. Искусство правильно мыслить, – М.: Просвещение, 1986.
15. Ивич А. Приключения изобретений, – М.: Дет. лит., 1990.
16. Кедров Б.М. О творчестве в науке и технике, – М.: Молодая гвардия, 1987.
17. Кудрявцев А.В. Методы интуитивного поиска технических решений (методы
анализа проблем и поиска решений в технике). – М.: Речной транспорт, 1991
18. Кэрролл Льюис. История с узелками. Перевод с английского Ю.А. Данилова. Под
редакцией Я.А. Смородинского. – М.: Мир. Редакция научно-популярной и научнофантастической литературы, 1973.
19. Микалко Майкл. Тренинг интеллекта. – СПб.: Питер, 2001.
20. Практический интеллект (под ред. Р.Дж. Стернберга). – СПб.: Питер, 2002.
21. Петров В. М. Основы теории решения изобретательских задач. Учебное пособие.
Издание 2-ое. Тель-Авив, 2002.
22. Саламатов Ю.П. Как стать изобретателем. 50 часов творчества. Книга для учителя. –
М: Просвещение, 1996.
23. Чуковский, 1970 – Чуковский К.И. От двух до пяти. – Л.: Детская литература,
1970. – Издание 21-е, исправленное и дополненное.
Об авторе
Латыпов Нурали Нурисламович. Родился в 1954 году в г. Маргилан Ферганской
области, закончил биологический и физический факультеты Ростовского государственного
университета, аспирантуру кафедры философии Московского Государственного
Университета им. М.В. Ломоносова, кандидат философских наук. Диссертация на тему
«Закономерности математизации науки».
Известен большинству граждан бывшего СССР как успешный игрок команды Знатоков
телевикторины «Что? Где? Когда?» (1980-х гг.), обладатель первой в истории Клуба
Знатоков «Хрустальной Совы». Автор креативной графики, иллюстратор (лауреат
международных конкурсов).
Работал политическим обозревателем при ЦК ВЛКСМ, объездил многие горячие точки
в качестве военного корреспондента. Входил в Инновационный Совет при премьер-министре
России И. Силаеве, работал советником по межрегиональным проблемам вице-премьера
Правительства РФ С. Шахрая, советником по вопросам инновационной политики – Мэра
Москвы Ю. Лужкова. Автор и ведущий познавательного цикла телепередач «Мнения
Знатоков» (ТВ-Столица, совместно с А. Вассерманом) (2008–2010). Авт. свид. на изобрет.
в области проведения интерактивных конкурсов для удалённых пользователей. В группе
Советников-экспертов ООО «ЛУКОЙЛ-Инжиниринг» (с февраля 2011).
Избранные книги:
Латыпов Н.Н., Бейлин В.А., Верешков Г.М. Вакуум, элементарные частицы и
Вселенная: В поисках физических и философских концепций XXI в. – М: Изд-во МГУ,
2001. – 232 с.
Латыпов Н.Н. Основы интеллектуального тренинга. Минута на размышление – СПб:
Питер, 2005. – 336 с.
Латыпов Н.Н. Философия в этюдах. – М: Московские учебники и картолитография,
2007. – 120 с.
Латыпов Н.Н. Зазеркалье. – М: Московские учебники и картолитография, 2010. – 156 с.
Латыпов Н.Н. Путеводитель по извилинам. Тренинг интеллекта. – М: Вече, 2010. – 408
с.
Вассерман А.А., Латыпов Н.Н. Самые интересные факты, люди и казусы всемирной
истории, отобранные знатоками. – М.: Астрель: Полиграфиздат, 2012. – 384 с.
Вассерман А.А., Латыпов Н.Н. Реакция Вассермана и Латыпова на мифы, легенды и
другие шутки истории. – М.: Астрель: Полиграфиздат, 2012. – 352 с.
Латыпов Н.Н., Вассерман А.А. Острая стратегическая недостаточность. Страна на
ПереПутье. – М.: Астрель: Полиграфиздат, 2012. – 448 с.
Латыпов Н.Н., Ёлкин С.В., Гаврилов Д.А. Инженерная эвристика. – М.: Астрель,
2012. – 320 с.
Латыпов Н.Н., Ёлкин С.В., Гаврилов Д.А. Самоучитель игры на извилинах. – М.: АСТ,
2012. – 320 с.
Вассерман А.А., Латыпов Н.Н. Монологи эпохи. Факты и факты. – М.: АСТ, 2013. – 352
с.
Латыпов Н.Н., Гаврилов Д.А., Ёлкин С.В. Турбулентное мышление. Зарядка для
интеллекта. – М.: АСТ, 2013. – 352 с.
Примечания
1 Мыслитель опирается правым локтем на левое колено.
2 Учитывая название игры («Колумбово яйцо») и подсказку (яйца – трёхмерные
материальные объекты неправильной шарообразной формы), легко понять: первый игрок для
победы должен поместить яйцо «точно в центр». Этого можно добиться, например,
осторожно надколов скорлупу – как это, по легенде, сделал Колумб, чтобы установить яйцо
вертикально. После этого можно отвечать противнику симметричными ходами. При этой
стратегии всегда будет возможность поставить ещё одно яйцо максимально близко к
центральному: если противник нашёл для этого место – значит, для Вас осталось свободное
место, симметричное найденному им.
3 Прямая теорема Пифагора: в прямоугольном треугольнике сумма квадратов двух
меньших сторон равна квадрату большей. Соответственно одна из обратных теорем
выглядит так: если в треугольнике сумма квадратов двух меньших сторон равна квадрату
большей, то треугольник прямоугольный (и прямой угол находится напротив большей
стороны).
4 Где мы есть – там плохо.
5 У каждой перчатки – две поверхности. Значит, у двух пар есть четыре поверхности,
так что на каждого из троих пациентов и на самого врача найдётся одна поверхность. Чистые
поверхности могут соприкасаться между собой, не загрязняясь. Да и грязные могут
соприкасаться между собой: им-то – в отличие от пациентов – хуже не будет. Врач сперва
надел одни перчатки на другие и обследовал одного пациента. Затем снял внешние перчатки,
при этом выворачивая их наизнанку, и во внутренних перчатках обследовал второго.
Наконец надел ранее снятые перчатки на те, что уже на нём, чистой стороной наружу и
обследовал третьего. Итак, врача касалась одна и та же поверхность перчаток, а каждого из
троих пациентов – чистая поверхность. Метод «от противного» тут проявился в возможности
соприкосновения грязных поверхностей.
6 Этот день – воскресенье. Проверьте, что будет послезавтра и после-послезавтра, а
также позавчера и поза-позавчера.
7 Шпицберген. Дословно – острые горы. Этот северный остров, ныне принадлежащий
Норвегии, содержит немалые залежи угля, по международному договору уже немногим
менее века разрабатываемые нашими шахтёрами.
8 Сочетание букв «орох» даёт в этой загадке горох, шорох, порох, ворох.
9 Фалес поймал момент, когда тень палки стала по длине равна самой палке. Скорее
всего, для этого он взял две равные палки, одну поставил вертикально, а другую положил
наземь и ждал, пока тень сравняется с нею. В этот момент он измерил длину тени пирамиды.
Очевидно, она равна высоте самой пирамиды. Правда, часть тени скрыта основанием
пирамиды – но его-то измерить (и добавить к длине тени половину размера) несложно!
Сходное наблюдение можно провести в любое другое время дня, измеряя соотношение
между высотой палки и длиной её тени. Но проще поймать момент, когда они равны, и не
возиться с расчётами пропорций.
10 Синтезатору предложили создать второй такой же синтезатор. Второму – третий и
так далее. Когда накопилось нужное число синтезаторов, каждому приказали сделать одну
деталь.
11 Поверхность у него велика по сравнению с массой. Поэтому сопротивление воздуха
быстро поглотит всю энергию, приданную Вашим броском. Сходные причины когда-то
заставили перейти от камней к стрелам, а потом от круглых пуль и снарядов к
продолговатым, стабилизированным вращением или оперением.
12 Он разделил год на 14 месяцев. Ведь число месяцев и продолжительность каждого
из них были предметами указов других императоров.
13 Друзья демонстративно «отравились», выпив остатки воды. После чего лодка была
вынуждена, горюя и сетуя, выбросить два «трупа» в окружающую воду.
14 Тара для стёкол делается прозрачной. Кстати, в США придумали делать и
прозрачный бинт – его не нужно каждый раз снимать, чтобы рассмотреть рану.
15 Обмакните монету в пиво и «приклейте» к внешней поверхности кружки
(цилиндрической!). Монета съедет по поверхности кружки до стола и останется так стоять.
16 Электрическую батарейку.
17 Первый бегун преодолевает большее сопротивление воздуха. За его спиной
создаётся разрежение. Это облегчает бег и позволяет сэкономить силы второму бегуну.
18 Левша держит карты в правой руке и сдаёт их левой. Так ему удобнее увидеть
несимметричную маркировку карт в нижнем и верхнем углах.
19 Высочайшей планетой называли в то время Сатурн: более отдалённые от Солнца
планеты ещё не были открыты. Галилей увидел кольца Сатурна, но в таком ракурсе, что не
смог их разделить в свой телескоп (и поэтому счёл большими спутниками, сопоставимыми
по размеру с самим Сатурном). Это сделали позднее другие астрономы. Гюйгенс открыл
спутник Сатурна Титан, где, возможно, спутник НАСА обнаружил удивительные бактерии,
«дышащие» водородом.
20 У кошек, как говорят, 9 жизней. 8 = 1 ′ 9–1, 70 = 8 ′ 9–2, 627 = 70 ′ 9–3 и т. д.
21 Когда обвиняемый думает о себе, он может рассуждать так: «Пусть второй
обвиняемый признается. Тогда, если я тоже признаюсь, то получу лёгкое наказание, а если не
признаюсь, буду сурово наказан. Пусть второй обвиняемый не признается. Тогда меня
освободят. В каждом случае лучше признаться». Но так рассуждая, оба обвиняемых
признают свою вину. В результате они оба получат наказание, хотя и лёгкое. А могли бы
быть освобождены, если бы оба не признали своей вины.
22 Можно сослаться на Я.И. Перельмана. Песчинки, не касаясь во время падения дна
сосуда, не оказывают на него давления. Можно думать поэтому, что в течение тех пяти
минут, пока длится пересыпание песка, чашка с часами должна быть легче и подняться
вверх. Опыт покажет, однако, другое. Чашка с часами качнётся вверх только в первое
мгновение, но затем в течение пяти минут весы будут сохранять равновесие до последнего
момента, когда чашка с часами качнётся вниз и весы снова придут в равновесие. Почему же
весы останутся пять минут в равновесии, несмотря на то, что часть песка, падая, не
оказывает на дно сосуда никакого давления? Прежде всего отметим: в течение каждой
секунды столько же песчинок покидает шейку часов, столько их достигает дна. Значит,
каждую секунду становятся «невесомыми» столько же песчинок, сколько ударяются о дно
сосуда. Каждой песчинке, делающейся невесомой, отвечает удар другой песчинки о дно.
Только в первый и последний моменты пятиминутного промежутка времени равновесие
весов (если они достаточно чувствительны) нарушится. В первый момент потому, что
некоторые песчинки уже покинут верхний сосуд часов, сделаются невесомыми, но ни одна
не успеет ещё удариться в дно нижнего сосуда: чашка с весами качнётся вверх. К концу
пятиминутного промежутка равновесие снова нарушится на мгновение: все песчинки уже
покинули верхний сосуд, новых невесомых песчинок нет, а удары о дно нижнего сосуда ещё
происходят – чашка с часами качнётся вниз. Затем снова наступит равновесие, на этот раз
окончательно. Что же касается сосуда с мухами, то вес сосуда в обоих случаях будет
одинаков – при полёте крылья мух оказывают на воздух давление, равное их весу.
23 Простой ответ (не вдаваясь в тонкости) гласит: обе обезьяны достигнут блока
одновременно, поскольку натяжения каната (а, значит, и ускорения, и скорости обезьян)
одинаковы с обеих сторон. В последнем случае раньше доберётся до блока более лёгкая
обезьяна, потому что её ускорение будет направлено вверх, тогда как более тяжёлой
обезьяны – вниз. Приведу ещё цитату из хорошей книги Я.А. Смородинского: «Как и многие
другие творения Кэрролла, его «обезьянья» задача породила многочисленные дискуссии и
споры. Ей посвящена обширная литература. Потешаясь над своими учёными коллегами –
профессорами физики Клифтоном и Прайсом, профессором химии Верной Харкортом и
лектором колледжа Христовой церкви Оксфордского университета Сэмпсоном, Кэрролл
сделал в своем дневнике следующую запись: «21 декабря, четверг (1893 г.). Получил ответ
профессора Клифтона к задаче «Обезьяна и груз». Весьма любопытно, сколь различных
мнений придерживаются хорошие математики. Прайс утверждает, что груз будет
подниматься с возрастающей скоростью, Клифтон (и Харкорт) считают, что груз будет
подниматься с такой же скоростью, как обезьяна, а Сэмпсон полагает, что груз будет
опускаться». Нашлись и такие, кто считал, что груз останется на месте. Споры по поводу
того, какое решение «обезьяньей» задачи Кэрролла следует считать единственно
правильным, время от времени возникают и поныне. (В действительности условия задачи
недоопределены и ответ зависит от дополнительных предположений, вводимых при решении
задачи.)»
24 Мудрец хотел довести до совершенства хотя бы одну из них!
25 «Змеи и драконы» – это корабли средневековых скандинавских воинов. На
форштевнях они вырезали из дерева головы этих грозных существ и называли корабли
соответственно – шнеккар (шнек – змея) или драккар (драк – дракон). У себя на родине эти
воины звались викингами (вик – боевой поход под руководством местных вождей). У нас их
чаще называли варягами – наёмниками: в голодной и нищей Скандинавии (как несколькими
веками позже – в такой же голодной и нищей Швейцарии) прокормиться было нечем, кроме
разве что рыбы, и те, кто хотел стабильной жизни, нанимались в войска всей Европы.
26 Металл при нагревании расширяется равномерно. Значит, форма «бублика» не
меняется. В частности, не меняется соотношение характерных размеров. Поскольку
увеличивается внешний диаметр, то увеличивается и внутренний, отверстие в «бублике»
становится больше.
27 Сложите кирпичи так: два кирпича один на другой и рядом с ними ещё один,
вплотную. «Пустота» над ним как раз равна кирпичу по объёму. Остается измерить линейкой
или рулеткой её диагональ.
28 Описанному условию соответствует любая кривая на сфере. Окружность – частный
случай: кривая, образованная пересечением сферы с плоскостью.
29 Ясно, что возраст и детей, и математика должен выражаться натуральными числами.
Причём ответ должен быть однозначным! Все числа от 33 (возраст Остапа Бендера) до 60
неоднозначно раскладываются на множители. Или сумма множителей больше 31 – числа
дней в месяце. Или содержат два одинаковых множителя – этот вариант не подходит, так как
есть средний сын! Остаётся единственный вариант: 1, 5, 8.
30 Ответ с точки зрения робота: «… И неземляне, и люди, когда у них спрашивают
пароль, всегда отвечают – либо правильно, либо неправильно… Я должен сделать вывод, что
любое существо, которое мне не отвечает, вообще не способно отвечать и можно на него не
обращать внимания… И на это большое животное, ползущее мимо меня, тоже можно не
обращать внимания…»
31 Драконовский – по имени Драконта (или Дракона).
32 Кардиган – вязаный жакет на пуговицах, без воротника и лацканов. Лорд Кардиган –
командующий лёгкой кавалерией Джеймс Томас Брюднелл – первым надел под свой мундир
жакет, позднее названный его именем.
33 Платоническое чувство – по имени Платона.
34 Лукулловы пиры, по имени Луция Лициния Лукулла.
35 Макиавеллиевский – в честь Никколо Макиавелли.
36 Задача расчётная. Посчитайте, сколько горячей и холодной воды (по объёму) втекает
в ванну (заданного объёма) за минуту, какой объём воды вытекает из ванны за минуту (если
считать, что скорость стока не зависит от уровня воды в ванне – например, вода сильно
тормозится узкой трубой) и составьте простое уравнение. В данном случае холодный кран в
минуту наливает 1/10 ванны, горячий – 1/8, то есть в сумме 9/40 ванны в минуту. Через слив
в минуту вытекает 1/5 ванны. Итого остаётся 1/40. Ванна заполнится за 40 минут.
37 63:36 = 6.
38…научиться кормить друг друга!
39 Эдисону «помогал» закон Архимеда. Величина объёма колбы входит в выражение
для выталкивающей – архимедовой – силы. А её можно измерить экспериментально.
40 Это всего лишь количество ударов классических часов с боем. 1 – удар, отмечающий
полчаса. Так что потом будут 1, 4, 1, 5 и т. д.
41 Это слово – инерция. От «in» – отрицание, «ars» – искусство, способность,
движущая сила.
42 Упившись как следует, купец мог произнести лишь некое мычание вроде «мум».
Официантам этого было достаточно, чтобы понять, чего хочет клиент.
43 Яма.
44 Трубу нужно прикрепить к днищу лодки. Тогда сила Архимеда уменьшит осадку
лодки, и она сможет двигаться по воде вместе с пассажиром.
45 Вода просачивается через капилляры в стенках глиняных сосудов и испаряется.
Испарение требует тепла, так что остаток воды охлаждается. Для ускорения испарения рабы
непрерывно обмахивали сосуды с водой опахалами.
46 Сын положил монету в 5 рублей под стол! Тем самым она оказалась под монетой в
10 рублей.
47… будто всё это написал он.
48…на воротах надпись «Приама нет».
49 Это – парик! У Людовика была заметная лысина. А из Германии привозили парики с
белокурыми волосами.
50 «…Они даже коврики для мышей делают!»
51 N+3, N+4, N+5 рублей легко получить, так как эти суммы на 3 отличаются от N,
N+1, N+2. Следовательно, так как проведены все шаги индукции, любое количество денег
может быть выдано имеющимися купюрами.
52 Потребности автомобилестроения и железнодорожного транспорта привели к
бурному строительству дорог, разработке залежей каменного угля, нефти, строительству
нефтеперерабатывающих заводов и т. д. Вся эта деятельность связана с разработкой недр и
активным строительством. По сути, повсеместно велись раскопки. Побочный результат: за
эти годы количество палеонтологических открытий заметно – в разы – возросло.
53 Спираль ДНК разделяется на отдельные маленькие спиральки, раскручиваемые
независимо. Затем каждый из кусочков подсоединяет к себе нужные элементы. В конце
процесса отдельные уже удвоенные кусочки вновь собираются в одну большую спираль.
54 В указанных словосочетаниях лишнее – продукты питания, все остальные
словосочетания – тавтологии.
55 Нужно сделать в трубке конусообразное отверстие с гибкими краями. Как только
мышь заберётся внутрь за приманкой, края распрямятся и не выпустят её наружу.
56 Законы геометрической оптики напоминают: в линзе (двояковыпуклой) получается
при определенных условиях перевёрнутое изображение. Именно такое изображение и даёт
круглая пробирка с водой. Слово «кофе» имеет горизонтальную ось симметрии, потому и
выглядит неизменным, в отличие от слова «чай». Аквариум такого эффекта не создаст, т. к.
не является линзой нужной формы.
57 Знаменитый писатель каждый день обедал в ресторане на первом этаже Эйфелевой
башни!
58 Ну, а если в дневнике стоял «кол»? Или написано, что его вообще выгоняют из
школы? Или что-то «такое» в нём написали приятели Вовочки? Варианты возможны.
59 Увидел инженер паутину на ветвях дерева. И придумал конструкцию висячего
моста.
60 Дидона разрезала острым ножом шкуру на множество тонких ремешков, связала их
и смогла отгородить весьма заметную площадь под строительство.
61 Известный ответ задачи: 4 мм. Вы присмотритесь, как стоят на полке рядом первый
и второй тома! Правда, мой партнёр по многим проектам Анатолий Вассерман ставит тома
справа налево – как если бы они составляли одну очень толстую книгу. Для его домашней
библиотеки верна как раз формулировка, использованная в журнале «Успехи физических
наук».
62 Мишек перевязали и сказали: бедные мишки болеют, поэтому их надо лечить! И их
дети оставляли в больнице – «лечиться».
63 Х приказал сделать на вилках гравировку «Украдено у Х». И пропажи прекратились.
64 Дробь.
65 В колодце.
66 Тень.
67 Ум. Правда, есть-то он у всех, но в разных дозах.
68 Прежде чем научить тебя говорить, мне придётся научить тебя молчать!
69 Четыре весельчака – просто музыкальный коллектив, выиграли же они свой гонорар.
70 Царь Пётр очень любил шахматы, играя в них, как и описано, везде.
71 Изобретение «думающих машин» высмеял (и напрасно!) Джонатан Свифт в
«Путешествиях Гулливера».
72 Екатерина указала носить белые перчатки и лорнеты будочникам – уличным
полицейским. Кто же из молодых дворян станет походить одеждой на представителей
низшего сословия?!
73 С самого верха башни капает через стальное сито расплавленный свинец. По пути
вниз капли успевают застыть правильными шариками. Остаётся отсортировать их по размеру
– и получится охотничья дробь.
74 Это знаменитая гравюра «Меланхолия» Альбрехта Дюрера.
75 Это микроволновая печь. В ней энергию электромагнитных волн поглощают и
преобразуют в тепло молекулы воды (жидкости), имеющиеся в любом продукте.
76 Упомянутые машины – различные типы «универсальных круговых самоходных
посадочных комбайнов», только последний из них – с «радиоуправлением». Почти
сельскохозяйственные роботы. Пока что таких нет. А жаль!
77 Труболёт – длинная труба на четырёх роликах, принцип движения – реактивный.
Сиденья расположены на самой трубе, поэтому летом они сильно нагреваются, а вот зимой
на них сидеть даже приятно.
78 Преподаватель отвечает: «Возьмите 3 тысячи сдачи».
79 Доктор, исследуя длинные подолы платьев, обнаружил на них огромное количество
болезнетворных микробов, собираемых при ходьбе.
80 Великий Ферма ещё в XVII веке утверждал (но не привёл своего доказательства),
что некое равенство невозможно при определённых условиях (xn+yn=zn при n, начиная с 3).
Доказательство в общем виде для всех показателей степени (весьма сложное) получил лишь
в середине 1990-х годов Э.Уайлс. Математика – одна из немногих наук, где утверждения о
невозможности удаётся строго доказать.
81 Пусть два монаха одновременно пускаются в путь по этой тропе – один вверх,
второй вниз. Конечно, в какой-то точке пути они встретятся. Вот эта точка и есть искомая (не
забывайте: монахи вышли в путь одновременно).
82 Мистическая идея состояла в том, что отпечаток твоего пальца значит больше, чем
сделанная рукой подпись. Разглядывая оттиски пальцев, Гершель понял (впрочем, он не был
первым – задолго до него об этом знали в Древнем Китае): папиллярные узоры на пальцах
людей не повторяются. Так что У. Гершель сделал много для возникновения серьезной науки
– дактилоскопии.
83 Разделение на 82 квадрата соответствует 82 клавишам (тонам) рояля. Каждый
участок сканируется звуковым лучом соответствующей частоты. Услышав звук, девочка
сразу его идентифицирует с определённой нотой (абсолютный слух!). Так установили место,
где она находится.
84 Зевс потребовал, чтобы Прометей вечно носил с собой кусочек скалы, окованный
цепью. Так возникли кольца с камнем – и до сих пор не вышли из моды.
85 Второй водитель так же спокойно говорит первому: «Сэр, когда дочитаете газету,
дайте и мне её почитать».
86 Джентльмен хочет выяснить: кто же бросает детей в воду?
87 Вот тут ответ зависит от Вашего вкуса: может быть голова, а может быть и книга.
88 К свежеевылупившимся цыплятам подсаживают более взрослых, у которых рефлекс
уже присутствует. Прочие устремляются к корму по примеру старших.
89 Лаборант Н. имел неплохой и весьма низкий голос, а, находясь в одиночестве, пел.
Но вот беда, слуха у него не было, и, зная об этом, лаборант вёл себя тихо в присутствии
посторонних, стесняясь этого своего недостатка. Оказалось, что акустические волны
существенно влияли на ход проводимой реакции.
90 В трубу до самого дня опустили верёвку, с некоторым избытком. Затем залили воду,
с тем, чтобы она покрывала суммарную длину гвоздя, гайки и шарика. Снаружи трубу
охладили жидким азотом и заморозили воду, а, вместе с ней все три предмета и конец
верёвки. Затем подождали, когда начнётся таяние, и вытащили за верёвку кусок льда с
вмороженными в него предметами. Оставшаяся в трубе вода постепенно испарилась.
Аппарат заработал, как новый.
91 Если кто полагает, что у этой задачи единственное решение, или хотя бы два, тот
заблуждается. Довольно очевидно, что надо один из стаканчиков поставить в другой…
92 Тот, кто выше ростом, – мать того, кто пониже.
93 Когда вода превратится в лёд. Впрочем, если до зимы далеко, можно аккуратно
натереть решето жиром или воском. Вода от них отталкивается, и над сеткой удержится слой
в пару сантиметров (точная высота зависит от размера ячеек).
94 375 г = 3/8 кг. Если знаменатель – степень двойки, всё можно решить
последовательными делениями пополам. Разделить килограммовый кусок сливочного масла
пополам, так, чтобы уравновесить правую и левую чаши весов (0.5 и 0.5 кг). Одну половину
убрать, другую вновь разделить пополам (0.25 и 0.25 кг). Повторить процедуру. Отделённую
часть в 0.125 кг на одной чашке соединить с ранее отложенными 0.25 кг.
95 Лишь один раз – первый. Любое следующее вычитание будет производиться из
меньшего числа.
96 Человек сидит у Вас на коленях.
97 Путников было трое: дед, отец и сын. Дед – отец отцу, отец – сын деду. Так и
набирается два отца и два сына.
98 Премьер-министр вытащил листок бумаги и, не глядя на него, скатал из него шарик
– и проглотил. Поскольку на оставшемся листке стояло «Уходите», то королю пришлось
признать: на проглоченном листке значилось «Останьтесь».
99 «Дурачок» был не таким уж дурачком: он понимал, что, пока он будет выбирать 50центовую монету, люди будут предлагать ему деньги на выбор, и его удивительное решение
будет вызывать интерес у всё новых «клиентов», если же он выберет пятидолларовую
купюру, предложения денег быстро прекратятся, и он тут же прекратит получать стабильный
«доход».
100 Поменяться верблюдами.
101 Если будете ехать некоторое время на постоянном расстоянии от впереди идущей
машины, то Ваш спидометр покажет скорость этой машины, совпадающую со скоростью
Вашего автомобиля.
102 Надо выйти из плоскости в пространство. Окружность вычерчивается: на
плоскости, на шаре, на конусе (центр в вершине конуса), кроме того, на плоскость можно
положить шашку произвольной высоты и утвердить на ней ножку циркуля.
103 Нельзя уже хотя бы потому, что происходит насыщение раствора. А даже если
искусственно повышать растворимость сахара (подогревая воду), делая раствор
пересыщенным, у любого стакана есть края (чай перельётся), как у любого насыщения –
предел.
104 Лопатой сгрести смесь в котелок. Залить водой, хорошенько перемешать.
Поваренная соль растворится, а песок пойдёт ко дну. Отстоять взвесь, чтобы песок осел на
дно. Слить раствор соли в другой котелок и выпарить воду на костре.
105 Пыль рассеивает свет беспорядочно во все стороны. Гладкая же поверхность
отражает в одном направлении – пятна света и воспринимаются как блеск.
106 Чтобы слушатель на земле отчётливо слышал звук, речь и пение, этот звук должен
падать на слушателя под углом от 45 до 60 градусов. Значит, чем выше громкоговоритель,
тем дальше от него должен находиться слушатель. То есть звук охватит больше площади, но
в центре её будет плохо слышно. Вдобавок, чем дальше слушатель, тем мощнее должен быть
громкоговоритель – а ведь энергию для его работы надо вложить в передаваемый со станции
сигнал.
107 Обрыв бумаги фиксируют по изменению напряжённости электростатического поля.
Трение бумаги при движении о части механизированной линии ещё никто не отменял, оно и
порождает статическое электричество.
108 Внутри круга радиусом десять метров собака может блуждать часами, проходя
многие километры. Кроме того, предмет, к которому она привязана, сам может быть
подвижным.
109 Опять же возможны варианты. Например, чашка может быть заполнена зёрнами
кофе. А некоторые современные серьги бывают и повыше чашек: один конец упрётся в дно,
а другой торчит на поверхности.
110 Всего три: собака, кошка и попугай.
111 Конечно, нет: через 72 часа после полуночи тоже будет полночь.
112 Якорь.
113 21:00.
114 Легавая собака (и охотник). Отсюда и небезызвестное оскорбление работников
милиции.
115 Почтовая марка того времени была сплошь чёрной, поэтому определить, погашена
она или нет, было очень трудно. Это позволяло людям использовать марки повторно. На
«красном пенни» штемпель гашения виден чётко.
116 В обоих случаях короли спали только одну ночь. Во Франции в 1582-м г., а в
Англии в 1752-м г. проведена замена календарей. До этих дат в соответствующих странах
использовался юлианский календарь, в котором накапливалась ошибка, и её нужно было
исправить, В 1582-м г. папа Григорий XIII приказал ввести новый календарь с меньшей
погрешностью (его назвали григорианским) и для согласования календарных дат с
астрономическими событиями исключить из текущего года 10 дней. Католические страны –
в том числе Франция – приняли поправки сразу, а протестантские, к которым относилась
Англия, долго обсуждали вопрос. Англия перешла на более точный григорианский
календарь только в 1752-м г., когда требуемая поправка составляла уже 11 дней.
117 Каину, поскольку кроме его родителей и брата в мире в тот момент никого из
людей не было.
118 Число зафиксированных ранений в голову увеличилось, но снизилась смертность.
Ранее, если осколок шрапнели попадал солдату в голову, он пробивал фуражку, и человек,
вероятнее всего умирал. Это фиксировалось как смерть, а не как ранение. После того, как
было предписано носить каски, удар шрапнели не убивал солдата, а всего лишь ранил.
Поэтому число ранений в голову увеличилось, а число смертей снизилось.
119 Он сварил яйцо, приравняв к массе яйца массу воды и опущенной в неё соли.
120 Того, кто успешно совершил самоубийство, уже не наказать.
121 Бормашина.
122 Король Георг IV изобрёл правый и левый сапог. До этого вся обувь могла
одеваться на любую ногу.
123 Может. Например, если человеку 50 лет, а его внуку или внучке 1 месяц.
124 Сферические и овальные яйца катились бы по прямой. Асимметричные же яйца, у
которых один конец тупее, а другой острее, при скатывании стремятся катиться по кругу.
Если яйцо лежит на краю обрыва или в другом ненадёжном месте, стремление катиться по
кругу, а не по прямой – большое преимущество.
125 Как бы точно ни была изготовлена гиря, всё же фактически вес её неизбежно
несколько отличается от официально обозначенного. Чтобы избежать этой неточности, гирю
намеренно делают несколько тяжелее требуемого веса. А затем от незначительного лишнего
веса избавляются спиливанием небольшого количества вкрапленного в гирю мягкого
металла – Меди или свинца.
126 Покойный сам не мог перемотать плёнку диктофона.
127 Крышка квадратного или прямоугольного люка может провалиться в люк, так как
длина диагонали люка больше длины стороны крышки. Крышка же круглого люка
провалиться не может, как её ни поверни. Поэтому круглые люки удобнее и безопаснее
квадратных.
128 Миша увидел, что у Пети грязное лицо и подумал, что у него также лицо грязное.
Петя, увидев чистое лицо Миши, подумал, что с его лицом также всё в порядке.
129 Это возможно, если двигаться с определённой скоростью (на скоростном
автомобиле или самолёте) в направлении, противоположном движению Земли. Чем ближе
человек находится к полюсу Земли, тем меньше требуется скорость, для того чтобы обогнать
Землю в её движении. Человек может перегнать Землю даже пешком, находясь от полюса на
расстоянии до 50 км. Идя на запад навстречу садящемуся Солнцу, он будет наблюдать
своеобразный восход Солнца.
130 Встряска содержимого бутылки приводит к тому, что из сильно газированной воды
выделяется растворённый углекислый газ. Давление внутри бутылки заметно возрастает (при
этом бутылка становится более упругой), крышка в свою очередь давит на удерживающую
её винтовую нарезку у горлышка бутылки. Трение крышки о нарезку пропорционально
этому давлению.
131 Удочку необходимо расположить по диагонали коробки длиной 4 фута и шириной
3 фута.
132 Вначале переправляются оба сына. Один из сыновей возвращается обратно к отцу.
Отец перебирается на противоположный берег к сыну. Отец остаётся на этом берегу, а сын
оттуда переправляется на исходный берег за братом, после чего они оба переправляются к
отцу.
133 Обычно предлагают такое решение: наполнить литровую банку; вылить её
содержимое в двухлитровую банку; наполнить литровую банку из трёхлитровой банки.
Однако можно разлить сок и быстрее: наполнить двухлитровую банку; наполнить из неё
литровую банку.
134 «Вы находитесь в своём городе?» – ответ «да» всегда означает, что Вы в городе
честных, кто бы Вам ни попался.
135 Тот, кто лучше воспитан и менее спесив.
136 Этиловый можно, метиловый нельзя. Оба спирта в организме преобразуются в
альдегиды – очень агрессивные соединения. Но этиловый спирт входит в цикл
внутриклеточного энергообмена. Поэтому есть фермент, быстро разлагающий этиловый
альдегид в менее опасные вещества. А вот для метилового альдегида нет подобного
переработчика, и сильный яд накапливается, разрушая прежде всего нервные клетки (даже
лёгкого отравления метиловым спиртом достаточно для слепоты), а затем убивая и прочие.
137 Заткнуть пальцем отверстие для иглы у шприца. Высыпать порошок в шприц.
Теплом рук сделать из снега воду и залить её в шприц. Вставить шток в шприц. Повернуть
шприц штоком вниз, отпустить палец, удалить воздух. Заткнуть пальцем отверстие для иглы,
оттянуть шток вниз. Вода при достаточно малом давлении кипит при соответственно низкой
температуре. Конечно, для обеззараживания этого не хватит – но на такой высоте микробов
почти нет. А для быстрого растворения порошка хватит пузырьков любой температуры.
138 Настоятель сказал, что в его монастыре будут служить обедни по богачу каждый
второй день.
139 Следует заявить: «Меня повесят».
140 Женщина дождалась, чтобы часовой ушёл в свою будку. Потом она проникла на
мост и пошла в сторону швейцарской границы. Она шла около 3-х минут, потом
остановилась, повернулась и пошла обратно, к Германии. Часовой вышел наружу и увидел
её. Когда она подошла к нему, он увидел, что у этой женщины нет разрешения на вход в
Германию, и поэтому направил её назад в Швейцарию.
141 Почтовый голубь.
142 Способ ношения монет был сугубо местным – отправляясь за покупками, фиджиец
клал монеты в рот. Медико-никелевые монеты оказались просто приятнее на вкус.
143 Столбы бросали в воду, чтобы посмотреть, куда их вынесет прибой. На этом месте
высаживались и строили дом – ведь это было, как считалось, указание предков и/или богов.
144 Чтобы показать имаму, что ни одно из его предложений не приемлемо, ходжа
сказал: «Прежде чем попасть сюда, я выспался у источника».
145 Копейка – неделимая единица счёта. Цена любого штучного товара должна быть
целым числом копеек. Число товаров каждого вида, заказанное братом, чётное. Значит, и
общая сумма заказа должна быть чётной. Буфетчица же назвала нечётную сумму.
146 В ту пору в Cеверной Америке в сельской местности ещё широко использовали
гужевой транспорт. В 1923-м Канада – бывшая английская колония, а тогда доминион –
перешла с левостороннего движения, обычного для Британской империи, на правостороннее.
Это не только подчеркнуло её новый – куда менее зависимый от метрополии – статус, но и
упростило перемещения между Канадой и её южным соседом: в США движение стало
правосторонним ещё задолго до появления автомобилей. Однако волы были приучены
двигаться всегда по левой для них обочине. В отличие от лошадей, волы практически не
поддаются переучиванию. Они отказались менять привычную полосу движения. Их
пришлось забивать. А было их так много, что рынок говядины буквально рухнул.
147 У большинства телег передние колёса меньшего диаметра, чем задние: это
упрощает конструкцию их крепления, обеспечивающего поворачивание при изменении
направления движения (во многих вариантах передние колёса на повороте вообще уходят
под телегу). При прохождении одного и того же расстояния меньшие колёса совершают
больше оборотов. Деревянная ось (ступица) и колесо образуют пару, ещё с древнейших
времён используемую для добывания огня трением. Поэтому вероятность загорания
переднего колеса больше, чем заднего.
148 Решение потребовало нескольких ходов рассуждений. Первое суждение: если
щепотка – то чего-то мелкого и твёрдого. Суждение второе: самые ходовые сыпучие
вещества – соль и сахар – отпадают, и для выпаривания той же соли (или сахара)
понадобились бы строения не такой высоты, зато куда более широкие. Значит, высота играет
роль. И тут я вспомнил про Антони ван Левенгука. Он получал линзы для микроскопов,
проливая расплавленное стекло в воду. Оседающие мельчайшие капли благодаря
поверхностному натяжению принимают на лету круглую форму. Правда, сопротивление
воздуха чуть (в пределах оптической точности) деформирует их – но для линз это даже
выгодно, поскольку определённые виды деформаций компенсируют некоторые искажения,
даваемые правильным стеклянным шариком. Суждение четвёртое: высокая башня защищает
искомое от ветра, когда его бросают или льют с вершины башни вниз. Суждение пятое: если
это массовое производство, необходимое разным странам и в разные времена, то это продукт
первой необходимости, но вряд ли еда. Теперь – после всех ограничений – ответ приходит
сам: это свинцовая дробь – она падала сверху в виде расплавленных капель, схватываясь в
воздухе и окончательно остывая при попадании в воду на дне бассейна, причём вода нужна,
чтобы удар при падении не деформировал свинцовый шарик. Решение фактически по
принципу бритвы Оккама: отсекается всё лишнее, а что остаётся, то и верно.
149 В физике есть очень полезный принцип наименьшего действия. В частности, свет
всегда распространяется по пути, требующему наименьшего времени (с учётом различий
скорости движения света в разных плотных средах). Представим себе, что железная дорога –
зеркало. Отразим в ней один из цехов. И соединим прямой, как движение света, линией это
отражение с другим цехом. Точка пересечения линии с железной дорогой укажет, где
расположить завод.
150 Множители содержат все латинские буквы. Значит, один из них имеет вид (x – x).
Очевидно, он равен нулю. Следовательно, равно нулю и всё произведение.
151 Девять монет делим на три равные кучки по три монеты. Первые три монеты
кладём на одну чашу весов, другие три монеты – на другую чашу весов. Если весы попрежнему уравновешены, то среди этих шести монет нет фальшивой. Поэтому снимаем с
весов шесть монет и приступаем к кучке, которую ещё не взвешивали. Берём произвольно из
оставшихся трёх монет две и кладём на ту и другую чашу. Если весы снова находятся в
равновесии, то оставшаяся девятая монета фальшивая. Если не находятся в равновесии, та,
что более лёгкая – фальшивая. Если же весы не находятся в равновесии уже после первого
взвешивания, значит, на одной из чаш среди трёх монет одна – фальшивая. Возьмём из более
лёгкой кучки две монеты и положим на весы. Если весы снова находятся в равновесии, то
оставшаяся монета из предыдущих трёх фальшивая. Если не находятся в равновесии, та, что
более лёгкая – фальшивая.
152 Говорят, что нижеследующее решение найдено перебором всех возможных
вариантов. Их довольно много, но всё же можно все их просмотреть без помощи
компьютера. Итак, прежде всего пронумеруем монеты. Для этого не обязательно что-то на
них писать – достаточно лишь помнить, куда какую монету по ходу работы перекладывают.
Для начала взвесим (1, 2, 3, 4) и (5, 6, 7, 8) монеты. Если левая чаша весов (с монетами 1, 2, 3,
4) тяжелее, то на шаге 2.1 взвешиваем (3, 8, 9) и (4, 6, 7) монеты. Если и тогда левая чашка
тяжелее, то – последнее взвешивание 3.1 – (1, 7, 8, 9) и (2, 4, 5, 6). Вывод из этой ветви: если
левая чашка тяжелее, то фальшивая монета с номером 6; если левая чашка легче, то
фальшивая монета – 7; если на чашках равенство, то фальшивая монета – 3. Но если левая
чашка во втором взвешивании легче, то последнее взвешивание 3.2 – (1, 7, 8, 9) и (2, 3, 5, 6),
и тогда: если левая чашка легче, то фальшивая монета – 8; если на чашках равенство, то
фальшивая – 4. Если же на чашках равенство во взвешивании 2.1, то – последнее
взвешивание 3.3 – (1, 7, 8, 9) и (2, 3, 4, 6); тогда: если слева тяжелее, то фальшивая – 1; если
слева легче, то фальшивая – 2, если на чашках равенство, то фальшивая – 5. Теперь вернёмся
к первому взвешиванию, и если левая чашка легче, то взвешиваем – 2.2 – (3, 8, 9) и (4, 6, 7)
монеты; тогда если левая чашка тяжелее, то последнее взвешивание (1, 7, 8, 9) и (2, 3, 5, 6), и
при этом: если левая тяжелее, то фальшивая монета – 8; если равенство, то фальшивая – 4.
Но если левая чашка во втором взвешивании легче, то последнее взвешивание (1, 7, 8, 9) и (2,
4, 5, 6); тогда, если слева тяжелее, то фальшивая – 7, если слева легче, то фальшивая – 6, если
равенство весов, то фальшивая – 3. Если же во взвешивании 2.2 равенство, то последнее
взвешивание (1, 7, 8, 9) и (2, 3, 4, 6), тогда: если слева тяжелее, то фальшивая – 2; если слева
легче, фальшивая – 1; если равенство, фальшивая 5. Наконец, если в первом взвешивании
равенство, то фальшивая – 9. И у нас есть в резерве взвешивание (даже два!), чтобы узнать,
легче она или тяжелее, чем настоящая.
Однако, можно решить задачу и логически. Возьмём и разделим девять монет на три
кучки по три монеты. Сравним вес любых двух кучек. Если эти две кучки равны по весу, то
дальше всё просто. Берём ту кучку, что осталась и сравниваем с любой из ранее взвешенных.
Так мы узнаём, что означает «фальшивость». Легче или тяжелее третья триада монет, чем
первая или вторая. Осталось третье взвешивание. Кладём на чаши весов по одной монете из
третьей кучки. Если весы уравновешены – оставшаяся вне весов монета фальшивая. Если
весы не в равновесии – фальшива та монета, что либо тяжелее, либо легче (как это выявлено
на втором взвешивании).
Но допустим, что при первом взвешивании первая порция из трёх монет не совпадает
по весу со второй порцией из трёх монет. Снимем любые три монеты с чаши весов, на
освободившуюся чашу положив ранее невзвешенные три монеты третьей порции. Если весы
в равновесии, фальшивая монета осталась в той тройке монет, что мы сняли, и уже из
первого взвешивания было видно, легче, или тяжелее фальшивая монета. Дальнейшее
решение очевидно, оно приведено абзацем выше. Допустим, однако, что мы снова не
угадали, и весы при втором взвешивании снова не в равновесии. Но это означает, что тройка
монет, которая два взвешивания находилась на весах содержит фальшивую монету, а те
монеты, которые мы снимали с весов – натуральные. Далее поступаем по аналогии. Кладём
на чаши весов по одной монете из этой самой кучки. Если весы уравновешены – оставшаяся
вне весов монета фальшивая. Если весы не в равновесии – фальшива та монета, что либо
тяжелее, либо легче (как это выявлено при предыдущих двух взвешиваниях).
153 Отложим в сторону тринадцатую монету, а остальные обозначим (буквами – так,
чтобы знающим английский язык легче было запомнить порядок взвешиваний) следующим
образом: FAKE MIND CLOT. Теперь последовательно кладём на весы четвёрки монет в
таком порядке: MA DO – LIKE, ME TO – FIND, FAKE – COIN. И уже просто найти
фальшивую монету, если она входит в эти двенадцать монет. К примеру, если результаты
трёх взвешиваний последовательно были: «слева легче», «равно», «слева легче», то
фальшивой может быть только монета «A», которая легче других. А что если фальшивой
окажется всё-таки отложенная нами, тринадцатая монета? Всё очень просто: в этом случае
при всех трёх взвешиваниях весы будут сбалансированы. К сожалению, в этом случае нам не
узнать, легче или тяжелее тринадцатая монета, но в условии такого требования и не было.
154 Разделим 10 монет на 2 равных кучки – по 5 монет. Положим на чаши весов.
Определим, в какой из этих кучек находится фальшивая монета. Теперь эту кучку делим на 3
кучки – в двух из них по две монеты, в третьей одна монета. Взвешиваем кучки, в которых
по две монеты. Если весы покажут равенство, то фальшивка в третьей кучке. Если покажут
неравенство, то фальшивая монета в кучке, которая легче. Теперь кладём на чаши весов по 1
монете из нужной кучки – более лёгкая монета, как задано в условии, фальшивая. Задача
решена.
155 Берём по два кольца и кладём каждую пару на свою чашу весов. Если чаши
находятся в равновесии, то оставшееся кольцо фальшивое. Если чаши не в равновесии, то
выбираем ту пару колец, что в сумме легче. Сравниваем кольца из выбранной пары на тех же
весах, выбираем более лёгкое – фальшивое.
156 Разница в ответах объясняется тем, что в реальности на обезьяну и груз действует
множество факторов. Можно ли пренебречь инерцией, силой трения в узле верёвка/блок,
весом верёвки, в избытке оказавшемся на одной из сторон, и т. д.? Проще рассмотреть
спускающуюся обезьяну. На неё действуют сила тяжести и сила трения со стороны верёвки.
Обезьяна начинает двигаться с ускорением – следовательно, сила трения меньше силы
тяжести. На верёвку со стороны обезьяны действует сила трения рук обезьяны (вниз) и сила
натяжения (вверх). В силу невесомости верёвки эти силы равны. На груз с противоположной
стороны действует сила тяжести, равная силе тяжести, действующей на обезьяну, и сила
натяжения нити, равная (в силу невесомости и нерастяжимости верёвки) силе натяжения со
стороны обезьяны. Подводя итог, видим: сила тяжести, действующая на груз, больше силы
натяжения, поскольку сила натяжения равна силе трения, а последняя меньше силы тяжести,
действующей на обезьяну. Вывод: груз будет опускаться с тем же ускорением, что и
обезьяна (относительно любой инерциальной системы отсчёта – например, связанной с
потолком). Значит, если обезьяна полезет вверх, и если блок и канат идеальные (блок без
трения и инерции, канат абсолютно нерастяжимый и невесомый), то груз будет подниматься
вверх, в общем, с той же скоростью, с какой поднимается вверх относительно пола обезьяна.
Если же канат считать имеющим вес, но по-прежнему абсолютно нерастяжимым, а блок попрежнему идеальным, да ещё предположить, что изначально обезьяна и груз находились на
одном уровне, то обезьяна поначалу не сможет взбираться наверх – она будет выбирать на
себя канат и под действием веса его избытка, оказавшегося по её сторону блока, опускаться
вниз.
157 Вы, конечно, думаете, что в накладе остался приятель завзятого театрала. Этот
ответ делает Вам честь, но показывает, что Вы слишком хорошего мнения о ростовщике.
Ведь ростовщик в отличие от приятеля имеет свой корыстный интерес. То есть при выкупе
залога придётся отдать ему больше, чем он дал под этот залог. Стало быть, в накладе – в
денежном выражение – остался театрал. Но чего не сделаешь из любви к искусству!
158 Зачем вообще сушат бездымный порох? Пироксилин – основной его компонент –
получают, обрабатывая целлюлозу смесью концентрированных азотной и серной кислот.
Если не отмыть непрореагировавшие остатки кислот, порох будет быстро разлагаться.
Можно, правда, нейтрализовать эти остатки другими реагентами, но продукты реакции тоже
нужно тщательно отмыть. Так что от воды никакими изменениями технологии не
избавиться. И остатки влаги приходится удалять из готового – легко воспламеняющегося –
пороха. При удалении влаги обычной сушкой – нагревом – случаются взрывы. Но есть и
другие способы. Можно, например, выморозить влагу – кристаллики льда постепенно
испаряются и при минусовой температуре. Но без лишних затрат энергии это возможно
лишь зимой – а порох делают круглый год. Вдобавок трещинки, остающиеся на месте
кристалликов, нарушают равномерность горения пороха, создают колебания давления
пламени – это чревато разрывом оружия. А что если промокнуть влагу? Звучит вроде бы
нелепо, но что делает промокашка? Впитывает влагу порами. Адсорбирует. Промокашка
твёрдая. А что обратно адсорбции? Абсорбция. Промокашка жидкая. Может быть, в самом
деле ввести в мокрый порох какое-то жидкое вещество, которое поглотит влагу? Именно
такое решение и предложил Дмитрий Иванович Менделеев: обезвоживать порох спиртом,
жадно поглощающим воду. Это совершенно безопасно, и с тех пор эта операция во всём
мире проводится по способу, разработанному автором периодического закона (изменения
свойств химических элементов).
159 Любая посуда правильной цилиндрической формы, если смотреть на неё сбоку,
представляет собой прямоугольник. Как известно, диагональ прямоугольника делит его на
две равные части. Точно так же цилиндр делится пополам эллипсом, чьи концы касаются
мест стыковки оснований и боковой стороны цилиндра. Из наполненной водой посуды
цилиндрической формы надо отливать воду до тех пор, пока поверхность воды с одной
стороны не достигнет угла посуды, где её дно смыкается со стенкой, а с другой стороны края
посуды, через который она выливается. В этот момент в посуде останется ровно половина
воды.
160 Необходимо перекинуть одну из досок в одном из внешних – тоже прямых – углов
рва параллельно диагонали квадратного поля, а затем, встав на эту доску, положить
оставшуюся на внутренний угол окружённого водой квадрата.
161 Ножниц в ящиках изначально было поровну. Если из второго взято столько,
сколько в первом осталось – значит, во втором осталось столько, сколько из первого взято.
То есть в общей сложности взято столько, сколько было в одном ящике. И столько же – 20
штук – осталось.
162 Совершенно очевидно, что раздвижение границ начального квадратного пруда
приведёт к тому, что дубы попадут в зону затопления. Однако и деревья сдвигать с места
нельзя. Стало быть, новый пруд должен располагаться таким образом, чтобы дубы, как и
прежде, оставались бы на периметре. Поскольку и новый пруд должен быть квадратным, то
все его стороны должны быть равны друг другу по длине, а все углы должны быть прямыми.
Это означает, что дубы должны находиться, как и прежде, на попарно параллельных прямых.
Проведём через вершины первого квадратного пруда четыре прямые, так, чтобы
соблюдались вышеприведённые ограничивающие рассуждения. Мы получим границы
нового квадратного пруда, в площадь которого вписана вдвое меньшая площадь старого
водоёма.
163 Глухота. Он стал плохо слышать и 40 с лишним лет преодолевал этот недуг.
164 Поскольку дотянуться до вкусных листьев невозможно, лоси выжидают, пока
дерево не повалят бобры – и уже после того объедают листву и ветки с лежащего дерева.
165 При получении платы за труд каменщики шептали на ухо выдающему деньги своё
слово, и каждый получал положенное ему жалованье.
166 Каждый раз, переворачивая прошлое, мы получаем настоящее. Откроем чёрный
ящик – там песочные часы.
167 Давно известно: прочность всей цепи определяется самым слабым её звеном.
Значит, все замки должны стать как бы звеньями цепи – тогда при отпирании любого из них
цепь разомкнётся. Зацепим один замок за конец цепи, второй за первый и так далее.
Последний замок зацепим не только за предпоследний, но и за лодку. Теперь дачнику,
отпирающему цепь, надо только позаботиться о том, чтобы остальные замки не попали в
воду: часть из них он оставит в составе цепи на причале, а часть положит в лодку на время
прогулки.
168 Надпись можно увидеть, если вода в реке вновь опустится до того минимального
уровня, который был во время особо жестоких засух, уносивших из-за неурожая тысячи
жизней.
169 Это – фотография! «Фотография мумифицирует время».
170…пламя от свечи замерзало, и его можно было отламывать, как сосульку.
Download